Download as docx, pdf, or txt
Download as docx, pdf, or txt
You are on page 1of 184

CONSTITUTIONAL LAW II

University of Baguio
(2nd Semester, SY 2017-1018)

GENERAL
PRINCIPLES ON THE
CONSITUTION
(Consolidated Case Digests)

1
1. The Bill of Rights and the Fundamental Powers
People of the Philippines vs. Andre Marti
G.R. No. 81561, January 18 1991

Facts:
The appellant and his common law wife, Shirley Reyes, went to the booth of the Manila Packing and
Export Forwarders in the Pistang Filipino Complex Ermita, Manila carrying with them four gift wrapped
packages to be sent in Zurich Switzerland. The proprietress, Anita Reyes (not related to Shirley Reyes) then
asked the appellant if he could examine and expect the packages however appellant refused, assuring her that
the packages simply contained books, cigars, and gloves and were just gifts to a friend. Anita no longer insisted.
Before delivery of appellant’s box to the bureau of Customs and or bureau of Post, Mr. Job Reyes, proprietor
and husband of Anita, following standard procedure opened the boxes for final inspection. When he opened a
peculiar odor emitted therefrom. He squeezed one of the bundles allegedly containing gloves and felt dried
leaves inside. Job prepared a letter reporting the shipment to the NBI and requesting laboratory examination
sample he extracted from the cellophane. Therefore, job and three NBI agents and a photographer went to the
Reyes’ office at Ermita. Job brought out the box in which appellants’ packages were places and in the presence
of the NBI agents, open the top flaps, removed the Styrofoam and took out the cellophane wrappers from
inside the gloves. Dried marijuana leaves are found inside the cellophane.

Issue: Whether or not there is violation of appellant’s constitutional right against unreasonable search and
seizure.

Ruling:
The Supreme Court held that it is not the NBI who made the search. Records of the case clearly indicate
that it was Mr. Job who made search and inspection of the said packages. Said inspection was reasonable and a
standard operating procedure on the part of Mr. Job as a precautionary measure before delivery of packages to
the Bureau of Custom or Post. If the search is made upon the request of law enforces, a warrant must generally
must be secured first if it to pass the test of constitutionality. However, if the search is made in the behest or
initiative of the proprietor of a private establishment for its own and private purpose, as in the case at bar, and
without the intervention of the police authorities, the right against unreasonable search and seizure cannot be
invoked for only the act of private individual, not the law enforcer, is involved.
In sum, the protection against unreasonable search and seizure cannot be extended to acts committed
by private individual as to bring it within the ambit of alleged unlawful intrusion by the government.
The alleged violation against unreasonable search and seizure may only invoked against the State by an
individual unjustly traduced by the exercise by the sovereign authority.
The judgment of conviction finding appellant guilty beyond reasonable doubt of the crime charged was
AFFIRMED.

Ermita Malate v City of Manila 20 SCRA 849 (1967)

Facts:
On June 13, 1963, the Municipal Board of Manila passed Ordinance No. 4760 with the following
provisions questioned for its violation of due process:
1. refraining from entertaining or accepting any guest or customer unless it fills out a prescribed form in
the lobby in open view;
2. prohibiting admission of less than 18 years old;
3. usurious increase of license fee to P4,500 and 6,000 o 150% and 200% respectively (tax issue also);
4. making unlawful lease or rent more than twice every 24 hours; and
5. cancellation of license for subsequent violation.

Ermita-Malate Hotel and Motel Operators Association, and one of its members Hotel del Mar Inc.
petitioned for the prohibition of Ordinance 4670 on June 14, 1963 to be applicable in the city of Manila.
2
They claimed that the ordinance was beyond the powers of the Manila City Board to regulate due to the
fact that hotels were not part of its regulatory powers. They also asserted that Section 1 of the challenged
ordinance was unconstitutional and void for being unreasonable and violative of due process insofar because it
would impose P6,000.00 license fee per annum for first class motels and P4,500.00 for second class motels;
there was also the requirement that the guests would fill up a form specifying their personal information.
There was also a provision that the premises and facilities of such hotels, motels and lodging houses
would be open for inspection from city authorites. They claimed this to be violative of due process for being
vague.
The law also classified motels into two classes and required the maintenance of certain minimum
facilities in first class motels such as a telephone in each room, a dining room or, restaurant and laundry. The
petitioners also invoked the lack of due process on this for being arbitrary.
The lower court declared the ordinance unconstitutional. Hence, this appeal by the city of Manila.

Issue: Whether Ordinance No. 4760 of the City of Manila is violative of the due process clause?

Held: No. Judgment reversed.

Ratio:
"The presumption is towards the validity of a law.” However, the Judiciary should not lightly set aside
legislative action when there is not a clear invasion of personal or property rights under the guise of police
regulation.
O'Gorman & Young v. Hartford Fire Insurance Co- Case was in the scope of police power. As underlying
questions of fact may condition the constitutionality of legislation of this character, the resumption of
constitutionality must prevail in the absence of some factual foundation of record for overthrowing the
statute." No such factual foundation being laid in the present case, the lower court deciding the matter on the
pleadings and the stipulation of facts, the presumption of validity must prevail and the judgment against the
ordinance set aside.”
There is no question but that the challenged ordinance was precisely enacted to minimize certain
practices hurtful to public morals, particularly fornication and prostitution. Moreover, the increase in the
licensed fees was intended to discourage "establishments of the kind from operating for purpose other than
legal" and at the same time, to increase "the income of the city government."
Police power is the power to prescribe regulations to promote the health, morals, peace, good order,
safety and general welfare of the people. In view of the requirements of due process, equal protection and other
applicable constitutional guaranties, however, the power must not be unreasonable or violative of due process.
There is no controlling and precise definition of due process. It has a standard to which the
governmental action should conform in order that deprivation of life, liberty or property, in each appropriate
case, be valid. What then is the standard of due process which must exist both as a procedural and a
substantive requisite to free the challenged ordinance from legal infirmity? It is responsiveness to the
supremacy of reason, obedience to the dictates of justice. Negatively put, arbitrariness is ruled out and
unfairness avoided.
Due process is not a narrow or "technical conception with fixed content unrelated to time, place and
circumstances," decisions based on such a clause requiring a "close and perceptive inquiry into fundamental
principles of our society." Questions of due process are not to be treated narrowly or pedantically in slavery to
form or phrase.
Nothing in the petition is sufficient to prove the ordinance’s nullity for an alleged failure to meet the
due process requirement.
Cu Unjieng case: Licenses for non-useful occupations are also incidental to the police power and the
right to exact a fee may be implied from the power to license and regulate, but in fixing amount of the license
fees the municipal corporations are allowed a much wider discretion in this class of cases than in the former,
and aside from applying the well-known legal principle that municipal ordinances must not be unreasonable,
oppressive, or tyrannical, courts have, as a general rule, declined to interfere with such discretion. Eg. Sale of
liquors.

Lutz v. Araneta- Taxation may be made to supplement the state’s police power.

3
In one case- “much discretion is given to municipal corporations in determining the amount," here the
license fee of the operator of a massage clinic, even if it were viewed purely as a police power measure.
On the impairment of freedom to contract by limiting duration of use to twice every 24 hours- It was
not violative of due process. 'Liberty' as understood in democracies, is not license; it is 'liberty regulated by
law.' Implied in the term is restraint by law for the good of the individual and for the greater good of the peace
and order of society and the general well-being.
Laurel- The citizen should achieve the required balance of liberty and authority in his mind through
education and personal discipline, so that there may be established the resultant equilibrium, which means
peace and order and happiness for all.
The freedom to contract no longer "retains its virtuality as a living principle, unlike in the sole case of
People v Pomar. The policy of laissez faire has to some extent given way to the assumption by the government
of the right of intervention even in contractual relations affected with public interest.
What may be stressed sufficiently is that if the liberty involved were freedom of the mind or the person,
the standard for the validity of governmental acts is much more rigorous and exacting, but where the liberty
curtailed affects at the most rights of property, the permissible scope of regulatory measure is wider.
On the law being vague on the issue of personal information, the maintenance of establishments, and
the “full rate of payment”- Holmes- “We agree to all the generalities about not supplying criminal laws with
what they omit but there is no canon against using common sense in construing laws as saying what they
obviously mean."

PEOPLE VS. JUDGE AYSON [175 SCRA 216; G.R. NO. 85215; 7 JUL 1989

Facts:
Felipe Ramos was a ticket freight clerk of the Philippine Airlines, assigned at its Baguio City station. It
was alleged that he was involved in irregularities in the sales of plane tickets, the PAL management notified
him of an investigation to be conducted. That investigation was scheduled in accordance with PAL's Code of
Conduct and Discipline, and the Collective Bargaining Agreement signed by it with the Philippine Airlines
Employees' Association (PALEA) to which Ramos pertained. A letter was sent by Ramos stating his willingness
to settle the amount of P76,000. The findings of the Audit team were given to him, and he refuted that he
misused proceeds of tickets also stating that he was prevented from settling said amounts. He proffered a
compromise however this did not ensue. Two months after a crime of estafa was charged against Ramos.
Ramos pleaded not guilty. Evidence by the prosecution contained Ramos’ written admission and statement, to
which defendants argued that the confession was taken without the accused being represented by a lawyer.
Respondent Judge did not admit those stating that accused was not reminded of his constitutional rights to
remain silent and to have counsel. A motion for reconsideration filed by the prosecutors was denied. Hence,
this appeal.

Issue:
Whether or Not the respondent Judge correct in making inadmissible as evidence the admission and
statement of accused.

Held:
No. Section 20 of the 1987 constitution provides that the right against self-incrimination (only to
witnesses other than accused, unless what is asked is relating to a different crime charged- not present in case
at bar).
This is accorded to every person who gives evidence, whether voluntarily or under compulsion of
subpoena, in any civil, criminal, or administrative proceeding. The right is not to "be compelled to be a witness
against himself.” It prescribes an "option of refusal to answer incriminating questions and not a prohibition of
inquiry." the right can be claimed only when the specific question, incriminatory in character, is actually put to
the witness. It cannot be claimed at any other time. It does not give a witness the right to disregard a subpoena,
to decline to appear before the court at the time appointed, or to refuse to testify altogether. It is a right that a
witness knows or should know. He must claim it and could be waived.
Rights in custodial interrogation as laid down in miranda v. Arizona: the rights of the accused include:
1) he shall have the right to remain silent and to counsel, and to be informed of such right.
4
2) nor force, violence, threat, intimidation, or any other means which vitiates the free will shall be used
against him.
3) any confession obtained in violation of these rights shall be inadmissible in evidence.

The individual may knowingly and intelligently waive these rights and agree to answer or make a
statement. But unless and until such rights and waivers are demonstrated by the prosecution at the trial, no
evidence obtained as a result of interrogation can be used against him.

Mirasol vs. DPWH

Facts:
On 19 February 1968, Secretary Antonio V. Raquiza of the Department of Public Works and
Communications issued AO 1, which, among others, prohibited motorcycles on limited access highways.
Accordingly, petitioners filed an Amended Petition on February 8, 2001 wherein petitioners sought the
declaration of nullity of the aforesaid administrative issuances. Moreover, petitioners prayed for the issuance of
a temporary restraining order and/or preliminary injunction to prevent the enforcement of the total ban on
motorcycles along the entire breadth of North and South Luzon Expressways and the Manila-Cavite (Coastal
Road) Toll Expressway under DO 215.

Issue:
Is DPWH Administrative Order No.1, DO 74 violative of the right to travel? Are all motorized vehicles
“created equal”?

Held:
DO 74 and DO 215 are void because the DPWH has no authority to declare certain expressways as
limited access facilities. Under the law, it is the DOTC which is authorized to administer and enforce all laws,
rules and regulations in the field of transportation and to regulate related activities. The DPWH cannot
delegate a power or function which it does not possess in the first place.
We find that it is neither warranted nor reasonable for petitioners to say that the only justifiable
classification among modes of transport is the motorized against the non-motorized. Not all motorized vehicles
are created equal. A 16-wheeler truck is substantially different from other light vehicles. The first may be
denied access to some roads where the latter are free to drive. Old vehicles may be reasonably differentiated
from newer models.46 We find that real and substantial differences exist between a motorcycle and other
forms of transport sufficient to justify its classification among those prohibited from plying the toll ways.
Amongst all types of motorized transport, it is obvious, even to a child, that a motorcycle is quite different from
a car, a bus or a truck. The most obvious and troubling difference would be that a two-wheeled vehicle is less
stable and more easily overturned than a four-wheeled vehicle.

CRISOSTOMO B. AQUINO vs. MUNICIPALITY OF MALAY AKLAN,


G.R. No. 211356; September 29, 2014

DOCTRINE:
Based on law and jurisprudence, the office of the mayor has quasi-judicial powers to order the closing
and demolition of establishments. This power granted by the LGC, as earlier explained, We believe, is not the
same power devolved in favor of the LGU under Sec. 17 (b)(2)(ii), as above-quoted, which is subject to review
by the DENR. The fact that the building to be demolished is located within a forestland under the
administration of the DENR is of no moment, for what is involved herein, strictly speaking, is not an issue on
environmental protection, conservation of natural resources, and the maintenance of ecological balance, but
the legality or illegality of the structure. Rather than treating this as an environmental issue then, focus should
not be diverted from the root cause of this debacle compliance.

FACTS:

5
Petitioner is the president and chief executive officer of Boracay Island West Cove Management
Philippines, Inc. (Boracay West Cove). On January 7, 2010, the company applied for a zoning compliance with
the municipal government of Malay, Aklan.2 While the company was already operating a resort in the area, and
the application sought the issuance of a building permit covering the construction of a three-storey hotel over a
parcel of land measuring 998 sqm. located in Sitio Diniwid, Barangay Balagab, Boracay Island, Malay, Aklan,
which is covered by a Forest Land Use Agreement for Tourism Purposes (FLAgT) issued by the Department of
Environment and Natural Resources (DENR) in favor of Boracay West Cove.
Through a Decision on Zoning dated January 20, 2010, the Municipal Zoning Administrator denied
petitioner’s application on the ground that the proposed construction site was within the “no build zone”
demarcated in Municipal Ordinance 2000-131 (Ordinance).
Petitioner appealed the denial action to the Office of the Mayor but despite follow up, no action was
ever taken by the respondent mayor. A Cease and Desist Order was issued by the municipal government,
enjoining the expansion of the resort, and on June 7, 2011, the Office of the Mayor of Malay, Aklan issued the
assailed EO 10, ordering the closure and demolition of Boracay West Cove’s hotel.
Petitioner filed a Petition for Certiorari with prayer for injunctive relief with the CA Alleging that the
order was issued and executed with grave abuse of discretion.
PETITIONER CONTENTION: The hotel cannot summarily be abated because it is not a nuisance per
se, given the hundred million peso-worth of capital infused in the venture. And the Municipality of Malay,
Aklan should have first secured a court order before proceeding with the demolition.
RESPONDENTS CONTENTION: The demolition needed no court order because the municipal mayor
has the express power under the Local Government Code (LGC) to order the removal of illegally constructed
buildings.

ISSUE:
Whether or not a judicial proceedings be conducted first before the LGU can order the closure and
demolition of the property in question.

RULING:
Generally, LGUs have no power to declare a particular thing as a nuisance unless such a thing is a
nuisance per se.
Despite the hotel’s classification as a nuisance per accidens, however, we still find in this case that the
LGU may nevertheless properly order the hotel’s demolition. This is because, in the exercise of police power
and the general welfare clause, property rights of individuals may be subjected to restraints and burdens in
order to fulfill the objectives of the government. Otherwise stated, the government may enact legislation that
may interfere with personal liberty, property, lawful businesses and occupations to promote the general
welfare.
Under the law, insofar as illegal constructions are concerned, the mayor can, after satisfying the
requirement of due notice and hearing, order their closure and demolition.
One such piece of legislation is the LGC, which authorizes city and municipal governments, acting
through their local chief executives, to issue demolition orders. Under existing laws, the office of the mayor is
given powers not only relative to its function as the executive official of the town; it has also been endowed with
authority to hear issues involving property rights of individuals and to come out with an effective order or
resolution thereon.20 Pertinent herein is Sec. 444 (b) (3) (vi) of the LGC, which empowered the mayor to order
the closure and removal of illegally constructed establishments for failing to secure the necessary permits.

CITY GOVERNMENT OF QUEZON CITY and CITY COUNCIL OF QUEZON CITY, petitioners, vs.
HON. JUDGE VICENTE G. ERICTA as Judge of the Court of First Instance of Rizal, Quezon City,
Branch XVIII; HIMLAYANG PILIPINO, INC., respondents.

Facts:
Section 9 of Ordinance No. 6118, S-64 provides that at least 6% of the total area of the memorial park
cemetery shall be set aside for the charity burial of deceased persons who are paupers and have been residents
of Quezon City for at least 5 years prior to their death. As such, the Quezon City engineer required the

6
respondent, Himlayang Pilipino Inc, to stop any further selling and/or transaction of memorial park lots in
Quezon City where the owners thereof have failed to donate the required 6% space intended for paupers burial.
The then Court of First Instance and its judge, Hon. Ericta, declared Section 9 of Ordinance No. 6118, S-
64 null and void.
Petitioners argued that the taking of the respondent’s property is a valid and reasonable exercise of
police power and that the land is taken for a public use as it is intended for the burial ground of paupers. They
further argued that the Quezon City Council is authorized under its charter, in the exercise of local police
power, ” to make such further ordinances and resolutions not repugnant to law as may be necessary to carry
into effect and discharge the powers and duties conferred by this Act and such as it shall deem necessary and
proper to provide for the health and safety, promote the prosperity, improve the morals, peace, good order,
comfort and convenience of the city and the inhabitants thereof, and for the protection of property therein.”
On the other hand, respondent Himlayang Pilipino, Inc. contended that the taking or confiscation of
property was obvious because the questioned ordinance permanently restricts the use of the property such that
it cannot be used for any reasonable purpose and deprives the owner of all beneficial use of his property.

Issue:
Is Section 9 of the ordinance in question a valid exercise of the police power?

Held:
No. The Sec. 9 of the ordinance is not a valid exercise of the police power.
Occupying the forefront in the bill of rights is the provision which states that ‘no person shall be
deprived of life, liberty or property without due process of law’ (Art. Ill, Section 1 subparagraph 1,
Constitution). On the other hand, there are three inherent powers of government by which the state interferes
with the property rights, namely-. (1) police power, (2) eminent domain, (3) taxation. These are said to exist
independently of the Constitution as necessary attributes of sovereignty.
An examination of the Charter of Quezon City (Rep. Act No. 537), does not reveal any provision that
would justify the ordinance in question except the provision granting police power to the City. Section 9 cannot
be justified under the power granted to Quezon City to tax, fix the license fee, and regulate such other business,
trades, and occupation as may be established or practiced in the City. The power to regulate does not include
the power to prohibit or confiscate. The ordinance in question not only confiscates but also prohibits the
operation of a memorial park cemetery.
Police power is defined by Freund as ‘the power of promoting the public welfare by restraining and
regulating the use of liberty and property’. It is usually exerted in order to merely regulate the use and
enjoyment of property of the owner. If he is deprived of his property outright, it is not taken for public use but
rather to destroy in order to promote the general welfare. In police power, the owner does not recover from the
government for injury sustained in consequence thereof.
Under the provisions of municipal charters which are known as the general welfare clauses, a city, by
virtue of its police power, may adopt ordinances to the peace, safety, health, morals and the best and highest
interests of the municipality. It is a well-settled principle, growing out of the nature of well-ordered and society,
that every holder of property, however absolute and may be his title, holds it under the implied liability that his
use of it shall not be injurious to the equal enjoyment of others having an equal right to the enjoyment of their
property, nor injurious to the rights of the community. A property in the state is held subject to its general
regulations, which are necessary to the common good and general welfare. Rights of property, like all other
social and conventional rights, are subject to such reasonable limitations in their enjoyment as shall prevent
them from being injurious, and to such reasonable restraints and regulations, established by law, as the
legislature, under the governing and controlling power vested in them by the constitution, may think necessary
and expedient. The state, under the police power, is possessed with plenary power to deal with all matters
relating to the general health, morals, and safety of the people, so long as it does not contravene any positive
inhibition of the organic law and providing that such power is not exercised in such a manner as to justify the
interference of the courts to prevent positive wrong and oppression.
However, in the case at hand, there is no reasonable relation between the setting aside of at least six (6)
percent of the total area of an private cemeteries for charity burial grounds of deceased paupers and the
promotion of health, morals, good order, safety, or the general welfare of the people. The ordinance is actually
a taking without compensation of a certain area from a private cemetery to benefit paupers who are charges of

7
the municipal corporation. Instead of building or maintaining a public cemetery for this purpose, the city
passes the burden to private cemeteries.
The expropriation without compensation of a portion of private cemeteries is not covered by Section
12(t) of Republic Act 537, the Revised Charter of Quezon City which empowers the city council to prohibit the
burial of the dead within the center of population of the city and to provide for their burial in a proper place
subject to the provisions of general law regulating burial grounds and cemeteries. When the Local Government
Code, Batas Pambansa Blg. 337 provides in Section 177 (q) that a Sangguniang panlungsod may “provide for
the burial of the dead in such place and in such manner as prescribed by law or ordinance” it simply authorizes
the city to provide its own city owned land or to buy or expropriate private properties to construct public
cemeteries. This has been the law and practice in the past. It continues to the present. Expropriation, however,
requires payment of just compensation. The questioned ordinance is different from laws and regulations
requiring owners of subdivisions to set aside certain areas for streets, parks, playgrounds, and other public
facilities from the land they sell to buyers of subdivision lots. The necessities of public safety, health, and
convenience are very clear from said requirements which are intended to insure the development of
communities with salubrious and wholesome environments. The beneficiaries of the regulation, in turn, are
made to pay by the subdivision developer when individual lots are sold to home-owners.
WHEREFORE, the petition for review is hereby DISMISSED. The decision of the respondent court is
affirmed.

OFFICE OF SOLICITOR GENERAL v. AYALA LAND INCORPORATED, GR No. 177056, 2009-09-


18

Facts:
Respondents Ayala Land, Robinsons, and Shangri-la maintain and operate shopping malls in various
locations in Metro Manila. Respondent SM Prime constructs, operates, and leases out commercial buildings
and other structures, among which, are SM City, Manila; SM Centerpoint, Sta. Mesa, Manila; SM City, North
Avenue, Quezon City; and SM Southmall, Las Piñas.
The shopping malls operated or leased out by respondents have parking facilities for all kinds of motor
vehicles, either by way of parking spaces inside the mall buildings or in separate buildings and/or adjacent lots
that are solely devoted for use as parking spaces.
Respondents expend for the maintenance and administration of their respective parking facilities.
The parking tickets or cards issued by respondents to vehicle owners contain the stipulation that
respondents shall not be responsible for any loss or damage to the vehicles parked in respondents' parking
facilities.
In 1999, the Senate Committees on Trade and Commerce and on Justice and Human Rights conducted
a joint investigation for the following purposes: (1) to inquire into the legality of the prevalent practice of
shopping malls of charging parking fees; (2) assuming arguendo... that the collection of parking fees was legally
authorized, to find out the basis and reasonableness of the parking rates charged by shopping malls; and (3) to
determine the legality of the policy of shopping malls of denying liability in cases of theft, robbery, or
carnapping,... by invoking the waiver clause at the back of the parking tickets.
In view of the foregoing, the Committees find that the collection of parking fees by shopping malls is
contrary to the National Building Code and is therefor [sic] illegal. While it is true that the Code merely
requires malls to provide parking spaces, without... specifying whether it is free or not, both Committees
believe that the reasonable and logical interpretation of the Code is that the parking spaces are for free.
Respondent SM Prime thereafter received information that, pursuant to Senate Committee Report No.
225, the DPWH Secretary and the local building officials of Manila, Quezon City, and Las Piñas intended to
institute, through the OSG, an action to enjoin respondent SM Prime and... similar establishments from
collecting parking fees, and to impose upon said establishments penal sanctions under Presidential Decree No.
1096, otherwise known as the National Building Code of the Philippines (National Building Code), and its
Implementing Rules and Regulations (IRR).
The RTC then held that there was no sufficient evidence to justify any award for damages.
The RTC finally decreed in its 29 May 2002 Joint Decision in Civil Cases No. 00-1208 and No. 00-1210
that:

8
FOR THE REASONS GIVEN, the Court declares that Ayala Land[,] Inc., Robinsons Land Corporation,
Shangri-la Plaza Corporation and SM Prime Holdings[,] Inc. are not obligated to provide parking spaces in
their malls for the use of their patrons or public in general,... free of charge.
WHEREFORE, premises considered, the instant appeals are DENIED. Accordingly, appealed Decision
is hereby AFFIRMED in toto.

ISSUE:
Whether mall operators should provide parking facilities, free of charge.

Ruling:
No. The petition of OSG to prohibit collection of parking fees is not a valid exercise of the police power
of State. It is not sufficient for the OSG to claim that “the power to regulate and control the use, occupancy, and
maintenance of buildings and structures carries with it the power to impose fees and, conversely, to control,
partially or, as in this case, absolutely, the imposition of such fees.” Firstly, the fees within the power of
regulatory agencies to impose are regulatory fees. It has been settled law in this jurisdiction that this broad and
all-compassing governmental competence to restrict rights of liberty and property carries with it the
undeniable power to collect a regulatory fee. It looks to the enactment of specific measures that govern the
relations not only as between individuals but also as between private parties and the political society. True, if
the regulatory agencies have the power to impose regulatory fees, then conversely, they also have the power to
remove the same. Even so, it is worthy to note that the present case does not involve the imposition by the
DPWH Secretary and local building officials of regulatory fees upon respondents; but the collection by
respondents of parking fees from persons who use the mall parking facilities. Secondly, assuming arguendo
that the DPWH Secretary and local building officials do have regulatory powers over the collection of parking
fees for the use of privately owned parking facilities, they cannot allow or prohibit such collection arbitrarily or
whimsically. Whether allowing or prohibiting the collection of such parking fees, the action of the DPWH
Secretary and local building officials must pass the test of classic reasonableness and propriety of the measures
or means in the promotion of the ends sought to be accomplished.
Without using the term outright, the OSG is actually invoking police power to justify the regulation by
the State, through the DPWH Secretary and local building officials, of privately owned parking facilities,
including the collection by the owners/operators of such facilities of parking fees from the public for the use
thereof. The Court finds, however, that in totally prohibiting respondents from collecting parking fees, the State
would be acting beyond the bounds of police power.
Police power is the power of promoting the public welfare by restraining and regulating the use of
liberty and property. It is usually exerted in order to merely regulate the use and enjoyment of the property of
the owner. The power to regulate, however, does not include the power to prohibit. A fortiori, the power to
regulate does not include the power to confiscate. Police power does not involve the taking or confiscation of
property, with the exception of a few cases where there is a necessity to confiscate private property in order to
destroy it for the purpose of protecting peace and order and of promoting the general welfare; for instance, the
confiscation of an illegally possessed article, such as opium and firearms.
When there is a taking or confiscation of private property for public use, the State is no longer
exercising police power, but another of its inherent powers, namely, eminent domain. Eminent domain enables
the State to forcibly acquire private lands intended for public use upon payment of just compensation to the
owner.
Normally, of course, the power of eminent domain results in the taking or appropriation of title to, and
possession of, the expropriated property; but no cogent reason appears why the said power may not be availed
of only to impose a burden upon the owner of condemned property, without loss of title and possession. It is a
settled rule that neither acquisition of title nor total destruction of value is essential to taking. It is usually in
cases where title remains with the private owner that inquiry should be made to determine whether the
impairment of a property is merely regulated or amounts to a compensable taking. A regulation that deprives
any person of the profitable use of his property constitutes a taking and entitles him to compensation, unless
the invasion of rights is so slight as to permit the regulation to be justified under the police power. Similarly, a
police regulation that unreasonably restricts the right to use business property for business purposes amounts
to a taking of private property, and the owner may recover therefor.
Although in the present case, title to and/or possession of the parking facilities remain/s with
respondents, the prohibition against their collection of parking fees from the public, for the use of said
9
facilities, is already tantamount to a taking or confiscation of their properties. The State is not only requiring
that respondents devote a portion of the latter’s properties for use as parking spaces, but is also mandating that
they give the public access to said parking spaces for free. Such is already an excessive intrusion into the
property rights of respondents. Not only are they being deprived of the right to use a portion of their properties
as they wish, they are further prohibited from profiting from its use or even just recovering therefrom the
expenses for the maintenance and operation of the required parking facilities.
In conclusion, the total prohibition against the collection by respondents of parking fees from persons
who use the mall parking facilities has no basis in the National Building Code or its IRR. The State also cannot
impose the same prohibition by generally invoking police power, since said prohibition amounts to a taking of
respondents’ property without payment of just compensation.
WHEREFORE, the instant Petition for Review on Certiorari is hereby DENIED. The Decision dated 25
January 2007 and Resolution dated 14 March 2007 of the Court of Appeals in CA-G.R. CV No. 76298,
affirming in toto the Joint Decision dated 29 May 2002 of the Regional Trial Court of Makati City, Branch 138,
in Civil Cases No. 00-1208 and No. 00-1210 are hereby AFFIRMED. No costs.

JOSE J. FERRER, JR. VS. CITY MAYOR HERBERT BAUTISTA G.R. No. 210551 June
30, 2015

Facts:
The City of Quezon passed two ordinances namely. The first one was the Socialized Housing Tax of QC
allowing the imposition of special assessment (1/2 of the assessed valued of land in excess of P100k). The
second one was Ordinance No. SP-2235, S-2013 on Garbage Collection Fees imposing fees depending on the
amount of the land or floor area). Jose Ferrer, as a property in Quezon City questioned the validity of the city
ordinances.
According to Ferrer: The city has no power to impose the tax. The SHT violates the rule on equality
because it burdens real property owners with expenses to provide funds for the housing of informal settlers.
The SHT is confiscatory or oppressive. Also, he assails the validity of the garbage fees imposition because: It
violates the rule on double taxation. It violates the rule on equality because the fees are collected from only
domestic households and not from restaurants, food courts, fast food chains, and other commercial dining
places that spew garbage much more than residential property owners.

Issue:
WON the ordinances were valid.

Held:
1st ordinance: Socialized Housing Tax of Quezon City is valid. Cities have the power to tax. It must be
noted that local government units such as cities has the power to tax. The collection for the socialized housing
tax is valid. It must be noted that the collections were made to accrue to the socialized housing programs and
projects of the city. The imposition was for a public purpose (exercise of power of taxation + police power). In
this case, there was both an exercise of the power to tax (primary) and police power (incidental). Removing
slum areas in Quezon City is not only beneficial to the underprivileged and homeless constituents but
advantageous to the real property owners as well. The situation will improve the value of the their property
investments, fully enjoying the same in view of an orderly, secure, and safe community, and will enhance the
quality of life of the poor, making them law-abiding constituents and better consumers of business products.
There is no violation of the rule on equality
Note: There is a substantial distinction between: real property owner and an informal settler. In fact,
the Supreme Court said that the disparity is so obvious. It is inherent in the power to tax that a State is free to
select the subjects of taxation. Inequities which result from a singling out of one particular class for taxation or
exemption infringe no constitutional limitation.
All these requisites are complied with: An ordinance based on reasonable classification does not violate
the constitutional guaranty of the equal protection of the law. The requirements for a valid and reasonable
classification are: (1) it must rest on substantial distinctions; (2) it must be germane to the purpose of the law;
(3) it must not be limited to existing conditions only; and (4) it must apply equally to all members of the same
class. The ordinance is not oppressive or confiscatory. The ordinance is also not oppressive since the tax rate
10
being imposed is consistent with the UDHA (Urban Development and Housing Act of 1992). While the law
authorizes LGUs to collect SHT on properties with an assessed value of more than P50,000.00, the questioned
ordinance only covers properties with an assessed value exceeding P100,000.00. As well, the ordinance
provides for a tax credit equivalent to the total amount of the special assessment paid by the property owner
beginning in the sixth (6th) year of the effectivity of the ordinance.

2nd ordinance: The imposition of garbage fee is invalid. Note: There was no violation of double taxation
but there was a violation of the rule on equity. There is no violation of double taxation: the garbage fees are not
taxes. In Progressive Development Corporation v. Quezon City, the Court declared that: "if the generating of
revenue is the primary purpose and regulation is merely incidental, the imposition is a tax; but if regulation is
the primary purpose, the fact that incidentally revenue is also obtained does not make the imposition a tax."
Contention of Ferrer: that the imposition of garbage fee is tantamount to double taxation because
garbage collection is a basic and essential public service that should be paid out from property tax, business
tax, transfer tax, amusement tax, community tax certificate, other taxes, and the IRA of the Quezon City
Government. All these are valid taxes. The garbage fees are license fees.

Footnote: In order to constitute double taxation in the objectionable or prohibited sense the same property
must be taxed twice when it should be taxed but once; both taxes must be imposed on the same property or
subject-matter, for the same purpose, by the same State, Government, or taxing authority, within the same
jurisdiction or taxing district, during the same taxing period, and they must be the same kind or character of
tax.
There is a violation of the rule on equality: no substantial distinction. There is no substantial distinction
between an occupant of a lot, on one hand, and an occupant of a unit in a condominium, socialized housing
project or apartment, on the other hand. Most likely, garbage output produced by these types of occupants is
uniform and does not vary to a large degree; thus, a similar schedule of fee is both just and equitable.

The garbage fees or rates are unjust and inequitable


A resident of a 200 sq. m. unit in a condominium or socialized housing project has to pay twice the
amount than a resident of a lot similar in size; unlike unit occupants, all occupants of a lot with an area of 200
sq. m. and less have to pay a fixed rate of Php100.00; and the same amount of garbage fee is imposed
regardless of whether the resident is from a condominium or from a socialized housing project.
The classifications are not germane to the purpose of the ordinance. The declared purpose is:
"promoting shared responsibility with the residents to attack their common mindless attitude in over-
consuming the present resources and in generating waste."
Instead of simplistically categorizing the payee into land or floor occupant of a lot or unit of a
condominium, socialized housing project or apartment, respondent City Council should have considered
factors that could truly measure the amount of wastes generated and the appropriate fee for its collection.
Factors include, among others, household age and size, accessibility to waste collection, population density of
the barangay or district, capacity to pay, and actual occupancy of the property.

SC:
→ Validity of Socialized Housing Tax of Quezon City is upheld..
→ Ordinance No. SP-2235, S-2013, which collects an annual garbage fee on all domestic households in Quezon
City, is unconstitutional and illegal.

PHILIPPINE BLOOMING MILLS EMPLOYMENT ORGANIZATION vs. PHILIPPINE BLOOMING


MILLS CO., INC.

Facts:
The petitioner Philippine Blooming Mills Employees Organization (PBMEO) is a legitimate labor union
composed of the employees of the respondent Philippine Blooming Mills Co., Inc., and petitioners Nicanor
Tolentino, Florencio Padrigano, Rufino Roxas, Mariano de Leon, Asencion Paciente, Bonifacio Vacuna,
Benjamin Pagcu and Rodulfo Munsod are officers and members of the petitioner Union. PBMEO decided to

11
stage a mass demonstration in front of Malacañang to express their grievances against the alleged abuses of the
Pasig Police.
Petitioners claim that on March 1, 1969, they decided to stage a mass demonstration at Malacañang on
March 4, 1969, in protest against alleged abuses of the Pasig police, to be participated in by the workers in the
first shift (from 6 A.M. to 2 P.M.) as well as those in the regular second and third shifts (from 7 A.M. to 4 P.M.
and from 8 A.M. to 5 P.M., respectively); and that they informed the respondent Company of their proposed
demonstration.
The Philippine Blooming Mills Inc., called for a meeting with the leaders of the PBMEO after learning
about the planned mass demonstration. During the meeting, the planned demonstration was confirmed by the
union. But it was stressed out by the union that the demonstration was not a strike against the company but
was in factual exercise of the laborers inalienable constitutional right to freedom of expression, freedom of
speech and freedom for petition for redress of grievances.
The company asked them to cancel the demonstration for it would interrupt the normal course of their
business which may result in the loss of revenue. This was backed up with the threat of the possibility that the
workers would lose their jobs if they pushed through with the rally.
A second meeting took place where the company reiterated their appeal that while the workers may be
allowed to participate, those from the 1st and regular shifts should not absent themselves to participate,
otherwise, they would be dismissed. Since it was too late to cancel the plan, the rally took place and the officers
of the PBMEO were eventually dismissed for a violation of the ‘No Strike and No Lockout’ clause of their
Collective Bargaining Agreement.
The lower court decided in favour of Philippine Blooming Mills Co., Inc., and the officers of the PBMEO
were found guilty of bargaining in bad faith. The PBMEO’s motion for reconsideration was subsequently
denied by the Court of Industrial Relations for being filed two days late.

Issue:
Whether or not to regard the demonstration against police officers, not against the employer, as a
violation of freedom expression in general and of their right of assembly and petition for redress of grievances.
Whether or not the collective bargaining agreement is an inhibition of the rights of free expression, free
assembly and petition of the employers

Held:
Property and property rights can be lost thru prescription; but human rights are imprescriptible. If
human rights are extinguished by the passage of time, then the Bill of Rights is a useless attempt to limit the
power of government and ceases to be an efficacious shield against the tyranny of officials, of majorities, of the
influential and powerful, and of oligarchs — political, economic or otherwise.
The demonstration held petitioners on March 4, 1969 before Malacañang was against alleged abuses of
some Pasig policemen, not against their employer, herein private respondent firm, said demonstrate was
purely and completely an exercise of their freedom expression in general and of their right of assembly and
petition for redress of grievances in particular before appropriate governmental agency, the Chief Executive,
again the police officers of the municipality of Pasig. They exercise their civil and political rights for their
mutual aid protection from what they believe were police excesses. As matter of fact, it was the duty of herein
private respondent firm to protect herein petitioner Union and its members from the harassment of local police
officers. It was to the interest herein private respondent firm to rally to the defense of, and take up the cudgels
for, its employees, so that they can report to work free from harassment, vexation or peril and as consequence
perform more efficiently their respective tasks enhance its productivity as well as profits.
To regard the demonstration against police officers, not against the employer, as evidence of bad faith
in collective bargaining and hence a violation of the collective bargaining agreement and a cause for the
dismissal from employment of the demonstrating employees, stretches unduly the compass of the collective
bargaining agreement, is “a potent means of inhibiting speech” and therefore inflicts a moral as well as mortal
wound on the constitutional guarantees of free expression, of peaceful assembly and of petition.
The collective bargaining agreement which fixes the working shifts of the employees, according to the
respondent Court Industrial Relations, in effect imposes on the workers the “duty … to observe regular working
hours.” The strain construction of the Court of Industrial Relations that a stipulated working shifts deny the
workers the right to stage mass demonstration against police abuses during working hours, constitutes a virtual

12
tyranny over the mind and life the workers and deserves severe condemnation. Renunciation of the freedom
should not be predicated on such a slender ground.
The respondent company is the one guilty of unfair labor practice. Because the refusal on the part of the
respondent firm to permit all its employees and workers to join the mass demonstration against alleged police
abuses and the subsequent separation of the eight (8) petitioners from the service constituted an
unconstitutional restraint on the freedom of expression, freedom of assembly and freedom petition for redress
of grievances, the respondent firm committed an unfair labor practice defined in Section 4(a-1) in relation to
Section 3 of Republic Act No. 875, otherwise known as the Industrial Peace Act. Section 3 of Republic Act No. 8
guarantees to the employees the right “to engage in concert activities for … mutual aid or protection”; while
Section 4(a-1) regards as an unfair labor practice for an employer interfere with, restrain or coerce employees
in the exercise their rights guaranteed in Section Three.
The Supreme Court set aside as null and void the orders of Court of Industrial Relations. The Supreme
Court also directed the re-instatement of the herein eight (8) petitioners, with full back pay from the date of
their separation from the service until re-instated, minus one day’s pay and whatever earnings they might have
realized from other sources during their separation from the service.

13
2. Due Process (Procedural, Substantive,
Administrative)
REPUBLIC OF THE PHILIPPINES vs. JENNIFER CAGANDAHAN
GR No. 166676, September 12, 2008

FACTS:
Jennifer Cagandahan filed before the Regional Trial Court Branch 33 of Siniloan, Laguna a Petition for
Correction of Entries in Birth Certificate of her name from Jennifer B. Cagandahan to Jeff Cagandahan and her
gender from female to male. It appearing that Jennifer Cagandahan is sufferingfrom Congenital Adrenal
Hyperplasia which is a rare medical condition where afflicted persons possess both male and female
characteristics. Jennifer Cagandahan grew up with secondary male characteristics. To further her petition,
Cagandahan presented in court the medical certificate evidencing that she is suffering from Congenital Adrenal
Hyperplasia which certificate is issued by Dr. Michael Sionzon of the Department of Psychiatry, University of
the Philippines-Philippine General Hospital, who, in addition, explained that “Cagandahan genetically is
female but because her body secretes male hormones, her female organs did not develop normally, thus has
organs of both male and female.” The lower court decided in her favor but the Office of the Solicitor General
appealed before the Supreme Court invoking that the same was a violation of Rules 103 and 108 of the Rules of
Court because the said petition did not implead the local civil registrar.

ISSUE:
Whether or not Cagandahan’s sex as appearing in her birth certificate be changed.

RULING:
The Supreme Court affirmed the decision of the lower court. It held that, in deciding the case, the
Supreme Court considered “the compassionate calls for recognition of the various degrees of intersex as
variations which should not be subject to outright denial.” The Supreme Court made use of the availale
evidence presented in court including the fact that private respondent thinks of himself as a male and as to the
statement made by the doctor that Cagandahan’s body produces high levels of male hormones (androgen),
which is preponderant biological support for considering him as being male.”
The Supreme Court further held that they give respect to (1) the diversity of nature; and (2) how an
individual deals with what nature has handed out. That is, the Supreme Court respects the respondent’s
congenital condition and his mature decision to be a male. Life is already difficult for the ordinary person. The
Court added that a change of name is not a matter of right but of judicial discretion, to be exercised in the light
of the reasons and the consequences that will follow.

ESTRADA VS SANDIGANBAYAN
369 SCRA 394

Facts: Petitioner Joseph Ejercito Estrada, the highest-ranking official to be prosecuted under RA 7080 (An Act
Defining and Penalizing the Crime of Plunder), 1 as amended by RA 7659, 2 wishes to impress upon us that the
assailed law is so defectively fashioned that it crosses that thin but distinct line which divides the valid from the
constitutionally infirm. He therefore makes a stringent call for this Court to subject the Plunder Law to the
crucible of constitutionality mainly because, according to him, (a) it suffers from the vice of vagueness; (b) it
dispenses with the "reasonable doubt" standard in criminal prosecutions; and, (c) it abolishes the element of
mens rea in crimes already punishable under The Revised Penal Code, all of which are purportedly clear
violations of the fundamental rights of the accused to due process and to be informed of the nature and cause
of the accusation against him.
That during the period from June, 1998 to January 2001, in the Philippines, and within the jurisdiction
of this Honorable Court, accused Joseph Ejercito Estrada, THE PRESIDENT OF THE REPUBLIC OF THE
PHILIPPINES, by himself AND/OR in CONNIVANCE/CONSPIRACY with his co-accused, WHO ARE

14
MEMBERS OF HIS FAMILY, RELATIVES BY AFFINITY OR CONSANGUINITY, BUSINESS ASSOCIATES,
SUBORDINATES AND/OR OTHER PERSONS, BY TAKING UNDUE ADVANTAGE OF HIS OFFICIAL
POSITION, AUTHORITY, RELATIONSHIP, CONNECTION, OR INFLUENCE, did then and there willfully,
unlawfully and criminally amass, accumulate and acquire BY HIMSELF DIRECTLY OR INDIRECTLY, ill-
gotten wealth in the aggregate amount or TOTAL VALUE of FOUR BILLION NINETY SEVEN MILLION
EIGHT HUNDRED FOUR THOUSAND ONE HUNDRED SEVENTY THREE AND SEVENTEEN CENTAVOS
(P4,097,804,173.17), more or less, THEREBY UNJUSTLY ENRICHING HIMSELF OR THEMSELVES AT THE
EXPENSE AND TO THE DAMAGE OF THE FILIPINO PEOPLE AND THE REPUBLIC OF PHILIPPINES
through ANY OR A combination OR A series of overt OR criminal acts, OR SIMILAR SCHEMES OR MEANS.
RESPECTIVELY OR A TOTAL OF MORE OR LESS ONE BILLION EIGHT HUNDRED FORTY SEVEN
MILLION FIVE HUNDRED SEVENTY EIGHT THOUSAND FIFTY SEVEN PESOS AND FIFTY CENTAVOS
(P1,847,578,057.50); AND BY COLLECTING OR RECEIVING, DIRECTLY OR INDIRECTLY, BY HIMSELF
AND/OR IN CONNIVANCE WITH JOHN DOES JANE DOES, COMMISSIONS OR PERCENTAGES BY
REASON OF SAID PURCHASES OF SHARES OF STOCK IN THE AMOUNT OF ONE HUNDRED EIGHTY
NINE MILLION SEVEN HUNDRED THOUSAND PESOS (P189,700,000.00) MORE OR LESS, FROM THE
BELLE CORPORATION WHICH BECAME PART OF THE DEPOSIT IN THE EQUITABLE BANK UNDER
THE ACCOUNT NAME 'JOSE VELARDE'

Issue: R.A. No. 7080 is unconstitutional on the following grounds:


I. IT VIOLATES THE DUE PROCESS CLAUSE FOR ITS VAGUENESS
II. IT VIOLATES THE CONSTITUTIONAL RIGHT OF THE ACCUSED TO KNOW THE NATURE AND
CAUSE OF THE ACCUSATION AGAINST HIM
III. IT VIOLATES THE DUE PROCESS CLAUSE AND THE CONSTITUTIONAL PRESUMPTION OF
INNOCENCE BY LOWERING THE QUANTUM OF EVIDENCE NECESSARY FOR PROVING THE
COMPONENT ELEMENTS OF PLUNDER
IV. IT IS BEYOND THE CONSTITUTIONAL POWER OF THE LEGISLATURE TO DELIMIT THE
REASONABLE DOUBT STANDARD AND TO ABOLISH THE ELEMENT OF MENS REA IN MALA IN
SE CRIMES BY CONVERTING THESE TO MALA PROHIBITA, IN VIOLATION OF THE DUE
PROCESS CONCEPT OF CRIMINAL RESPONSIBILITY.

Held: PREMISES CONSIDERED, this Court holds that RA 7080 otherwise known as the Plunder Law, as
amended by RA 7659, is CONSTITUTIONAL. Consequently, the petition to declare the law unconstitutional is
DISMISSED for lack of merit. SO ORDERED.

Ratio:

In view of vagueness and ambiguity


Congress is not restricted in the form of expression of its will, and its inability to so define the words
employed in a statute will not necessarily result in the vagueness or ambiguity of the law so long as the
legislative will is clear, or at least, can be gathered from the whole act, which is distinctly expressed in the
Plunder Law. Moreover, it is a well-settled principle of legal hermeneutics that words of a statute will be
interpreted in their natural, plain and ordinary acceptation and signification, 7 unless it is evident that the
legislature intended a technical or special legal meaning to those words 8 The intention of the lawmakers —
who are, ordinarily, untrained philologists and lexicographers — to use statutory phraseology in such a manner
is always presumed. Thus, Webster's New Collegiate Dictionary contains the following commonly accepted
definition of the words "combination" and "series:"
Combination — the result or product of combining; the act or process of combining. To combine is to
bring into such close relationship as to obscure individual characters.
Series — a number of things or events of the same class coming one after another in spatial and
temporal succession.

Verily, had the legislature intended a technical or distinctive meaning for "combination" and "series," it
would have taken greater pains in specifically providing for it in the law. As for "pattern," we agree with the
observations of the Sandiganbayan 9 that this term is sufficiently defined in Sec. 4, in relation to Sec. 1, par. (d),
and Sec. 2. . . under Sec. 1 (d) of the law, a 'pattern' consists of at least a combination or series of overt or
15
criminal acts enumerated in subsections (1) to (6) of Sec. 1 (d). Secondly, pursuant to Sec. 2 of the law, the
pattern of overt or criminal acts is directed towards a common purpose or goal which is to enable the public
officer to amass, accumulate or acquire ill-gotten wealth. And thirdly, there must either be an 'overall unlawful
scheme' or 'conspiracy' to achieve said common goal. As commonly understood, the term 'overall unlawful
scheme' indicates a 'general plan of action or method' which the principal accused and public officer and others
conniving with him, follow to achieve the aforesaid common goal. In the alternative, if there is no such overall
scheme or where the schemes or methods used by multiple accused vary, the overt or criminal acts must form
part of a conspiracy to attain a common goal.
With more reason, the doctrine cannot be invoked where the assailed statute is clear and free from
ambiguity, as in this case. The test in determining whether a criminal statute is void for uncertainty is whether
the language conveys a sufficiently definite warning as to the proscribed conduct when measured by common
understanding and practice. It must be stressed, however, that the "vagueness" doctrine merely requires a
reasonable degree of certainty for the statute to be upheld — not absolute precision or mathematical exactitude,
as petitioner seems to suggest.
Hence, it cannot plausibly be contended that the law does not give a fair warning and sufficient notice of
what it seeks to penalize. Under the circumstances, petitioner's reliance on the "void-for-vagueness" doctrine is
manifestly misplaced. The doctrine has been formulated in various ways, but is most commonly stated to the
effect that a statute establishing a criminal offense must define the offense with sufficient definiteness that
persons of ordinary intelligence can understand what conduct is prohibited by the statute.

In view of due process


On the second issue, petitioner advances the highly stretched theory that Sec. 4 of the Plunder Law
circumvents the immutable obligation of the prosecution to prove beyond reasonable doubt the predicate acts
constituting the crime of plunder when it requires only proof of a pattern of overt or criminal acts showing
unlawful scheme or conspiracy. The running fault in this reasoning is obvious even to the simplistic mind. In a
criminal prosecution for plunder, as in all other crimes, the accused always has in his favor the presumption of
innocence which is guaranteed by the Bill of Rights, and unless the State succeeds in demonstrating by proof
beyond reasonable doubt that culpability lies, the accused is entitled to an acquittal.
What the prosecution needs to prove beyond reasonable doubt is only a number of acts sufficient to
form a combination or series which would constitute a pattern and involving an amount of at least
P50,000,000.00. There is no need to prove each and every other act alleged in the Information to have been
committed by the accused in furtherance of the overall unlawful scheme or conspiracy to amass, accumulate or
acquire ill- gotten wealth.

In view of mens rea


As regards the third issue, again we agree with Justice Mendoza that plunder is a malum in se which
requires proof of criminal intent. Thus, he says, in his Concurring Opinion — . . . Precisely because the
constitutive crimes are mala in se the element of mens rea must be proven in a prosecution for plunder. It is
noteworthy that the amended information alleges that the crime of plunder was committed "willfully,
unlawfully and criminally." It thus alleges guilty knowledge on the part of petitioner.
[With the government] terribly lacking the money to provide even the most basic services to its people,
any form of misappropriation or misapplication of government funds translates to an actual threat to the very
existence of government, and in turn, the very survival of the people it governs over. Viewed in this context, no
less heinous are the effect and repercussions of crimes like qualified bribery, destructive arson resulting in
death, and drug offenses involving government official, employees or officers, that their perpetrators must not
be allowed to cause further destruction and damage to society. Indeed, it would be absurd to treat prosecutions
for plunder as though they are mere prosecutions for violations of the Bouncing Check Law (B.P. Blg. 22) or of
an ordinance against jaywalking, without regard to the inherent wrongness of the acts.
To clinch, petitioner likewise assails the validity of RA 7659, the amendatory law of RA 7080, on
constitutional grounds. Suffice it to say however that it is now too late in the day for him to resurrect this long
dead issue, the same having been eternally consigned by People v. Echegaray 38 to the archives of
jurisprudential history. The declaration of this Court therein that RA 7659 is constitutionally valid stands as a
declaration of the State, and becomes, by necessary effect, assimilated in the Constitution now as an integral
part of it.

16
In view of presumption of innocence
At all events, let me stress that the power to construe law is essentially judicial. To declare what the law
shall be is a legislative power, but to declare what the law is or has been is judicial. Statutes enacted by
Congress cannot be expected to spell out with mathematical precision how the law should be interpreted under
any and all given situations. The application of the law will depend on the facts and circumstances as adduced
by evidence which will then be considered, weighed and evaluated by the courts. Indeed, it is the
constitutionally mandated function of the courts to interpret, construe and apply the law as would give flesh
and blood to the true meaning of legislative enactments.
A construction should be rejected if it gives to the language used in a statute a meaning that does not
accomplish the purpose for which the statute was enacted and that tends to defeat the ends that are sought to
be attained by its enactment. Viewed broadly, "plunder involves not just plain thievery but economic
depredation which affects not just private parties or personal interests but the nation as a whole." Invariably,
plunder partakes of the nature of "a crime against national interest which must be stopped, and if possible,
stopped permanently."

In view of estoppel
Petitioner is not estopped from questioning the constitutionality of R.A. No. 7080. The case at bar has
been subject to controversy principally due to the personalities involved herein. The fact that one of petitioner's
counsels was a co-sponsor of the Plunder Law and petitioner himself voted for its passage when he was still a
Senator would not in any put him in estoppel to question its constitutionality. The rule on estoppel applies to
questions of fact, not of law. Moreover, estoppel should be resorted to only as a means of preventing injustice.
To hold that petitioner is estopped from questioning the validity of R.A. No. 7080 because he had earlier voted
for its passage would result in injustice not only to him, but to all others who may be held liable under this
statute.

What is RICO
Racketeer Influenced and Corrupt Organizations Act is a United States federal law that provides for
extended criminal penalties and a civil cause of action for acts performed as part of an ongoing criminal
organization. RICO was enacted by section 901(a) of the Organized Crime Control Act of 1970 (Pub.L. 91-452,
84 Stat. 922, enacted October 15, 1970). RICO is codified as Chapter 96 of Title 18 of the United States Code, 18
U.S.C. § 1961–1968. While its intended use was to prosecute the Mafia as well as others who were actively
engaged in organized crime, its application has been more widespread.

In view of facial challenge


A facial challenge is allowed to be made to a vague statute and to one which is overbroad because of
possible "chilling effect" upon protected speech. The theory is that "[w]hen statutes regulate or proscribe
speech and no readily apparent construction suggests itself as a vehicle for rehabilitating the statutes in a single
prosecution, the transcendent value to all society of constitutionally protected expression is deemed to justify
allowing attacks on overly broad statutes with no requirement that the person making the attack demonstrate
that his own conduct could not be regulated by a statute drawn with narrow specificity.'
This rationale does not apply to penal statutes. Criminal statutes have general in terrorem effect
resulting from their very existence, and, if facial challenge is allowed for this reason alone, the State may well
be prevented from enacting laws against socially harmful conduct. In the area of criminal law, the law cannot
take chances as in the area of free speech.

In view of burden of proof (accused) according to PANGANIBAN, J.


In sum, the law must be proven to be clearly and unequivocally repugnant to the Constitution before
this Court may declare its unconstitutionality. To strike down the law, there must be a clear showing that what
the fundamental law prohibits, the statute allows to be done. 40 To justify the nullification of the law, there
must be a clear, unequivocal breach of the Constitution; not a doubtful, argumentative implication. 41 Of some
terms in the law which are easily clarified by judicial construction, petitioner has, at best, managed merely to
point out alleged ambiguities. Far from establishing, by clear and unmistakable terms, any patent and glaring
conflict with the Constitution, the constitutional challenge to the Anti-Plunder law must fail. For just as the
accused is entitled to the presumption of innocence in the absence of proof beyond reasonable doubt, so must a
law be accorded the presumption of constitutionality without the same requisite quantum of proof.
17
Petitioner now concludes that the Anti-Plunder Law "eliminates proof of each and every component
criminal act of plunder by the accused and limits itself to establishing just the pattern of over or criminal acts
indicative of unlawful scheme or conspiracy."
All told, the above explanation is in consonance with what is often perceived to be the reality with
respect to the crime of plunder — that "the actual extent of the crime may not, in its breadth and entirety, be
discovered, by reason of the 'stealth and secrecy' in which it is committed and the involvement of 'so many
persons here and abroad and [the fact that it] touches so many states and territorial units."'
"The constitutionality of laws is presumed. To justify nullification of a law, there must be a clear and
unequivocal breach of the Constitution, not a doubtful or argumentative implication; a law shall not be
declared invalid unless the conflict with the Constitution is clear beyond a reasonable doubt. 'The presumption
is always in favor of constitutionality . . . To doubt is to sustain.'

In view of burden of proof (State) according to KAPUNAN, J.


The Constitution guarantees both substantive and procedural due process as well as the right of the
accused to be informed of the nature and cause of the accusation against him. A criminal statute should not be
so vague and uncertain that "men of common intelligence must necessarily guess as to its meaning and differ as
to its application. There are three distinct considerations for the vagueness doctrine. First, the doctrine is
designed to ensure that individuals are properly warned ex ante of the criminal consequences of their conduct.
This "fair notice" rationale was articulated in United States v. Harriss: The constitutional requirement of
definiteness is violated by a criminal statute that fails to give a person of ordinary intelligence fair notice that
his contemplated conduct is forbidden by the statute. The underlying principle is that no man shall be held
criminally responsible for conduct which he could not reasonably understand to be proscribed.
While the dictum that laws be clear and definite does not require Congress to spell out with
mathematical certainty the standards to which an individual must conform his conduct, it is necessary that
statutes provide reasonable standards to guide prospective conduct. And where a statute imposes criminal
sanctions, the standard of certainty is higher. The penalty imposable on the person found guilty of violating
R.A. No. 7080 is reclusion perpetua to death. Given such penalty, the standard of clarity and definiteness
required of R.A. No. 7080 is unarguably higher than that of other laws.
It has been incorrectly suggested that petitioner cannot mount a "facial challenge" to the Plunder Law,
and that "facial" or "on its face" challenges seek the total invalidation of a statute. Fr. Bernas, for his part,
pointed to several problematical portions of the law that were left unclarified. He posed the question: "How can
you have a 'series' of criminal acts if the elements that are supposed to constitute the series are not proved to be
criminal?" The meanings of "combination" and "series" as used in R.A. No. 7080 are not clear.
To quote Fr. Bernas again: "How can you have a 'series' of criminal acts if the elements that are
supposed to constitute the series are not proved to be criminal?" Because of this, it is easier to convict for
plunder and sentence the accused to death than to convict him for each of the component crimes otherwise
punishable under the Revised Penal Code and other laws which are bailable offenses. The resultant absurdity
strikes at the very heart if the constitutional guarantees of due process and equal protection.
The component acts constituting plunder, a heinous crime, being inherently wrongful and immoral, are
patently mala in se, even if punished by a special law and accordingly, criminal intent must clearly be
established together with the other elements of the crime; otherwise, no crime is committed. By eliminating
mens rea, R.A. 7080 does not require the prosecution to prove beyond reasonable doubt the component acts
constituting plunder and imposes a lesser burden of proof on the prosecution, thus paying the way for the
imposition of the penalty of reclusion perpetua to death on the accused, in plain violation of the due process
and equal protection clauses of the Constitution.
It obfuscates the mind to ponder that such an ambiguous law as R.A. No. 7080 would put on the
balance the life and liberty of the accused against whom all the resources of the State are arrayed. It could be
used as a tool against political enemies and a weapon of hate and revenge by whoever wields the levers of
power.

In view of due process according to YNARES-SANTIAGO, J.


It is an ancient maxim in law that in times of frenzy and excitement, when the desire to do justice is
tarnished by anger and vengeance, there is always the danger that vital protections accorded an accused may be
taken away.

18
Substantive due process dictates that there should be no arbitrariness, unreasonableness or ambiguity
in any law which deprives a person of his life or liberty. The trial and other procedures leading to conviction
may be fair and proper. But if the law itself is not reasonable legislation, due process is violated. Thus, an
accused may not be sentenced to suffer the lethal injection or life imprisonment for an offense understood only
after judicial construction takes over where Congress left off, and interpretation supplies its meaning.
The Constitution guarantees both substantive and procedural due process as well as the right of the
accused to be informed of the nature and cause of the accusation against him. Substantive due process requires
that a criminal statute should not be vague and uncertain. More explicitly — That the terms of a penal statute. .
. must be sufficiently explicit to inform those who are subject to it what conduct on their part will render them
liable to penalties, is a well-recognized requirement, consonant alike with ordinary notions of fair play and the
settled rules of law. And a statute which either forbids or requires the doing of an act in terms so vague that
men of common intelligence must necessarily guess at its meaning and differ as to its application, violates the
first essential of due process.
In its early formulation, the overbreadth doctrine states that a governmental purpose to control or
prevent activities constitutionally subject to regulation may not be achieved by means which sweep
unnecessarily broadly and thereby invade the area of protected freedoms. 9
A statute, especially one involving criminal prosecution, must be definite to be valid. A statute is vague
or overbroad, in violation of the due process clause, where its language does not convey sufficiently definite
warning to the average person as to the prohibited conduct. A statute is unconstitutionally vague if people of
common intelligence must necessarily guess at its meaning.
In malversation or bribery under the Revised Penal Code, the criminal intent is an important element of
the criminal acts. Under the Plunder Law, it is enough that the acts are committed. Equally disagreeable is the
provision of the Plunder Law which does away with the requirement that each and every component of the
criminal act of plunder be proved and instead limits itself to proving only a pattern of overt acts indicative of
the unlawful scheme or conspiracy. 18 In effect, the law seeks to penalize the accused only on the basis of a
proven scheme or conspiracy, and does away with the rights of the accused insofar as the component crimes
are concerned. In other words, R.A. No. 7080 circumvents the obligation of the prosecution to prove beyond
reasonable doubt every fact necessary to constitute the crime of plunder, because the law requires merely proof
of a pattern of overt acts showing an unlawful scheme or conspiracy.
I agree with petitioner's concern over the danger that the trial court may allow the specifications of
details in an information to validate a statute inherently void for vagueness. An information cannot rise higher
than the statute upon which it is based. Not even the construction by the Sandiganbayan of a vague or
ambiguous provision can supply the missing ingredients of the Plunder Law. The right of an accused to be
informed of the nature and cause of the accusation against him is most often exemplified in the care with which
a complaint or information should be drafted. However, the clarity and particularity required of an information
should also be present in the law upon which the charges are based. If the penal law is vague, any particularity
in the information will come from the prosecutor. The prosecution takes over the role of Congress.

In view of vagueness according to SANDOVAL-GUTIERREZ, J.


As a basic premise, we have to accept that even a person accused of a crime possesses inviolable rights
founded on the Constitution which even the welfare of the society as a whole cannot override. The rights
guaranteed to him by the Constitution are not subject to political bargaining or to the calculus of social interest.
Thus, no matter how socially-relevant the purpose of a law is, it must be nullified if it tramples upon the basic
rights of the accused.
When Section 4 of R.A. No. 7080 mandates that it shall not be necessary for the prosecution to prove
each and every criminal act done by the accused, the legislature, in effect, rendered the enumerated "criminal
acts" under Section 1 (d) merely as means and not as essential elements of plunder. This is constitutionally
infirmed and repugnant to the basic idea of justice and fair play. As a matter of due process, the prosecution is
required to prove beyond reasonable doubt every fact necessary to constitute the crime with which the
defendant is charged. The State may not specify a lesser burden of proof for an element of a crime. 8 With more
reason, it should not be allowed to go around the principle by characterizing an essential element of plunder
merely as a "means" of committing the crime. For the result is the reduction of the burden of the prosecution to
prove the guilt of the accused beyond reasonable doubt.
In short, all that R.A. No. 7080 requires is that each Justice must be convinced of the existence of a
"combination or series." As to which criminal acts constitute a combination or series, the Justices need not be
19
in full agreement. Surely, this would cover-up a wide disagreement among them about just what the accused
actually did or did not do. Stated differently, even if the Justices are not unified in their determination on what
criminal acts were actually committed by the accused, which need not be proved under the law, still, they could
convict him of plunder.
The Special Prosecution Division Panel defines it as "at least three of the acts enumerated under Section
1(d) thereof." 33 But it can very well be interpreted as only one act repeated at least three times. And the Office
of the Solicitor General, invoking the deliberations of the House of Representatives, contends differently. It
defines the term series as a "repetition" or pertaining to "two or more."
A statute which is so vague as to permit the infliction of capital punishment on acts already punished
with lesser penalties by clearly formulated law is unconstitutional. The vagueness cannot be cured by judicial
construction.
In fine, I can only stress that the one on trial here is not Mr. Estrada, but R.A. No. 7080. The issue
before this Court is not the guilt or innocence of the accused, but the constitutionality of the law. I vote to grant
the petition, not because I favor Mr. Estrada, but because I look beyond today and I see that this law can pose a
serious threat to the life, liberty and property of anyone who may come under its unconstitutional provisions.
As a member of this Court, my duty is to see to it that the law conforms to the Constitution and no other. I
simply cannot, in good conscience, fortify a law that is patently unconstitutional.

MENDOZA, J., concurring in the judgment:


Before I explain my vote, I think it necessary to restate the basic facts.
Petitioner Joseph Ejercito Estrada was President of the Philippines until January 20, 2001 when he was
forced to vacate the presidency by people power and then Vice President Gloria Macapagal-Arroyo succeeded
him in office.[1] He was charged, in eight cases filed with the Sandiganbayan, with various offenses committed
while in office, among them plunder, for allegedly having amassed ill-gotten wealth in the amount of P4.1
billion, more or less. He moved to quash the information for plunder on the ground that R.A. No. 7080,
otherwise called the Anti-Plunder Law, is unconstitutional and that the information charges more than one
offense.
In its resolution dated July 9, 2001, the Sandiganbayan denied petitioners motion, along with those
filed by his co-accused, Edward Serapio, and his son, Jose Jinggoy Estrada. Petitioner brought this petition for
certiorari and prohibition under Rule 65 to set aside the Sandiganbayans resolution principally on the ground
that the Anti-Plunder Law is void for being vague and overbroad. We gave due course to the petition and
required respondents to file comments and later heard the parties in oral arguments on September 18, 2001
and on their memoranda filed on September 28, 2001 to consider the constitutional claims of petitioner.

I. THE ANTI-PLUNDER LAW


The Anti-Plunder Law (R.A. No. 7080) was enacted by Congress on July 12, 1991 pursuant to the
constitutional mandate that the State shall maintain honesty and integrity in the public service and take
positive and effective measures against graft and corruption.[2] Section 2 of the statute provides:
Definition of the Crime of Plunder; Penalties. Any public officer who, by himself or in connivance with
members of his family, relatives by affinity or consanguinity, business associates, subordinates or other
persons, amasses, accumulates or acquires ill-gotten wealth through a combination or series of overt or
criminal acts as described in Section 1(d) hereof in the aggregate amount or total value of at least Fifty million
pesos (P50,000,000.00) shall be guilty of the crime of plunder and shall be punished by reclusion perpetua to
death. Any person who participated with the said public officer in the commission of an offense contributing to
the crime of plunder shall likewise be punished for such offense. In the imposition of penalties, the degree of
participation and the attendance of mitigating and extenuating circumstances, as provided by the Revised
Penal Code, shall be considered by the court. The court shall declare any and all ill-gotten wealth and their
interests and other incomes and assets including the properties and shares of stocks derived from the deposit
or investment thereof forfeited in favor of the State. (As amended by Sec. 12, R.A. No. 7659).
The term ill-gotten wealth is defined in 1(d) as follows:
Ill-gotten wealth, means any asset, property, business enterprise or material possession of any
person within the purview of Section Two (2) hereof, acquired by him directly or indirectly through
dummies, nominees, agents, subordinates and/or business associates by any combination or series of
the following means or similar schemes:

20
1) Through misappropriation, conversion, misuse, or malversation of public funds or raids on the
public treasury.
2) By receiving, directly or indirectly, any commission, gift, share, percentage, kickbacks or any other
form of pecuniary benefit from any person and/or entity in connection with any government
contract or project or by reason of the office or position of the public officer concerned;
3) By the illegal or fraudulent conveyance or disposition of assets belonging to the National
Government or any of its subdivisions, agencies or instrumentalities or government-owned or
controlled corporations and their subsidiaries.
4) By obtaining, receiving or accepting directly or indirectly any shares of stock, equity or any other
form of interest or participation including the promise of future employment in any business
enterprise or undertaking;
5) By establishing agricultural, industrial or commercial monopolies or other combinations and/or
implementation of decrees and orders intended to benefit particular persons or special interests; or
6) By taking undue advantage of official position, authority, relationship, connection or influence to
unjustly enrich himself or themselves at the expense and to the damage and prejudice of the Filipino
people and the Republic of the Philippines.

Section 4 of the said law states:


Rule of Evidence. For purposes of establishing the crime of plunder, it shall not be necessary to
prove each and every criminal act done by the accused in furtherance of the scheme or conspiracy to
amass, accumulate or acquire ill-gotten wealth, it being sufficient to establish beyond reasonable doubt
a pattern of overt or criminal acts indicative of the overall unlawful scheme or conspiracy.

II. ANTI-PLUNDER LAW NOT TO BE JUDGED


ON ITS FACE
The amended information against petitioner charges violations of 2, in relation to 1(d)(1)(2), of the
statute. It reads:
AMENDED INFORMATION
The undersigned Ombudsman Prosecutor and OIC-Director, EPIB, Office of the Ombudsman, hereby
accuses former President of the Republic of the Philippines, Joseph Ejercito Estrada a.k.a Asiong Salonga and
a.k.a Jose Velarde, together with Jose Jinggoy Estrada, Charlie Atong Ang, Edward Serapio, Yolanda T.
Ricaforte, Alma Alfaro, John Doe a.k.a. Eleuterio Tan or Eleuterio Ramos Tan or Mr. Uy, Jane Doe a.k.a. Delia
Rajas, and John Does & Jane Does, of the crime of plunder, defined and penalized under R.A. No. 7080, as
amended by Sec. 12 of R.A. No. 7659, committed as follows:
That during the period from June, 1998 to January, 2001, in the Philippines, and within the jurisdiction
of this Honorable Court, accused Joseph Ejercito Estrada, then a public officer, being then the President of the
Republic of the Philippines, by himself and/or in connivance/conspiracy with his co-accused, who are
members of his family, relatives by affinity or consanguinity, business associates, subordinates and/or other
persons, by taking undue advantage of his official position, authority, relationship, connection, or influence,
did then and there wilfully, unlawfully and criminally amass, accumulate and acquire by himself, directly or
indirectly, ill-gotten wealth in the aggregate amount or total value of four billion ninety seven million eight
hundred four thousand one hundred seventy three pesos and seventeen centavos [P4,097,804,173.17], more or
less, thereby unjustly enriching himself or themselves at the expense and to the damage of the Filipino people
and the Republic of the Philippines, through any or a combination or aseries of overt or criminal acts, or
similar schemes or means, described as follows:
a) by receiving or collecting, directly or indirectly, on several instances, money in the aggregate amount of
five hundred forty-five million pesos (P545,000,000.00), more or less, from illegal gambling in the
form of gift, share, percentage, kickback or any form of pecuniary benefit, by himself and/or in
connivance with co-accused Charlie Atong Ang, Jose Jinggoy Estrada, Yolanda T. Ricaforte, Edward
Serapio, and John Does and Jane Does, in consideration of toleration or protection of illegal gambling;
b) by diverting, receiving, misappropriating, converting or misusing directly or indirectly, for his or their
personal gain and benefit, public funds in the amount of ONE HUNDRED THIRTY MILLION PESOS
[P130,000,000.00], more or less, representing a portion of the two hundred million pesos
[P200,000,000.00] tobacco excise tax share allocated for the Province of Ilocos Sur under R.A. No.
7171, by himself and/or in connivance with co-accused Charlie Atong Ang, Alma Alfaro, John Doe a.k.a.
21
Eleuterio Tan or Eleuterio Ramos Tan or Mr. Uy, and Jane Doe a.k.a. Delia Rajas, and other John Does
and Jane Does;
c) by directing, ordering and compelling, for his personal gain and benefit, the Government Service
Insurance System (GSIS) to purchase 351,878,000 shares of stocks, more or less, and the Social
Security System (SSS), 329,855,000 shares of stocks, more or less, of the Belle Corporation in the
amount of more or less one billion one hundred two million nine hundred sixty five thousand six
hundred seven pesos and fifty centavos [P1,102,965,607.50] and more or less seven hundred forty four
million six hundred twelve thousand and four hundred fifty pesos [P744,612,450.00], respectively, or a
total of more or less one billion eight hundred forty seven million five hundred seventy eight thousand
fifty seven pesos and fifty centavos [P1,847,578,057.50]; and by collecting or receiving, directly or
indirectly, by himself and/or in connivance with John Does and Jane Does, commissions or percentages
by reason of said purchases of shares of stock in the amount of one hundred eighty nine million seven
hundred thousand pesos [P189,700,000.00], more or less, from the Belle Corporation which became
part of the deposit in the Equitable-PCI Bank under the account name Jose Velarde;
d) by unjustly enriching himself from commissions, gifts, shares, percentages, kickbacks, or any form of
pecuniary benefits, in connivance with John Does and Jane Does, in the amount of more or less three
billion two hundred thirty three million one hundred four thousand one hundred seventy three pesos
and seventeen centavos [P3,233,104,173.17] and depositing the same under his account name Jose
Velarde at the Equitable-PCI Bank.

CONTRARY TO LAW.
Manila for Quezon City, Philippines, 18 April 2001
But, although this is a prosecution under 2, in relation to 1(d)(1)(2), what we are seeing here is a
wholesale attack on the validity of the entire statute. Petitioner makes little effort to show the alleged invalidity
of the statute as applied to him. His focus is instead on the statute as a whole as he attacks on their face not
only 1(d)(1)(2) of the statute but also its other provisions which deal with plunder committed by illegal or
fraudulent disposition of government assets (1(d)(3)), acquisition of interest in business (1(d)(4)), and
establishment of monopolies and combinations or implementation of decrees intended to benefit particular
persons or special interests (1(d)(5)).
These other provisions of the statute are irrelevant to this case. What relevance do questions regarding
the establishment of monopolies and combinations, or the ownership of stocks in a business enterprise, or the
illegal or fraudulent dispositions of government property have to the criminal prosecution of petitioner when
they are not even mentioned in the amended information filed against him? Why should it be important to
inquire whether the phrase overt act in 1(d) and 2 means the same thing as the phrase criminal act as used in
the same provisions when the acts imputed to petitioner in the amended information are criminal acts? Had
the provisions of the Revised Penal Code been subjected to this kind of line-by-line scrutiny whenever a portion
thereof was involved in a case, it is doubtful if we would have the jurisprudence on penal law that we have
today. The prosecution of crimes would certainly have been hampered, if not stultified. We should not even
attempt to assume the power we are asked to exercise. The delicate power of pronouncing an Act of Congress
unconstitutional is not to be exercised with reference to hypothetical cases . . . . In determining the sufficiency
of the notice a statute must of necessity be examined in the light of the conduct with which a defendant is
charged.[3]
Nonetheless, it is contended that because these provisions are void for being vague and overbroad, the
entire statute, including the part under which petitioner is being prosecuted, is also void. And if the entire
statute is void, there is no law under which he can be prosecuted for plunder. Nullum crimen sine lege, nullum
poena sine lege.
Two justifications are advanced for this facial challenge to the validity of the entire statute. The first is
that the statute comes within the specific prohibitions of the Constitution and, for this reason, it must be given
strict scrutiny and the normal presumption of constitutionality should not be applied to it nor the usual judicial
deference given to the judgment of Congress.[4] The second justification given for the facial attack on the Anti-
Plunder Law is that it is vague and overbroad.[5]
We find no basis for such claims either in the rulings of this Court or of those of the U.S. Supreme
Court, from which petitioners counsel purports to draw for his conclusions. We consider first the claim that the
statute must be subjected to strict scrutiny.

22
A. Test of Strict Scrutiny Not Applicable to Penal Statutes
Petitioner cites the dictum in Ople v. Torres[6] that when the integrity of a fundamental right is at
stake, this Court will give the challenged law, administrative order, rule or regulation stricter scrutiny and that
It will not do for authorities to invoke the presumption of regularity in the performance of official duties. As
will presently be shown, strict scrutiny, as used in that decision, is not the same thing as the strict scrutiny
urged by petitioner. Much less did this Court rule that because of the need to give stricter scrutiny to laws
abridging fundamental freedoms, it will not give such laws the presumption of validity.
Petitioner likewise cites the most celebrated footnote in [American] constitutional law, i.e., footnote 4 of
the opinion in United States v. Carolene Products Co.,[7] in which it was stated:
There may be narrower scope for operation of the presumption of constitutionality when legislation
appears on its face to be within a specific prohibition of the Constitution, such as those of the first ten
amendments, which are deemed equally specific when held to be embraced within the Fourteenth.
It is unnecessary to consider now whether legislation which restricts those political processes which can
ordinarily be expected to bring about repeal of undesirable legislation, is to be subjected to more exacting
judicial scrutiny under the general prohibitions of the Fourteenth Amendment than are most other types of
legislation.
Nor need we inquire whether similar considerations enter into the review of statutes directed at
particular religious, or national, or racial minorities: whether prejudice against discrete and insular minorities
may be a special condition, which tends seriously to curtail the operation of those political processes ordinarily
to be relied upon to protect minorities, and which may call for a correspondingly more searching judicial
inquiry.
Again, it should be noted that what the U.S. Supreme Court said is that there may be narrower scope for
the operation of the presumption of constitutionality for legislation which comes within the first ten
amendments to the American Federal Constitution compared to legislation covered by the Fourteenth
Amendment Due Process Clause. The American Court did not say that such legislation is not to be presumed
constitutional, much less that it is presumptively invalid, but only that a narrower scope will be given for the
presumption of constitutionality in respect of such statutes. There is, therefore, no warrant for petitioners
contention that the presumption of constitutionality of a legislative act is applicable only where the Supreme
Court deals with facts regarding ordinary economic affairs, not where the interpretation of the text of the
Constitution is involved.[8]
What footnote 4 of the Carolene Products case posits is a double standard of judicial review: strict
scrutiny for laws dealing with freedom of the mind or restricting the political process, and deferential or
rational basis standard of review for economic legislation. As Justice (later Chief Justice) Fernando explained
in Malate Hotel and Motel Operators Assn v. The City Mayor,[9] this simply means that if the liberty involved
were freedom of the mind or the person, the standard for the validity of governmental acts is much more
rigorous and exacting, but where the liberty curtailed affects what are at the most rights of property, the
permissible scope of regulatory measures is wider.
Hence, strict scrutiny is used today to test the validity of laws dealing with the regulation of speech,
gender, or race and facial challenges are allowed for this purpose. But criminal statutes, like the Anti-Plunder
Law, while subject to strict construction, are not subject to strict scrutiny. The two (i.e., strict construction and
strict scrutiny) are not the same. The rule of strict construction is a rule of legal hermeneutics which deals with
the parsing of statutes to determine the intent of the legislature. On the other hand, strict scrutiny is a standard
of judicial review for determining the quality and the amount of governmental interest brought to justify the
regulation of fundamental freedoms. It is set opposite such terms as deferential review and intermediate
review.
Thus, under deferential review, laws are upheld if they rationally further a legitimate governmental
interest, without courts seriously inquiring into the substantiality of such interest and examining the
alternative means by which the objectives could be achieved. Under intermediate review, the substantiality of
the governmental interest is seriously looked into and the availability of less restrictive alternatives are
considered. Under strict scrutiny, the focus is on the presence of compelling, rather than substantial,
governmental interest and on the absence of less restrictive means for achieving that interest.[10]
Considering these degrees of strictness in the review of statutes, how many criminal laws can survive
the test of strict scrutiny to which petitioner proposes to subject them? How many can pass muster if, as
petitioner would have it, such statutes are not to be presumed constitutional? Above all, what will happen to
the States ability to deal with the problem of crimes, and, in particular, with the problem of graft and
23
corruption in government, if criminal laws are to be upheld only if it is shown that there is a compelling
governmental interest for making certain conduct criminal and if there is no other means less restrictive than
that contained in the law for achieving such governmental interest?

B. Vagueness and Overbreadth Doctrines, as Grounds for Facial Challenge,


Not Applicable to Penal Laws
Nor do allegations that the Anti-Plunder Law is vague and overbroad justify a facial review of its
validity. The void-for-vagueness doctrine states that a statute which either forbids or requires the doing of an
act in terms so vague that men of common intelligence must necessarily guess at its meaning and differ as to its
application, violates the first essential of due process of law.[11] The overbreadth doctrine, on the other hand,
decrees that a governmental purpose may not be achieved by means which sweep unnecessarily broadly and
thereby invade the area of protected freedoms.[12]
A facial challenge is allowed to be made to a vague statute and to one which is overbroad because of
possible chilling effect upon protected speech. The theory is that [w]hen statutes regulate or proscribe speech
and no readily apparent construction suggests itself as a vehicle for rehabilitating the statutes in a single
prosecution, the transcendent value to all society of constitutionally protected expression is deemed to justify
allowing attacks on overly broad statutes with no requirement that the person making the attack demonstrate
that his own conduct could not be regulated by a statute drawn with narrow specificity.[13]The possible harm
to society in permitting some unprotected speech to go unpunished is outweighed by the possibility that the
protected speech of others may be deterred and perceived grievances left to fester because of possible
inhibitory effects of overly broad statutes.
This rationale does not apply to penal statutes. Criminal statutes have general in terrorem effect
resulting from their very existence, and, if facial challenge is allowed for this reason alone, the State may well
be prevented from enacting laws against socially harmful conduct. In the area of criminal law, the law cannot
take chances as in the area of free speech.
The overbreadth and vagueness doctrines then have special application only to free speech cases. They
are inapt for testing the validity of penal statutes. As the U.S. Supreme Court put it, in an opinion by Chief
Justice Rehnquist, we have not recognized an overbreadth doctrine outside the limited context of the First
Amendment.[14] In Broadrick v. Oklahoma,[15] the Court ruled that claims of facial overbreadth have been
entertained in cases involving statutes which, by their terms, seek to regulate only spoken words and, again,
that overbreadth claims, if entertained at all, have been curtailed when invoked against ordinary criminal laws
that are sought to be applied to protected conduct. For this reason, it has been held that a facial challenge to a
legislative Act is the most difficult challenge to mount successfully, since the challenger must establish that no
set of circumstances exists under which the Act would be valid.[16] As for the vagueness doctrine, it is said that
a litigant may challenge a statute on its face only if it is vague in all its possible applications. A plaintiff who
engages in some conduct that is clearly proscribed cannot complain of the vagueness of the law as applied to
the conduct of others.[17]
In sum, the doctrines of strict scrutiny, overbreadth, and vagueness are analytical tools developed for
testing on their faces statutes in free speech cases or, as they are called in American law, First Amendment
cases. They cannot be made to do service when what is involved is a criminal statute. With respect to such
statute, the established rule is that one to whom application of a statute is constitutional will not be heard to
attack the statute on the ground that impliedly it might also be taken as applying to other persons or other
situations in which its application might be unconstitutional.[18] As has been pointed out, vagueness
challenges in the First Amendment context, like overbreadth challenges typically produce facial invalidation,
while statutes found vague as a matter of due process typically are invalidated [only] as applied to a particular
defendant.[19] Consequently, there is no basis for petitioners claim that this Court review the Anti-Plunder
Law on its face and in its entirety.

C. Anti-Plunder Law Should be Construed As Applied


Indeed, on its face invalidation of statutes results in striking them down entirely on the ground that
they might be applied to parties not before the Court whose activities are constitutionally protected.[20] It
constitutes a departure from the case and controversy requirement of the Constitution and permits decisions to
be made without concrete factual settings and in sterile abstract contexts.[21] But, as the U.S. Supreme Court
pointed out in Younger v. Harris:[22]

24
[T]he task of analyzing a proposed statute, pinpointing its deficiencies, and requiring correction of these
deficiencies before the statute is put into effect, is rarely if ever an appropriate task for the judiciary. The
combination of the relative remoteness of the controversy, the impact on the legislative process of the relief
sought, and above all the speculative and amorphous nature of the required line-by-line analysis of detailed
statutes,...ordinarily results in a kind of case that is wholly unsatisfactory for deciding constitutional questions,
whichever way they might be decided.
This is the reason on its face invalidation of statutes has been described as manifestly strong medicine,
to be employed sparingly and only as a last resort,[23] and is generally disfavored.[24] In determining the
constitutionality of a statute, therefore, its provisions which are alleged to have been violated in a case must be
examined in the light of the conduct with which the defendant is charged.[25]
This brings me to the question whether, as applied, 2, in relation to 1(d)(1)(2), of the Anti-Plunder Law
is void on the ground of vagueness and overbreadth.

III. ANTI-PLUNDER LAW NEITHER VAGUE NOR OVERBROAD


As earlier noted, the case against petitioner Joseph Ejercito Estrada in the Sandiganbayan is for
violation of 2, in relation to 1(d)(1)(2), of the Anti-Plunder Law, which, so far as pertinent, provide:
SEC. 2. Definition of the Crime of Plunder; Penalties. Any public officer who, by himself or in
connivance with members of his family, relatives by affinity or consanguinity, business associates,
subordinates or other persons, amasses, accumulates or acquires ill-gotten wealth through a
combination or series of overt or criminal acts as described in Section 1(d) hereof in the aggregate
amount or total value of at least Fifty million pesos (P50,000,000.00) shall be guilty of the crime of
plunder and shall be punished by reclusion perpetua to death....

SEC. 1. Definition of Terms. ...


(d) Ill-gotten wealth, means any asset, property, business enterprise or material possession of any
person within the purview of Section Two (2) hereof, acquired by him directly or indirectly through
dummies, nominees, agents, subordinates and/or business associates by any combination or series of
the following means or similar schemes:
1) Through misappropriation, conversion, misuse, or malversation of public funds or raids on the
public treasury.
2) By receiving, directly or indirectly, any commission, gift, share, percentage, kickbacks or any other
form of pecuniary benefit from any person and/or entity in connection with any government
contract or project or by reason of the office or position of the public officer concerned;

The charge is that in violation of these provisions, during the period June 1998 to January 2001,
petitioner, then the President of the Philippines, willfully, unlawfully, and criminally amassed wealth in the
total amount of P4,097,804,173.17, more or less, through a combination or series of overt or criminal acts, to
wit: (1) by receiving or collecting the total amount of P545,000,000.00, more or less, from illegal gambling by
himself and/or in connivance with his co-accused named therein, in exchange for protection of illegal
gambling; (2) by misappropriating, converting, or misusing, by himself or in connivance with his co-accused
named therein, public funds amounting to P130,000,000.00, more or less, representing a portion of the share
of the Province of Ilocos Sur in the tobacco excise tax; (3) by ordering the GSIS and the SSS to buy shares of
stocks of the Belle Corp., worth P1,102,965,607.50 and P744,612,450.00 respectively, or the total amount of
P1,847,578,057.50, for which he received as commission the amount of P189,700,000.00, more or less, from
Belle Corp.; (4) by unjustly enriching himself from commissions, gifts, shares, percentages, and kickbacks in
the amount of P3,233,104,173.17, which he deposited in the Equitable-PCI Bank under the name of Jose
Velarde.
Anyone reading the law in relation to this charge cannot possibly be mistaken as to what petitioner is
accused of in Criminal Case No. 26558 of the Sandiganbayan. But, repeatedly, petitioner complains that the law
is vague and deprives him of due process. He invokes the ruling in Connally v. General Constr. Co.[26] that a
statute which either forbids or requires the doing of an act in terms so vague that men of common intelligence
must necessarily guess at its meaning and differ as to its application, violates the first essential of due process
of law. He does this by questioning not only 2, in relation to 1(d)(1)(2), as applied to him, but also other
provisions of the Anti-Plunder Law not involved in this case. In 55 out of 84 pages of discussion in his

25
Memorandum, petitioner tries to show why on their face these provisions are vague and overbroad by asking
questions regarding the meaning of some words and phrases in the statute, to wit:
1. Whether series means two, three, or four overt or criminal acts listed in 1(d) in view of the alleged
divergence of interpretation given to this word by the Ombudsman, the Solicitor General, and the
Sandiganbayan, and whether the acts in a series should be directly related to each other;
2. Whether combination includes two or more acts or at least two of the means or similar schemes
mentioned in 1(d);
3. Whether pattern as used in 1(d) must be related to the word pattern in 4 which requires that it be
indicative of an overall unlawful scheme or conspiracy;
4. Whether overt means the same thing as criminal;
5. Whether misuse of public funds is the same as illegal use of public property or technical malversation;
6. Whether raids on the public treasury refers to raids on the National Treasury or the treasury of a
province or municipality;
7. Whether the receipt or acceptance of a gift, commission, kickback, or pecuniary benefits in connection
with a government contract or by reason of his office, as used in 1(d)(2), is the same as bribery in the
Revised Penal Code or those which are considered corrupt practices of public officers;
8. Whether illegal or fraudulent conveyance or disposition of assets belonging to the National
Government, as used in 1(d)(3), refers to technical malversation or illegal use of public funds or
property in the Revised Penal Code;
9. Whether mere ownership of stocks in a private corporation, such as a family firm engaged in fishing, is
prohibited under 1(d)(4);
10. Whether the phrase monopolies or other combinations in restraint of trade in 1(d)(5) means the same
thing as monopolies and combinations in restraint of trade in the Revised Penal Code because the latter
contemplates monopolies and combinations established by any person, not necessarily a public officer;
and
11. Whether under 1(d)(5) it is the public officer who intends to confer benefit on a particular person by
implementing a decree or it is the decree that is intended to benefit the particular person and the public
officer simply implements it.

Many more questions of this tenor are asked in the memorandum of petitioner[27] as well as in the
dissent of MR. JUSTICE KAPUNAN. Not only are they irrelevant to this case, as already pointed out. It is also
evident from their examination that what they present are simply questions of statutory construction to be
resolved on a case-to-case basis. Consider, for example, the following words and phrases in 1(d) and 2:

A. Combination or series of overt or criminal acts


Petitioner contends that the phrase combination or series of overt, or criminal acts in 1(d) and 2 should
state how many acts are needed in order to have a combination or a series. It is not really required that this be
specified. Petitioner, as well as MR. JUSTICE KAPUNAN, cites the following remarks of Senators Gonzales and
Taada during the discussion of S. No. 733 in the Senate:
SENATOR GONZALES. To commit the offense of plunder, as defined in this Act while constituting a
single offense, it must consist of a series of overt or criminal acts, such as bribery, extortion, malversation of
public funds, swindling, falsification of public documents, coercion, theft, fraud, and illegal exaction, and graft
or corrupt practices act and like offenses. Now, Mr. President, I think, this provision, by itself, will be vague. I
am afraid that it might be faulted for being violative of the due process clause and the right to be informed of
the nature and cause of accusation of an accused. Because, what is meant by series of overt or criminal acts? I
mean, would 2, 3, 4 or 5 constitute a series? During the period of amendments, can we establish a minimum of
overt acts like, for example, robbery in band? The law defines what is robbery in band by the number of
participants therein.
In this particular case, probably, we can statutorily provide for the definition of series so that two, for
example, would that be already a series? Or, three, what would be the basis for such a determination?
SENATOR TAADA. I think, Mr. President, that would be called for, this being a penal legislation, we
should be very clear as to what it encompasses; otherwise, we may contravene the constitutional provision on
the right of the accused to due process.[28]

26
But, as the later discussion in the Senate shows, the senators in the end reached a consensus as to the
meaning of the phrase so that an enumeration of the number of acts needed was no longer proposed. Thus, the
record shows:

SENATOR MACEDA. In line with our interpellations that sometimes one or maybe even two acts may
already result in such a big amount, on line 25, would the Sponsor consider deleting the words a series
of overt or. To read, therefore: or conspiracy COMMITTED by criminal acts such. Remove the idea of
necessitating a series. Anyway, the criminal acts are in the plural.
SENATOR TAADA. That would mean a combination of two or more of the acts mentioned in this.
THE PRESIDENT. Probably, two or more would be . . .
SENATOR MACEDA. Yes, because a series implies several or many; two or more.
SENATOR TAADA: Accepted, Mr. President.
....
THE PRESIDENT: If there is only one, then he has to be prosecuted under the particular crime. But
when we say acts of plunder there should be, at least, two or more.
SENATOR ROMULO: In other words, that is already covered by existing laws, Mr. President.[29]

Indeed, the record shows that no amendment to S. No. 733 was proposed to this effect. To the contrary,
Senators Gonzales and Taada voted in favor of the bill on its third and final reading on July 25, 1989.The
ordinary meaning of the term combination as the union of two things or acts was adopted, although in the case
of series, the senators agreed that a repetition of two or more times of the same thing or act would suffice, thus
departing from the ordinary meaning of the word as a group of usually three or more things or events standing
or succeeding in order and having a like relationship to each other, or a spatial or temporal succession of
persons or things, or a group that has or admits an order of arrangement exhibiting progression.[30]
In the Bicameral Conference Committee on Justice meeting held on May 7, 1991, the same meanings
were given to the words combination and series. Representative Garcia explained that a combination is
composed of two or more of the overt or criminal acts enumerated in 1(d), while a series is a repetition of any of
the same overt or criminal acts. Thus:

REP. ISIDRO: I am just intrigued again by our definition of plunder. We say, THROUGH A
COMBINATION OR SERIES OF OVERT OR CRIMINAL ACTS AS MENTIONED IN SECTION ONE
HEREOF. Now when we say combination, we actually mean to say, if there are two or more means, we
mean to say that number one and two or number one and something else are included, how about a
series of the same act? For example, through misappropriation, conversion, misuse, will these be
included also?
....

REP. ISIDRO: When we say combination, it seems that


THE CHAIRMAN (REP. GARCIA): Two.
REP. ISIDRO: Not only two but we seem to mean that two of the enumerated means not twice of one
enumeration.
THE CHAIRMAN (REP. GARCIA): No, no, not twice.
REP. ISIDRO: Not twice?
THE CHAIRMAN (REP. GARCIA): Yes, combination is not twice but combination, two acts.
REP. ISIDRO: So in other words, thats it. When we say combination, we mean, two different acts. It can
not be a repetition of the same act.
THE CHAIRMAN (REP. GARCIA): That be referred to series. Yeah.
REP. ISIDRO: No, no. Supposing one act is repeated, so there are two.
THE CHAIRMAN (REP. GARCIA): A series.
REP. ISIDRO: Thats not [a] series. Its a combination. Because when we say combination or series, we
seem to say that two or more, di ba?
THE CHAIRMAN (REP. GARCIA): Yes, this distinguishes it, really, from ordinary crimes. That is why, I
said, that is a very good suggestion because if it is only one act, it may fall under ordinary crime but we
have here a combination or series of overt or criminal acts. So. . .
....
27
REP. ISIDRO: When you say combination, two different?
THE CHAIRMAN (REP. GARCIA): Yes.
THE CHAIRMAN (SEN. TAADA): Two different. . . .
REP. ISIDRO: Two different acts.
THE CHAIRMAN (REP. GARCIA): For example, ha. . .
REP. ISIDRO: Now a series, meaning, repetition. . .[31]

Thus, resort to the deliberations in Congress will readily reveal that the word combination includes at
least two different overt or criminal acts listed in R.A. No. 7080, such as misappropriation (1(d)(1)) and taking
undue advantage of official position (1(d)(6)). On the other hand, series is used when the offender commits the
same overt or criminal act more than once. There is no plunder if only one act is proven, even if the ill-gotten
wealth acquired thereby amounts to or exceeds the figure fixed by the law for the offense (now
P50,000,000.00). The overt or criminal acts need not be joined or separated in space or time, since the law
does not make such a qualification. It is enough that the prosecution proves that a public officer, by himself or
in connivance with others, amasses wealth amounting to at least P50 million by committing two or more overt
or criminal acts.
Petitioner also contends that the phrase series of acts or transactions is the subject of conflicting
decisions of various Circuit Courts of Appeals in the United Sates. It turns out that the decisions concerned a
phrase in Rule 8(b) of the Federal Rules of Criminal Procedure which provides:
(b) Joinder of Defendants: Two or more defendants may be charged in the same indictment or
information if they are alleged to have participated in the same act or transaction or in the same series of acts
or transactions constituting an offense or offenses. Such defendants may be charged in one or more counts
together or separately and all of the defendants need not be charged on each count. (Emphasis added)
The fact that there is a conflict in the rulings of the various courts does not mean that Rule 8(b) is void
for being vague but only that the U.S. Supreme Court should step in, for one of its essential functions is to
assure the uniform interpretation of federal laws.
We have a similar provision in Rule 3, 6 of the 1997 Code of Civil Procedure. It reads:
SEC. 6. Permissive joinder of parties. All persons in whom or against whom any right to relief in
respect to or arising out of the same transaction or series of transactions is alleged to exist, whether
jointly, severally, or in the alternative, may, except as otherwise provided in these Rules, join as
plaintiffs or be joined as defendants in one complaint, where any question of law or fact common to all
such plaintiffs or to all such defendants may arise in the action; but the court may make such orders as
may be just to prevent any plaintiff or defendant from being embarrassed or put to expense in
connection with any proceedings in which he may have no interest. (Emphasis added)

This provision has been in our Rules of Court since 1940 but it has never been thought of as vague. It
will not do, therefore, to cite the conflict of opinions in the United States as evidence of the vagueness of the
phrase when we do not have any conflict in this country.

B. Pattern of overt or criminal acts


Petitioner contends that it is not enough that there be at least two acts to constitute either a
combination or series because 4 also mentions a pattern of overt or criminal acts indicative of the overall
scheme or conspiracy, and pattern means an arrangement or order of things or activity.
A pattern of overt or criminal acts is required in 4 to prove an unlawful scheme or conspiracy. In such a
case, it is not necessary to prove each and every criminal act done in furtherance of the scheme or conspiracy so
long as those proven show a pattern indicating the scheme or conspiracy. In other words, when conspiracy is
charged, there must be more than a combination or series of two or more acts. There must be several acts
showing a pattern which is indicative of the overall scheme or conspiracy. As Senate President Salonga
explained, if there are 150 constitutive crimes charged, it is not necessary to prove beyond reasonable doubt all
of them. If a pattern can be shown by proving, for example, 10 criminal acts, then that would be sufficient to
secure conviction.[32]
The State is thereby enabled by this device to deal with several acts constituting separate crimes as just
one crime of plunder by allowing their prosecution by means of a single information because there is a
common purpose for committing them, namely, that of amassing, accumulating or acquiring wealth through
28
such overt or criminal acts. The pattern is the organizing principle that defines what otherwise would be
discreet criminal acts into the single crime of plunder.
As thus applied to petitioner, the Anti-Plunder Law presents only problems of statutory construction,
not vagueness or overbreadth. In Primicias v. Fugoso,[33] an ordinance of the City of Manila, prohibiting the
holding of parades and assemblies in streets and public places unless a permit was first secured from the city
mayor and penalizing its violation, was construed to mean that it gave the city mayor only the power to specify
the streets and public places which can be used for the purpose but not the power to ban absolutely the use of
such places. A constitutional doubt was thus resolved through a limiting construction given to the ordinance.
Nor is the alleged difference of opinion among the Ombudsman, the Solicitor General, and the
Sandiganbayan as to the number of acts or crimes needed to constitute plunder proof of the vagueness of the
statute and, therefore, a ground for its invalidation. For sometime it was thought that under Art. 134 of the
Revised Penal Code convictions can be had for the complex crime of rebellion with murder, arson, and other
common crimes. The question was finally resolved in 1956 when this Court held that there is no such complex
crime because the common crimes were absorbed in rebellion.[34] The point is that Art. 134 gave rise to a
difference of opinion that nearly split the legal profession at the time, but no one thought Art. 134 to be vague
and, therefore, void.
Where, therefore, the ambiguity is not latent and the legislative intention is discoverable with the aid of
the canons of construction, the void for vagueness doctrine has no application.
In Connally v. General Constr. Co.[35] the test of vagueness was formulated as follows:
[A] statute which either forbids or requires the doing of an act in terms so vague that men of common
intelligence must necessarily guess at its meaning and differ as to its application, violates the first essential of
due process of law.
Holmess test was that of the viewpoint of the bad man. In The Path of the Law, Holmes said:
If you want to know the law and nothing else, you must look at it as a bad man, who cares only for the
material consequences which such knowledge enables him to predict, not as a good one, who finds his
reasons for conduct, whether inside the law or outside of it, in the vaguer sanctions of conscience.[36]

Whether from the point of view of a man of common intelligence or from that of a bad man, there can
be no mistaking the meaning of the Anti-Plunder Law as applied to petitioner.

IV. PLUNDER A COMPLEX CRIME REQUIRING PROOF OF MENS REA


Petitioner argues that, in enacting the statute in question, Congress eliminated the element of mens rea,
or the scienter, thus reducing the burden of evidence required for proving the crimes which are mala in se.[37]
There are two points raised in this contention. First is the question whether the crime of plunder is a
malum in se or a malum prohibitum. For if it is a malum prohibitum, as the Ombudsman and the Solicitor
General say it is,[38] then there is really a constitutional problem because the predicate crimes are mainly
mala in se.

A. Plunder A Malum In Se Requiring Proof of Mens Rea


Plunder is a malum in se, requiring proof of criminal intent. Precisely because the constitutive crimes
are mala in se the element of mens rea must be proven in a prosecution for plunder. It is noteworthy that the
amended information alleges that the crime of plunder was committed willfully, unlawfully and criminally. It
thus alleges guilty knowledge on the part of petitioner.
In support of his contention that the statute eliminates the requirement of mens rea and that is the
reason he claims the statute is void, petitioner cites the following remarks of Senator Taada made during the
deliberation on S. No. 733:
SENATOR TAADA. . . . And the evidence that will be required to convict him would not be evidence for
each and every individual criminal act but only evidence sufficient to establish the conspiracy or scheme to
commit this crime of plunder.[39]
However, Senator Taada was discussing 4 as shown by the succeeding portion of the transcript quoted
by petitioner:
SENATOR ROMULO: And, Mr. President, the Gentleman feels that it is contained in Section 4, Rule of
Evidence, which, in the Gentlemans view, would provide for a speedier and faster process of attending
to this kind of cases?
SENATOR TAADA. Yes, Mr. President . . .[40]
29
Seator Taada was only saying that where the charge is conspiracy to commit plunder, the prosecution
need not prove each and every criminal act done to further the scheme or conspiracy, it being enough if it
proves beyond reasonable doubt a pattern of overt or criminal acts indicative of the overall unlawful scheme or
conspiracy. As far as the acts constituting the pattern are concerned, however, the elements of the crime must
be proved and the requisite mens rea must be shown.
Indeed, 2 provides that any person who participated with the said public officer in the commission of an
offense contributing to the crime of plunder shall likewise be punished for such offense. In the imposition of
penalties, the degree of participation and the attendance of mitigating and extenuating circumstances, as
provided by the Revised Penal Code, shall be considered by the court.
The application of mitigating and extenuating circumstances in the Revised Penal Code to prosecutions
under the Anti-Plunder Law indicates quite clearly that mens rea is an element of plunder since the degree of
responsibility of the offender is determined by his criminal intent. It is true that 2 refers to any person who
participates with the said public officers in the commission of an offense contributing to the crime of plunder.
There is no reason to believe, however, that it does not apply as well to the public officer as principal in the
crime. As Justice Holmes said: We agree to all the generalities about not supplying criminal laws with what
they omit, but there is no canon against using common sense in construing laws as saying what they obviously
mean.[41]
Finally, any doubt as to whether the crime of plunder is a malum in se must be deemed to have been
resolved in the affirmative by the decision of Congress in 1993 to include it among the heinous crimes
punishable by reclusion perpetua to death. Other heinous crimes are punished with death as a straight penalty
in R.A. No. 7659. Referring to these groups of heinous crimes, this Court held in People v. Echagaray:[42]
The evil of a crime may take various forms. There are crimes that are, by their very nature, despicable,
either because life was callously taken or the victim is treated like an animal and utterly dehumanized
as to completely disrupt the normal course of his or her growth as a human being. . . . Seen in this light,
the capital crimes of kidnapping and serious illegal detention for ransom resulting in the death of the
victim or the victim is raped, tortured, or subjected to dehumanizing acts; destructive arson resulting in
death; and drug offenses involving minors or resulting in the death of the victim in the case of other
crimes; as well as murder, rape, parricide, infanticide, kidnapping and serious illegal detention, where
the victim is detained for more than three days or serious physical injuries were inflicted on the victim
or threats to kill him were made or the victim is a minor, robbery with homicide, rape or intentional
mutilation, destructive arson, and carnapping where the owner, driver or occupant of the carnapped
vehicle is killed or raped, which are penalized by reclusion perpetua to death, are clearly heinous by
their very nature.

There are crimes, however, in which the abomination lies in the significance and implications of the
subject criminal acts in the scheme of the larger socio-political and economic context in which the state finds
itself to be struggling to develop and provide for its poor and underprivileged masses. Reeling from decades of
corrupt tyrannical rule that bankrupted the government and impoverished the population, the Philippine
Government must muster the political will to dismantle the culture of corruption, dishonesty, greed and
syndicated criminality that so deeply entrenched itself in the structures of society and the psyche of the
populace. [With the government] terribly lacking the money to provide even the most basic services to its
people, any form of misappropriation or misapplication of government funds translates to an actual threat to
the very existence of government, and in turn, the very survival of the people it governs over. Viewed in this
context, no less heinous are the effects and repercussions of crimes like qualified bribery, destructive arson
resulting in death, and drug offenses involving government officials, employees or officers, that their
perpetrators must not be allowed to cause further destruction and damage to society.
The legislative declaration in R.A. No. 7659 that plunder is a heinous offense implies that it is a malum
in se. For when the acts punished are inherently immoral or inherently wrong, they are mala in se[43]and it
does not matter that such acts are punished in a special law, especially since in the case of plunder the
predicate crimes are mainly mala in se. Indeed, it would be absurd to treat prosecutions for plunder as though
they are mere prosecutions for violations of the Bouncing Check Law (B.P. Blg. 22) or of an ordinance against
jaywalking, without regard to the inherent wrongness of the acts.

B. The Penalty for Plunder


30
The second question is whether under the statute the prosecution is relieved of the duty of proving
beyond reasonable doubt the guilt of the defendant. It is contended that, in enacting the Anti-Plunder Law,
Congress simply combined several existing crimes into a single one but the penalty which it provided for the
commission of the crime is grossly disproportionate to the crimes combined while the quantum of proof
required to prove each predicate crime is greatly reduced.
We have already explained why, contrary to petitioners contention, the quantum of proof required to
prove the predicate crimes in plunder is the same as that required were they separately prosecuted. We,
therefore, limit this discussion to petitioners claim that the penalty provided in the Anti-Plunder Law is grossly
disproportionate to the penalties imposed for the predicate crimes. Petitioner cites the following examples:
For example, please consider the following combination or series of overt or criminal acts (assuming
the P50 M minimum has been acquired) in light of the penalties laid down in the Penal Code:

a. One act of indirect bribery (penalized under Art. 211 of the Revised Penal Code with prision
correccional in its medium and maximum periods),
combined with
one act of fraud against the public treasury (penalized under Art. 213 of the Revised Penal Code with
prision correccional in its medium period to prision mayor in its minimum period,
- equals -
plunder (punished by reclusion perpetua to death plus forfeiture of assets under R.A. 7080)

b. One act of prohibited transaction (penalized under Art. 215 of the revised Penal Code with prision
correccional in its minimum period or a fine ranging from P200 to P1,000 or both),
combined with
one act of establishing a commercial monopoly (penalized under Art. 186 of Revised Penal Code with
prision correccional in its minimum period or a fine ranging from P200 to P6,000, or both),
-equals-
plunder (punished by reclusion perpetua to death, and forfeiture of assets under R.A. 7080.

c. One act of possession of prohibited interest by a public officer (penalized with prision correccional
in its minimum period or a fine of P200 to P1,000, or both under Art. 216 of the Revised Penal Code),
combined with
one act of combination or conspiracy in restraint of trade (penalized under Art. 186 of the Revised penal
Code with prision correccional in its minimum period, or a fine of P200 to P1,000, or both,
- equals -
plunder, punished by reclusion perpetua to death, and forfeiture of assets)[44]

But this is also the case whenever other special complex crimes are created out of two or more existing
crimes. For example, robbery with violence against or intimidation of persons under Art. 294, par. 5 of the
Revised Penal Code is punished with prision correccional in its maximum period (4 years, 2 months, and 1
day) to prision mayor in its medium period (6 years and 1 day to 8 years). Homicide under Art. 249 of the
same Code is punished with reclusion temporal (12 years and 1 day to 20 years). But when the two crimes are
committed on the same occasion, the law treats them as a special complex crime of robbery with homicide and
provides the penalty of reclusion perpetua to death for its commission. Again, the penalty for simple rape
under Art. 266-B of the Revised Penal Code is reclusion perpetua, while that for homicide under Art. 249 it is
reclusion temporal (12 years and 1 day to 20 years). Yet, when committed on the same occasion, the two are
treated as one special complex crime of rape with homicide and punished with a heavier penalty of reclusion
perpetua to death. Obviously, the legislature views plunder as a crime as serious as robbery with homicide or
rape with homicide by punishing it with the same penalty. As the explanatory note accompanying S. No. 733
explains:
Plunder, a term chosen from other equally apt terminologies like kleptocracy and economic treason,
punishes the use of high office for personal enrichment, committed thru a series of acts done not in the public
eye but in stealth and secrecy over a period of time, that may involve so many persons, here and abroad, and
which touch so many states and territorial units. The acts and/or omissions sought to be penalized do not
involve simple cases of malversation of public funds, bribery, extortion, theft and graft but constitute the
plunder of an entire nation resulting in material damage to the national economy. The above-described crime
31
does not yet exist in Philippine statute books. Thus, the need to come up with a legislation as a safeguard
against the possible recurrence of the depravities of the previous regime and as a deterrent to those with
similar inclination to succumb to the corrupting influences of power.
Many other examples drawn from the Revised Penal Code and from special laws may be cited to show
that, when special complex crimes are created out of existing crimes, the penalty for the new crime is heavier.
______________________

To recapitulate, had R.A. No. 7080 been a law regulating speech, I would have no hesitation examining
it on its face on the chance that some of its provisions even though not here before us are void. For then the risk
that some state interest might be jeopardized, i.e., the interest in the free flow of information or the prevention
of chill on the freedom of expression, would trump any marginal interest in security.
But the Anti-Plunder Law is not a regulation of speech. It is a criminal statute designed to combat graft
and corruption, especially those committed by highly-placed public officials. As conduct and not speech is its
object, the Court cannot take chances by examining other provisions not before it without risking vital interests
of society. Accordingly, such statute must be examined only as applied to the defendant and, if found valid as to
him, the statute as a whole should not be declared unconstitutional for overbreadth or vagueness of its other
provisions. Doing so, I come to the following conclusions:
1. That the validity of R.A. No. 7080, otherwise known as the Anti-Plunder Law, cannot be determined by
applying the test of strict scrutiny in free speech cases without disastrous consequences to the States
effort to prosecute crimes and that, contrary to petitioners contention, the statute must be presumed to
be constitutional;
2. That in determining the constitutionality of the Anti-Plunder Law, its provisions must be considered in
light of the particular acts alleged to have been committed by petitioner;
3. That, as applied to petitioner, the statute is neither vague nor overbroad;
4. That, contrary to the contention of the Ombudsman and the Solicitor General, the crime of plunder is a
malum in se and not a malum prohibitum and the burden of proving each and every predicate crime is
on the prosecution.

For these reasons, I respectfully submit that R.A. No. 7080 is valid and that, therefore, the petition
should be dismissed.

SECRETARY OF JUSTICE VS LANTION


343 SCRA 377 (2000)

Facts:
This is a petition for review of a decision of the Manila Regional Trial Court (RTC). The Department of
Justice received a request from the Department of Foreign Affairs for the extradition of respondent Mark
Jimenez to the U.S. The Grand Jury Indictment. The warrant for his arrest, and other supporting documents
for said extradition were attached along with the request. Charges include:
1. Conspiracy to commit offense or to defraud the US
2. Attempt to evade or defeat tax
3. Fraud by wire, radio, or television
4. False statement or entries
5. Election contribution in name of another

The Department of Justice (DOJ), through a designated panel proceeded with the technical evaluation
and assessment of the extradition treaty which they found having matters needed to be addressed. Respondent,
then requested for copies of all the documents included in the extradition request and for him to be given
ample time to assess it. The Secretary of Justice denied request on the following grounds:
1. He found it premature to secure him copies prior to the completion of the evaluation. At that point in
time, the DOJ is in the process of evaluating whether the procedures and requirements under the
relevant law (PD 1069 Philippine Extradition Law) and treaty (RP-US Extradition Treaty) have been
complied with by the Requesting Government. Evaluation by the DOJ of the documents is not a

32
preliminary investigation like in criminal cases making the constitutionally guaranteed rights of the
accused in criminal prosecution inapplicable.
2. The U.S. requested for the prevention of unauthorized disclosure of the information in the documents.
3. The department is not in position to hold in abeyance proceedings in connection with an extradition
request, as Philippines is bound to Vienna Convention on law of treaties such that every treaty in force
is binding upon the parties.

Mark Jimenez then filed a petition against the Secretary of Justice. RTC presiding Judge Lantion
favored Jimenez. Secretary of Justice was made to issue a copy of the requested papers, as well as conducting
further proceedings. Thus, this petition is now at bar.

ISSUE: Whether or not to uphold a citizen’s basic due process rights or the government’s ironclad duties
under a treaty.

RULING:
The human rights of person, whether citizen or alien , and the rights of the accused guaranteed in our
Constitution should take precedence over treaty rights claimed by a contracting state. The duties of the
government to the individual deserve preferential consideration when they collide with its treaty obligations to
the government of another state. This is so although we recognize treaties as a source of binding obligations
under generally accepted principles of international law incorporated in our Constitution as part of the law of
the land.
The doctrine of incorporation is applied whenever municipal tribunals are confronted with situation in
which there appears to be a conflict between a rule of international law and the provision of the constitution or
statute of the local state.
Petitioner (Secretary of Justice) is ordered to furnish Mark Jimenez copies of the extradition request
and its supporting papers, and to grant him (Mark Jimenez) a reasonable period within which to file his
comment with supporting evidence.
“Under the Doctrine of Incorporation, rules of international law form part of the law of the land and no
further legislative action is needed to make such rules applicable in the domestic sphere.
“The doctrine of incorporation is applied whenever municipal tribunals are confronted with situations
in which there appears to be a conflict between a rule of international law and the provisions of the constitution
or statute of the local state.
“Efforts should first be exerted to harmonize them, so as to give effect to both since it is to be presumed
that municipal law was enacted with proper regard for the generally accepted principles of international law in
observance of the incorporation clause in the above cited constitutional provision.
“In a situation, however, where the conflict is irreconcilable and a choice has to be made between a rule
of international law and a municipal law, jurisprudence dictates that municipal law should be upheld by the
municipal courts, for the reason that such courts are organs of municipal law and are accordingly bound by it in
all circumstances.
“The fact that international law has been made part of the law of the land does not pertain to or imply
the primacy of international law over national or municipal law in the municipal sphere. The doctrine of
incorporation, as applied in most countries, decrees that rules of international law are given equal standing
with, but are not superior to, national legislative enactments. Accordingly, the principle lex posterior derogate
priori takes effect – a treaty may repeal a statute and a statute may repeal a treaty. In states where the
Constitution is the highest law of the land, such as the Republic of the Philippines, both statutes and treaties
may be invalidated if they are in conflict with the constitution.

CUDIA VS SUPERINTENDENT OF THE PHILIPPINE MILITARY ACADEMY


751 SCRA 469 (2015)

Aldrin Jeff Cudia was a member of the Philippine Military Academy (PMA) Siklab Diwa Class of 2014.
On November 14, 2013, Cudia’s class had a lesson examination in their Operations Research (OR) subject the
schedule of which was from 1:30pm to 3pm.

33
However, after he submitted his exam paper, Cudia made a query to their OR teacher. Said teacher,
then asked Cudia to wait for her. Cudia complied and as a result, he was late for his next class (English). Later,
the English teacher reported Cudia for being late.
In his explanation, Cudia averred that he was late because his OR class was dismissed a bit late. The
tactical officer (TO) tasked to look upon the matter concluded that Cudia lied when he said that their OR class
was dismissed late because the OR teacher said she never dismissed her class late. Thus, Cudia was meted with
demerits and touring hours because of said infraction.
Cudia did not agree with the penalty hence he asked the TO about it. Not content with the explanation
of the TO, Cudia said he will be appealing the penalty he incurred to the senior tactical officer (STO). The TO
then asked Cudia to write his appeal.
In his appeal, Cudia stated that his being late was out of his control because his OR class was dismissed
at 3pm while his English class started at 3pm also. To that the TO replied: that on record, and based on the
interview with the teachers concerned, the OR teacher did not dismiss them (the class) beyond 3pm and the
English class started at 3:05pm, not 3pm; that besides, under PMA rules, once a student submitted his
examination paper, he is dismissed from said class and may be excused to leave the classroom, hence, Cudia
was in fact dismissed well before 3pm; that it was a lie for Cudia to state that the class was dismissed late
because again, on that day in the OR class, each student was dismissed as they submit their examination, and
were not dismissed as a class; that if Cudia was ordered by the teacher to stay, it was not because such
transaction was initiated by the teacher, rather, it was initiated by Cudia (because of his query to the teacher),
although there were at least two students with Cudia at that time querying the teacher, the three of them
cannot be considered a “class”; Cudia could just have stated all that instead of saying that his class was
dismissed a bit late, hence he lied. The STO sustained the decision of the TO.
Later, the TO reported Cudia to the PMA’s Honor Committee (HC) for allegedly violating the Honor
Code. Allegedly, Cudia lied in his written appeal when he said his class was dismissed late hence, as a result, he
was late for his next class.
The Honor Code is PMA’s basis for the minimum standard of behavior required of their cadets. Any
violation thereof may be a ground to separate a cadet from PMA.
Cudia submitted an explanation to the HC. Thereafter, the HC, which is composed of nine (9) cadets,
conducted an investigation. After two hearings and after the parties involved were heard and with their
witnesses presented, the HC reconvened and the members cast their vote. The initial vote was 8-1: 8 found
Cudia guilty and 1 acquitted Cudia. Under PMA rules (Honor System), a dissenting vote means the acquittal of
Cudia. However, they also have a practice of chambering where the members, particularly the dissenter, are
made to explain their vote. This is to avoid the “tyranny of the minority”. After the chambering, the dissenter
was convinced that his initial “not guilty vote” was improper, hence he changed the same and the final vote
became 9-0. Thus, Cudia was immediately placed inside PMA’s holding center.
Cudia appealed to the HC chairman but his appeal was denied. Eventually, the Superintendent of the
PMA ordered the dismissal of Cudia from the PMA.
Cudia and several members of his family then sent letters to various military officers requesting for a re-
investigation. It was their claim that there were irregularities in the investigation done by the HC. As a result of
such pleas, the case of Cudia was referred to the Cadet Review and Appeals Board of PMA (CRAB).
Meanwhile, Cudia’s family brought the case to the Commission on Human Rights (CHR) where it was
alleged that PMA’s “sham” investigation violated Cudia’s rights to due process, education, and privacy of
communication.
Eventually, the CRAB ruled against Cudia. This ruling was affirmed by the AFP Chief of Staff. But on the
other hand, the CHR found in favor of Cudia.
PMA averred that CHR’s findings are at best recommendatory. Cudia filed a petition for certiorari,
prohibition, and mandamus before the Supreme Court. PMA opposed the said petition as it argued that the
same is not proper as a matter of policy and that the court should avoid interfering with military matters.

ISSUES:
1. Whether or not Cudia’s petitions is proper.
2. Whether or not the PMA can validly dismiss Cudia based on its findings.

HELD:
I. Mandamus is not proper
34
Mandamus will not prosper in this case. Cudia’s prayer that PMA should be compelled to reinstate him
as well as to give him his supposed academic awards is not proper. The Courts, even the Supreme Court, cannot
compel PMA to do so because the act of restoring Cudia’s rights and entitlements as a cadet as well as his
awards is a discretionary act. Mandamus cannot be availed against an official or government agency, in this
case PMA, whose duty requires the exercise of discretion or judgment. Further, such act which PMA was
sought by Cudia to perform is within PMA’s academic freedom as an educational institution – and such
performance is beyond the jurisdiction of courts.

Certiorari is allowed
The petition for certiorari is allowed because the issue herein is whether or not PMA and its responsible
officers acted with grave abuse of discretion when it dismissed Cudia. Under the Constitution, that is the duty
of the courts to decide actual controversies and to determine whether or not a government branch or
instrumentality acted with grave abuse of discretion. Thus, PMA cannot argue that judicial intervention into
military affairs is not proper as a matter of policy. Suffice it to say that judicial non-interference in military
affairs is not an absolute rule.

On the civil liberties of PMA cadets


One of the arguments raised by PMA is that cadets, when they enrolled in the PMA, have surrendered
parts of their civil and political liberties. Hence, when they are disciplined and punished by the PMA, said
cadets cannot question the same, much less, question it in the courts. in short, they cannot raise due process.
On this, the SC held that such argument is wrong. It is true that a PMA cadet, by enrolling at PMA, must
be prepared to subordinate his private interests for the proper functioning of the educational institution he
attends to, one that is with a greater degree than a student at a civilian public school. However, a cadet facing
dismissal from PMA, whose private interests are at stake (life, liberty, property) which includes his honor, good
name, and integrity, is entitled to due process. No one can be deprived of such without due process of law and
the PMA, even as a military academy, is not exempt from such strictures. Thus, when Cudia questioned in court
the manner upon which he was dismissed from the PMA, such controversy may be inquired upon by the courts.
(Author’s note: PMA, in essence, raised that due process, as contemplated by the Constitution, is not
needed in dismissing a cadet yet, as can be seen in the below discussion, PMA presented evidence that due
process was, in fact, complied with.)

II. Yes. It is within PMA’s right to academic freedom to decide whether or not a cadet is still worthy to be part
of the institution. Thus, PMA did not act with grave abuse of discretion when it dismissed Cudia. In fact, Cudia
was accorded due process. In this case, the investigation of Cudia’s Honor Code violation followed the
prescribed procedure and existing practices in the PMA. He was notified of the Honor Report submitted by his
TO. He was then given the opportunity to explain the report against him. He was informed about his options
and the entire process that the case would undergo. The preliminary investigation immediately followed after
he replied and submitted a written explanation. Upon its completion, the investigating team submitted a
written report together with its recommendation to the HC Chairman. The HC thereafter reviewed the findings
and recommendations. When the honor case was submitted for formal investigation, a new team was assigned
to conduct the hearing. During the formal investigation/hearing, he was informed of the charge against him
and given the right to enter his plea. He had the chance to explain his side, confront the witnesses against him,
and present evidence in his behalf. After a thorough discussion of the HC voting members, he was found to
have violated the Honor Code. Thereafter, the guilty verdict underwent the review process at the Academy level
– from the OIC of the HC, to the SJA (Staff Judge Advocate), to the Commandant of Cadets, and to the PMA
Superintendent. A separate investigation was also conducted by the HTG (Headquarters Tactics Group). Then,
upon the directive of the AFP-GHQ (AFP-General Headquarters) to reinvestigate the case, a review was
conducted by the CRAB. Further, a Fact-Finding Board/Investigation Body composed of the CRAB members
and the PMA senior officers was constituted to conduct a deliberate investigation of the case. Finally, he had
the opportunity to appeal to the President. Sadly for him, all had issued unfavorable rulings. And there is no
reason for the SC to disturb the findings of facts by these bodies.

Academic freedom of the PMA


Cudia would argue that there is no law providing that a guilty finding by the HC may be used by the
PMA to dismiss or recommend the dismissal of a cadet from the PMA; that Honor Code violation is not among
35
those listed as justifications for the attrition of cadets considering that the Honor Code and the Honor System
(manner which PMA conducts investigation of Honor Code violations) do not state that a guilty cadet is
automatically terminated or dismissed from service.
Such argument is not valid. Even without express provision of a law, the PMA has regulatory authority
to administratively dismiss erring cadets. Further, there is a law (Commonwealth Act No. 1) authorizing the
President to dismiss cadets. Such power by the President may be delegated to the PMA Superintendent, who
may exercise direct supervision and control over the cadets.
Further, as stated earlier, such power by the PMA is well within its academic freedom. Academic
freedom or, to be precise, the institutional autonomy of universities and institutions of higher learning has
been enshrined in the Constitution.
The essential freedoms of academic freedom on the part of schools are as follows;
a. the right to determine who may teach;
b. the right to determine what may be taught;
c. the right to determine how it shall be taught;
d. the right to determine who may be admitted to study.

The Honor Code is just but one way for the PMA to exercise its academic freedom. If it determines that
a cadet violates it, then it has the right to dismiss said cadet. In this case, based on its findings, Cudia lied –
which is a violation of the Honor Code.

But Cudia’s lie is not even that big; is dismissal from the PMA really warranted?
The PMA Honor Code does not distinguish between a big lie and a minor lie. It punishes any form of
lying. It does not have a gradation of penalties. In fact, it is the discretion of the PMA as to what penalty may be
imposed. When Cudia enrolled at PMA, he agreed to abide by the Honor Code and the Honor System. Thus,
while the punishment may be severe, it is nevertheless reasonable and not arbitrary, and, therefore, not in
violation of due process -also considering that Cudia, as a cadet, must have known all of these.

GO VS COLEGIO DE SAN JUAN DE LETRAN


683 SCRA 358

Facts;
In October 2001, Mr. George Isleta, the head of Letran’s auxiliary Services department, received an
information that certain fraternities were recruiting new members among the Letran’s high school students,
together with the list of the allegedly involved students. The school conducted medical examinations on the
students involved and on November 20,2002 the school physician reported that 6 students bore injuries on
the posterior portions or their thighs, Mr. Rosarda, the Assistant Prefect for discipline, conferred with the
students and asked for their explanations in writing. Four students admitted that they were neophytes of the
Tau Gamma Fraternity and were present in a hazing rite held in Tondo, manila. They also identified the senior
members of the fraternity present at their hazing. These included Kim Go, a fourth year highschool student. In
the meantime, the school’s security officer prepared an incident report that the Tau Gamma Fraternity has
been recruiting members from Letran’s high school department. He had spoken to one of the neophytes and
obtain a list of eighteen members of the fraternity currently enrolled at the high school department. Kim’s
name was also in the list. Mr. Rosarda has informed kim’s mother, that her son is a fraternity member whereas
she expressed her disbelief stating that her son has always been in constant supervision. Kim responded a
written statement dated December 19,2001 and denied that he was a fraternity member.
In time, the respondents found that twenty nine of their students, including Kim, were fraternity
members. The respondents found substantial basis in the neophytes statements that Kim was a senior
fraternity member. Based on their disciplinary rules, the father Prefect for Discipline Rev. Rhommel
Hernandez recommended for the dismissal of the fraternity members from the highschool department rolls.
On January 2002, the petitioners filed a complaint for damages before the RTC of Caloocan City claiming that
the repsondents had unlawfully dismissed Kim. They refused to accept that Kim was a fraternity member. They
likewise insisted that due process has not been observed. Sps Go sought compensation for the business
opportunity losses” they suffered while personally attending Kim’s disciplinary case. The RTC ruled that Kim
was dismissed without due process, his membership in the fraternity was not duly proven and the school had
36
no authority to dismiss Kim from school. The court of appeals disagreed with the RTC and reversed the
decision. Hence, the petition.

ISSUE:
Whether or not repsondent’s had unlawfully dismissed Kim from the high School department.

RULING:
No.
We see no merit in this argument as the petitioners apparently hew to an erroneous view of
administrative due process. Jurisprudence has clarified that administrative due process cannot be fully
equated with due process in the strict judicial sense. The very nature of due process negates the any concept of
inflexible procedures universally applicable to every imaginable situation. Thus we are hard pressed to believe
that Kim’s denial of his fraternity membership before formal notice was given worked against his interest in the
disciplinary case. What matters purpose is notice of what is to be explained, not in the form in which the notice
is given. The raison d’etre of the written notice rule is to inform the student of the disciplinary charge against
him and to enable him to suitably prepare a defense. The records show that as early as November 23, 2001, it
was already plain to the petitoners that the subject matter of the case against Kim was his alleged fraternity
membership. Thus, the time Mr. Rosarda spoke to Kim and asked for his written explanation in December
2001, Kim has had enough time to prepare his response to this plain charge. The essence of due process, it
bears repeating is simply the opportunity to be heard. And Kim had been heard. His written explanation was
received, indeed even solicited by the respondents. Thus, he cannot claim that he was denied the right to
adduce evidence in his behalf.
In Licup v. San Carlos University, the Court held that when a student commits a serious breach of
discipline or fails to maintain the required academic standard, he forfeits his contractual right, and the court
should not review the discretion of university authorities.
As a final point, the Court of Appeals correctly held that there were no further bases to hold the
respondents liable for moral or exemplary damages. Our study of the records confirms that the respondents did
not act with bad faith, malice, fraud, or improper or wilful motive or conduct in disciplining Kim.
Wherefore, premises considered we hereby affirm the decision dated May 27, 2005 of the Court of
Appeals.

G.R. No. 211833, April 07, 2015


FERDINAND R. VILLANUEVA, PRESIDING JUDGE, MCTC, COMPOSTELA-NEW BATAAN,
COMPOSTELA VALLEY PROVINCE, Petitioner, v. JUDICIAL AND BAR COUNCIL, Respondent.

FACTS:
The petitioner was appointed on September 18, 2012 as the Presiding Judge of the Municipal Circuit
Trial Court, Compostela-New Bataan, Poblacion, Compostela Valley Province, Region XI, which is a first-level
court. On September 27, 2013, he applied for the vacant position of Presiding Judge in the following Regional
Trial Courts (RTCs): Branch 31, Tagum City; Branch 13, Davao City; and Branch 6, Prosperidad, Agusan Del
Sur In a letter2 dated December 18, 2013, JBC's Office of Recruitment, Selection and Nomination, informed
the petitioner that he was not included in the list of candidates for the said stations. On the same date, the
petitioner sent a letter, through electronic mail, seeking reconsideration of his non-inclusion in the list of
considered applicants and protesting the inclusion of applicants who did not pass the prejudicature
examination. The petitioner was informed by the JBC Executive Officer, through a letter3 dated February 3,
2014, that his protest and reconsideration was duly noted by the JBC en banc. However, its decision not to
include his name in the list of applicants was upheld due to the JBC's long-standing policy of opening the
chance for promotion to second-level courts to, among others, incumbent judges who have served in their
current position for at least five years, and since the petitioner has been a judge only for more than a year, he
was excluded from the list. This caused the petitioner to take recourse to this Court

ISSUE:

37
WON the writ of certiorari and prohibition cannot issue to prevent the JBC from performing its
principal function under the Constitution to recommend appointees to the Judiciary because the JBC is not a
tribunal exercising judicial or quasi-judicial function

HELD:
The remedies of certiorari and prohibition are tenable. "The present Rules of Court uses two special
civil actions for determining and correcting grave abuse of discretion amounting to lack or excess of
jurisdiction.
In this case, it is clear that the JBC does not fall within the scope of a tribunal, board, or officer
exercising judicial or quasi-judicial functions. In the process of selecting and screening applicants, the JBC
neither acted in any judicial or quasi-judicial capacity nor assumed unto itself any performance of judicial or
quasi-judicial prerogative. However, since the formulation of guidelines and criteria, including the policy that
the petitioner now assails, is necessary and incidental to the exercise of the JBC's constitutional mandate, a
determination must be made on whether the JBC has acted with grave abuse of discretion amounting to lack or
excess of jurisdiction in issuing and enforcing the said policy.
Besides, the Court can appropriately take cognizance of this case by virtue of the Court's power of
supervision over the JBC. Jurisprudence provides that the power of supervision is the power of oversight, or
the authority to see that subordinate officers perform their duties.
Following this definition, the supervisory authority of the Court over the JBC is to see to it that the JBC
complies with its own rules and procedures. Thus, when the policies of the JBC are being attacked, then the
Court, through its supervisory authority over the JBC, has the duty to inquire about the matter and ensure that
the JBC complies with its own rules.
The remedy of mandamus cannot be availed of by the petitioner in assailing JBC's policy. It is essential
to the issuance of a writ of mandamus that the applicant should have a clear legal right to the thing demanded
and it must be the imperative duty of the respondent to perform the act required. The remedy of mandamus, as
an extraordinary writ, lies only to compel an officer to perform a ministerial duty, not a discretionary one.14
Clearly, the use of discretion and the performance of a ministerial act are mutually exclusive. Clearly, to be
included as an applicant to second-level judge is not properly compellable by mandamus inasmuch as it
involves the exercise of sound discretion by the JBC.
The petition for declaratory relief is improper. "An action for declaratory relief should be filed by a
person interested under a deed, a will, a contract or other written instrument, and whose rights are affected by
a statute, an executive order, a regulation or an ordinance. The relief sought under this remedy includes the
interpretation and determination of the validity of the written instrument and the judicial declaration of the
parties' rights or duties thereunder."
In this case, the petition for declaratory relief did not involve an unsound policy. Rather, the petition
specifically sought a judicial declaration that the petitioner has the right to be included in the list of applicants
although he failed to meet JBC's five-year requirement policy. Again, the Court reiterates that no person
possesses a legal right under the Constitution to be included in the list of nominees for vacant judicial
positions. The opportunity of appointment to judicial office is a mere privilege, and not a judicially enforceable
right that may be properly claimed by any person
Furthermore, the instant petition must necessarily fail because this Court does not have original
jurisdiction over a petition for declaratory relief even if only questions of law are involved.18 The special civil
action of declaratory relief falls under the exclusive jurisdiction of the appropriate RTC pursuant to Section
1919 of Batas Pambansa Blg. 129, as amended by R.A.No. 7691.
Therefore, by virtue of the Court's supervisory duty over the JBC and in the exercise of its expanded
judicial power, the Court assumes jurisdiction over the present petition. But in any event, even if the Court will
set aside procedural infirmities.

Elena Daycaico VS Social Security System

Facts:
Elena Dycaico seeks to reverse the Decision of the Court of Appeals that affirmed the decision of Social
Security Commission denying her claim for survivor’s pension which accrues from the death of her husband,
Bonifacio Dycaico. Bonifacio Dycaico became a member of SSS and designated Elena Dycaico and their eight
38
children as beneficiaries therein. At that time, Bonifacio and Elena lived together as husband and wife without
the benefit of marriage. Nine years after, Bonifacio was considered retired and began receiving his monthly
pension from the SSS. He continued to receive the monthly pension until he passed away. A few months prior
to his death, however, Bonifacio married the petitioner.
Shortly after Bonifacio’s death, the petitioner filed with the SSS an application for survivor’s pension.
Her application, however, was denied on the ground that they were not living under the benefit of marriage
when Bonifacio became a member of SSS. The basis was Section 12-B(d) of Republic Act (Rep. Act) No. 8282
which reads: Sec. 12-B. Retirement Benefits.

(d) Upon the death of the retired member, his primary beneficiaries as of the date of his retirement
shall be entitled to receive the monthly pension. …

An appeal was made to the Court of Appeals but it was, likewise, denied. The same Court ruled that that
since the petitioner was merely the common-law wife of Bonifacio at the time of his retirement, his designation
of the petitioner as one of his beneficiaries is void. The petitioner claims that there is no merit to the decision of
Court of Appeals as the SSS law does is silent denying the beneficiary’s claim for survivor pension.

Issue:
Whether or not there is a violation to equal protection clause of the Constitution.

HELD:
As illustrated by the petitioners case, the proviso as of the date of his retirement in Section 12-B(d) of
Rep. Act No. 8282 which qualifies the term primary beneficiaries results in the classification of dependent
spouses as primary beneficiaries into two groups:
1) Those dependent spouses whose respective marriages to SSS members were contracted prior to the
latter’s retirement; and
2) Those dependent spouses whose respective marriages to SSS members were contracted after the latter’s
retirement.

Underlying these two classifications of dependent spouses is that their respective marriages are valid. In
other words, both groups are legitimate or legal spouses. The distinction between them lies solely on the date
the marriage was contracted. The petitioner belongs to the second group of dependent spouses, i.e., her
marriage to Bonifacio was contracted after his retirement. As such, she and those similarly situated do not
qualify as primary beneficiaries under Section 12-B(d) of Rep. Act No. 8282 and, therefore, are not entitled to
survivors pension under the same provision by reason of the subject proviso.
The petitioner belongs to the second group of dependent spouses, i.e., her marriage to Bonifacio was
contracted after his retirement. She and those similarly situated are undoubtedly discriminated against as the
proviso as of the date of his retirement disqualifies them from being considered primary beneficiaries for the
purpose of entitlement to survivors pension.
Generally, a statute based on reasonable classification does not violate the constitutional guaranty of
the equal protection clause of the law. With respect to Rep. Act No. 8282, in particular, as a social security law,
it is recognized that it is permeated with provisions that draw lines in classifying those who are to receive
benefits. Congressional decisions in this regard are entitled to deference as those of the institution charged
under our scheme of government with the primary responsibility for making such judgments in light of
competing policies and interests.
However, classifying dependent spouses and determining their entitlement to survivors pension based
on whether the marriage was contracted before or after the retirement of the other spouse, regardless of the
duration of the said marriage, bears no relation to the achievement of the policy objective of the law, i.e.,
provide meaningful protection to members and their beneficiaries against the hazard of disability, sickness,
maternity, old age, death and other contingencies resulting in loss of income or financial burden." The nexus
of the classification to the policy objective is vague and flimsy. Put differently, such classification of dependent
spouses is not germane to the aforesaid policy objective.
A marriage contracted by a retired SSS member after the said age may still last for more than ten years,
assuming the member lives up to over seventy (70) years old. In such a case, it cannot be said that the marriage
was a sham or was entered into solely for the purpose of enabling one spouse to obtain the financial benefits
39
due upon the death of the other spouse. Nonetheless, the said surviving spouse is not entitled to survivors
pension because he or she is not a primary beneficiary as of the date of retirement of the SSS member following
Section 12-B(d) of Rep. Act No. 8282.
Further, the classification of dependent spouses on the basis of whether their respective marriages to
the SSS member were contracted prior to or after the latter’s retirement for the purpose of entitlement to
survivors pension does not rest on real and substantial distinctions. It is arbitrary and discriminatory. It is too
sweeping because the proviso as of the date of his retirement, which effectively disqualifies the dependent
spouses whose respective marriages to the retired SSS member were contracted after the latter’s retirement as
primary beneficiaries, unfairly lumps all these marriages as sham relationships or were contracted solely for
the purpose of acquiring benefits accruing upon the death of the other spouse. The proviso thus unduly
prejudices the rights of the legal surviving spouse, like the petitioner, and defeats the avowed policy of the law
to provide meaningful protection to members and their beneficiaries against the hazards of disability, sickness,
maternity, old age, death, and other contingencies resulting in loss of income or financial burden.

REPUBLIC VS ALBIOS
707 SCRA 584 (2013)

Facts:
On October 22, 2004, Fringer, an American citizen, and Albios were married before Judge Ofelia I. Calo
of the Metropolitan Trial Court, Branch 59, Mandaluyong City (MeTC).
On December 6, 2006, Albios filed with the RTC a petition for declaration of nullity[4] of her marriage
with Fringer. She alleged that immediately after their marriage, they separated and never lived as husband and
wife because they never really had any... intention of entering into a married state or complying with any of
their essential marital obligations. She described their marriage as one made in jest and, therefore, null and
void ab initio.
Summons was served on Fringer but he did not file his answer. On September 13, 2007, Albios filed a
motion to set case for pre-trial and to admit her pre-trial brief.
At the pre-trial, only Albios, her counsel and the prosecutor appeared.
Fringer did not attend the hearing despite being duly notified of the schedule.
The RTC declared the marriage void ab initio
The RTC was of the view that the parties married each other for convenience only. Giving credence to
the testimony of Albios, it stated that she contracted Fringer to enter into a marriage to enable her to acquire
American citizenship; that in consideration thereof, she agreed... to pay him the sum of $2,000.00... she did
not pay him the $2,000.00 because he never processed her petition for... citizenship.
The OSG filed an appeal before the CA.
The CA affirmed the RTC ruling which found that the essential requisite of consent was lacking.

Issues:
Is a marriage, contracted for the sole purpose of acquiring American citizenship in consideration of
$2,000.00, void ab initio on the ground of lack of consent?

Ruling:
The marriage is valid
The Court resolves in the negative
In 1975, the seminal case of Bark v. Immigration and Naturalization Service,[15] established the
principal test for determining the presence of marriage fraud in immigration cases. It ruled that a "marriage is
a sham if the bride and groom did not... intend to establish a life together at the time they were married." This
standard was modified with the passage of the Immigration Marriage Fraud Amendment of 1986 (IMFA),
which now requires the couple to instead demonstrate that the marriage was not "entered into for... the
purpose of evading the immigration laws of the United States." The focus, thus, shifted from determining the
intention to establish a life together, to determining the intention of evading immigration laws.
Under said Article 2, for consent to be valid, it must be (1) freely given and (2) made in the presence of a
solemnizing officer. A "freely given" consent requires that the contracting parties willingly and deliberately
enter into the marriage. Consent must be real in the... sense that it is not vitiated nor rendered defective by any
40
of the vices of consent under Articles 45 and 46 of the Family Code, such as fraud, force, intimidation, and
undue influence.[24] Consent must also be conscious or intelligent, in that the... parties must be capable of
intelligently understanding the nature of, and both the beneficial or unfavorable consequences of their act
Based on the above, consent was not lacking between Albios and Fringer. In fact, there was real consent
because it was not vitiated nor rendered defective by any vice of consent. Their consent was also conscious and
intelligent as they understood the nature and the... beneficial and inconvenient consequences of their marriage,
as nothing impaired their ability to do so. That their consent was freely given is best evidenced by their
conscious purpose of acquiring American citizenship through marriage. Such plainly demonstrates that they...
willingly and deliberately contracted the marriage. There was a clear intention to enter into a real and valid
marriage so as to fully comply with the requirements of an application for citizenship. There was a full and
complete understanding of the legal tie that would be... created between them, since it was that precise legal tie
which was necessary to accomplish their goal.

SPOUSES CARLOS S. ROMUALDEZ and ERLINDA R. ROMUALDEZ, petitioners,


vs.COMMISSION ON ELECTIONS and DENNIS GARAY, respondents.
G.R. No. 167011 April 30, 2008
CHICO-NAZARIO, J.

Facts:
Garay and Apostol filed a complaint against Sps. Romualdez for violation of the OEC and RA 8189 or
Voter’s Registration Act of 1996 for making false information as to their residence in their applications as new
voters in Burauen, Leyte.
The Complaint-Affidavit contained a prayer that a preliminary investigation be conducted by the
COMELEC, and if the evidence so warrants, the corresponding Information against petitioners be filed before
the Regional Trial Court (RTC) for the prosecution of the same.
Sps. Romualdez contends that they intend to reside in Burauen, Leyte since 1989. On May 2000, they
took actual residence in Burauen by leasing for 5 years the house of Renomeron.
The Complaint-Affidavit contained a prayer that a preliminary investigation be conducted by the
COMELEC, and if the evidence so warrants, the corresponding Information against petitioners be filed before
the Regional Trial Court (RTC) for the prosecution of the same.

Issue: WON due process was violated.

Held: No.
First, the Complaint-Affidavit filed by private respondent with the COMELEC is couched in a language
which embraces the allegations necessary to support the charge for violation of Section 10(g) and (j), in relation
to Section 45(j) of Republic Act No. 8189.
Petitioners cannot be said to have been denied due process on the claim that the election offenses
charged against them by private respondent are entirely different from those for which they stand to be accused
of before the RTC, as charged by the COMELEC. In the first place, there appears to be no incongruity between
the charges as contained in the Complaint-Affidavit and the Information filed before the RTC, notwithstanding
the denomination by private respondent of the alleged violations to be covered by Section 261(y)(2) and
Section 261(y)(5) of the Omnibus Election Code and Section 12 of Republic Act No. 8189. Evidently, the
Information directed to be filed by the COMELEC against petitioners, and which were, in fact, filed with the
RTC, were based on the same set of facts as originally alleged in the private respondent’s Complaint-Affidavit.
In Lacson, we underscored the elementary rule that the jurisdiction of a court is determined by the
allegations in the Complaint or Information, and not by the evidence presented by the parties at the trial.
Indeed, in Lacson, we articulated that the real nature of the criminal charge is determined not from the caption
or preamble of the Information nor from the specification of the provision of law alleged to have been violated,
they being conclusions of law, but by the actual recital of facts in the Complaint or Information.
Petitioners’ reliance on Lacson, however, does not support their claim of lack of due process because, as
we have said, the charges contained in private respondent’s Complaint-Affidavit and the charges as directed by
the COMELEC to be filed are based on the same set of facts. In fact, the nature of the criminal charges in
private respondent’s Complaint-Affidavit and that of the charges contained in the Information filed with the
41
RTC, pursuant to the COMELEC Resolution En Banc are the same, such that, petitioners cannot claim that
they were not able to refute or submit documentary evidence against the charges that the COMELEC filed with
the RTC. Petitioners were afforded due process because they were granted the opportunity to refute the
allegations in private respondent’s Complaint-Affidavit. On 2 April 2001, in opposition to the Complaint-
Affidavit, petitioners filed a Joint Counter-Affidavit with Motion to Dismiss with the Law Department of the
COMELEC. They similarly filed a Memorandum before the said body. Finding that due process was not
dispensed with under the circumstances in the case at bar, we agree with the stance of the Office of the Solicitor
General that petitioners were reasonably apprised of the nature and description of the charges against them. It
likewise bears stressing that preliminary investigations were conducted whereby petitioners were informed of
the complaint and of the evidence submitted against them. They were given the opportunity to adduce
controverting evidence for their defense. In all these stages, petitioners actively participated.

SOUTHERN HEMISPHERE ENGAGEMENT NETWORK, INC. and ATTY. SOLIMAN M. SANTOS,


JR. v. ANTI-TERRORISM COUNCIL, et al.

CONSOLIDATED WITH: G.R. No. 178554; G.R. No. 178581; G.R. No. 178890; G.R. No. 179157;
G.R. No. 179461

FACTS:
Six petitions for certiorari and prohibition were filed challenging the constitutionality of RA 9372,
otherwise known as the Human Security Act. Impleaded as respondents in the various petitions are the Anti-
Terrorism Council composed of, at the time of the filing of the petitions, Executive Secretary Eduardo Ermita
as Chairperson, Justice Secretary Raul Gonzales as Vice Chairperson, and Foreign Affairs Secretary Alberto
Romulo, Acting Defense Secretary and National Security Adviser Norberto Gonzales, Interior and Local
Government Secretary Ronaldo Puno, and Finance Secretary Margarito Teves as members. All the petitions,
except that of the IBP, also impleaded Armed Forces of the Philippines (AFP) Chief of Staff Gen. Hermogenes
Esperon and Philippine National Police (PNP) Chief Gen. Oscar Calderon.

ISSUE:
Whether or not the petition should prosper?

HELD:
Section 1, Rule 65 of the Rules of Court provides: Section 1. Petition for certiorari. When any tribunal,
board or officer exercising judicial or quasi-judicial functions has acted without or in excess of its or his
jurisdiction, or with grave abuse of discretion amounting to lack or excess of jurisdiction, and there is no
appeal, nor any plain, speedy, and adequate remedy in the ordinary course of law, a person aggrieved thereby
may file a verified petition in the proper court, alleging the facts with certainty and praying that judgment be
rendered annulling or modifying the proceedings of such tribunal, board or officer, and granting such
incidental reliefs as law and justice may require.
Parenthetically, petitioners do not even allege with any modicum of particularity how respondents acted
without or in excess of their respective jurisdictions, or with grave abuse of discretion amounting to lack or
excess of jurisdiction.

***
Petitioners assail for being intrinsically vague and impermissibly broad the definition of the crime of
terrorism under RA 9372 in that terms like "widespread and extraordinary fear and panic among the populace"
and "coerce the government to give in to an unlawful demand" are nebulous, leaving law enforcement agencies
with no standard to measure the prohibited acts.
A statute or act suffers from the defect of vagueness when it lacks comprehensible standards that men
of common intelligence must necessarily guess at its meaning and differ as to its application. It is repugnant to
the Constitution in two respects: (1) it violates due process for failure to accord persons, especially the parties
targeted by it, fair notice of the conduct to avoid; and (2) it leaves law enforcers unbridled discretion in
carrying out its provisions and becomes an arbitrary flexing of the Government muscle. The overbreadth
doctrine, meanwhile, decrees that a governmental purpose to control or prevent activities constitutionally
42
subject to state regulations may not be achieved by means which sweep unnecessarily broadly and thereby
invade the area of protected freedoms.
As distinguished from the vagueness doctrine, the overbreadth doctrine assumes that individuals will
understand what a statute prohibits and will accordingly refrain from that behavior, even though some of it is
protected.
Distinguished from anas-applied challenge which considers only extant facts affecting real litigants, a
facial invalidation is an examination of the entire law, pinpointing its flaws and defects, not only on the basis of
its actual operation to the parties, but also on the assumption or prediction that its very existence may cause
others not before the court to refrain from constitutionally protected speech or activities.
Justice Mendoza accurately phrased the subtitle in his concurring opinion that the vagueness and
overbreadth doctrines, as grounds for a facial challenge, are not applicable to penal laws. A litigant cannot thus
successfully mount a facial challenge against a criminal statute on either vagueness or overbreadth grounds.
Since a penal statute may only be assailed for being vague as applied to petitioners, a limited vagueness
analysis of the definition of "terrorism" in RA 9372 is legally impermissible absent an actual or imminent
charge against them.
In insisting on a facial challenge on the invocation that the law penalizes speech, petitioners contend
that the element of "unlawful demand" in the definition of terrorism must necessarily be transmitted through
some form of expression protected by the free speech clause.
Before a charge for terrorism may be filed under RA 9372, there must first be a predicate crime actually
committed to trigger the operation of the key qualifying phrases in the other elements of the crime, including
the coercion of the government to accede to an "unlawful demand." Given the presence of the first element, any
attempt at singling out or highlighting the communicative component of the prohibition cannot re-categorize
the unprotected conduct into a protected speech.
Petitioners notion on the transmission of message is entirely inaccurate, as it unduly focuses on just one
particle of an element of the crime. Almost every commission of a crime entails some mincing of words on the
part of the offender like in declaring to launch overt criminal acts against a victim, in haggling on the amount of
ransom or conditions, or in negotiating a deceitful transaction.
As earlier reflected, petitioners have established neither an actual charge nor a credible threat of
prosecution under RA 9372. Even a limited vagueness analysis of the assailed definition of "terrorism" is thus
legally impermissible. The Court reminds litigants that judicial power neither contemplates speculative
counseling on a statutes future effect on hypothetical scenarios nor allows the courts to be used as an extension
of a failed legislative lobbying in Congress.

DISMISSED

Landmark Case: RH BILL CASE -- Imbong vs Ochoa et.al. G.R. No. 204819 April 8, 2014

FACTS:
Concerned citizens and the Catholic Church had petitioned for the constitutionality of the Reproductive
Health Bill. Republic Act (R.A.) No. 10354, otherwise known as the Responsible Parenthood and Reproductive
Health Act of 2012 (RH Law), was enacted by Congress on December 21, 2012. Challengers from various
sectors of society are questioning the constitutionality of the said Act. The petitioners are assailing the
constitutionality of RH Law on the following grounds:

ISSUE:
Whether or not the RH Law is “void-for-vagueness” in violation of the due process clause of the
Constitution.
Whether or not the RH Law violates the right to equal protection of the law.

HELD:
The RH Law does not violate the due process clause of the Constitution as the definitions of several
terms as observed by the petitioners are not vague.

43
The definition of “private health care service provider” must be seen in relation to Section 4(n) of the
RH Law which defines a “public health service provider”. The “private health care institution” cited under
Section 7 should be seen as synonymous to “private health care service provider.
The terms “service” and “methods” are also broad enough to include providing of information and
rendering of medical procedures. Thus, hospitals operated by religious groups are exempted from rendering
RH service and modern family planning methods (as provided for by Section 7 of the RH Law) as well as from
giving RH information and procedures.
The RH Law also defines “incorrect information”. Used together in relation to Section 23 (a)(1), the
terms “incorrect” and “knowingly” connote a sense of malice and ill motive to mislead or misrepresent the
public as to the nature and effect of programs and services on reproductive health.
To provide that the poor are to be given priority in the government’s RH program is not a violation of
the equal protection clause. In fact, it is pursuant to Section 11, Article XIII of the Constitution, which states
that the State shall prioritize the needs of the underprivileged, sick elderly, disabled, women, and children and
that it shall endeavor to provide medical care to paupers.
The RH Law does not only seek to target the poor to reduce their number, since Section 7 of the RH Law
prioritizes poor and marginalized couples who are suffering from fertility issues and desire to have children. In
addition, the RH Law does not prescribe the number of children a couple may have and does not impose
conditions upon couples who intend to have children. The RH Law only seeks to provide priority to the poor.
The exclusion of private educational institutions from the mandatory RH education program under
Section 14 is valid. There is a need to recognize the academic freedom of private educational institutions
especially with respect to religious instruction and to consider their sensitivity towards the teaching of
reproductive health education.

Agabon vs. NLRC / Riviera Home - GR No. 158693 Case Digest

FACTS:
Petitioners were employed by Riviera Home as gypsum board and cornice installers from January 1992
to February 23, 1999 when they were dismissed for abandonment of work. Petitioners filed a complaint for
illegal dismissal and was decided in their favor by the Labor Arbiter. Riviera appealed to the NLRC contending
just cause for the dismissal because of petitioner’s abandonment of work. NLRC ruled there was just cause and
petitioners were not entitled to backwages and separation pay. The CA in turn ruled that the dismissal was not
illegal because they have abandoned their work but ordered the payment of money claims.

ISSUE:
Whether or not petitioners were illegally dismissed.

RULING:
To dismiss an employee, the law required not only the existence of a just and valid cause but also
enjoins the employer to give the employee the right to be heard and to defend himself. Abandonment is the
deliberate and unjustified refusal of an employee to resume his employment. For a valid finding or
abandonment, two factors are considered: failure to report for work without a valid reason; and, a clear
intention to sever employer-employee relationship with the second as the more determinative factor which is
manifested by overt acts from which it may be deduced that the employees has no more intention to work.
Where the employer had a valid reason to dismiss an employee but did not follow the due process
requirement, the dismissal may be upheld but the employer will be penalized to pay an indemnity to the
employee. This became known as the Wenphil Doctrine of the Belated Due process Rule.
Art. 279 means that the termination is illegal if it is not for any of the justifiable or authorized by law.
Where the dismissal is for a just cause, the lack of statutory due process should not nullify the dismissal but the
employer should indemnify the employee for the violation of his statutory rights. The indemnity should be
stiffer to discourage the abhorrent practice of “dismiss now, pay later” which we sought to deter in Serrano
ruling. The violation of employees’ rights warrants the payment of nominal damages.

El Banco Espanol-Filipino vs. Vicente Palanca G.R. No. L-11390, March 26, 1918
44
El Banco Espanol-Filipino vs. Palanca
G.R. No. L-11390, March 26, 1918

* DUE PROCESS IN FORECLOSURE PROCEEDINGS: Property is always assumed to be in the possession of


its owner, in person or by agent; and he may be safely held, under certain conditions, to be affected with
knowledge that proceedings have been instituted for its condemnation and sale.

FACTS:
Engracio Palanca Tanquinyeng y Limquingco mortgaged various parcels of real property in Manila to El
Banco Espanol-Filipino. Afterwards, Engracio returned to China and there he died on January 29, 1810
without returning again to the Philippines. The mortgagor then instituted foreclosure proceeding but since
defendant is a non-resident, it was necessary to give notice by publication. The Clerk of Court was also directed
to send copy of the summons to the defendant’s last known address, which is in Amoy, China. It is not shown
whether the Clerk complied with this requirement. Nevertheless, after publication in a newspaper of the City of
Manila, the cause proceeded and judgment by default was rendered. The decision was likewise published and
afterwards sale by public auction was held with the bank as the highest bidder. On August 7, 1908, this sale was
confirmed by the court. However, about seven years after the confirmation of this sale, a motion was made by
Vicente Palanca, as administrator of the estate of the original defendant, wherein the applicant requested the
court to set aside the order of default and the judgment, and to vacate all the proceedings subsequent thereto.
The basis of this application was that the order of default and the judgment rendered thereon were void
because the court had never acquired jurisdiction over the defendant or over the subject of the action.

ISSUE:
* Whether or not the lower court acquired jurisdiction over the defendant and the subject matter of the
action
* Whether or not due process of law was observed

RULING:
On Jurisdiction
The word “jurisdiction” is used in several different, though related, senses since it may have reference
(1) to the authority of the court to entertain a particular kind of action or to administer a particular kind of
relief, or it may refer to the power of the court over the parties, or (2) over the property which is the subject to
the litigation.
The sovereign authority which organizes a court determines the nature and extent of its powers in
general and thus fixes its competency or jurisdiction with reference to the actions which it may entertain and
the relief it may grant.

How Jurisdiction is Acquired


Jurisdiction over the person is acquired by the voluntary appearance of a party in court and his
submission to its authority, or it is acquired by the coercive power of legal process exerted over the person.
Jurisdiction over the property which is the subject of the litigation may result either from a seizure of
the property under legal process, whereby it is brought into the actual custody of the law, or it may result from
the institution of legal proceedings wherein, under special provisions of law, the power of the court over the
property is recognized and made effective. In the latter case the property, though at all times within the
potential power of the court, may never be taken into actual custody at all. An illustration of the jurisdiction
acquired by actual seizure is found in attachment proceedings, where the property is seized at the beginning of
the action, or some subsequent stage of its progress, and held to abide the final event of the litigation. An
illustration of what we term potential jurisdiction over the res, is found in the proceeding to register the title of
land under our system for the registration of land. Here the court, without taking actual physical control over
the property assumes, at the instance of some person claiming to be owner, to exercise a jurisdiction in rem
over the property and to adjudicate the title in favor of the petitioner against the entire world.
In the terminology of American law the action to foreclose a mortgage is said to be a proceeding quasi in
rem, by which is expressed the idea that while it is not strictly speaking an action in rem yet it partakes of that
nature and is substantially such. The expression "action in rem" is, in its narrow application, used only with
reference to certain proceedings in courts of admiralty wherein the property alone is treated as responsible for
45
the claim or obligation upon which the proceedings are based. The action quasi rem differs from the true action
in rem in the circumstance that in the former an individual is named as defendant, and the purpose of the
proceeding is to subject his interest therein to the obligation or lien burdening the property. All proceedings
having for their sole object the sale or other disposition of the property of the defendant, whether by
attachment, foreclosure, or other form of remedy, are in a general way thus designated. The judgment entered
in these proceedings is conclusive only between the parties.
It is true that in proceedings of this character, if the defendant for whom publication is made appears,
the action becomes as to him a personal action and is conducted as such. This, however, does not affect the
proposition that where the defendant fails to appear the action is quasi in rem; and it should therefore be
considered with reference to the principles governing actions in rem.

G.R. Nos. L-39516-17 January 28, 1975


ROSARIO CASTILLO and SONIA VILLASANTA, petitioners,
vs.
THE HONORABLE JUDGE CELESTINO JUAN, respondent.

FACTS:
In this certiorari proceedings, petitioners, two young maidens who are the offended parties in two rape
cases, assail the actuation of respondent Judge and seek his disqualification on the ground of bias and
prejudice. What was done by him, according to their strongly-worded petition, was in disregard of the highly-
prized ideal in adjudication, likewise a due process requirement, that a litigant "is entitled to nothing less than
the cold neutrality of an impartial judge."
On two separate occasions on August 15 and 27, 1974, in the secrecy of his chambers he informed
petitioners of the weakness of their cases, the likelihood of a verdict of acquittal in favor of the accused, and
impressed upon them that it would be to their advantage to settle, as the most he could do on their behalf was
to have such accused indemnify them. This move, according to him, would assure their being spared from the
embarrassment occasioned by suits of this character, clearly prejudicial to their future. These conversations
took place even before the prosecution had finished presenting its evidence, one of the petitioners not having
testified as yet. Respondent Judge could not very well deny that he did invite them to confer with him, but he
would impress on this Court that their version should not be let credence and that he was prompted to act thus
from the best of motives, "as an act of charity" and as a "clear attempt to humanize justice.

ISSUE:
Whether or not the respondent judge violated due process.

HELD:
In every litigation, perhaps much more so in criminal cases, the manner and attitude of a trial judge are
crucial to everyone concerned, the offended party, no less than the accused. It is not for him to indulge or even
to give the appearance of catering to the at times human failing of yielding to first impressions. He is to refrain
from reaching hasty conclusions or prejudging matters. It would be deplorable if he lays himself open to the
suspicion of reacting to feelings rather than to facts, of being imprisoned in the net of his own sympathies and
predilections. It must be obvious to the parties as well as the public that he follows the traditional mode of
adjudication requiring that he hear both sides with patience and understanding to keep the risk of reaching an
unjust decision at a minimum. It is not necessary that he should possess marked proficiency in law, but it is
essential that he is to hold the balance true. What is equally important is that he should avoid any conduct that
casts doubt on his impartially. What has been said is not merely a matter of judicial ethics. It is impressed with
constitutional significance.
It is now beyond dispute that due process cannot be satisfied in the absence of that degree of objectivity
on the part of a judge sufficient to reassure litigants of his being fair and being just. Thereby there is the
legitimate expectation that the decision arrived at would be the application of the law to the facts as found by a
judge who does not play favorites. For him, the parties stand on equal footing.
It has been said, in fact, that due process of law requires a hearing before an impartial and disinterested
tribunal, and that every litigant is entitled to nothing less than the cold neutrality of an impartial Judge.

46
The ponente, should strive to be at all times "wholly free, disinterested, impartial and independent.
Elementary due process requires a hearing before an impartial and disinterested tribunal. A judge has both the
duty of rendering a just decision and the duty of doing it in a manner completely free from suspicion as to its
fairness and as to his integrity."
Thereby, it is made clear to the occupants of the bench that outside of pecuniary interest, relationship
or previous participation in the matter that calls for adjudication, there may be other causes that could
conceivably erode the trait of objectivity, thus calling for inhibition. That is to betray a sense of realism, for the
factors that lead to preferences or predilections are many and varied. It is well, therefore, that if any such
should make its appearance and prove difficult to resist, the better course for a judge is to disqualify himself.
That way, he avoids being misunderstood. His reputation for probity and objectivity is preserved. What is even
more important, the ideal of an impartial administration of justice is lived up to. Thus is due process
vindicated.

Espleta v Avelino (1975) 62 SCRA 395


J. Fernando

Facts:
Shell’s counsel Bellaflor forwarded an oral motion for the revocation of appearance of Espleta’s witness
Montano for cross-examination and the conlusion of her testimony. She was unable to appear in one of the
trials due to her an audit for a job in the Department of Local Government at the day she was supposed to
finish her testimony and cross-examination. Judge Avelino accepted this proposal and even allowed Shell to
present its rebuttal witness for Espeleta’s testimony. The judge granted this request to the prejudice of
Espeleta. The magistrate also did not consider Espeleta’s counsel’s letter for postponement. The judge told
parties to submit documentary evidence afterwards but rejected the ones from Montano’s testimony due to her
being stricken from the records.
In essence, there was partiality on the part of the judge.

Issue:
Whether the concept of fairness that is basic to procedural due process would be satisfied if the right to
be heard of petitioner was revoked by the respondent Judge?

Held:
No. Petition for certiorari granted.
Espeleta presented Montano as an accountant to testify for the reduced balance to Shell in the form of
14,000 from Shell’s proposed amount of 22,000. The deductions included payment for damage due to gasoline
leakages.
Under the circumstances, the stress on the absence of procedural due process is understandable for as a
result of the order of respondent Judge now sought to be set aside, there is more than just a probability that
petitioner would be condemned to pay before he had been fully heard. The trial didn’t satisfy the standard for a
judicious inquiry, because there was a mockery of the requirement that the litigants should be given full
opportunity to sustain their claims and have their evidence considered and weighted. The petitioner can assert
due process.
By saying that the postponement of the counsel was a delay on the administration of justice was not in
line with Justice Carson’s saying that “a sound discretion in this regard should be exercised by the trial judge,
and the highly commendable desire for the dispatch of business should not be permitted to turn the scales of
justice rather than accede to a reasonable request for a continuance.”
Due heed must be paid to procedural due process mandate.
Ching Hong So- when a party litigant without malice or fault is not prepared for trial, the court can
exceed the discretion on it by law in denying to him the opportunity to prepare and obtain due process.
However, the controlling doctrine can be seen Capitol Subdivision v Negros where liberality must be
exercised in postponing trial to obtain material evidence and prevent miscarriage of justice.
The discretion for denying motions is allowed but such must be exercised with a view to substantial
justice.

47
In Luciano v Tan, procedural due process requires the infirmity consisting in a refusal to grant
postmonet then subsequently cured by the court reopening the case to comply with such a requirement.
The judge in this case must have reconsidered the denial.
Bellaflor’s pleadings displayed a total lack of awareness of due process in the implications of his petition
because of his assertion that the constitional question was essential factual.

G.R. No. L-30290 February 24, 1975


VICENTE SALANDANAN, Petitioner,
-versus-
HON. TITO V. TIZON, Judge of the Court of First Instance of Bataan, and ANTUSA M. MAGNO,
Respondents.

FACTS:
On Aril 25, 1967, a complaint for ejectment and recovery of possession filed by respondent Antusa M.
Magno against petitioner in the Court of First Instance of Bataan presided by respondent Judge, the alleged
cause of action being predicated on the sale and conveyance of what was denominated petitioner’s “real right,
interest and participation” over a parcel of land located at Culis, Hermosa, Bataan, and the sale of one male
carabao.
On June 22, 1967, petitioner set forth as a special and affirmative defense the allegation that he was as
far back as 1960 “an agricultural leasehold tenant over the landholding in question,” the rental being agreed
upon at fifty cavans but that previous to the filing of the action, he had been agitating for the reduction of the
annual rentals of the land without success, private respondent having warned him that if he persisted, she
would get back the land if defendant insisted. The action therefore was one of harassment and one moreover
beyond the jurisdiction of respondent Judge involving as it did a tenancy dispute.
It was the failure of respondent Judge to accede to the plea of petitioner that a tenancy dispute being involved,
the matter was beyond its jurisdiction, certainly an infirmity attended by fatal consequence, that led to the
institution of this certiorari proceeding.
As if that were not enough, respondent Judge, again heedless of the procedural right to due process, an even
more imperative requirement in litigation of this character, went ahead and decided the case in favor of private
respondent, Antusa M. Magno, without according a hearing to petitioner.

ISSUE:
Whether or not the respondent judge has jurisdiction on the case.
Whether or not “due process” was violated by respondent judge.

HELD:
We hold that when the factual question of the existence of a leasehold tenancy relation between the
parties is raised, in an ejectment case, which if true, would vest original and exclusive jurisdiction over the case
in the court of agrarian relations and not in the municipal court, it is essential that the court of first instance,
hold a preliminary hearing and receive the evidence solely on the facts that would show or disprove the
existence of the alleged leasehold tenancy. On the basis of such evidence, the court would then determine
whether or not it has jurisdiction, and summarize the facts in an order upholding its jurisdiction and that of the
municipal court or declaring the lack thereof.
Without holding a hearing then on the question of jurisdiction respondent Judge allowed the case to
proceed and, on the basis of the evidence submitted by private respondent, without petitioner being heard,
rendered the challenged decision. The due process failing is thus apparent.
If procedural deficiency were taken into account, it appears that respondent Judge had much to answer
for. Nor is it a matter of proceeding according to doctrinal requirements alone that vitiated his actuation. The
due process mandate was likewise aid scant respect, considering the circumstances of the case, more
specifically, petitioner being a pauper litigant.

G.R. No. L-49360 December 14, 1979


FILEMON DAVID, petitioner,
48
vs.
HON. GREGORIO U. AQUILIZAN, FELOMENO JUGAR and RICARDO JUGAR, respondents.

FACTS:
David has a large parcel of land in Polomolok, Cotabato. He let Felomeno Jugar and Ricardo Jugar tend
and caretake separate portions of his land in 1971. The land is estimated to be yielding 60-70 cavans of corn
cobs and the share agreed upon is 50-50. In 1973, David withdrew the land from the brothers and has not
allowed them to go back there. The brothers prayed for reinstatement but David refused to do so. David denied
that the brothers were his tenants. He said that Ricardo was his tractor driver before but he resigned to take
care of his dad and to work for DOLE. Fewlomeno on the other hand surrendered the portion of the land he
was tending to continue his faith healing. J Aquilizan handled the case filed by the brothers against David and
after three months he rendered a decision in favor of the brothers without any hearing. David averred he was
denied due process. J Aquilizan admitted that there was indeed no hearing conducted but he said the decision
has already become final and executory as the period for appeal has already lapsed.

ISSUE:
Whether or not David is entitled to an appeal.

HELD:
The SC ruled in favor of David. A decision rendered without a hearing is null and void and may be
attacked directly or collaterally. The decision is null and void for want of due process. And it has been held that
a final and executory judgment may be set aside with a view to the renewal of the litigation when the judgment
is void for lack of due process of law. In legal contemplation, it is as if no judgment has been rendered at all.

CASE DIGEST : US vs TURIBIO


G.R. No. L-5060 January 26, 1910 THE UNITED STATES, plaintiff-appellee, vs. LUIS TORIBIO, defendant-
appellant.

Facts: Respondent Toribio is an owner of carabao, residing in the town of Carmen in the province of Bohol. The
trial court of Bohol found that the respondent slaughtered or caused to be slaughtered a carabao without a
permit from the municipal treasurer of the municipality wherein it was slaughtered, in violation of Sections 30
and 33 of Act No. 1147, an Act regulating the registration, branding, and slaughter of Large Cattle. The act
prohibits the slaughter of large cattle fit for agricultural work or other draft purposes for human consumption.
The respondent counters by stating that what the Act is (1) prohibiting is the slaughter of large cattle in
the municipal slaughter house without a permit given by the municipal treasurer. Furthermore, he contends
that the municipality of Carmen has no slaughter house and that he slaughtered his carabao in his dwelling, (2)
the act constitutes a taking of property for public use in the exercise of the right of eminent domain without
providing for the compensation of owners, and it is an undue and unauthorized exercise of police power of the
state for it deprives them of the enjoyment of their private property.

Issue: Whether or not Act. No. 1147, regulating the registration, branding and slaughter of large cattle, is an
undue and unauthorized exercise of police power.

Held: It is a valid exercise of police power of the state.

Facts: The Supreme Court Said sections 30 and 33 of the Act prohibit and penalize the slaughtering or causing
to be slaughtered for human consumption of large cattle at any place without the permit provided for in section
30.
Where the language of a statute is fairly susceptible of two or more constructions, that construction
should be adopted which will most tend to give effect to the manifest intent of the lawmaker and promote the
object for which the statute was enacted, and a construction should be rejected which would tend to render
abortive other provisions of the statute and to defeat the object which the legislator sought to attain by its
enactment

49
The Supreme Court also said that if they will follow the contention of Toribio it will defeat the purpose
of the law.
The police power rests upon necessity and the right of self-protection and if ever the invasion of private
property by police regulation can be justified, The Supreme Court think that the reasonable restriction placed
upon the use of carabaos by the provision of the law under discussion must be held to be authorized as a
reasonable and proper exercise of that power.
The Supreme Court cited events that happen in the Philippines like an epidemic that wiped 70-100% of
the population of carabaos. The Supreme Court also said that these animals are vested with public interest for
they are fundamental use for the production of crops. These reasons satisfy the requisites of a valid exercise of
police power.
The Supreme Court finally said that article 1147 is not an exercise of the inherent power of eminent
domain. The said law does not constitute the taking of carabaos for public purpose; it just serve as a mere
regulation for the consumption of these private properties for the protection of general welfare and public
interest.

Churchill & Tait v. Rafferty 32 Phil. 580 (1915)


In re: Police power of the State, Lawful Subject of police power

This is an appeal from a judgment of the Court of First Instance of Manila. The case involves a dual
question one involving the power of the court to restrain by injunction the collection of the tax in question and
the other relating to the power of the Collector of Internal Revenue to remove any sign, signboard, or billboard
upon the ground that the same is offensive to the sight or is otherwise a nuisance.
The focus of this digest is to highlight the cases’ latter aspect as correlated to the police power of the
State.

Facts
Appellees, Francis A. Churchill and Stewart Tait are involved in the advertising business, particularly in
billboard advertising. Their billboards located upon private lands in the Province of Rizal were removed upon
complaints and by the orders of the defendant Collector of Internal Revenue by virtue of the provisions of
subsection (b) of section 100 of Act No. 2339.
Appellees, in their supplementary complaint challenge the power of the of the Collector of Internal
Revenue to remove any sign, signboard, or billboard upon the ground that the same is offensive to the sight or
is otherwise a nuisance and maintain that the billboards in question “in no sense constitute a nuisance and are
not deleterious to the health, morals, or general welfare of the community, or of any persons.” Defendant
Collector of Internal Revenue avers that after due investigation made upon the complaints of the British and
German Consuls, the defendant “decided that the billboard complained of was and still offensive to the sight
and is otherwise a nuisance.”

Issue
Was the enactment assailed by the plaintiffs was a legitimate exercise of the police power of the
Government?

Held
The High Court is of the opinion that unsightly advertisements or signs, signboards, or billboards which
are offensive to the sight, are not disassociated from the general welfare of the public. This is not establishing a
new principle, but carrying a well- recognized principle to further application. Moreover, if the police power
may be exercised to encourage a healthy social and economic condition in the country, and if the comfort and
convenience of the people are included within those subjects, everything which encroaches upon such territory
is amenable to the police power. Judgment reversed.

RUBI et.al vs The Provincial Board of Mindoro

50
FACTS: Rubi and various other Manguianes (Mangyans) in the province of Mindoro were ordered by the
provincial governor of Mindoro to remove their residence from their native habitat and to established
themselves on a reservation in Tigbao, still in the province of Mindoro, and to remain there, or be punished by
imprisonment if they escaped. Manguianes had been ordered to live in a reservation made to that end and for
purposes of cultivation under certain plans. The Manguianes are a Non-Christian tribe who were considered to
be of “very low culture”.
One of the Manguianes, a certain Dabalos, escaped from the reservation but was later caught and was
placed in prison at Calapan, solely because he escaped from the reservation. An application for habeas corpus
was made on behalf by Rubi and other Manguianes of the province, alleging that by virtue of the resolution of
the provincial board of Mindoro creating the reservation, they had been illegally deprived of their liberty. In
this case, the validity of Section 2145 of the Administrative Code, which provides: “With the prior approval of
the Department Head, the provincial governor of any province in which non-Christian inhabitants are found
is authorized, when such a course is deemed necessary in the interest of law and order, to direct such
inhabitants to take up their habitation on sites on unoccupied public lands to be selected by him and
approved by the provincial board,” was challenged.

ISSUE: Whether or not Section 2145 of the Administrative Code constitutes undue delegation. Whether or not
the Manguianes are being deprived of their liberty.

HELD:
I. No. By a vote of five to four, the Supreme Court sustained the constitutionality of this section of the
Administrative Code. Under the doctrine of necessity, who else was in a better position to determine whether
or not to execute the law but the provincial governor. It is optional for the provincial governor to execute the
law as circumstances may arise. It is necessary to give discretion to the provincial governor. The Legislature
may make decisions of executive departments of subordinate official thereof, to whom it has committed the
execution of certain acts, final on questions of fact.

II. No. Among other things, the term “non-Christian” should not be given a literal meaning or a religious
signification, but that it was intended to relate to degrees of civilization. The term “non-Christian” it was said,
refers not to religious belief, but in a way to geographical area, and more directly to natives of the Philippine
Islands of a low grade of civilization. In this case, the Manguianes were being reconcentrated in the reservation
to promote peace and to arrest their seminomadic lifestyle. This will ultimately settle them down where they
can adapt to the changing times.
The Supreme Court held that the resolution of the provincial board of Mindoro was neither
discriminatory nor class legislation, and stated among other things: “. . . one cannot hold that the liberty of the
citizen is unduly interfered with when the degree of civilization of the Manguianes is considered. They are
restrained for their own good and the general good of the Philippines. Nor can one say that due process of law
has not been followed. To go back to our definition of due process of law and equal protection of the laws, there
exists a law; the law seems to be reasonable; it is enforced according to the regular methods of procedure
prescribed; and it applies alike to all of a class.”

Rubi vs Provincial Board of Mindoro


G.R. No. 14078, March 07, 1919

Facts: The provincial board, by Resolution No. 25, selected a site in the sitio of Tigbao on Naujan Lake for the
permanent settlement of Mangyanes in Mindoro. Pursuant to the provisions of section 2145 of the revised
Administrative Code, all the Mangyans in the vicinities of the townships of Naujan and Pola and the Mangyans
east of the Baco River including those in the districts of Dulangan and Rubi’s place in Calapan are directed to
take up their habitation on the site of Tigbao, Naujan Lake.
This is an application for habeas corpus in favor of Rubi and other Manguianes of the Province of
Mindoro. It is alleged that the Mangyanes are being illegally deprived of their liberty by the provincial officials
of that province. Rubi and his companions are said to be held on the reservation established at Tigbao,
Mindoro, against their will, and one Dabalos is said to be held under the custody of the provincial sheriff in the
prison at Calapan for having run away from the reservation.
51
Issue: Whether Section 2145 of the Administrative Code deprives a person of his liberty of abode and is
therefore unconstitutional.

Held: No. Section 2145 of the Administrative Code does not deprive a person of his liberty without due process
of law and does not deny to him the equal protection of the laws, and that confinement in reservations in
accordance with said section does not constitute slavery and involuntary servitude. The court further ruled that
section 2145 of the Administrative Code is a legitimate exertion of the police power and thus constitutional.
Petitioners are not unlawfully imprisoned or restrained of their liberty. Habeas corpus can, therefore,
not issue.One cannot hold that the liberty of the citizen is unduly interfered with when the degree of civilization
of the. Manguianes is considered. They are restrained for their own good and the general good of the
Philippines. Nor can one say that due process of law has not been followed. To go back to our definition of due
process of law and equal protection of the laws, there exists a law; the law seems to be reasonable; it is enforced
according to the regular methods of procedure prescribed; and it applies alike to all of a class.The public policy
of the Government of the Philippine Islands is shaped with a view to benefit the Filipino people as a whole. The
Manguianes, in order to fulfill this governmental policy, must be confined for a time, as we have said, for their
own good and the good of the country.

PP vs Juan F. Fajardo et.al

Facts: On 15 August 1950, during the incumbency of Juan F. Fajardo as mayor of the municipality of Baao,
Camarines Sur, the municipal council passed Ordinance 7, series of 1950, providing that "any person or
persons who will construct or repair a building should, before constructing or repairing, obtain a written
permit from the Municipal Mayor," that "a fee of not less than P2.00 should be charged for each building
permit and P1.00 for each repair permit issued," and that any violation of the provisions of the ordinance shall
make the violator liable to pay a fine of not less than P25 nor more than P50 or imprisonment of not less than
12 days nor more than 24 days or both, at the discretion of the court; and that if said building destroys the view
of the Public Plaza or occupies any public property, it shall be removed at the expense of the owner of the
building or house. 4 years later, after the term of Fajardo as mayor had expired, he and his son-in-law, Pedro
Babilonia, filed a written request with the incumbent municipal mayor for a permit to construct a building
adjacent to their gasoline station on a parcel of land registered in Fajardo's name, located along the national
highway and separated from the public plaza by a creek. On 16 January 1954, the request was denied, for the
reason among others that the proposed building would destroy the view or beauty of the public plaza. On 18
January 1954, Fajardo and Babilonia reiterated their request for a building permit, but again the request was
turned down by the mayor. Whereupon, Fajardo and Babilonia proceeded with the construction of the building
without a permit, because they needed a place of residence very badly, their former house having been
destroyed by a typhoon and hitherto they had been living on leased property. On 26 February 1954, Fajardo
and Babilonia were charged before and convicted by the justice of the peace court of Baao, Camarines Sur, for
violation of Ordinance 7. Fajardo and Babilonia appealed to the Court of First Instance (CDI), which affirmed
the conviction, and sentenced both to pay a fine of P35 each and the costs, as well as to demolish the building in
question because it destroys the view of the public plaza of Baao. From this decision, Fajardo and Babilonia
appealed to the Court of Appeals, but the latter forwarded the records to the Supreme Court because the appeal
attacks the constitutionality of the ordinance in question.

Issue: Whether the refusal of the Mayor of Baao to issue a building permit on the ground that the proposed
Constitutional Law II, 2005 ( 7 ) Narratives (Berne Guerrero) building would destroy the view of the public
plaza is an undue deprivation of the use of the property in question, and thus a taking without due
compensation.

Held: The refusal of the Mayor of Baao to issue a building permit to Fajardo and Babilonia was predicated on
the ground that the proposed building would "destroy the view of the public plaza" by preventing its being seen
from the public highway. Even thus interpreted, the ordinance is unreasonable and oppressive, in that it
operates — to permanently deprive the latter of the right to use their own property; hence, it oversteps the
bounds of police power, and amounts to a taking of the property without just compensation. But while property
52
may be regulated in the interest of the general welfare such as to regard the beautification of neighbourhood as
conducive to the comfort and happiness of residents), and in its pursuit, the State may prohibit structures
offensive to the sight, the State may not, under the guise of police power, permanently divest owners of the
beneficial use of their property and practically confiscate them solely to preserve or assure the aesthetic
appearance of the community. As the case now stands, every structure that may be erected on Fajardo's land,
regardless of its own beauty, stands condemned under the ordinance in question, because it would interfere
with the view of the public plaza from the highway. Fajardo would, in effect, be constrained to let their land
remain idle and unused for the obvious purpose for which it is best suited, being urban in character.

Santiago vs Alalayan et.al.

FACTS: In 1961, Republic Act No. 3043 (An Act to Further Amend Commonwealth Act Numbered One
Hundred Twenty, as Amended by Republic Act Numbered Twenty Six Hundred and Forty One) was passed.
This law amended the charter of NAPOCOR (National Power Corporation). Section 3 of RA 3043 provides
that:
a. contractors being supplied by NAPOCOR shall not exceed an annual profit of 12%;
b. if they do, they shall refund such excess to their customers;
c. that NAPOCOR has the power to renew all existing contracts with franchise holders for the supply of
energy.

Santiago Alalayan and the Philippine Power and Development Company (PPDC) assailed the said
provision.They averred that Section 3 is a rider because first, it was not included in the title of the amending
law nor was it included in the amended law. Second, the main purpose of RA 3043 was to increase the capital
stock of NAPOCOR hence Alalayan et al believed that Section 3 was not germane to RA 3043.

ISSUE: Whether or not Section 3 of RA 3043 is constitutional.

HELD: Yes. The Supreme Court simply ruled that the Constitution does not require Congress to employ in the
title of an enactment, language of such precision as to mirror, fully index or catalogue all the contents and the
minute details therein. It suffices if the title should serve the purpose of the constitutional demand that it
inform the legislators, the persons interested in the subject of the bill, and the public, of the nature, scope and
consequences of the proposed law and its operation. And this, to lead them to inquire into the body of the bill,
study and discuss the same, take appropriate action thereon, and, thus, prevent surprise or fraud upon the
legislators.

Jesus P. Morfe vs Amelito R. Mutuc 22 SCRA 424, January 31, 1968


Facts: Section 7 of Republic Act No. 3019 (R.A. 3019), provides that “every public officer, ...after his
assumption to office and within the month of January of every other year thereafter, as well as upon the
termination of his position, shall prepare and file with the head of the office to which he belongs, a true detailed
and sworn statement of assets and liabilities, including a statement of the amounts and sources of his income,
the amounts of his personal and family expenses and the amount of income taxes paid for the next preceding
calendar year...”Jesus Morfe, disputing that such requirement is violative of due process as an oppressive
exercise of police power and as an unlawful invasion of the constitutional right to privacy, implicit in the ban
against unreasonable search and seizure construed together with the prohibition against self-incrimination,
filed a petition for declaratory relief before the Court of First Instance (CFI) of Pangasinan. After the
submission of pleadings and stipulation of facts, the CFI found for Morfe, affirming that the requirement of
periodical submission of such sworn statement of assets and liabilities exceeds the permissible limit of the
police power and is thus offensive to the due process clause – hence, Section 7 of R.A. 3019 is unconstitutional.
Aggrieved, Executive Secretary Amelito Mutuc appealed the decision of the CFI before the Supreme Court.

Issue: Whether or not, the requirement of periodical submission of the sworn statement of assets and
liabilities, pursuant to R.A. 3019, exceeds the permissible limit of the State’s police power and is thus offensive
to the due process clause?
53
Ruling: No. Nothing can be clearer than that R.A. 3019 was precisely aimed at curtailing and minimizing the
opportunities for official corruption and maintaining a standard of honesty in the public service. It is intended
to further promote morality in public administration. A public office must indeed be a public trust. Nobody can
cavil at its objective; the goal to be pursued commands the assent of all. The conditions then prevailing called
for norms of such character. The times demanded such a remedial device. In the absence of a factual
foundation, the presumption of a statute’s validity must prevail over mere pleadings and stipulation of facts
(Ermita-Malate Hotel, et. al. v. Mayor of Manila). While in the attainment of attainment of such public good,
no infringement of constitutional rights is permissible, there must be a showing, clear, categorical, and
undeniable that what the Constitution condemns, the statute allows. While the soundness of the assertion that
a public office is a public trust and as such not amounting to property in its usual sense cannot be denied, there
can be no disputing the proposition that from the standpoint of the security of tenure guaranteed by the
Constitution the mantle of protection afforded by due process could rightfully be invoked.

US vs Salaveria

FACTS:
The municipal council of Orion, Bataan, enacted, on February 28, 1917, an ordinance which, among
other things, prohibited the playing of panguingue on days not Sundays or legal holidays, and penalized the
violation thereof by a casero [housekeeper] by a fine of not less than P10 nor more than P200, and by jugadores
[gamblers] by a fine of not less than P5 nor more than P200. The justice of the peace of Orion, when this
ordinance went into effect, was Prudencio Salaveria, now the defendant and appellant. Notwithstanding his
official station, on the evening of March 8,1917, not a Sunday or legal holiday, seven persons including the
justice of the peace and his wife were surprised by the police while indulging in a game of panguingue in the
house of the justice of the peace. The chief of police took possession of the cards, the counters (sigayes), a tray,
and P2.07 in money, used in the game.

ISSUE:
Whether or not Ordinance No. 3 of Orion, Bataan, is found to be valid.

HELD:
Wherefore, although panguingue is not entirely a game of chance, since it is a proper subject for
regulation by municipal authorities acting under their delegated police power, whose laudable intention is to
improve the public morals and promote the prosperity of their people, their action should be upheld by the
courts. Ordinance No. 3 of Orion, Bataan, is found to be valid.

Ordinance No. 3
Third. — The games known as "Panguingue" "Manilla," "Jung-kiang," "Paris-Paris," "Poker," "Tute,"
"Burro," and "Treinta-y-uno" shall be allowed only on Sundays an official holidays.
The following penalties shall be imposed upon those who play the above games on days other than Sundays
and official holidays:
a. For the owner of the house: A fine of from Ten to Two hundred pesos, or subsidiary imprisonment in
case of insolvency at the rate of one peso a day.
b. For the gamblers: A fine of from Five to Two hundred pesos each or subsidiary imprisonment in case of
insolvency at the rate of one peso a day.

General Welfare Clause (sec. 2238, Adm. Code of 1917):


The municipal council shall enact such ordinances and make such regulations, not repugnant to law, as
may be necessary to carry into effect and discharge the powers an duties conferred upon it by law an such as
shall seem necessary and proper to provide for the health and safety, promote the prosperity, improve the
morals, peace, good order, comfort, and convenience of the municipality and the inhabitants thereof, and for
the protection of property therein.

54
BARTOLOME CAUNCA v. JULIA SALAZAR

FACTS:
Estelita Flores, 21, orphan of father and mother, illiterate, was brought to Manila to work as a maid and
stayed in the house of Julia Salazar. when her cousin Bartolome Caunca went to pay her a visit, Estelita
manifested her earnest desire to go along with him, but was prevented by Julia Salazar and Estrella Justo, both
demanding the condition that the sum of P83.85 advanced for the fare and other transportation expenses of
Estelita from Buruanga to Manila be paid first before she could leave the house of Julia Salazar although there
is no evidence that any physical force has been used to prevent her from leaving the house, Estelita failed to
leave it.

ISSUE:
Whether or Not an employment agency has the right to restrain and detain a maid without returning
the advance payment it gave?

RULING:
An employment agency, regardless of the amount it may advance to a prospective employee or maid,
has absolutely no power to curtail her freedom of movement. The fact that no physical force has been exerted to
keep her in the house of the respondent does not make less real the deprivation of her personal freedom of
movement, freedom to transfer from one place to another, freedom to choose one’s residence. Freedom may be
lost due to external moral compulsion, to found or groundless fear, to erroneous belief in the existence of an
imaginary power of an impostor to cause harm if not blindly obeyed, to any other psychological element that
may curtail the mental faculty of choice or the unhampered exercise of the will. If the actual effect of such
psychological spell is to place a person at the mercy of another, the victim is entitled to the protection of courts
of justice as much as the individual who is illegally deprived of liberty by duress or physical coercion.

AGUSTIN VS EDU

FACTS:
Agustin is the owner of a Volkswagen Beetle Car. He is assailing the validity of Letter of Instruction No
229 which requires all motor vehicles to have early warning devices particularly to equip them with a pair of
“reflectorized triangular early warning devices”. Agustin is arguing that this order is unconstitutional, harsh,
cruel and unconscionable to the motoring public. Cars are already equipped with blinking lights which is
already enough to provide warning to other motorists. And that the mandate to compel motorists to buy a set
of reflectorized early warning devices is redundant and would only make manufacturers and dealers instant
millionaires.

ISSUE:
Whether or not the said is EO is valid.

RULING:
YES, the court held that the letter of Instruction No.229 as amended as well as the implementing rules
and regulations were valid and constitutional as a valid measure of police power. The Vienna Convention on
Road signs and signals and the United Nations Organization was ratified by the Philippine local legislation for
the installation of road safety signs and devices. It cannot be disputed then that this Declaration of Principle
found in the Constitution possesses relevance, between the International law and municipal law in applying the
rule municipal law prevails. In the exercise of police power, the EO is a valid exercise of such power. Police
power, according to the case of Edu v Ericta, which cited J. Taney, is nothing more or less than the power of
government inherent in every sovereignty. The case also says that police power is state authority to enact
legislation that may interfere with personal liberty or property to promote the general welfare.

TANADA VS TUVERA

55
FACTS:
Lorenzo M. Tanada, et. al. invoked due process in demanding the disclosure of a number of Presidential
Decrees which they claimed had not been published as required by Law. The government argued that while
publication was necessary as a rule, it was not so when it was otherwise provided, as when the decrees
themselves declared that they were to become effective immediately upon approval. The court decided on April
24, 1985 in affirming the necessity for publication of some of the decrees. The court ordered the respondents to
publish in the official gazette all unpublished Presidential Issuances which are of general force and effect. The
petitioners suggest that there should be no distinction between laws of general applicability and those which
are not. The publication means complete publication, and that publication must be made in the official gazette.
In a comment required by the solicitor general, he claimed first that the motion was a request for an advisory
opinion and therefore be dismissed. And on the clause “unless otherwise provided” in Article 2 of the new civil
code meant that the publication required therein was not always imperative, that the publication when
necessary, did not have to be made in the official gazette.

Issues:
Whether or not all laws shall be published in the official gazette.

Held:
Article 2 of the Civil Code provides that “laws shall take effect after fifteen days following the
completion of their publication in the Official Gazette, unless it is otherwise provided ” The Court has ruled
that publication in the Official Gazette is necessary in those cases where the legislation itself does not provide
for its effectivity date-for then the date of publication is material for determining its date of effectivity, which is
the fifteenth day following its publication-but not when the law itself provides for the date when it goes into
effect. Article 2 does not preclude the requirement of publication in the Official Gazette, even if the law itself
provides for the date of its effectivity.
The publication of all presidential issuances “of a public nature” or “of general applicability” is
mandated by law. Obviously, presidential decrees that provide for fines, forfeitures or penalties for their
violation or otherwise impose a burden or. the people, such as tax and revenue measures, fall within this
category. Other presidential issuances which apply only to particular persons or class of persons such as
administrative and executive orders need not be published on the assumption that they have been circularized
to all concerned.
Publication is, therefore, mandatory.

ANG TIBAY VS CIR

FACTS:
Teodoro Toribio owns and operates Ang Tibay, a leather company which supplies the Philippine Army.
Due to alleged shortage of leather, Toribio caused the lay off of a number of his employees. However, the
National Labor Union, Inc. (NLU) questioned the validity of said lay off as it averred that the said employees
laid off were members of NLU while no members of the rival labor union National Workers Brotherhood
(NWB) were laid off. NLU claims that NWB is a company dominated union and Toribio was merely busting
NLU.
The case reached the Court of Industrial Relations (CIR) where Toribio and NWB won. Eventually, NLU
went to the Supreme Court invoking its right for a new trial on the ground of newly discovered evidence. The
Supreme Court agreed with NLU. The Solicitor General, arguing for the CIR, filed a motion for reconsideration.

ISSUE:
Whether or not the National Labor Union, Inc. is entitled to a new trial.

HELD:
Yes. The records show that the newly discovered evidence or documents obtained by NLU, which they
attached to their petition with the SC, were evidence so inaccessible to them at the time of the trial that even
with the exercise of due diligence they could not be expected to have obtained them and offered as evidence in
the Court of Industrial Relations. Further, the attached documents and exhibits are of such far-reaching
56
importance and effect that their admission would necessarily mean the modification and reversal of the
judgment rendered (said newly obtained records include books of business/inventory accounts by Ang Tibay
which were not previously accessible but already existing).

GUZMAN VS NU

FACTS:
Diosdado Guzman and two others complained that the National University (NU) barred them from
enrolling in the said university. NU argued that their failure to enroll was due to the students’ fault. It was
alleged that Guzman et al spearheaded illegal mass actions within the university premises; that such mass
actions were violative of school policies; that due to their mass actions, Guzman et al incurred bad grades; that
Guzman et al hated NU anyway so why should they be allowed to enroll; that it is in the best interest of both
parties for the students not to be enrolled.

ISSUE:
Whether or not NU deprived Guzman et al of due process.

RULING:
Guzman et al were deprived of due process. In the first place, NU never showed which school policies or
duly published rules did Guzman et al violate upon which they may be expelled from. NU failed to show that it
conducted any sort of proceedings (not necessarily a trial type one) to determine Guzman et al’s liability or
alleged participation in the said mass actions.
Under the Education Act of 1982, Guzman et al, as students, have the right among others “to freely
choose their field of study subject to existing curricula and to continue their course therein up to graduation,
except in case of academic deficiency, or violation of disciplinary regulations.” Guzman et al were being denied
this right, or being disciplined, without due process, in violation of the Manual of Regulations for Private
Schools which provides that “no penalty shall be imposed upon any student except for cause as defined in the
Manual and/or in the school rules and regulations as duly promulgated and only after due investigation shall
have been conducted.”

57
3. Equal Protection
BIRAOGO VS PTC

FACTS:
Pres. Aquino signed E. O. No. 1 establishing Philippine Truth Commission of 2010 (PTC) dated July 30,
2010.
PTC is a mere ad hoc body formed under the Office of the President with the primary task to investigate
reports of graft and corruption committed by third-level public officers and employees, their co-principals,
accomplices and accessories during the previous administration, and to submit its finding and
recommendations to the President, Congress and the Ombudsman. PTC has all the powers of an investigative
body. But it is not a quasi-judicial body as it cannot adjudicate, arbitrate, resolve, settle, or render awards in
disputes between contending parties. All it can do is gather, collect and assess evidence of graft and corruption
and make recommendations. It may have subpoena powers but it has no power to cite people in contempt,
much less order their arrest. Although it is a fact-finding body, it cannot determine from such facts if probable
cause exists as to warrant the filing of information in our courts of law. Petitioners asked the Court to declare it
unconstitutional and to enjoin the PTC from performing its functions.

ISSUE:
Whether or not EO No. 1 is unconstitutional?

RULING:
E.O No. 1 should be struck down as it is violative of the equal protection clause. The Chief Executive’s
power to create the Ad hoc Investigating Committee cannot be doubted. Having been constitutionally granted
full control of the Executive Department, to which respondents belong, the President has the obligation to
ensure that all executive officials and employees faithfully comply with the law. With AO 298 as mandate, the
legality of the investigation is sustained. Such validity is not affected by the fact that the investigating team and
the PCAGC had the same composition, or that the former used the offices and facilities of the latter in
conducting the inquiry.

KABATAAN PARTYLIST VS COMELEC

FACTS:
RA 10367 mandates the COMELEC to implement a mandatory biometrics registration system for new
voters in order to establish a clean, complete, permanent, and updated list of voters through the adoption of
biometric technology. COMELEC issued Resolution No. 9721 as amended by Resolutions No. 9863 and 10013.
Among others, the said Resolution provides that: “the registration records of voters without biometrics data
who failed to submit for validation on or before the last day of filing of applications for registration for the
purpose of the May 9, 2016 National and Local Elections shall be deactivated.
Herein petitioners filed the instant petition with application for temporary restraining order (TRO)
and/or writ of preliminary mandatory injunction (WPI) assailing the constitutionality of the biometrics
validation requirement imposed under RA 10367, as well as COMELEC Resolution Nos. 9721, 9863, and 10013,
all related thereto.

ISSUES:
Whether or not the statutory requirement of biometrics validation is an unconstitutional requirement of
literacy and property

RULING:
NO. The Court held that biometrics validation is not a “qualification” to the exercise of the right of
suffrage, but a mere aspect of the registration procedure, of which the State has the right to reasonably
regulate.

58
The Court reiterated their ruling in several cases that registration regulates the exercise of the right of
suffrage. It is not a qualification for such right. The process of registration is a procedural limitation on the
right to vote.
Thus, although one is deemed to be a “qualified elector,” he must nonetheless still comply with the
registration procedure in order to vote.
Thus, unless it is shown that a registration requirement rises to the level of a literacy, property or other
substantive requirement as contemplated by the Framers of the Constitution -that is, one which propagates a
socio-economic standard which is bereft of any rational basis to a person’s ability to intelligently cast his vote
and to further the public good -the same cannot be struck down as unconstitutional, as in this case.

GARCIA VS DRILON

FACTS:
Jesus Garcia (husband) admitted having an affair with a bank manager. His infidelity emotionally
wounded private respondent which spawned several quarrels that left respondent wounded. Petitioner also
unconscionably beat up their daughter, Jo-ann.
Rosalie Jaype-Garcia filed, for herself and in behalf of her minor children for a Temporary Protection
Order against her husband, Jesus C. Garcia pursuant to R.A. 9262. She claimed to be a victim of physical
abuse; emotional, psychological, and economic violence as a result of marital infidelity on the part of
petitioner, with threats of deprivation of custody of her children and of financial support. The husband now,
assails the constitutionality of RA 9262 as being violative of the equal protection clause.

ISSUE:
Whether there is a violation of equal protection clause.

HELD:
R.A. 9262 does not violate the guaranty of equal protection of the laws.
Equal protection simply requires that all persons or things similarly situated should be treated alike,
both as to rights conferred and responsibilities imposed. R.A. 9262 is based on a valid classification as shall
hereinafter be discussed and, as such, did not violate the equal protection clause by favoring women over men
as victims of violence and abuse to whom the State extends its protection.
There is likewise no merit to the contention that R.A. 9262 singles out the husband or father as the
culprit. As defined above, VAWC may likewise be committed “against a woman with whom the person has or
had a sexual or dating relationship.” Clearly, the use of the gender-neutral word “person” who has or had a
sexual or dating relationship with the woman encompasses even lesbian relationships.
R.A. 9262 is based on a valid classification as such, did not violate the equal protection clause by
favoring women over men as victims of violence and abuse to whom the State extends its protection. The
unequal power relationship between women and men; the fact that women are more likely than men to be
victims of violence; and the widespread gender bias and prejudice against women all make for real differences
justifying the classification under the law. As Justice McIntyre succinctly states, “the accommodation of
differences … is the essence of true equality.”

GSIS v. MONTESCLAROS

FACTS:
Milagros assail unconstitutionality of section 18 PD 1146 being violative of due process and equal
protection clause. When her husband died, she filed in GSIS for claim for survivorship pension. GSIS denied
claim, it said surviving spouse has no right of survivorship pension if the surviving spouse contracted the
marriage with the pensioner within three years before the pensioner qualified for the pension.

ISSUE:
Whether or not Section 18 PD 1146 is constitutional.

59
HELD:
There is denial of due process when it out-rightly denies the claim for survivorship. There is outright
confiscation of benefits due the surviving spouse without giving her an opportunity to be heard. There is also
violation of equal protection. A proviso requiring certain number of years of togetherness in marriage before
the employee’s death is valid to prevent sham marriages contracted for monetary gains. Here, it is 3 years
before pensioner qualified for the pension. Under this, even if the dependent spouse married the pensioner
more than 3 years before the pensioner’s death, the dependent spouse would still not receive survivorship
pension if the marriage took place within 3 years before the pensioner qualified for pension. The object of
prohibition is vague. There is no reasonable connection between the means employed and the purpose
intended.

TABLARIN vs. GUTIERREZ

FACTS:
The petitioners sought to enjoin the Secretary of Education, Culture and Sports, the Board of Medical
Education and the Center for Educational Measurement from enforcing a requirement the taking and passing
of the NMAT as a condition for securing certificates of eligibility for admission, from proceeding with accepting
applications for taking the NMAT and from administering the NMAT as scheduled on 26 April 1987 and in the
future. The trial court denied said petition and the NMAT was conducted and administered as scheduled.
The NMAT, an aptitude test, is considered as an instrument toward upgrading the selection of
applicants for admission into the medical schools and its calculated to improve the quality of medical education
in the country. The cutoff score for the successful applicants, based on the scores on the NMAT, shall be
determined every year by the Board of Medical Education after consultation with the Association of Philippine
Medical Colleges. The NMAT rating of each applicant, together with the other admission requirements as
presently called for under existing rules, shall serve as a basis for the issuance of the prescribed certificate of
eligibility for admission into the medical colleges.

ISSUE:
Whether or not Section 5 (a) and (f) of Republic Act No. 2382, as amended, and MECS Order No. 52, s.
1985 are constitutional.

HELD:
Yes. We conclude that prescribing the NMAT and requiring certain minimum scores therein as a
condition for admission to medical schools in the Philippines, do not constitute an unconstitutional imposition.
The police power, it is commonplace learning, is the pervasive and non-waivable power and authority of the
sovereign to secure and promote all the important interests and needs — in a word, the public order — of the
general community. An important component of that public order is the health and physical safety and well-
being of the population, the securing of which no one can deny is a legitimate objective of governmental effort
and regulation. Perhaps the only issue that needs some consideration is whether there is some reasonable
relation between the prescribing of passing the NMAT as a condition for admission to medical school on the
one hand, and the securing of the health and safety of the general community, on the other hand. This question
is perhaps most usefully approached by recalling that the regulation of the practice of medicine in all its
branches has long been recognized as a reasonable method of protecting the health and safety of the public.

ANTONIO SERRANO VS ALLANT MARITIME SERVICES, INC.

FACTS:
Petitioner Antonio Serrano was hired by respondents Gallant Maritime Services, Inc. and Marlow
Navigation Co., Inc., under a POEA-approved contract of employment for 12 months, as Chief Officer, with the
basic monthly salary of US$1,400, plus $700/month overtime pay, and 7 days paid vacation leave per month.
On the date of his departure, Serrano was constrained to accept a downgraded employment contract
upon the assurance and representation of respondents that he would be Chief Officer by the end of April
1998.Respondents did not deliver on their promise to make Serrano Chief Officer.
60
Hence, Serrano refused to stay on as second Officer and was repatriated to the Philippines, serving only
two months and 7 days, leaving an unexpired portion of nine months and twenty-three days.
Upon complaint filed by Serrano before the Labor Arbiter (LA), the dismissal was declared illegal. On
appeal, the NLRC modified the LA decision based on the provision of RA 8042.
Serrano filed a Motion for Partial Reconsideration, but this time he questioned the constitutionality of
the last clause in the 5th paragraph of Section 10 of RA 8042.

ISSUES:
1. Whether or not the subject clause violates Section 10, Article III of the Constitution on non-impairment
of contracts;
2. Whether or not the subject clause violates Section 1, Article III of the Constitution, and Section 18,
Article II and Section 3, Article XIII on labor as a protected sector.

HELD:
On the first issue.
The answer is in the negative. Petitioner’s claim that the subject clause unduly interferes with the
stipulations in his contract on the term of his employment and the fixed salary package he will receive is not
tenable.
The subject clause may not be declared unconstitutional on the ground that it impinges on the
impairment clause, for the law was enacted in the exercise of the police power of the State to regulate a
business, profession or calling, particularly the recruitment and deployment of OFWs, with the noble end in
view of ensuring respect for the dignity and well-being of OFWs wherever they may be employed.

On the second issue.


The answer is in the affirmative. To Filipino workers, the rights guaranteed under the foregoing
constitutional provisions translate to economic security and parity.
Upon cursory reading, the subject clause appears facially neutral, for it applies to all OFWs. However, a
closer examination reveals that the subject clause has a discriminatory intent against, and an invidious impact
on, OFWs at two levels:
 First, OFWs with employment contracts of less than one year vis-à-vis OFWs with employment
contracts of one year or more;
 Second, among OFWs with employment contracts of more than one year; and
 Third, OFWs vis-à-vis local workers with fixed-period employment;

The subject clause singles out one classification of OFWs and burdens it with a peculiar disadvantage.
Thus, the subject clause in the 5th paragraph of Section 10 of R.A. No. 8042 is violative of the right of
petitioner and other OFWs to equal protection. The subject clause “or for three months for every year of the
unexpired term, whichever is less” in the 5th paragraph of Section 10 of Republic Act No. 8042 is DECLARED
UNCONSTITUTIONAL.

CENTRAL BANK EMPLOYEES ASSOCIATION V. BSP

Doctrines:
(Equal Protection Clause)
Elements of valid class legislation:
1) must rest on substantial distinctions;
2) must be germane to the purposes of the law;
3) must not be limited to existing conditions only;
4) must apply equally to all members of the same class

Relative Constitutionality. The fact that a statute is constitutional at first does not mean it is
constitutional forever. The subsequent changes in the original circumstance surrounding the law would affect
its validity.

61
FACTS:
The new Central Bank Act took effect and gave way for the creation of Bangko Sentral ng
Pilipinas.Other Governmental Financial Institutions (GFIs) also amended their charters.
After almost 8 years following the amendment of the GFIs’ charters, BSP’s employees, through
petitioner, filed a petition for prohibition against the BSP and the Executive Secretary to restrain the
respondents from further implementing the last proviso in Sec. 15, Art. II of the New Central Bank Act (i.e., the
exemption from the Salary Standardization Law (SSL) of all employees with salary grade of 19 and the non-
exemption of those having a salary grade under 19). They alleged its constitutionality for being an invalid “class
legislation”.

Petitioner’s Contentions:
1) The said proviso violates equal protection clause because only the officers of the BSP (those holding the
salary grade of 19 and up) are exempted from the SSL.
2) Those belonging from 19 and up and those 19 below do not really differ from one other in terms of the
nature of work and expertise.
3) Other GFIs, which are the same as the BSP, exempt all their rank-and-file personnel from SSL without
any distinction.

BSP’s contention:
The proviso is not unconstitutional as it can stand the constitutional test, provided it is construed in
harmony with other provisions of the same law, such as the mandate of the Monetary Board to “establish
professionalism and excellence at all levels in accordance with sound principles of management.”

Solicitor General, on behalf of respondent Executive Secretary:


The proviso is not unconstitutional as the classification is based on actual and real differentiation, even
as it adheres to the enunciated policy of the new SB Act to establish professionalism and excellence within the
BSP subject to prevailing laws and policies of the national government.

ISSUES:
WON the proviso is unconstitutional for being violative of equal protection clause.

HELD:
Yes, the proviso is unconstitutional for being violative of the equal protection clause.
Equal protection clause does not prevent the Legislature from establishing classes of individuals or
objects upon which different rules shall operate – so long as the classification is not unreasonable. Equality of
operation of statutes does not mean indiscriminate operation on persons themselves, but on persons according
to the circumstances surrounding them. It guarantees equality, not identity of rights.
In the case at bar, it is clear in the legislative deliberations that the exemption of officers (SG 20 and
above) from the SSL was intended to address the BSP’s lack of competitiveness in terms of attracting
competent officers and executives. It was not intended to discriminate against the rank-and-file and the
resulting discrimination or distinction has a rational basis and is not palpably, purely, and entirely arbitrary in
the legislative sense. However, in the subsequent passages of the amendment on the charters of other GFI, the
surrounding circumstances of the case changed.
The subsequent amendments of the other GFIs’ charter (i.e., express authorization to determine and
institute its own compensation and wage structure, and explicit exemption – without distinction as to salary
grade or position – all employees of the GFI from the SSL) resulted to the oppressive results of Congress’
inconsistent and unequal policy towards the BSP rank-and-file and those of the seven other GFI. In the case at
bar, it is precisely the fact that as regards the exemption from the SSL, there are no characteristics peculiar only
to the seven GFIs or their rank-and-file so as to justify the exemption which BSP rank-and-file employees were
denied (not to mention the anomaly of the SEC getting one). The distinction made by the law is not only
superficial, but also arbitrary. It is not based on substantial distinctions that make real differences between the
BSP rank-and-file and the seven other GFIs.
The subsequent grant to the rank-and-file of the seven other GFIs and continued denial to the BSP
rank-and-file employees of the exemption from SSL breached the latter’s right to equal protection.

62
The equal protection clause does not demand absolute equality but it requires that all persons shall be
treated alike, under like circumstances and conditions both as to privileges conferred and liabilities enforced.

PEOPLE VS CAYAT

FACTS:
Accused Cayat, a native of Baguio, Benguet, Mountain Province, and a member of the non-Christian
tribes, was found guilty of violating sections 2 and 3 of Act No. 1639 for having acquired and possessed one
bottle of A-1-1 gin, an intoxicating liquor, which is not a native wine. The law made it unlawful for any native of
the Philippines who is a member of a non-Christian tribe within the meaning of Act 1397 to buy, receive, have
in his possession, or drink any ardent spirits, ale, beer, wine or intoxicating liquors of any kind, other than the
so-called native wines and liquors which the members of such tribes have been accustomed to prior to the
passage of the law. Cayat challenges the constitutionality of Act 1639 on the grounds that it is discriminatory
and denies the equal protection of the laws, violates due process clause, and is an improper exercise of police
power.

ISSUE:
Whether or not there is discriminatory and denial of equal protection of the laws

HELD:
It is an established principle of constitutional law that the guaranty of the equal protection of the laws is
not violated by a legislation based on reasonable classification. (1) must rest on substantial distinctions; (2)
must be germane to the purposes of the law; (3) must not be limited to existing conditions only; and (4) must
apply equally to all members of the same class.
Act No. 1639 satisfies these requirements. The classification rests on real or substantial, not merely
imaginary or whimsical distinctions. It is not based upon “accident of birth or parentage,” as counsel for the
appellant asserts, but upon the degree of civilization and culture. “The term ‘non-Christian tribes’ refers, not to
religious belief but in a way, to the geographical area and more directly, to natives of the Philippine Islands of a
low grade of civilization, usually living in tribal relationship apart from settled communities.” (Rubi vs.
Provincial Board of Mindora, supra.) This distinction is unquestionably reasonable, for the Act was intended to
meet the peculiar conditions existing in the non-Christian tribes
The prohibition enshrined in Act 1397 is designed to insure peace and order in and among non-
Christian tribes. It applies equally to all members of the class evident from perusal thereof. That it may be
unfair in its operation against a certain number of non-Christians by reason of their degree of culture, is not an
argument against the equality of its application.

ICHONG VS HERNANDEZ

Conflict with fundamental law; Police Power


LAO H. ICHONG, in his own behalf and in behalf of other alien residents, corporations and
partnerships adversely affected. by Republic Act No. 1180, petitioner, JAIME HERNANDEZ, Secretary of
Finance, and MARCELINO SARMIENTO, City Treasurer of Manila, respondents.

FACTS:
Driven by aspirations for economic independence and national security, the Congress enacted Act No.
1180 entitled “An Act to Regulate the Retail Business.” The main provisions of the Act, among others, are:
1) Prohibition against persons, not citizens of the Philippines, and against associations, among others,
from engaging directly or indirectly in the retail trade; and
2) Prohibition against the establishment or opening by aliens actually engaged in the retail business of
additional stores or branches of retail business.

Lao H. Ichong, in his own behalf and on behalf of other alien residents, corporations and partnerships
adversely affected by the said Act, brought an action to obtain a judicial declaration, and to enjoin the Secretary
63
of Finance, Jaime Hernandez, and all other persons acting under him, particularly city and municipal
treasurers, from enforcing its provisions. Petitioner attacked the constitutionality of the Act, contending that:
It denies to alien residents the equal protection of the laws and deprives of their liberty and
property without due process of law.
1) The subject of the Act is not expressed or comprehended in the title thereof.
2) The Act violates international and treaty obligations of the Republic of the Philippines.

ISSUES:
Whether or not a law may invalidate or supersede treaties or generally accepted principles.

Discussions:
A generally accepted principle of international law, should be observed by us in good faith. If a treaty
would be in conflict with a statute then the statute must be upheld because it represented an exercise of the
police power which, being inherent could not be bargained away or surrendered through the medium of a
treaty.

HELD:
Yes, a law may supersede a treaty or a generally accepted principle. In this case, the Supreme Court saw
no conflict between the raised generally accepted principle and with RA 1180. The equal protection of the law
clause “does not demand absolute equality amongst residents; it merely requires that all persons shall be
treated alike, under like circumstances and conditions both as to privileges conferred and liabilities enforced”;
and, that the equal protection clause “is not infringed by legislation which applies only to those persons falling
within a specified class, if it applies alike to all persons within such class, and reasonable grounds exist for
making a distinction between those who fall within such class and those who do not.”

VILLEGAS vs. HUI CHIUNG TSAI PAO HO

FACTS:
The Municipal Board of Manila enacted Ordinance 6537 requiring aliens (except those employed in the
diplomatic and consular missions of foreign countries, in technical assistance programs of the government and
another country, and members of religious orders or congregations) to procure the requisite mayor’s permit so
as to be employed or engage in trade in the City of Manila. The permit fee is P50, and the penalty for the
violation of the ordinance is 3 to 6 months’ imprisonment or a fine of P100 to P200, or both.

ISSUE:
Whether the ordinance imposes a regulatory fee or a tax.

HELD:
The ordinance’s purpose is clearly to raise money under the guise of regulation by exacting P50 from
aliens who have been cleared for employment. The amount is unreasonable and excessive because it fails to
consider difference in situation among aliens required to pay it, i.e. being casual, permanent, part-time, rank-
and-file or executive.
The Ordinance was declared invalid as it is arbitrary, oppressive and unreasonable, being applied only
to aliens who are thus deprived of their rights to life, liberty and property and therefore violates the due process
and equal protection clauses of the Constitution. Further, the ordinance does not lay down any criterion or
standard to guide the Mayor in the exercise of his discretion, thus conferring upon the mayor arbitrary and
unrestricted powers.

VERA vs. CUEVAS

FACTS:
Private respondents herein, are engaged in the manufacture, sale and distribution of filled milk
products throughout the Philippines. The products of private respondent, Consolidated Philippines Inc. are
64
marketed and sold under the brand Darigold whereas those of private respondent, General Milk Company
(Phil.), Inc., under the brand "Liberty;" and those of private respondent, Milk Industries Inc., under the brand
"Dutch Baby." Private respondent, Institute of Evaporated Filled Milk Manufacturers of the Philippines, is a
corporation organized for the principal purpose of upholding and maintaining at its highest the standards of
local filled milk industry, of which all the other private respondents are members.
CIR required the respondents to withdraw from the market all of their filled milk products which do not
bear the inscription required by Section 169 of the Tax Code within fifteen (15) days from receipt of the order.
Failure to comply will result to penalties. Section 169 talks of the inscription to be placed in skimmed milk
wherein all condensed skimmed milk and all milk in whatever form, from which the fatty part has been
removed totally or in part, sold or put on sale in the Philippines shall be clearly and legibly marked on its
immediate containers, and in all the language in which such containers are marked, with the words, "This milk
is not suitable for nourishment for infants less than one year of age," or with other equivalent words.
The CFI Manila ordered the CIR to perpetually restrain from requiring the respondents to print on the
labels of their product the words "This milk is not suitable for nourishment for infants less than one year of
age.". Also, it ordered the Fair Trade Board to perpetually restrain from investigating the respondents related
to the manufacture/sale of their filled milk products.

ISSUE:
Whether or not skimmed milk is included in the scope of Section 169 of the Tax Code.

HELD:
No, Section 169 of the Tax Code is not applicable to filled milk. The use of specific and qualifying terms
"skimmed milk" in the headnote and "condensed skimmed milk" in the text of the cited section, would restrict
the scope of the general clause "all milk, in whatever form, from which the fatty pat has been removed totally or
in part." In other words, the general clause is restricted by the specific term "skimmed milk" under the familiar
rule of ejusdem generis that general and unlimited terms are restrained and limited by the particular terms
they follow in the statute.
The difference, therefore, between skimmed milk and filled milk is that in the former, the fatty part has
been removed while in the latter, the fatty part is likewise removed but is substituted with refined coconut oil
or corn oil or both. It cannot then be readily or safely assumed that Section 169 applies both to skimmed milk
and filled milk. It cannot then be readily or safely assumed that Section 169 applies both to skimmed milk and
filled milk. Also, it has been found out that "the filled milk products of the petitioners (now private
respondents) are safe, nutritious, wholesome and suitable for feeding infants of all ages" (p. 44, Rollo) and that
"up to the present, Filipino infants fed since birth with filled milk have not suffered any defects, illness or
disease attributable to their having been fed with filled milk."
Hence, applying Section 169 to it would cause a deprivation of property without due process of law.

DUMLAO vs. COMELEC

FACTS:
Petitioner Patricio Dumlao, is a former Governor of Nueva Vizcaya, who has filed his certificate of
candidacy for said position of Governor in the forthcoming elections of January 30, 1980. Petitioner Dumlao
specifically questions the constitutionality of section 4 of Batas Pambansa Blg. 52 as discriminatory and
contrary to the equal protection and due process guarantees of the Constitution which provides that “….Any
retired elective provincial city or municipal official who has received payment of the retirement benefits to
which he is entitled under the law and who shall have been 65 years of age at the commencement of the term of
office to which he seeks to be elected shall not be qualified to run for the same elective local office from which
he has retired.” He likewise alleges that the provision is directed insidiously against him, and is based on
“purely arbitrary grounds, therefore, class legislation.

ISSUE:
Whether or not 1st paragraph of section 4 of BP 22 is valid.

HELD:
65
In the case of a 65-year old elective local official, who has retired from a provincial, city or municipal
office, there is reason to disqualify him from running for the same office from which he had retired, as provided
for in the challenged provision. The need for new blood assumes relevance. The tiredness of the retiree for
government work is present, and what is emphatically significant is that the retired employee has already
declared himself tired and unavailable for the same government work, but, which, by virtue of a change of
mind, he would like to assume again. It is for this very reason that inequality will neither result from the
application of the challenged provision. Just as that provision does not deny equal protection, neither does it
permit of such denial.
The equal protection clause does not forbid all legal classification. What is proscribes is a classification
which is arbitrary and unreasonable. That constitutional guarantee is not violated by a reasonable classification
based upon substantial distinctions, where the classification is germane to the purpose of the low and applies
to all those belonging to the same class.
WHEREFORE, the first paragraph of section 4 of Batas Pambansa Bilang 52 is hereby declared valid.

PJA V. PRADO

FACTS: The petitioners in this case are members of the lower courts. They assailed the constitutionality of
Section 35 of R.A. No. 7354 as implemented by the Philippine Postal Corporation through its Circular No.92-
28. It withdrew the franking privilege from the SC, the CA, the RTC, the Metropolitan TC, the
Municipal TC, and the Land Registration Commission and Registers of Deeds, along with
certain other government offices.
The petitioners assailed the constitutionality of R.A. No. 7354 on the grounds that:
1) its title embraces more than one subject and does not express its purposes;
2) it did not pass the required readings in both Houses of Congress and printed copies of the bill in its
final form were not distributed among the members before its passage; and
3) it is discriminatory and encroaches on the independence of the Judiciary.

ISSUE: Whether or not Section 35 of R.A. No. 7354, as implemented by the Philippine Postal Corporation
through its Circular No.92-28, violates the equal protection clause.

RULING: Yes. The Supreme Court declared Section 35 of R.A. No. 7354, as implemented by the Philippine
Postal Corporation through its Circular No.92-28, as unconstitutional.
The petitioners alleged that R.A. No. 7354 is discriminatory while on the other hand the respondents
counter that there is no discrimination because the law is based on a valid classification in accordance with the
equal protection clause.
Equal protection simply requires that all persons or things similarly situated should be treated alike,
both as to rights conferred and responsibilities imposed. The equal protection clause does not require the
universal application of the laws on all persons or things without distinction. What the clause requires is
equality among equals as determined according to a valid classification. By classification is
meant the grouping of persons or things similar to each other in certain particulars and
different from all others in these same particulars.
The only acceptable reason for the grant of the franking privilege was the perceived need of the grantee
for the accommodation for a smoother flow of communication between the government and the people. There
is no question that if there is any major branch of the government that needs the privilege, it is the Judicial
Department.
The respondents point out that available data from the Postal Service Office show that from January
1988 to June 1992, the total volume of frank mails amounted to P90, 424,175.00. Of this amount, frank mails
from the Judiciary and other agencies whose functions include the service of judicial processes, such as the
intervenor, the Department of Justice and the Office of the Ombudsman, amounted to P86, 481,759. Frank
mails coming from the Judiciary amounted to P73, 574,864.00, and those coming from the petitioners reached
the total amount of P60, 991,431.00.
The respondents' conclusion is that because of this considerable volume of mail from the Judiciary, the
franking privilege must be withdrawn from it. The respondents are in effect saying that the franking

66
privilege should be extended only to those who do not need it very much but not to those who
need it badly (especially the courts of justice).
If the problem of the respondents is the loss of revenues from the franking privilege, the
remedy is to withdraw it altogether from all agencies of government, including those who do
not need it. The problem is not solved by retaining it for some and withdrawing it from others, especially
where there is no substantial distinction between those favored, which may or may not need it at all, and the
Judiciary, which definitely needs it. There is no reason why it should not recognize a similar and in fact greater
need on the part of the Judiciary for such privilege.

CENIZA V. COMELEC

FACTS: Pursuant to Batas Bilang 51, the COMELEC adopted Resolution No. 1421 which
effectively bars voters in chartered cities, highly urbanized (those earning above P40 M) cities,
and component cities from voting in provincial elections. The City of Mandaue, on the other hand, is a
component city NOT a chartered one or a highly urbanized one. So when COMELEC added Mandaue to the list
of 20 cities that cannot vote in provincial elections, Ceniza, in behalf of the other members of DOERS
(Democracy or Extinction: Resolved to Succeed) questioned the constitutionality of BB 51 and the COMELEC
resolution.
They said that the regulation/restriction of voting being imposed is a curtailment of the
right to suffrage. Further, petitioners claim that political and gerrymandering motives were behind the
passage of Batas Blg. 51 and Section 96 of the Charter of Mandaue City. They contend that the Province of Cebu
is politically and historically known as an opposition bailiwick and of the total 952,716 registered voters in the
province, close to one-third (1/3) of the entire province of Cebu would be barred from voting for the provincial
officials of the province of Cebu. Ceniza also said that the constituents of Mandaue never ratified their charter.
Ceniza likewise aver that Sec 3 of BB 885 insofar as it classifies cities including Cebu City as
highly urbanized as the only basis for not allowing its electorate to vote for the provincial
officials is inherently and palpably unconstitutional in that such classification is not based on
substantial distinctions germane to the purpose of the law which in effect provides for and regulates
the exercise of the right of suffrage, and therefore such unreasonable classification amounts to a denial of equal
protection.

ISSUE: Whether or not there is a violation of equal protection.

RULING: The thrust of the 1973 Constitution is towards the fullest autonomy of local government units. In
the Declaration of Principles and State Policies, it is stated that “The State shall guarantee and promote the
autonomy of local government units to ensure their fullest development as self-reliant communities.
The petitioner’s allegation of gerrymandering is of no merit; it has no factual or legal
basis. The Constitutional requirement that the creation, division, merger, abolition, or alteration of the
boundary of a province, city, municipality, or barrio should be subject to the approval by the majority of the
votes cast in a plebiscite in the governmental unit or units affected is a new requirement that came into being
only with the 1973 Constitution. It is prospective in character and therefore cannot affect the
creation of the City of Mandaue which came into existence on 21 June 1969.
The classification of cities into highly urbanized cities and component cities on the basis
of their regular annual income is based upon substantial distinction. The revenue of a city would
show whether or not it is capable of existence and development as a relatively independent social, economic,
and political unit. It would also show whether the city has sufficient economic or industrial activity as to
warrant its independence from the province where it is geographically situated. Cities with smaller income
need the continued support of the provincial government thus justifying the continued participation of the
voters in the election of provincial officials in some instances.
The petitioners also contend that the voters in Mandaue City are denied equal protection
of the law since the voters in other component cities are allowed to vote for provincial officials.
The contention is without merit. The practice of allowing voters in one component city to vote for
provincial officials and denying the same privilege to voters in another component city is a matter of legislative
discretion which violates neither the Constitution nor the voter’s right of suffrage.
67
NUNEZ V. SANDIGANBAYAN

FACTS: Nuñez assails the validity of the PD 1486 creating the Sandiganbayan as amended by PD
1606. He was accused before the Sandiganbayan of estafa through falsification of public and commercial
documents committed in connivance with his other co-accused, all public officials, in several cases.
It is the claim of Nuñez that PD1486, as amended, is violative of the due process, equal protection, and
ex post facto clauses of the Constitution. He claims that the Sandiganbayan proceedings violates
Nuñez’s right to equal protection, because – appeal as a matter of right became minimized into
a mere matter of discretion; – appeal likewise was shrunk and limited only to questions of law, excluding a
review of the facts and trial evidence; and there is only one chance to appeal conviction, by certiorari to the SC,
instead of the traditional two chances; while all other estafa indictees are entitled to appeal as a matter of right
covering both law and facts and to two appellate courts, i.e., first to the CA and thereafter to the SC.

ISSUE: Whether or not the creation of Sandiganbayan violates equal protection insofar as appeals would be
concerned.

RULING: The SC ruled against Nuñez. The 1973 Constitution had provided for the creation of a
special court that shall have original jurisdiction over cases involving public officials charged
with graft and corruption. The constitution specifically makes mention of the creation of a special court,
the Sandiganbayan, precisely in response to a problem, the urgency of which cannot be denied, namely,
dishonesty in the public service. It follows that those who may thereafter be tried by such court ought to have
been aware as far back as January 17, 1973, when the present Constitution came into force, that a different
procedure for the accused therein, whether a private citizen as petitioner is or a public official,
is not necessarily offensive to the equal protection clause of the Constitution. Further, the
classification therein set forth met the standard requiring that it “must be based on substantial distinctions
which make real differences; it must be germane to the purposes of the law; it must not be limited to existing
conditions only, and must apply equally to each member of the class.” Further still, decisions in the
Sandiganbayan are reached by a unanimous decision from 3 justices – a showing that decisions therein are
more conceivably carefully reached than other trial courts.

VILLEGAS V. SUBIDO

FACTS: Then Metro Manila Mayor Antonio Villegas approved the appointing of 91 women
street sweepers in the City of Manila. But the appointing would still have to be approved by the Office of
Civil Service Commission under Commissioner Abelardo Subido. Subido refused to extend approval to
such appointments on the ground that appointing women to manual labor is against
Memorandum Circular No. 18 series of 1964. Subido pointed out that putting women workers with men
workers outside under the heat of the sun and placing them under manual labor exposes them to contempt and
ridicule and constitutes a violation of the traditional dignity and respect accorded Filipino womanhood.
Villegas however pointed out that the said Memo has already been set aside by the Office of the President hence
the same is no longer in effect.

ISSUE: Whether or not the appointment of said women workers should be confirmed by the Civil Service
Commissioner.

RULING: Yes, the appointments must be confirmed. The basis of Subido was not on any law or rule
but simply on his own concept of what policy to pursue, in this instance in accordance with his own
personal predilection. Here he appeared to be unalterably convinced that to allow women laborers to work
outside their offices as street sweepers would run counter to Filipino tradition. A public official must be able to
point to a particular provision of law or rule justifying the exercise of a challenged authority.
Nothing is better settled in the law than that a public official exercises power, not rights. The
government itself is merely an agency through which the will of the state is expressed and enforced. Its officers
68
therefore are likewise agents entrusted with the responsibility of discharging its functions. As such there is no
presumption that they are empowered to act. There must be a delegation of such authority, either express or
implied. In the absence of a valid grant, they are devoid of power. It must be conceded that departmental zeal
may not be permitted to outrun the authority conferred by statute. Neither the high dignity of the office nor the
righteousness of the motive then is an acceptable substitute. Otherwise the rule of law becomes a myth. Such
an eventuality, we must take all pains to avoid.

69
4. Search and Seizures
*Art. III, Sec. 2, 3(2)
AAA V. CARBONELL

FACTS: Petitioner worked as a secretary at the Arzadon Automotive and Car Service Center. On May 27, 2001
at about 6:30 p.m., Arzadon asked her to deliver a book to an office located at another building but when she
returned to their office, the lights had been turned off and the gate was closed. Nevertheless, she went inside to
get her handbag. On her way out, she saw Arzadon standing beside a parked van holding a pipe. He told her to
go near him and upon reaching his side; he threatened her with the pipe and forced her to lie on the pavement.
He removed her pants and underwear, and inserted his penis into her vagina. She wept and cried out for help
but to no avail because there was nobody else in the premises. When she discovered that she was pregnant as a
consequence of the rape, she narrated the incident to her parents. On July 24, 2002, petitioner filed a
complaint for rape against Arzadon.
On September 16, 2002, Assistant City Prosecutor Imelda Cosalan issued a Resolution finding probable
cause and recommending the filing of an information for rape. However, she failed to attend the next hearing
hence, the case was provisionally dismissed.
On March 5, 2003, petitioner filed another Affidavit-Complaint with a comprehensive account of the
alleged rape incident. The investigating prosecutor issued a Resolution finding that a prima facie case of rape
exists and recommending the filing of the information.
An Information for rape was filed before the RTC, Branch 27, San Fernando, La Union on February 6,
2004, docketed as Criminal Case No. 6415. Thereafter, Arzadon filed a Motion to Hold in Abeyance All Court
Proceedings Including the Issuance of a Warrant of Arrest and to Determine Probable Cause for the Purpose of
Issuing a Warrant of Arrest. On March 18, 2004, respondent Judge Antonio A. Carbonell granted
the motion and directed petitioner and her witnesses to take the witness stand for
determination of probable cause. Instead of taking the witness stand, petitioner filed a motion
for reconsideration claiming that the documentary evidence sufficiently established the
existence of probable cause.
Meanwhile, on December 16, 2005, respondent Judge Carbonell issued the assailed Order
dismissing Criminal Case No. 6983 for lack of probable cause. Petitioner’s motion for reconsideration
was denied hence, this petition.

ISSUE: Whether or not respondent Judge Carbonell acted with grave abuse of discretion in dismissing
Criminal Case No. 6983 for lack of probable cause.

RULING: The Supreme Court ruled in the affirmative. Respondent Judge Carbonell dismissed the
case for lack of probable cause on the ground that petitioner and her witnesses failed to comply
with his orders to take the witness stand.
He claims that under Section 2, Article III of the 1987 Constitution, no warrant of arrest shall issue
except upon probable cause to be determined personally by the judge after examination under oath or
affirmation of the complainant and the witnesses he may produce.
In the leading case of Soliven v. Makasiar the Court explained that this constitutional
provision does not mandatorily require the judge to personally examine the complainant and
her witnesses. Instead, he may opt to personally evaluate the report and supporting documents
submitted by the prosecutor or he may disregard the prosecutors report and require the
submission of supporting affidavits of witnesses.
In the case of Webb v. De Leon, before issuing warrants of arrest, judges merely determine the
probability, not the certainty, of guilt of an accused. In doing so, judges do not conduct a de novo hearing to
determine the existence of probable cause. They just personally review the initial determination of
the prosecutor finding a probable cause to see if it is supported by substantial evidence.
Indeed, what the law requires as personal determination on the part of the judge is that he should not
rely solely on the report of the investigating prosecutor. If the report, taken together with the supporting

70
evidence, is sufficient to sustain a finding of probable cause, it is not compulsory that a personal examination of
the complainant and his witnesses be conducted.
In this case, respondent Judge Carbonell dismissed Criminal Case No. 6983 without taking
into consideration the June 11, 2003 Resolution of 2nd Assistant Provincial Prosecutor Georgina Hidalgo,
the October 13, 2003 Resolution of the panel of prosecutors, and the July 1, 2005 Resolution of the Department
of Justice, all of which sustain a finding of probable cause against Arzadon. Moreover, he failed to
evaluate the evidence in support thereof. Respondent judge’s finding of lack of probable cause was
premised only on the complainants and her witnesses’ absence during the hearing scheduled by
the respondent judge for the judicial determination of probable cause.
After a careful examination of the records, we find that there is sufficient evidence to establish probable
cause. The gravamen of rape is the carnal knowledge by the accused of the private complainant under any of
the circumstances provided in Article 335 of the Revised Penal Code, as amended. It is clear therefore that
respondent Judge Carbonell gravely abused his discretion in dismissing Criminal Case No. 6983 for lack of
probable cause on the ground that petitioner and her witnesses failed to take the witness stand. Considering
there is ample evidence and sufficient basis on record to support a finding of probable cause, it was
unnecessary for him to take the further step of examining the petitioner and her witnesses. Moreover, he erred
in holding that petitioner’s absences in the scheduled hearings were indicative of a lack of interest in
prosecuting the case. In fact, the records show that she has relentlessly pursued the same.

STONEHILL V. DIOKNO

FACTS: Upon application of the officers of the government, several judges issued, on different
dates, a total of 42 search warrants against petitioners and/or the corporations of which they
were officers. The search warrants directed to any peace officer to search the persons above-named and/or
the premises of their offices, warehouses and/or residences, and to seize and take possession of their
personal property such as books of accounts, financial records, vouchers, correspondence, receipts, ledgers,
journals, portfolios, credit journals, typewriters, and other documents and/or papers showing all business
transactions including disbursements receipts, balance sheets and profit and loss statements and Bobbins
(cigarette wrappers). Those items were described as "the subject of the offense; stolen or embezzled and
proceeds or fruits of the offense," or "used or intended to be used as the means of committing the offense,"
which is in "violation of Central Bank Laws, Tariff and Customs Laws, Internal Revenue (Code) and the
Revised Penal Code."
The petitioners alleged that the search warrants are null and void, as contravening the Constitution and
the Rules of Court — because, inter alia:
1) they do not describe with particularity the documents, books and things to be seized;
2) cash money, not mentioned in the warrants, were actually seized;
3) the warrants were issued to fish evidence against the aforementioned petitioners in deportation cases
filed against them;
4) the searches and seizures were made in an illegal manner; and
5) the documents, papers and cash money seized were not delivered to the courts that issued the warrants,
to be disposed of in accordance with law.

The petitioners filed with the Supreme Court an action for certiorari, prohibition, mandamus and
injunction, and prayed that, pending final disposition of the present case, a writ of preliminary injunction be
issued restraining Respondents-Prosecutors, their agents and /or representatives from using the effects seized
as aforementioned or any copies thereof, in their deportation cases and that, in due course, thereafter, decision
be rendered quashing the contested search warrants and declaring the same null and void.

ISSUE: Whether the search warrants in question, and the searches and seizures made are valid or not.

RULING: The Supreme Court ruled that the search warrants issued were illegal and thus, it
violated the constitutional provision against unreasonable search. Two points must be stressed in
connection with this constitutional mandate, namely: (1) that no warrant shall issue but upon probable cause,

71
to be determined by the judge in the manner set forth in said provision; and (2) that the warrant shall
particularly describe the things to be seized.
None of these requirements has been complied with in the contested warrants. The search
warrants were issued upon applications stating that the natural and juridical person therein named had
committed a "violation of Central Ban Laws, Tariff and Customs Laws, Internal Revenue (Code)
and Revised Penal Code." In other words, no specific offense had been alleged in said
applications. As a consequence, it was impossible for the judges who issued the warrants to have found the
existence of probable cause, for the same presupposes the introduction of competent proof that the party
against whom it is sought has performed particular acts, or committed specific omissions, violating a given
provision of our criminal laws. The applications involved in this case do not allege any specific acts performed
by the petitioners.
Thus, the warrants authorized the search for and seizure of records pertaining to all
business transactions of petitioners herein, regardless of whether the transactions were legal
or illegal. The warrants sanctioned the seizure of all records of the petitioners and the aforementioned
corporations, whatever their nature, thus openly contravening the explicit command of our Bill of
Rights — that the things to be seized be particularly described — as well as tending to defeat its
major objective: the elimination of general warrants.
The Supreme Court ruled that the warrants for the search of three (3) residences of the petitioners, as
specified in the Resolution of June 29, 1962, are null and void and that the searches and seizures therein made
are illegal.

VALEROSO V. CA

FACTS: This case is a Letter-Appeal of Senior Inspector Jerry C. Valeroso (Valeroso) praying that the
February 22, 2008 Decision and June 30, 2008 Resolution be set aside and a new one be entered acquitting
him of the crime of illegal possession of firearm and ammunition.
The testimonies of the witnesses of the defense, which the Court gave more credence, are as follows:
On July 10, 1996, Valeroso was sleeping inside a room in the boarding house of his
children located at Sagana Homes, Barangay New Era, Quezon City. He was awakened by four
(4) heavily armed men in civilian attire who pointed their guns at him and pulled him out of the
room. The raiding team tied his hands and placed him near the faucet (outside the room) then
went back inside, searched and ransacked the room. Moments later, an operative came out of
the room and exclaimed, "Hoy, may nakuha akong baril sa loob!"
Disuanco informed Valeroso that there was a standing warrant for his arrest. However, the raiding team
was not armed with a search warrant.
Timbol testified that he issued to Valeroso a Memorandum Receipt dated July 1, 1993 covering the
subject firearm and its ammunition, upon the verbal instruction of Col. Angelito Moreno.
On May 6, 1998, the Regional Trial Court (RTC), Branch 97, Quezon City, convicted Valeroso as charged and
sentenced him to suffer the indeterminate penalty of four (4) years, two (2) months and one (1) day, as
minimum, to six (6) years, as maximum. The gun subject of the case was further ordered confiscated in favor of
the government.
On appeal, the Court of Appeals (CA) affirmed the RTC decision but the minimum term of the
indeterminate penalty was lowered to four (4) years and two (2) months. On petition for review, the SC
affirmed the CA decision. Valeroso filed a Motion for Reconsideration which was denied with finality on June
30, 2008. Valeroso’s Letter-Appeal focused on his breached constitutional rights against unreasonable search
and seizure.

ISSUE: whether or not the warrantless search and seizure of Valeroso’s firearm and ammunition is valid.

RULING: The Supreme Court ruled in the negative. When an arrest is made, it is reasonable for the
arresting officer to search the person arrested in order to remove any weapon that the latter might use in order
to resist arrest or effect his escape. Otherwise, the officer’s safety might well be endangered, and the arrest itself
frustrated. In addition, it is entirely reasonable for the arresting officer to search for and seize any evidence on
the arrestee’s person in order to prevent its concealment or destruction. Moreover, in lawful arrests, it
72
becomes both the duty and the right of the apprehending officers to conduct a warrantless
search not only on the person of the suspect, but also in the permissible area within the latter’s
reach. The phrase "within the area of his immediate control" means the area from within which he might gain
possession of a weapon or destructible evidence.
In the present case, Valeroso was arrested by virtue of a warrant of arrest allegedly for kidnapping with
ransom. At that time, Valeroso was sleeping inside the boarding house of his children. He was awakened by the
arresting officers who were heavily armed. They pulled him out of the room, placed him beside the
faucet outside the room, tied his hands, and then put him under the care of Disuanco. The other
police officers remained inside the room and ransacked the locked cabinet where they found
the subject firearm and ammunition. With such discovery, Valeroso was charged with illegal possession
of firearm and ammunition.
The arresting officers served the warrant of arrest without any resistance from Valeroso. They placed
him immediately under their control by pulling him out of the bed, and bringing him out of the
room with his hands tied. To be sure, the cabinet which, according to Valeroso, was locked, could
no longer be considered as an "area within his immediate control" because there was no way
for him to take any weapon or to destroy any evidence that could be used against him.
The arresting officers would have been justified in searching the person of Valeroso, as well as the tables
or drawers in front of him, for any concealed weapon that might be used against the former. But under the
circumstances obtaining, there was no comparable justification to search through all the desk
drawers and cabinets or the other closed or concealed areas in that room itself.
In this case, search was made in the locked cabinet which cannot be said to have been
within Valeroso’s immediate control. Thus, the search exceeded the bounds of what may be
considered as an incident to a lawful arrest.
Nor can the warrantless search in this case be justified under the "plain view doctrine." The doctrine is
usually applied where a police officer is not searching for evidence against the accused, but nonetheless
inadvertently comes across an incriminating object. The police officers were inside the boarding house of
Valeroso’s children, because they were supposed to serve a warrant of arrest issued against Valeroso. In other
words, the police officers had a prior justification for the intrusion. Consequently, any evidence that they would
inadvertently discover may be used against Valeroso. However, in this case, the police officers did not
just accidentally discover the subject firearm and ammunition; they actually searched for
evidence against Valeroso.

LUZ V. PEOPLE

FACTS: PO2 Emmanuel L. Alteza, who was then assigned at the Sub-Station 1 of the Naga City Police
Station as a traffic enforcer, testified that on March 10, 2003 at around 3:00 o’clock in the morning, he saw
the accused, who was coming from the direction of Panganiban Drive and going to Diversion
Road, Naga City, driving a motorcycle without a helmet; that this prompted him to flag down
the accused for violating a municipal ordinance which requires all motorcycle drivers to wear
helmet (sic) while driving said motor vehicle; that he invited the accused to come inside their sub-
station since the place where he flagged down the accused is almost in front of the said sub-station; that while
he and SPO1 Rayford Brillante were issuing a citation ticket for violation of municipal ordinance, he noticed
that the accused was uneasy and kept on getting something from his jacket; that he was alerted and so, he told
the accused to take out the contents of the pocket of his jacket as the latter may have a weapon inside it; that
the accused obliged and slowly put out the contents of the pocket of his jacket which was a nickel-like tin or
metal container about two (2) to three (3) inches in size, including two (2) cellphones, one (1) pair of scissors
and one (1) Swiss knife; that upon seeing the said container, he asked the accused to open it; that after the
accused opened the container, he noticed a cartoon cover and something beneath it; and that upon his
instruction, the accused spilled out the contents of the container on the table which turned out to be four (4)
plastic sachets, the two (2) of which were empty while the other two (2) contained suspected shabu.
Arraigned on 2 July 2003, petitioner, assisted by counsel, entered a plea of not guilty to the charge of
illegal possession of dangerous drugs. During trial, Police Officer 3 (PO3) Emmanuel Alteza and a forensic
chemist testified for the prosecution. On the other hand, petitioner testified for himself and raised the defense
of planting of evidence and extortion.
73
The RTC convicted petitioner of illegal possession of dangerous drug committed on 10
March 2003. It found the prosecution evidence sufficient to show that he had been lawfully arrested for a
traffic violation and then subjected to a valid search, which led to the discovery on his person of two plastic
sachets later found to contain shabu. The RTC also found his defense of frame-up and extortion to be weak,
self-serving and unsubstantiated. Upon review, the CA affirmed the RTCs Decision.
On 12 September 2011, petitioner filed under Rule 45 the instant Petition for Review on Certiorari dated
1 September 2011.

ISSUE: Whether or not the arrest and the search and seizure of the alleged shabu is valid.

RULING: The Supreme Court ruled in the negative. First, there was no valid arrest of
petitioner. When he was flagged down for committing a traffic violation, he was not, ipso facto
and solely for this reason, arrested.
Arrest is the taking of a person into custody in order that he or she may be bound to answer for the
commission of an offense. It is effected by an actual restraint of the person to be arrested or by that person’s
voluntary submission to the custody of the one making the arrest. It is enough that there be an intention on the
part of one of the parties to arrest the other, and that there be an intent on the part of the other to submit,
under the belief and impression that submission is necessary. Also, under R.A. 4136, or the Land
Transportation and Traffic Code, the general procedure for dealing with a traffic violation is not the arrest of
the offender, but the confiscation of the driver’s license of the latter.
At the time that he was waiting for PO3 Alteza to write his citation ticket, petitioner could
not be said to have been under arrest. There was no intention on the part of PO3 Alteza to arrest him,
deprive him of his liberty, or take him into custody. As found by the trial court, PO3 Alteza himself testified
that the only reason they went to the police sub-station was that petitioner had been flagged down almost in
front of that place. Hence, it was only for the sake of convenience that they were waiting there. There was no
intention to take petitioner into custody.
According to City Ordinance No. 98-012, which was violated by petitioner, the failure to wear a crash
helmet while riding a motorcycle is penalized by a fine only. Under the Rules of Court, a warrant of
arrest need not be issued if the information or charge was filed for an offense penalized by a
fine only.
Second, there being no valid arrest, the warrantless search that resulted from it was
likewise illegal. The following are the instances when a warrantless search is allowed: (i) a warrantless search
incidental to a lawful arrest; (ii) search of evidence in plain view; (iii) search of a moving vehicle; (iv) consented
warrantless search; (v) customs search; (vi) a stop and frisk search; and (vii) exigent and emergency
circumstances. None of the above-mentioned instances, especially a search incident to a lawful arrest, are
applicable to this case.
The evidence seized, although alleged to be inadvertently discovered, was not in plain
view. It was actually concealed inside a metal container inside petitioner’s pocket. Clearly, the
evidence was not immediately apparent.
Neither was there a consented warrantless search. It must be voluntary in order to validate an otherwise
illegal search; that is, the consent must be unequivocal, specific, intelligently given and uncontaminated by any
duress or coercion. While the prosecution claims that petitioner acceded to the instruction of PO3 Alteza, this
alleged accession does not suffice to prove valid and intelligent consent. Whether consent to the search was in
fact voluntary is a question of fact to be determined from the totality of all the circumstances. Relevant to this
determination are the following characteristics of the person giving consent and the environment in which
consent is given: (1) the age of the defendant; (2) whether the defendant was in a public or a secluded location;
(3) whether the defendant objected to the search or passively looked on; (4) the education and intelligence of
the defendant; (5) the presence of coercive police procedures; (6) the defendants belief that no incriminating
evidence would be found; (7) the nature of the police questioning; (8) the environment in which the
questioning took place; and (9) the possibly vulnerable subjective state of the person consenting.
In this case, all that was alleged was that petitioner was alone at the police station at three in the
morning, accompanied by several police officers. These circumstances weigh heavily against a finding of valid
consent to a warrantless search.

74
Neither does the search qualify under the stop and frisk rule. While the rule normally applies when a
police officer observes suspicious or unusual conduct, which may lead him to believe that a criminal act may be
afoot, the stop and frisk is merely a limited protective search of outer clothing for weapons.
The subject items seized during the illegal arrest are inadmissible.

MARTINEZ V. PEOPLE

FACTS: At around 9:15 in the evening of December 29, 2007, PO2 Roberto Soque (PO2 Soque), PO2
Alejandro Cepe (PO2 Cepe) and PO3Edilberto Zeta (PO3 Zeta), who were all assigned to the Station Anti-
Illegal Drugs (SAID) Section of the Malate Police Station 9 (Police Station 9), conducted a routine foot patrol
along Balingkit Street, Malate, Manila. In the process, they heard a man shouting "Putang-ina mo!
Limang-daan na ba ito?"
For purportedly violating Section 844 of the Revised Ordinance of the City of Manila (Manila City
Ordinance) which punishes breaches of the peace, the man, later identified as Ramon, was
apprehended and asked to empty his pockets. In the course thereof, the police officers were
able to recover from him a small transparent plastic sachet containing white crystalline
substance suspected to be shabu. PO2 Soque confiscated the sachet and brought Ramon to Police Station
9 where the former marked the item with the latter’s initials, "RMG." There, Police Superintendent Ferdinand
Ricafrente Quirante (PSupt Quirante) prepared a request for laboratory examination which, together with the
specimen, was brought by PO2 Soque to the PNP Crime Laboratory for examination.
Forensic Chemist Police Senior Inspector Erickson Calabocal (PSInsp Calabocal) examined the
specimen which contained 0.173 gram of white crystalline substance and found the same positive for methyl
amphetamine hydrochloride (or shabu).
Consequently, Ramon was charged with possession of dangerous drugs under Section 11(3), Article II of
RA 9165 through an Information dated January 3, 2008 which states:
That on or about December 29, 2007, in the City of Manila, Philippines, the said accused,
without being authorized by law to possess any dangerous drug, did then and there willfully, unlawfully
and knowingly have in his possession and under his custody and control one (1) heat sealed
transparent plastic sachet containing ZERO POINT ONE SEVEN THREE (0.173) gram of
white crystalline substance containing methyl amphetamine hydrochloride known as SHABU, a
dangerous drug.

In defense, Ramon denied the charge and gave his version of the incident. He narrated that on
December 29, 2007, at around 4:00 in the afternoon, while walking along Balingkit Street to borrow a welding
machine from one Paez Garcia, a man in civilian clothing approached and asked him if he is Ramon Goco.
Upon affirming his identity, he was immediately handcuffed by the man who eventually introduced himself as a
police officer. Together, they boarded a tricycle (sidecar) where the said officer asked him if he was carrying
illegal drugs. Despite his denial, he was still brought to a precinct to be detained. Thereafter, PO2 Soque
propositioned Ramon and asked for ₱20,000.00 in exchange for his release. When Ramon’s wife, Amalia Goco,
was unable to produce the ₱20,000.00 which PO2 Soque had asked for, he (Ramon) was brought to the Manila
City Hall for inquest proceedings.

ISSUE: Whether or not the CA erred in affirming the Decision of the RTC convicting Ramon of the crime of
possession of dangerous drugs.

RULING: Yes, the petition is meritorious. Enshrined in the fundamental law is a person’s right against
unwarranted intrusions by the government. Section 2, Article III of the 1987 Philippine Constitution
(Constitution) states that:
Section 2.The right of the people to be secure in their persons, houses, papers, and effects
against unreasonable searches and seizures of whatever nature and for any purpose shall be inviolable,
and no search warrant or warrant of arrest shall issue except upon probable cause to be determined
personally by the judge after examination under oath or affirmation of the complainant and the
witnesses he may produce, and particularly describing the place to be searched and the persons or
things to be seized.
75
Effects secured by government authorities in contravention of the foregoing are
rendered inadmissible in evidence for any purpose, in any proceeding.
The "exclusionary rule” of Section 2 is not, however, an absolute and rigid one. As found in
jurisprudence, the traditional exceptions are customs searches, searches of moving vehicles, seizure of evidence
in plain view, consented searches, "stop and frisk" measures and searches incidental to a lawful arrest. The
search incidental to a lawful arrest is of particular significance to this.
A valid warrantless arrest which justifies a subsequent search is one that is carried out
under the parameters of Section 5(a), Rule 113 of the Rules of Court which requires that the
apprehending officer must have been spurred by probable cause to arrest a person caught in
flagrante delicto. To be sure, the term probable cause has been understood to mean a reasonable ground of
suspicion supported by circumstances sufficiently strong in themselves to warrant a cautious man's belief that
the person accused is guilty of the offense with which he is charged. With respect to arrests, it is such facts and
circumstances which would lead a reasonably discreet and prudent man to believe that an offense has been
committed by the person sought to be arrested.
Records show that PO2 Soque arrested Ramon for allegedly violating Section 844 of the
Manila City Ordinance which penalizes the following acts: (1) making, countenancing, or assisting in
making any riot, affray, disorder, disturbance, or breach of the peace; (2) assaulting, beating or using personal
violence upon another without just cause in any public place; (3) uttering any slanderous, threatening or
abusive language or expression or exhibiting or displaying any emblem, transparency, representation, motto,
language, device, instrument, or thing; and (4) doing any act, in any public place, meeting or procession,
tending to disturb the peace or excite a riot, or collect with other persons in a body or crowd for any unlawful
purpose, or disturbance or disquiet any congregation engaged in any lawful assembly. Evidently, the
gravamen of these offenses is the disruption of communal tranquillity.
Thus, to justify a warrantless arrest based on the same, it must be established that the apprehension
was effected after a reasonable assessment by the police officer that a public disturbance was happening.
It cannot be said that the act of shouting in a thickly-populated place, with many people
conversing with each other on the street, would constitute any of the acts punishable under
Section 844 of the Manila City Ordinance as above-quoted. Ramon was not making or assisting in any
riot, affray, disorder, disturbance, or breach of the peace; he was not assaulting, beating or using personal
violence upon another; and, the words he allegedly shouted – "Putang-ina mo! Limang-daan na ba
ito?" – are not slanderous, threatening or abusive, and thus, could not have tended to disturb
the peace or excite a riot considering that at the time of the incident, Balingkit Street was still
teeming with people and alive with activity.
Furthermore, no one present at the place of arrest ever complained that Ramon’s
shouting disturbed the public. On the contrary, a disinterested member of the community even testified
that Ramon was merely standing in front of the store of a certain Mang Romy when a man in civilian clothes,
later identified as PO2 Soque, approached Ramon, immediately handcuffed and took him away.
The Court observes that these facts and circumstances could not have engendered a well-founded belief
that any breach of the peace had been committed by Ramon at the time that his warrantless arrest was effected.
No probable cause existed to justify Ramon’s warrantless arrest.
Consequently, as it cannot be said that Ramon was validly arrested the warrantless
search that resulted from it was also illegal. Thus, the subject shabu purportedly seized from
Ramon is inadmissible in evidence for being the proverbial fruit of the poisonous tree as
mandated by the above discussed constitutional provision.

CABALLES VS. COURT OF APPEALS, G.R. No. 136292, 373 SCRA 221, January 15, 2002

PONENTE: PUNO

FACTS:
Sgt. Victorino Noceja and Pat. Alex de Castro, while on a routine patrol in a Barangay in Laguna,
spotted a passenger jeep unusually covered with "kakawati" leaves. Suspecting that the jeep was loaded with
smuggled goods, the two police officers flagged down the vehicle. With appellant's alleged consent, the police
76
officers checked the cargo and they discovered bundles of galvanized conductor wires exclusively owned by
National Power Corporation (NPC). Thereafter, appellant and the vehicle with the high-voltage wires were
brought to the Pagsanjan Police Station. Danilo Cabale took pictures of the appellant and the jeep loaded with
the wires which were turned over to the Police Station Commander of Pagsanjan, Laguna. Appellant was
incarcerated for 7 days in the Municipal jail. In defense, appellant interposed denial and alibi. Thus, the court a
quo rendered judgment finding the accused guilty beyond reasonable doubt of the crime of Theft. On appeal,
the Court of Appeals affirmed the judgment of conviction.

ISSUE:
Whether or not the warrantless search and seizure made by the police officers, and the admissibility of
the evidence obtained by virtue thereof was valid.

RULING:
NO. Enshrined in our Constitution is the inviolable right of the people to be secure in their persons and
properties against unreasonable searches and seizures, as defined under Section 2, Article III. The exclusionary
rule under Section 3(2), Article III of the Constitution bars the admission of evidence obtained in violation of
such right.
The constitutional proscription against warrantless searches and seizures is not absolute but admits of
certain exceptions, namely: (1) warrantless search incidental to a lawful arrest recognized under Section 12,
Rule 126 of the Rules of Court and by prevailing jurisprudence; (2) seizure of evidence in plain view;9 (3)
search of moving vehicles;10 (4) consented warrantless search; (5) customs search; (6) stop and frisk situations
(Terry search);12 and (7) exigent and emergency circumstances.
In cases where warrant is necessary, the steps prescribed by the Constitution and reiterated in the Rules
of Court must be complied with. In the exceptional events where warrant is not necessary to effect a valid
search or seizure, or when the latter cannot be performed except without a warrant, what constitutes a
reasonable or unreasonable search or seizure is purely a judicial question, determinable from the uniqueness of
the circumstances involved, including the purpose of the search or seizure, the presence or absence of probable
cause, the manner in which the search and seizure was made, the place or thing searched and the character of
the articles procured.

THE PEOPLE OF THE PHILIPPINES, vs. LEILA JOHNSON Y REYES, 348 SCRA 526, Dec. 18,
2000

FACTS:
Accused-appellant Leila Reyes Johnson was, at the time of the incident, 58 years old, a widow, and a
resident of Ocean Side, California, U.S.A. She is a former Filipino citizen who was naturalized as an American
on June 16, 1968 and had since been working as a registered nurse, taking care of geriatric patients and those
with Alzheimers disease, in convalescent homes in the United States.
On June 16, 1998, she arrived in the Philippines to visit her son’s family in Calamba, Laguna. She was
due to fly back to the United States on July 26. On July 25, she checked in at the Philippine Village Hotel to
avoid the traffic on the way to the Ninoy Aquino International Airport (NAIA) and checked out at 5:30 p.m. the
next day, June 26, 1998.
At around 7:30 p.m. of that day, Olivia Ramirez was on duty as a lady frisker at Gate 16 of the NAIA
departure area. Her duty was to frisk departing passengers, employees, and crew and check for weapons,
bombs, prohibited drugs, contraband goods, and explosives.
When she frisked accused-appellant Leila Johnson, a departing passenger bound for the United States
via Continental Airlines CS-912, she felt something hard on the latter’s abdominal area. Upon inquiry, Mrs.
Johnson explained she needed to wear two panty girdles as she had just undergone an operation as a result of
an ectopic pregnancy.
Not satisfied with the explanation, Ramirez reported the matter to her superior, SPO4 Reynaldo
Embile, saying Sir, hindi po ako naniniwalang panty lang po iyon. (Sir, I do not believe that it is just a panty.)
She was directed to take accused-appellant to the nearest women’s room for inspection. Ramirez took accused-
appellant to the rest room, accompanied by SPO1 Rizalina Bernal. Embile stayed outside.

77
Inside the women’s room, accused-appellant was asked again by Ramirez what the hard object on her
stomach was and accused-appellant gave the same answer she had previously given. Ramirez then asked her to
bring out the thing under her girdle. Accused-appellant brought out three plastic packs, which Ramirez then
turned over to Embile, outside the women’s room.
The confiscated packs, marked as Exhibits C-1, C-2 and C-3, contained a total of 580.2 grams of a
substance which was found by NBI Chemist George de Lara to be methamphetamine hydrochloride or shabu.
Embile took accused-appellant and the plastic packs to the 1st Regional Aviation and Security Office
(1st RASO) at the arrival area of the NAIA, where accused-appellants passport and ticket were taken and her
luggage opened. Pictures were taken and her personal belongings were itemized.
In her defense, accused-appellant alleged that she was standing in line at the last boarding gate when
she was approached by Embile and two female officers. She claimed she was handcuffed and taken to the
women’s room. There, she was asked to undress and was then subjected to a body search. She insisted that
nothing was found on her person. She was later taken to a room filled with boxes, garbage, and a chair. Her
passport and her purse containing $850.00 and some change were taken from her, for which no receipt was
issued to her. After two hours, she said, she was transferred to the office of a certain Col. Castillo.
After another two hours, Col. Castillo and about eight security guards came in and threw two white
packages on the table. They told her to admit that the packages were hers. But she denied knowledge and
ownership of the packages. She was detained at the 1st RASO office until noon of June 28, 1999 when she was
taken before a fiscal for inquest. She claimed that throughout the period of her detention, from the night of
June 26 until June 28, she was never allowed to talk to counsel nor was she allowed to call the U.S. Embassy or
any of her relatives in the Philippines.
Since the accused contends that her constitutional right was violated specifically Art 3 Sec. 12. But The
trial court rendered a decision that the accused was guilty beyond reasonable doubt and her rights was not
violated because the arrest was in flagrante delicto pursuant to a valid search made on her person according to
Rule 113 of Criminal Procedure. The CA affirmed the decision and lift the charges to Reclusion Perpetua.

ISSUE:
Whether or not the accused appellant is entitled art. 3 sec. 12 of Constitution which requires a valid
search warrant and seizure?

HELD:
No. The supreme court ruled that art. 3 sec. 12 cannot be applied because of the certain exceptions,
namely: (1) warrantless search incidental to a lawful arrest recognized under Section 12, Rule 126 of the Rules
of Court and by prevailing jurisprudence; (2) seizure of evidence in plain view;9 (3) search of moving
vehicles;10 (4) consented warrantless search; (5) customs search; (6) stop and frisk situations (Terry search);12
and (7) exigent and emergency circumstances.
Persons may lose the protection of the search and seizure clause by exposure of their persons or
property to the public in a manner reflecting a lack of subjective expectation of privacy, which expectation
society is prepared to recognize as reasonable. Such recognition is implicit in airport security procedures. With
increased concern over airplane hijacking and terrorism has come increased security at the nation’s airports.
Passengers attempting to board an aircraft routinely pass through metal detectors; their carry-on baggage as
well as checked luggage are routinely subjected to x-ray scans. Should these procedures suggest the presence of
suspicious objects, physical searches are conducted to determine what the objects are. There is little question
that such searches are reasonable, given their minimal intrusiveness, the gravity of the safety interests
involved, and the reduced privacy expectations associated with airline travel. Indeed, travelers are often
notified through airport public address systems, signs, and notices in their airline tickets that they are subject
to search and, if any prohibited materials or substances are found, such would be subject to seizure. These
announcements place passengers on notice that ordinary constitutional protections against warrantless
searches and seizures do not apply to routine airport procedures.
The packs of methamphetamine hydrochloride having thus been obtained through a valid warrantless
search, they are admissible in evidence against the accused-appellant herein. Corollarily, her subsequent arrest,
although likewise without warrant, was justified since it was effected upon the discovery and recovery of shabu
in her person in flagrante delicto.

78
The Court is convinced that the requirements of the law in order that a person may be validly charged
with and convicted of illegal possession of a dangerous drug in violation of R.A. No. 6425, as amended, have
been complied with by the prosecution in this case. The decision of the trial court must accordingly be upheld.
As regards the fine imposed by the trial court, it has been held that courts may fix any amount within
the limits established by law. Considering that five hundred eighty point two (580.2) grams of shabu were
confiscated from accused-appellant, the fine imposed by the trial court may properly be reduced to
P50,000.00.

SOCIAL JUSTICE SECRETARY V. DANGEROUS DRUG BOARD

FACTS:
In 2002, Republic Act No. 9165 or the Comprehensive Dangerous Drugs Act of 2002 was implemented.
Section 36 thereof requires mandatory drug testing of candidates for public office, students of secondary and
tertiary schools, officers and employees of public and private offices, and persons charged before the
prosecutor’s office with certain offenses.
In December 2003, COMELEC issued Resolution No. 6486, prescribing the rules and regulations on
the mandatory drug testing of candidates for public office in connection with the May 10, 2004 synchronized
national and local elections. Aquilino Pimentel, Jr., a senator and a candidate for re-election in the May
elections, filed a Petition for Certiorari and Prohibition under Rule 65. In it, he seeks (1) to nullify Sec. 36(g) of
RA 9165 and COMELEC Resolution No. 6486 dated December 23, 2003 for being unconstitutional in that they
impose a qualification for candidates for senators in addition to those already provided for in the 1987
Constitution; and (2) to enjoin the COMELEC from implementing Resolution No. 6486.
According to Pimentel, the Constitution only prescribes a maximum of five (5) qualifications for one to
be a candidate for, elected to, and be a member of the Senate. He says that both the Congress and COMELEC,
by requiring, via RA 9165 and Resolution No. 6486, a senatorial aspirant, among other candidates, to undergo
a mandatory drug test, create an additional qualification that all candidates for senator must first be certified
as drug free. He adds that there is no provision in the Constitution authorizing the Congress or COMELEC to
expand the qualification requirements of candidates for senator.

ISSUE: Whether or not Sec 36 of RA 9165 and Resolution 6486 are constitutional.

HELD:
No. Pimentel’s contention is valid. Accordingly, Sec. 36 of RA 9165 is unconstitutional. It is basic that if
a law or an administrative rule violates any norm of the Constitution, that issuance is null and void and has no
effect. The Constitution is the basic law to which all laws must conform; no act shall be valid if it conflicts with
the Constitution. In the discharge of their defined functions, the three departments of government have no
choice but to yield obedience to the commands of the Constitution. Whatever limits it imposes must be
observed.
The provision “[n]o person elected to any public office shall enter upon the duties of his office until he
has undergone mandatory drug test” is not tenable as it enlarges the qualifications. COMELEC cannot, in the
guise of enforcing and administering election laws or promulgating rules and regulations to implement Sec. 36,
validly impose qualifications on candidates for senator in addition to what the Constitution prescribes. If
Congress cannot require a candidate for senator to meet such additional qualification, the COMELEC, to be
sure, is also without such power. The right of a citizen in the democratic process of election should not be
defeated by unwarranted impositions of requirement not otherwise specified in the Constitution.

JESSE LUCAS V. JESUS LUCAS, 2011

FACTS:
On July 26, 2007, petitioner, Jesse U. Lucas, filed a Petition to Establish Illegitimate Filiation (with
Motion for the Submission of Parties to DNA Testing) [2] before the Regional Trial Court (RTC), Branch 72,
Valenzuela City. Petitioner narrated that, sometime in 1967, his mother, Elsie Uy (Elsie), migrated to Manila
from Davao and stayed with a certain Ate Belen (Belen) who worked in a prominent nightspot in Manila. Elsie
79
would oftentimes accompany Belen to work. On one occasion, Elsie got acquainted with respondent, Jesus S.
Lucas, at Belen’s workplace, and an intimate relationship developed between the two. Elsie eventually got
pregnant and, on March 11, 1969, she gave birth to petitioner, Jesse U. Lucas. The name of petitioner’s father
was not stated in petitioner’s certificate of live birth. However, Elsie later on told petitioner that his father is
respondent. On August 1, 1969, petitioner was baptized at San Isidro Parish, Taft Avenue, Pasay City.
Respondent allegedly extended financial support to Elsie and petitioner for a period of about two years. When
the relationship of Elsie and respondent ended, Elsie refused to accept respondents offer of support and
decided to raise petitioner on her own. While petitioner was growing up, Elsie made several attempts to
introduce petitioner to respondent, but all attempts were in vain.
Attached to the petition were the following: (a) petitioners certificate of live birth; (b) petitioners
baptismal certificate; (c) petitioners college diploma, showing that he graduated from Saint Louis University in
Baguio City with a degree in Psychology; (d) his Certificate of Graduation from the same school; (e) Certificate
of Recognition from the University of the Philippines, College of Music; and (f) clippings of several articles
from different newspapers about petitioner, as a musical prodigy.
RTC held that the ruling on the grounds relied upon by petitioner for filing the petition is premature
considering that a full-blown trial has not yet taken place. The court stressed that the petition was sufficient in
form and substance. It noted that the new Rule on DNA Evidence [11] allows the conduct of DNA testing,
whether at the courts instance or upon application of any person who has legal interest in the matter in
litigation.
The CA reversed the RTC because it did not acquire jurisdiction over the person of respondent, as no
summons had been served on him. Respondents special appearance could not be considered as voluntary
appearance because it was filed only for the purpose of questioning the jurisdiction of the court over
respondent.

ISSUE:
Whether or not an order for DNA testing is considered as unreasonable search and seizure.

HELD:
No. a DNA testing order is not a matter of right. There should be first a finding of probable cause before
the court will grant the DNA testing. The Supreme Court adopted the decision of SC Louisiana that Although a
paternity action is civil, not criminal, the constitutional prohibition against unreasonable searches and seizures
is still applicable, and a proper showing of sufficient justification under the particular factual circumstances of
the case must be made before a court may order a compulsory blood test.
The same condition precedent should be applied in our jurisdiction to protect the putative father from
mere harassment suits. Thus, during the hearing on the motion for DNA testing, the petitioner must present
prima facie evidence or establish a reasonable possibility of paternity.
Notwithstanding these, it should be stressed that the issuance of a DNA testing order remains
discretionary upon the court. The court may, for example, consider whether there is absolute necessity for the
DNA testing. If there is already preponderance of evidence to establish paternity and the DNA test result would
only be corroborative, the court may, in its discretion, disallow a DNA testing.

PEOPLE vs. NAZARENO VILLAREAL

G.R. No. 201363, March 18, 2013, 693 SCRA 549

FACTS: PO3 Renato de Leon was driving his motorcycle on his way home along 5th Avenue when he saw
Nazareno from a distance of about 8 to 10 meters, holding and scrutinizing in his hand a plastic sachet of
shabu. Thus, PO3 de Leon alighted from his motorcycle and approached Nazareno whom he recognized as
someone he had previously arrested from illegal drug possession. Upon seeing PO3 de Leon, Nazareno tried to
escape but was quickly apprehended with the help of a tricycle driver. PO3 de Leon was able board Nazareno
on to his motorcycle and confiscate the plastic sachet of shabu in his possession.

ISSUE: Whether or not the warrantless arrest was valid.

80
HELD. NO. Nazareno’s acts of walking along the street and holding something in his hands even if they
appeared to be dubious, coupled with his previous criminal charge for the same offense, are not by themselves
sufficient to incite suspicion of criminal activity or to create probable cause enough to justify a warrantless
arrest under Section 5 of Rule 113.
“Probable Cause” has been understood to mean a reasonable ground of suspicion supported by
circumstances sufficiently strong in themselves to warrant a cautious man’s belief that the person accused is
guilty of the offense with which he is charged.

QUA CHEE GAN V. DEPORTATION BOARD, 1963

FACTS:
Qua Chee Gan, James Uy, Daniel Dy alias Dee Pac, Chan Tiong Yu, Chua Chu Tian, Chua Lim Pao alias
Jose Chua, and Basilio King were charged before the Deportation Board with having purchased $130,000.00
without the necessary licensing from the Central Bank and having clandestinely remitted the same to Hong
Kong.
After the filing of the deportation charges, a warrant of arrest was issued for Qua Chee Gan, et al.
pending investigation.
They were granted provisional liberty upon their filing of a surely bond for P10,000.00 and a cash bond
for P10,000.00.
Qua Chee Gan, et al. filed a joint motion to dismiss the charges on the ground, among others, that such
charges are not legal grounds for deportation ad that the Board has not jurisdiction over such charges. The
motion to dismiss was denied.
Qua Chee Gan, et al. then filed a petition for habeas corpus and/or prohibition.

TRIAAL COURT: Upheld the validity of the delegation by the President to the Deportation Board the power to
conduct investigations for the purpose of determining whether the stay of an alien in this country would be
injurious to the security, welfare and interest of the State.
The court also sustained the power of the deportation Board to issue warrant of arrest and fix bonds for
the alien's temporary release pending investigation on the theory that the power to arrest and fix the amount of
the bond of the arrested alien is essential to and complement the power to deport aliens.

ISSUES:
1) WoN the President has the power to deport aliens and if such power is validly delegated to the
Deportation Board. –YES.
2) WoN the authority to deport aliens includes the power to order the arrest of such aliens. –YES. BUT
only when there is already an ORDER OF DEPORTATION.

HELD:
1) CA No. 613 expressly grants the Commissioner of Immigration the power to effect the arrest and
expulsion of an alien, after previous determination by the Board of Commissioners, but such power was
not intended to be delimited to the Immigration Commissioner as Sec. 69 of the Administrative Code,
although not expressly conferring such power, lays down the procedure for such deportation
proceedings for the President.
Therefore, the deportation of an undesirable alien may be effected in 2 ways:
a. By order of the President, after due investigation, pursuant to Section 69 of the Revised
Administrative Code, and
b. By the Commissioner of Immigration, upon recommendation by the Board of
Commissioners, under Section 37 of Commonwealth Act No. 613.

And although the charges against Qua Chee Gan are not enumerated in CA No. 613, the act of
profiteering, hoarding or black-marketing of U.S. dollars, in violation of the Central Bank regulations,
which is tantamount to economic sabotage, is a ground for deportation under the provisions of Republic
Act 503 amending Section 37 of the Philippine Immigration Act of 1940.

81
2) Under EO No. 69, it is required that the alien charged in deportation proceedings shall file a bond with
the Commissioner of Immigration in order to secure their appearance. However, the same did not
authorize the arrest of the alien pending investigation.
It was in EO No. 398, that the Board was authorized motu proprio or upon the filing of formal
charges by the Special Prosecutor of the Board, to issue the warrant for the arrest of the alien
complained of and to hold him under detention during the investigation unless he files a bond for his
provisional release in such amount and under such conditions as may be prescribed by the Chairman of
the Board.
However, Section 69 of the Revised Administrative Code, upon whose authority the President's
power to deport is predicated, does NOT provide for the exercise of the power to arrest.
Moreover, the right of an individual to be secure in his person is guaranteed by Sec. 1 Art III of
the Constitution: “...no warrants shall issue but upon probable cause, to be determined by the judge
after examination under oath or affirmation of the complainant and the witnesses he may produce...”
Rodriguez, et al. v. Villamiel, et al. expands the requirement — "to be determined by the judge"
— to any public officer who may be authorized by the Legislature to make such determination, and
thereafter issue the warrant of arrest.
Therefore, the arrest of a foreigner, which is necessary to carry into effect the power of
deportation is valid only when there is already an order of deportation. However, during the
investigation, it is not indispensable that the alien be arrested.
It is enough that a bond be required to insure the appearance of the alien during the
investigation, as was authorized in EO69.

BRICCIO Ricky A. POLLO V. CONSTANTINO-DAVID, 2011

FACTS:
Respondent CSC Chair Constantino-David received an anonymous letter complaint alleging of an
anomaly taking place in the Regional Office of the CSC. The respondent then formed a team and issued a memo
directing the team “to back up all the files in the computers found in the Mamamayan Muna (PALD) and Legal
divisions.”
Several diskettes containing the back-up files sourced from the hard disk of PALD and LSD computers
were turned over to Chairperson David. The contents of the diskettes were examined by the CSC’s Office for
Legal Affairs (OLA). It was found that most of the files in the 17 diskettes containing files copied from the
computer assigned to and being used by the petitioner, numbering about 40 to 42 documents, were draft
pleadings or letters in connection with administrative cases in the CSC and other tribunals. On the basis of this
finding, Chairperson David issued the Show-Cause Order, requiring the petitioner, who had gone on extended
leave, to submit his explanation or counter-affidavit within five days from notice.
In his Comment, petitioner denied the accusations against him and accused the CSC Officials of “fishing
expedition” when they unlawfully copied and printed personal files in his computer.
He was charged of violating R.A. No. 6713 (Code of Conduct and Ethical Standards for Public Officials
and Employees). He assailed the formal charge and filed an Omnibus Motion ((For Reconsideration, to
Dismiss and/or to Defer) assailing the formal charge as without basis having proceeded from an illegal search
which is beyond the authority of the CSC Chairman, such power pertaining solely to the court.
The CSC denied the omnibus motion and treated the motion as the petitioner’s answer to the charge. In
view of the absence of petitioner and his counsel, and upon the motion of the prosecution, petitioner was
deemed to have waived his right to the formal investigation which then proceeded ex parte
The petitioner was dismissed from service. He filed a petition to the CA which was dismissed by the
latter on the ground that it found no grave abuse of discretion on the part of the respondents. He filed a motion
for reconsideration which was further denied by the appellate court. Hence, this petition.

ISSUE:
WON the search conducted by the CSC on the computer of the petitioner constituted an illegal search
and was a violation of his constitutional right to privacy.

HELD:
82
NO. The search conducted on his office computer and the copying of his personal files was lawful and
did not violate his constitutional right.

Ratio Decidendi
In this case, the Court had the chance to present the cases illustrative of the issue raised by the
petitioner.
O’Connor v. Ortega 480 U.S. 709 (1987), the Court categorically declared that “[i]ndividuals do not lose Fourth
Amendment rights merely because they work for the government instead of a private employer.” In O’Connor
the Court recognized that “special needs” authorize warrantless searches involving public employees for work-
related reasons. The Court thus laid down a balancing test under which government interests are weighed
against the employee’s reasonable expectation of privacy. This reasonableness test implicates neither probable
cause nor the warrant requirement, which are related to law enforcement.
Social Justice Society (SJS) v. Dangerous Drugs Board G.R. Nos. 157870, 158633 and 161658,
November 3, 2008, 570 SCRA 410, 427, (citing Ople v. Torres, G.R. No. 127685, July 23, 1998, 293 SCRA 141,
169), recognized the fact that there may be such legitimate intrusion of privacy in the workplace.
The Court ruled that the petitioner did not have a reasonable expectation of privacy in his office and
computer files.
As to the second point of inquiry, the Court answered in the affirmative. The search authorized by the
CSC Chair, the copying of the contents of the hard drive on petitioner’s computer reasonable in its inception
and scope.
The Court noted that unlike in the case of Anonymous Letter-Complaint against Atty. Miguel Morales,
Clerk of Court, Metropolitan Trial Court of Manila A.M. Nos. P-08-2519 and P-08-2520, November 19, 2008,
571 SCRA 361, the case at bar involves the computer from which the personal files of the petitioner were
retrieved is a government-issued computer, hence government property the use of which the CSC has absolute
right to regulate and monitor.

OPLE V. TORRES

FACTS:
Administrative Order No. 308, entitled "Adoption of a National Computerized Identification Reference
System," was issued by President Fidel Ramos On December 12, 1996.
Senator Blas F. Ople filed a petition seeking to invalidate A.O. No. 308 on several grounds. One of them
is that: The establishment of a National Computerized Identification Reference System requires a legislative
act. The issuance of A.O. No. 308 by the President is an unconstitutional usurpation of the legislative powers of
congress. Petitioner claims that A.O. No. 308 is not a mere administrative order but a law and hence, beyond
the power of the President to issue. He alleges that A.O. No. 308 establishes a system of identification that is
all-encompassing in scope, affects the life and liberty of every Filipino citizen and foreign resident, and more
particularly, violates their right to privacy.
On this point, respondents counter-argue that: A.O. No. 308 was issued within the executive and
administrative powers of the president without encroaching on the legislative powers of congress.

ISSUE:
Whether or not A.O. No. 308 violates the right of privacy.

HELD:
YES. Yes.
The right to privacy as such is accorded recognition independently of its identification with liberty; in
itself, it is fully deserving of constitutional protection.
The right of privacy is guaranteed in several provisions of the Constitution:
"Sections 3 (1), 1, 2, 6, 8 and 17 of the Bill of Rights
"Sec. 3. The privacy of communication and correspondence shall be inviolable except upon lawful order
of the court, or when public safety or order requires otherwise as prescribed by law."
"Sec. 1. No person shall be deprived of life, liberty, or property without due process of law, nor shall any
person be denied the equal protection of the laws."
83
"Sec. 2. The right of the people to be secure in their persons, houses, papers, and effects against
unreasonable searches and seizures of whatever nature and for any purpose shall be inviolable, and no search
warrant or warrant of arrest shall issue except upon probable cause to be determined personally by the judge
after examination under oath or affirmation of the complainant and the witnesses he may produce, and
particularly describing the place to be searched and the persons or things to be seized."
"Sec. 6. The liberty of abode and of changing the same within the limits prescribed by law shall not be
impaired except upon lawful order of the court. Neither shall the right to travel be impaired except in the
interest of national security, public safety, or public health, as may be provided by law."
"Sec. 8. The right of the people, including those employed in the public and private sectors, to form
unions, associations, or societies for purposes not contrary to law shall not be abridged."
"Sec. 17. No person shall be compelled to be a witness against himself."
The right to privacy is a fundamental right guaranteed by the Constitution, hence, it is the burden of
government to show that A.O. No. 308 is justified by some compelling state interest and that it is narrowly
drawn. A.O. No. 308 is predicated on two considerations: (1) the need to provide our citizens and foreigners
with the facility to conveniently transact business with basic service and social security providers and other
government instrumentalities and (2) the need to reduce, if not totally eradicate, fraudulent transactions and
misrepresentations by persons seeking basic services. It is debatable whether these interests are compelling
enough to warrant the issuance of A.O. No. 308.
But what is not arguable is the broadness, the vagueness, the overbreadth of A.O. No. 308 which if
implemented will put our people's right to privacy in clear and present danger. The possibilities of abuse and
misuse of the PRN, biometrics and computer technology are accentuated when we consider that the individual
lacks control over what can be read or placed on his ID, much less verify the correctness of the data encoded.
They threaten the very abuses that the Bill of Rights seeks to prevent.
The petition is granted and declared the Administrative Order No. 308 entitled "Adoption of a National
Computerized Identification Reference System" null and void for being unconstitutional.

KILUSANG MAYO UNO V. DIRECTOR GENERAL NEDA

FACTS:
President Arroyo issued Executive Order 450 which requires all government agencies and controlled
corporations to have a uniform identification card; the director-general of the national economic development
authority was tasked to implement this order. The information required to be in the said identification card
would be: name, home address, sex, picture, signature, date of birth, place of birth, marital status, names of
parents, height, weight, two index fingers and two thumb marks, any prominent distinguishing features like
moles and others, tax identification number (TIN). The petitioners argued that the said executive order
usurped legislative functions and violates the right of privacy. Petitioners alleged that EO 450 is contrary to law
because it violated the principle handed down by the Court in Ople v Torres and RA 8282 or the Social Security
Act of 1997. The order according to the petitioners was also going to use funds that are not appropriated by the
Congress, it was also issued without a public hearing. The order was also violating the constitutional provision
of equal protection of the laws because it discriminates and penalizes those who do not have an id. The
petitioners also argue that the order violates the right to privacy by allowing for the access of the personal data
of the owner without his or her consent.

ISSUE:
Whether or not EO 450 usurped legislative functions and violated the citizen’s right to privacy.

Held:
NO. The Supreme Court ruled that the petition had no merit. The said order only applies to government
agencies who are already issuing identification cards even before the said order was implemented. The
purposes of the order were to: reduce costs, achieve efficiency and reliability, convenience to the people served
by the government entities and insure compatibility. Section 17 Article VII of the Constitution also provides for
the President to have control to all executive departments, bureaus and offices. This constitutional power of the
President is self-executing and does not need implementing legislation. This power of course is limited to
executive branch of the government and does not extend to other branches or independent constitutional
84
commissions. EO 450 does not violate the right to privacy since no citizen particularly government employee
has complained upon the showing of information on their identification cards, even the petitioners have not
made any complaint about their own identification cards. EO 450 also issues identification cards that only have
14 data about the owner much less than what is issued upon Supreme Court employees.

Gamboa v. Chan 677 SCRA 385

FACTS
Gamboa alleged that the Philippine National Police in Ilocos Norte (PNP–Ilocos Norte) conducted a
series of surveillance operations against her and her aides, and classified her as someone who keeps a Private
Army Group (PAG). Purportedly without the benefit of data verification, PNP–Ilocos Norte forwarded the
information gathered on her to the Zeñarosa Commission, thereby causing her inclusion in the Report’s
enumeration of individuals maintaining PAGs. Contending that her right to privacy was violated and her
reputation maligned and destroyed, Gamboa filed a Petition for the issuance of a writ of habeas data against
respondents in their capacities as officials of the PNP-Ilocos Norte.

ISSUE
Whether or not the petition for the issuance of writ of habeas data is proper when the right to privacy is
invoked as opposed to the state’s interest in preserving the right to life, liberty or security.

RULING
NO.
The writ of habeas data is an independent and summary remedy designed to protect the image, privacy,
honor, information, and freedom of information of an individual, and to provide a forum to enforce one’s right
to the truth and to informational privacy. It seeks to protect a person’s right to control information regarding
oneself, particularly in instances in which such information is being collected through unlawful means in order
to achieve unlawful ends. It must be emphasized that in order for the privilege of the writ to be granted, there
must exist a nexus between the right to privacy on the one hand, and the right to life, liberty or security on the
other.
In this case, the Court ruled that Gamboa was unable to prove through substantial evidence that her
inclusion in the list of individuals maintaining PAGs made her and her supporters susceptible to harassment
and to increased police surveillance. In this regard, respondents sufficiently explained that the investigations
conducted against her were in relation to the criminal cases in which she was implicated. As public officials,
they enjoy the presumption of regularity, which she failed to overcome. [T]he state interest of dismantling
PAGs far outweighs the alleged intrusion on the private life of Gamboa, especially when the collection and
forwarding by the PNP of information against her was pursuant to a lawful mandate. Therefore, the privilege of
the writ of habeas data must be denied.

Rhonda Vivares vs St. Theresa’s College, 737 SCRA 92

FACTS
In January 2012, Angela Tan, a high school student at St. Theresa’s College (STC), uploaded on
Facebook several pictures of her and her classmates (Nenita Daluz and Julienne Suzara) wearing only their
undergarments.
Thereafter, some of their classmates reported said photos to their teacher, Mylene Escudero. Escudero,
through her students, viewed and downloaded said pictures. She showed the said pictures to STC’s Discipline-
in-Charge for appropriate action.
Later, STC found Tan et al to have violated the student’s handbook and banned them from “marching”
in their graduation ceremonies scheduled in March 2012.
The issue went to court but despite a TRO (temporary restraining order) granted by the Cebu RTC
enjoining the school from barring the students in the graduation ceremonies, STC still barred said students.
Subsequently, Rhonda Vivares, mother of Nenita, and the other mothers filed a petition for the issuance
of the writ of habeas data against the school. They argued, among others, that:
85
1. The privacy setting of their children’s Facebook accounts was set at “Friends Only.” They, thus, have a
reasonable expectation of privacy which must be respected.
2. The photos accessed belong to the girls and, thus, cannot be used and reproduced without their consent.
Escudero, however, violated their rights by saving digital copies of the photos and by subsequently
showing them to STC’s officials. Thus, the Facebook accounts of the children were intruded upon;
3. The intrusion into the Facebook accounts, as well as the copying of information, data, and digital
images happened at STC’s Computer Laboratory;

They prayed that STC be ordered to surrender and deposit with the court all soft and printed copies of
the subject data and have such data be declared illegally obtained in violation of the children’s right to privacy.
The Cebu RTC eventually denied the petition. Hence, this appeal.

ISSUE: Whether or not the petition for writ of habeas data is proper.

HELD: Yes, it is proper but in this case, it will not prosper.


Contrary to the arguments of STC, the Supreme Court ruled that:
1. The petition for writ of habeas data can be availed of even if this is not a case of extralegal killing or
enforced disappearance; and
2. The writ of habeas data can be availed of against STC even if it is not an entity engaged in the
business of “gathering, collecting, or storing data or information regarding the person, family, home
and correspondence of the aggrieved party”.

First, the Rule on Habeas Data does not state that it can be applied only in cases of extralegal killings or
enforced disappearances. Second, nothing in the Rule would suggest that the habeas data protection shall be
available only against abuses of a person or entity engaged in the business of gathering, storing, and collecting
of data.

Right to Privacy on Social Media (Online Networking Sites)


The Supreme Court ruled that if an online networking site (ONS) like Facebook has privacy tools, and
the user makes use of such privacy tools, then he or she has a reasonable expectation of privacy (right to
informational privacy, that is). Thus, such privacy must be respected and protected.
In this case, however, there is no showing that the students concerned made use of such privacy tools.
Evidence would show that that their post (status) on Facebook were published as “Public”.
Facebook has the following settings to control as to who can view a user’s posts on his “wall” (profile
page):
a) Public – the default setting; every Facebook user can view the photo;
b) Friends of Friends – only the user’s Facebook friends and their friends can view the photo;
c) Friends – only the user’s Facebook friends can view the photo;
d) Custom – the photo is made visible only to particular friends and/or networks of the Facebook user;
and
e) Only Me – the digital image can be viewed only by the user.

The default setting is “Public” and if a user wants to have some privacy, then he must choose any setting
other than “Public”. If it is true that the students concerned did set the posts subject of this case so much so
that only five people can see them (as they claim), then how come most of their classmates were able to view
them. This fact was not refuted by them. In fact, it was their classmates who informed and showed their
teacher, Escudero, of the said pictures. Therefore, it appears that Tan et al never use the privacy settings of
Facebook hence, they have no reasonable expectation of privacy on the pictures of them scantily clad.
STC did not violate the students’ right to privacy. The manner which the school gathered the pictures
cannot be considered illegal. As it appears, it was the classmates of the students who showed the picture to
their teacher and the latter, being the recipient of said pictures, merely delivered them to the proper school
authority and it was for a legal purpose, that is, to discipline their students according to the standards of the
school (to which the students and their parents agreed to in the first place because of the fact that they enrolled
their children there).

86
*Search Warrants:
Rule 12, Rules of Court
Rules 113, Sec. 3, Rules of Court
PEDRO PACIS vs. MANUEL R. PAMARAN, 56 SCRA 16

FACTS:
Respondent Ricardo Santos brought into this country Mercury automobile, model 1957. Petitioner
Pedro Pacis, acting Collector of Customs, on July 22, 1964 received from the Administrator, General Affairs
Administration of the Department of National Defense, a letter to the effect that the Land Transportation
Commission reported that such automobile was a “hot car.” By virtue thereof, petitioner, through his
subordinates, looked into the records of his office. Thus he did ascertain that although the amount of P311.00
was already paid for customs duty, the amount collectible on said car should be P2,500.00, more or less. Based
on such discrepancy, on July 22, 1964, he instituted seizure proceedings and issued a warrant of seizure and
detention. On the strength thereof, the automobile was taken while it was parked on Economia Street, Manila,
and was then brought to the General Affairs Administration compound.
Then on August 26, 1964, respondent Ricardo Santos, through counsel, wrote to the petitioner asking
that such warrant of seizure and detention issued against his car be withdrawn or dissolved and the car
released on his contention that the issuance of the warrant was unauthorized. Petitioner denied the request for
the release of the car and adverting that the petitioner had, under the law, authority to issue such warrant of
seizure and detention. As a result, respondent on September 15, 1964 filed a criminal complaint for usurpation
of judicial functions with the City Fiscal of Manila.

ISSUE:
May petitioner effect the seizure without any search warrant issued by a component court.?

RULING:
The Tariff and Customs Code does not require said warrant in the instant case. The Code authorizes
persons having police authority under Section 2203 of the Tariff and Customs Code to enter, pass through or
search any land, enclosure, warehouse, store or building, not being a dwelling house and also to inspect, search
and examine any vessel or aircraft and any trunk, package, box or envelope or any person on board, or stop and
search and examine any vehicle, beast or person suspected of holding or conveying any dutiable or prohibited
article introduced into the Philippines contrary to law, without mentioning the need of a search warrant in said
cases. But in the search of a dwelling house, the Code provides that said “dwelling house may be entered and
searched only upon warrant issued by a judge or justice of the peace …”
In this case the subject car was seized in a parking lot, not a dwelling house.

Lim vs. Ponce De Leon, 66 SCRA 299

FACTS:
Taha sold to a certain Alberto Timbangcaya a motor launch named M/L "SAN RAFAEL". A year later or
on April 9, 1962 Alberto Timbangcaya filed a complaint with the Office of the Provincial Fiscal of Palawan
alleging that after the sale Jikil Taha forcibly took away the motor launch from him.
Fiscal Francisco Ponce de Leon, upon being informed that the motor launch was in Balabac, Palawan,
wrote the Provincial Commander of Palawan requesting him to direct the detachment commander-in Balabac
to impound and take custody of the motor launch.
On June 26, 1962, Fiscal Ponce de Leon reiterated his request to the Provincial Commander to impound
the motor launch, explaining that its subsequent sale to a third party, plaintiff-appellant Delfin Lim, cannot
prevent the court from taking custody of the same. 2 So, on July 6, 1962 upon order of the Provincial
Commander, defendant-appellee Orlando Maddela, Detachment Commander of Balabac, Palawan, seized the
motor launch "SAN RAFAEL" from plaintiff-appellant Delfin Lim and impounded it.

87
Delfin Lim and Jikil Taha filed a case against Fiscal Francisco Ponce de Leon and Orlando Maddela,
alleging that on July 6, 1962 Orlando Maddela entered the premises of Delfin Lim without a search warrant
and then and there took away the hull of the motor launch without his consent; that he effected the seizure
upon order of Fiscal Ponce de Leon who knew fully well that his office was not vested with authority to order
the seizure of a private property; that said motor launch was purchased by Delfin Lim from Jikil Taha in
consideration of Three Thousand Pesos (P3,000.00), Two Thousand Pesos (P2,000.00) of which has been
given to Jikil Taha as advance payment; that as a consequence of the unlawful seizure of the motor launch, its
sale did not materialize; and that since July 6, 1962, the said motor launch had been moored at the Balabac
Bay, Palawan and because of exposure to the elements it had become worthless and beyond repair.
In their answer, defendants-appellees denied the material allegations of the complaint and as
affirmative defenses alleged that the motor launch in question which was sold by Jikil Taha to Alberto
Timbangcaya on April 29, 1961 was sometime in April 1962, forcibly taken with violence upon persons and with
intent to gain by Jikil Taha from Alfredo Timbangcaya without the latter's knowledge and consent, thus giving
rise to the filing of a criminal charge of robbery against Jikil Taha; that Fiscal Ponce de Leon, in his capacity as
Acting Provincial Fiscal of Palawan ordered Orlando Maddela to seize and impound the motor launch "SAN
RAFAEL", for being the corpus delicti of the robbery; and that Orlando Maddela merely obeyed the orders of
his superior officer to impound said launch. By way of counterclaim, defendants-appellees alleged that because
of the malicious and groundless filing of the complaint by plaintiffs-appellants, they were constrained to
engage the services of lawyers, each of them paying P500.00 as attorney's fees; and that they suffered moral
damages in the amount of P5,000.00 each and actual damages in the amount of P500.00 each. They also
prayed that each of them awarded exemplary damages in the amount of P1,000.00.

ISSUES:
Whether or not defendant-appellee Fiscal Ponce de Leon had the power to order the seizure of the
motor launch in question without a warrant of search and seizure even if the same was admittedly the corpus
delicti of the crime.

HELD:
No, the power to issue a search warrant is vested in a judge or magistrate and in no other officer and no
search and seizure can be made without a proper warrant.
Whether or not defendants-appellees are civilly liable to plaintiffs-appellants for damages allegedly
suffered by them granting that the seizure of the motor launch was unlawful.
To be liable under Article 32 of the New Civil Code it is enough that there was a violation of the
constitutional rights of the plaintiffs and it is not required that defendants should have acted with malice or
bad faith. Except for Madella who was merely acting under orders.

RATIO DICIDENDI:
Since in the present case defendants-appellees seized the motor launch without a warrant, they have
violated the constitutional right of plaintiffs-appellants against unreasonable search and seizure.
Under the old Constitution 7 the power to issue a search warrant is vested in a judge or magistrate and
in no other officer and no search and seizure can be made without a proper warrant. At the time the act
complained of was committed, there was no law or rule that recognized the authority of Provincial Fiscals to
issue a search warrant. In his vain attempt to justify the seizure of the motor launch in question without a
warrant Fiscal Ponce de Leon invoked the provisions of Republic Act No. 732, which amended Sections 1674
and 1687 of the Revised Administrative Code. But there is nothing in said law which confers upon the
provincial fiscal; the authority to issue warrants, much less to order without warrant the seizure of a personal
property even if it is the corpus delicti of a crime. True, Republic Act No. 732 has broadened the power of
provincial fiscals to conduct preliminary investigations, but said law did not divest the judge or magistrate of
its power to determine, before issuing the corresponding warrant, whether or not probable cause exists
therefor. 8
We are not prepared to sustain his defense of good faith. To be liable under Article 32 of the New Civil
Code it is enough that there was a violation of the constitutional rights of the plaintiffs and it is not required
that defendants should have acted with malice or bad faith.
But defendant-appellee Orlando Maddela cannot be held accountable because he impounded the motor
launch upon the order of his superior officer. While a subordinate officer may be held liable for executing
88
unlawful orders of his superior officer, there are certain circumstances which would warrant Maddela's
exculpation from liability. The records show that after Fiscal Ponce de Leon made his first request to the
Provincial Commander on June 15, 1962 Maddela was reluctant to impound the motor launch despite repeated
orders from his superior officer. 21 It was only after he was furnished a copy of the reply of Fiscal Ponce de
Leon, dated June 26, 1962, to the letter of the Provincial Commander, justifying the necessity of the seizure of
the motor launch on the ground that the subsequent sale of the launch to Delfin Lim could not prevent the
court from taking custody of the same, 22 that he impounded the motor launch on July 6, 1962. With said letter
coming from the legal officer of the province, Maddela was led to believe that there was a legal basis and
authority to impound the launch. Then came the order of his superior officer to explain for the delay in the
seizure of the motor launch. 23 Faced with a possible disciplinary action from his Commander, Maddela was
left with no alternative but to seize the vessel. In the light of the above circumstances. We are not disposed to
hold Maddela answerable for damages.

PONSICA VS IGNALAGA, 152 SCRA 647

FACTS:
The chief issue raised by the petitioners in this case is whether or not Section 143 of the Local
Government Code1granting power to the municipal mayor to conduct preliminary investigations and order the
arrest of the accused, was repealed by the 1985 Rules on Criminal Procedure promulgated by this Court; and is,
in addition, unconstitutional as vesting the power to conduct preliminary investigations in an official who
cannot be deemed a "neutral and detached magistrate" within the contemplation of Section 3, Article IV of the
1973 Constitution. The issue is hereby resolved adversely to the petitioners, with the stressed qualification that
the mayor's power to order arrest ceased to exist as of February 2, 1987 when the new Constitution was ratified
by the Filipino people, and that, in any event, the investigation actually conducted by respondent mayor in the
case at bar was fatally defective.

RULING:
While it is true that the mayors do "exercise general supervision over units and elements of the INP
stationed or assigned in their respective jurisdictions," they are not themselves directly involved in police work
and cannot in any sense be described, as the petitioners do, as being deeply involved in law enforcement
functions. And even if that "deep involvement" be conceded, it does not follow that this would necessarily
preclude their assuming "the cold neutrality of an impartial judge" in conducting preliminary investigations of
persons suspected of crimes.
As the law now stands, the mayor may no longer conduct preliminary investigation, the authority to do
so being limited under Section 2, Rule 1 1 2 of the Rules of Court to (1) provincial or city fiscals and their
assistants; (2) judges of the Municipal Trial Courts and Municipal Circuit Trial Courts; (3) national and
regional state prosecutors; and (d) such other officers as may be authorized by law. But only "the judge" may
issue search and arrest warrants after due determination of probable cause.

SALAZAR V. ACHACOSO, 183 SCRA 145

Facts:
Rosalie Tesoro of Pasay City in a sworn statement filed with the POEA, charged petitioner with illegal
recruitment. Public respondent Atty. Ferdinand Marquez sent petitioner a telegram directing him to appear to
the POEA regarding the complaint against him. On the same day, after knowing that petitioner had no license
to operate a recruitment agency, public respondent Administrator Tomas Achacoso issued a Closure and
Seizure Order No. 1205 to petitioner. It stated that there will a seizure of the documents and paraphernalia
being used or intended to be used as the means of committing illegal recruitment, it having verified that
petitioner has— (1) No valid license or authority from the Department of Labor and Employment to recruit and
deploy workers for overseas employment; (2) Committed/are committing acts prohibited under Article 34 of
the New Labor Code in relation to Article 38 of the same code. A team was then tasked to implement the said
Order. The group, accompanied by media men and Mandaluyong policemen, went to petitioner’s residence.
They served the order to a certain Mrs. For a Salazar, who let them in. The team confiscated assorted costumes.
89
Petitioner filed with POEA a letter requesting for the return of the seized properties, because she was not given
prior notice and hearing. The said Order violated due process. She also alleged that it violated Sec 2 of the Bill
of Rights, and the properties were confiscated against her will and were done with unreasonable force and
intimidation.

Issue:
Whether or Not the Philippine Overseas Employment Administration (or the Secretary of Labor) can
validly issue warrants of search and seizure (or arrest) under Article 38 of the Labor Code

Held:
Under the new Constitution, “. . . no search warrant or warrant of arrest shall issue except upon
probable cause to be determined personally by the judge after examination under oath or affirmation of the
complainant and the witnesses he may produce, and particularly describing the place to be searched and the
persons or things to be seized”. Mayors and prosecuting officers cannot issue warrants of seizure or arrest. The
Closure and Seizure Order was based on Article 38 of the Labor Code. The Supreme Court held, “We reiterate
that the Secretary of Labor, not being a judge, may no longer issue search or arrest warrants. Hence, the
authorities must go through the judicial process. To that extent, we declare Article 38, paragraph (c), of the
Labor Code, unconstitutional and of no force and effect… The power of the President to order the arrest of
aliens for deportation is, obviously, exceptional. It (the power to order arrests) cannot be made to extend to
other cases, like the one at bar. Under the Constitution, it is the sole domain of the courts.” Furthermore, the
search and seizure order was in the nature of a general warrant. The court held that the warrant is null and
void, because it must identify specifically the things to be seized.
WHEREFORE, the petition is GRANTED. Article 38, paragraph (c) of the Labor Code is declared
UNCONSTITUTIONAL and null and void. The respondents are ORDERED to return all materials seized as a
result of the implementation of Search and Seizure Order No. 1205.

THE PRESIDENTIAL ANTI-DOLLAR SALTING TASK FORCE vs COURT OF APPEALS 171 SCRA
348

FACTS:
On March 12, 1985, State Prosecutor Jose B. Rosales, who is assigned with the Presidential Anti-Dollar
Salting Task Force, issued search warrants Nos. 156, 157, 158, 159, 160 and 161 against the petitioners Karamfil
Import-Export Co., Inc., P & B Enterprises Co., Inc., Philippine Veterans Corporation, Philippine Veterans
Development Corporation, Philippine Construction Development Corporation, Philippine Lauan Industries
Corporation, Inter-trade Development (Alvin Aquino), Amelili U. Malaquiok Enterprises and
Jaime P. Lucman Enterprises.
The application for the issuance of said search warrants was filed by Atty. Napoleon Gatmaytan of the Bureau
of Customs who is a deputized member of the PADS Task Force. Attached to the said application is the affidavit
of Josefin M. Castro who is an operative and investigator of the PADS Task Force. Said Josefin M. Castro is
likewise the sole deponent in the purported deposition to support the application for the issuance of the six (6)
search warrants involved in this case. The application filed by Atty. Gatmaytan, the affidavit and deposition of
Josefin M. Castro are all dated March 12, 1985. Shortly thereafter, the private respondent (the petitioner) went
to the Regional Trial Court on a petition to enjoin the implementation of the search warrants in question. On
April 16, 1985, the lower court issued the first of its challenged Orders, and held:
WHEREFORE, in view of all the foregoing, the Court hereby declares Search Warrant Nos. 156,
157, 158, 159, 160, and 161 to be null and void. Accordingly, the respondents are hereby ordered to
return and surrender immediately all the personal properties and documents seized by them from the
petitioners by virtue of the aforementioned search warrants. On August 21, 1985, the trial court denied
reconsideration. On April 4, 1986, the Presidential Anti-Dollar Salting Task Force went to the
respondent Court of Appeals to contest, on certiorari, the twin Orders of the lower court. In ruling
initially for the Task Force, the Appellate.

Court held:

90
Herein petitioner is a special quasi-judicial body with express powers enumerated under PD 1936 to
prosecute foreign exchange violations defined and punished under P.D. No. 1883. The petitioner, in exercising
its quasi-judicial powers, ranks with the Regional Trial Courts, and the latter in the case at bar had no
jurisdiction to declare the search warrants in question null and void. Besides as correctly pointed out by the
Assistant Solicitor General the decision of the Presidential Anti-Dollar Salting Task Force is appealable to the
Office of the President.
On November 12, 1986, Karamfil Import-Export Co., Inc. sought a reconsideration, on the question
primarily of whether or not the Presidential Anti-Dollar Salting Task Force is "such other responsible officer'
countenanced by the 1973 Constitution to issue warrants of search and seizure. The Court of Appeals, on
Karamfil's motion, reversed itself and issued its Resolution, dated September 1987, and subsequently, its
Resolution, dated May 20, 1988, denying the petitioner's motion for reconsideration.
In submitting that it is a quasi-judicial entity, the petitioner states that it is endowed with "express
powers and functions under PD No. 1936, to prosecute foreign exchange violations as defined and punished
under PD No. 1883." "By the very nature of its express powers as conferred by the laws," so it is contended,
"which are decidedly quasi-judicial or discretionary function, such as to conduct preliminary investigation on
the charges of foreign exchange violations, issue search warrants or warrants of arrest, hold departure orders,
among others, and depending upon the evidence presented, to dismiss the charges or to file the corresponding
information in court of Executive Order No. 934, PD No. 1936 and its Implementing Rules and Regulations
effective August 26, 1984, petitioner exercises quasi-judicial power or the power of adjudication ."
The Court of Appeals, in its Resolution now assailed, was of the opinion that "the grant of quasi-judicial
powers to petitioner did not diminish the regular courts' judicial power of interpretation. The right to interpret
a law and, if necessary to declare one unconstitutional, exclusively pertains to the judiciary. In assuming this
function, courts do not proceed on the theory that the judiciary is superior to the two other coordinate
branches of the government, but solely on the theory that they are required to declare the law in every case
which come before them." In its petition to this Court, the petitioner alleges that in so issuing the Resolutions
above-mentioned, the respondent Court of Appeals "committed grave abuse of discretion and/or acted in
excess of its appellate jurisdiction."

ISSUE:
Whether or not The Presidential Anti-Dollar Salting Task Force is a quasi-judicial body, and one
coequal in rank and standing with the Regional Trial Court, and accordingly, beyond the latter's jurisdiction

RULING:
No. This Court finds the Appellate Court to be in error, since what the petitioner puts to question is the
Regional Trial Court's act of assuming jurisdiction over the private respondent's petition below and its
subsequent countermand of the Presidential Anti-Dollar Salting Task Force's orders of search and seizure, for
the reason that the presidential body, as an entity (allegedly) coordinate and co-equal with the Regional Trial
Court, was (is) not vested with such a jurisdiction. An examination of the Presidential Anti- Dollar Salting Task
Force's petition shows indeed its recognition of judicial review (of the acts of Government) as a basic privilege
of the courts. Its objection, precisely, is whether it is the Regional Trial Court, or the superior courts, that may
undertake such a review.
As we have observed, the question is whether or not the Presidential Anti-Dollar Salting Task Force is,
in the first place, a quasi-judicial body, and one whose decisions may not be challenged before the regular
courts, other than the higher tribunals, the Court of Appeals and this Court. A quasi-judicial body has been
defined as "an organ of government other than a court of law and other than a legislature, which affects the
rights of private parties through either adjudication or rule making." As may be seen, it is the basic function of
these bodies to adjudicate claims and/or to determine rights, and unless its decision are seasonably appealed to
the proper reviewing authorities, the same attain finality and become executory. A perusal of the Presidential
Anti-Dollar Salting Task Force's organic act, Presidential Decree No. 1936, as amended by Presidential Decree
No. 2002, convinces the Court that the Task Force was not meant to exercise quasi-judicial functions, that is, to
try and decide claims and execute its judgments. As the President's arm called upon to combat the vice of
"dollar salting" or the black marketing and salting of foreign exchange, it is tasked alone by the Decree to
handle the prosecution of such activities, but nothing more.
The Court sees nothing in the provisions of Presidential Decree No. 1936 (except with respect to the
Task Force's powers to issue search warrants) that will reveal a legislative intendment to confer it with quasi-
91
judicial responsibilities relative to offenses punished by Presidential Decree No. 1883. Its undertaking, as we
said, is simply, to determine whether or not probable cause exists to warrant the filing of charges with the
proper court, meaning to say, to conduct an inquiry preliminary to a judicial recourse, and to recommend
action "of appropriate authorities". It is not unlike a fiscal's office that conducts a preliminary investigation to
determine whether or not prima facie evidence exists to justify haling the respondent to court, and yet, while it
makes that determination, it cannot be said to be acting as a quasi-court. For it is the courts, ultimately, that
pass judgment on the accused, not the fiscal.
If the Presidential Anti-Dollar Salting Task Force is not, hence, a quasi-judicial body, it cannot be said
to be co-equal or coordinate with the Regional Trial Court. There is nothing in its enabling statutes that would
demonstrate its standing at par with the said court.
In that respect, we do not find error in the respondent Court of Appeal's resolution sustaining the
assumption of jurisdiction by the court a quo.

RATIO:
A quasi-judicial body has been defined as "an organ of government other than a court of law and
other than a legislature, which affects the rights of private parties through either adjudication or rule
making."

MALALOAN V CA, 232 SCRA 249

FACTS:
Members of the CAPCOM, armed with a search warrant issued by respondent RTC Judge of Kalookan
City in connection with an alleged violation of P.D. 1866 perpetrated in Quezon City, proceeded to the site
where a labor seminar was then taking place. Firearms, explosive materials and subversive documents, among
others, were seized and taken during the search. Petitioners, all EILER Instructors, were indicted.

ISSUE(S):
Whether or not the issuance of search warrants and/or warrants of arrest is subject to jurisdictional
restrictions.

RULING:
NO. It may be conceded, as a matter of policy, that where a criminal case is pending, the court wherein
it was filed, or the assigned branch thereof, has primary jurisdiction to issue the search warrant; and where no
such criminal case has yet been filed, that the executive judges or their lawful substitutes in the areas and for
the offenses contemplated in Circular No. 19 shall have primary jurisdiction.
Petition is DENIED and the assailed judgment of respondent Court of Appeals is AFFIRMED.

ILANO V CA, 244 SCRA 346

FACTS:
The PNP-NARCOM, NCR applied for a search warrant before the Regional Trial Court of Kalookan City
to search petitioner’s house and parlor in Quezon City. After personally examining the applicant and his
witness, the presiding judge issued the search warrants.

ISSUE:
Whether or not a court may validly issue a search warrant for a place outside its jurisdiction.

RULING:
YES. While the trial court which has territorial jurisdiction over the place has primary authority to issue
search warrants therefor, any court of competent jurisdiction when necessitated and justified by compelling
considerations of urgency, subject, time and place, may issue a search warrant covering a place outside its
territorial jurisdiction.
Petition for review is DENIED.
92
JOSE BURGOS VS. CHIEF OF STAFF, DECEMBER 26, 1984

FACTS:
Respondent Judge Ernani Cruz-Pano issued two (2) search warrants to search the premises of the
“Metropolitan Mail” and “We Forum”. Various equipment’s, paraphernalia and written documents were
searched. Written literature alleged to be in the possession and control of petitioner Jose Burgos, Jr.
publisher-editor of “We Forum” newspaper was seized.
The validity of these warrants were questioned through a petition for certiorari and so that the articles
and equipment may be returned to the petitioners.

ISSUE:
Whether or not the warrant of arrest is valid to justify the seizure of the items.

HELD:
The law provides that “no search warrant or warrant of arrest shall issue except upon probable cause to
be determined by the judge, or such other responsible officer as may be authorized by law, after examination
under oath or affirmation of the complainant and the witnesses he may produce, and particularly describing
the place to be searched and the persons or things to be seized”.
Probable cause for a search is defined as such facts and circumstances which would lead a reasonably
discreet and prudent man to believe that an offense has been committed and that the objects sought in
connection with the offense are in the place sought to be searched. In the case, the reason for the seizure must
be well stated, as well as the specifications and the particularities of the alleged subversive material that the
petitioner has published or is intending to publish. Mere generalization will not suffice. Thus, the broad
statement in Col. Abadilla’s application is a mere conclusion of law and does not satisfy the requirements of
probable cause. The warrant is constitutionally objectionable because they are in the nature of general
warrants. The search warrants were declared null and void.

QUINTERO VS. NBI, 162 SCRA 467, JUNE 23, 1988

FACTS:
Petitioner is a delegate of the First District of Leyte to the 1971 Constitutional Convention. He delivered
a privileged speech at the plenary session and disclosed that certain persons are bribing some delegates. He
eventually released from his hospital bed a sworn statement the names of the persons who gave him the
money, implicating the First Lady among others. On the basis of a search warrant, agents of the respondent
National Bureau of Investigation raided petitioner’s house and a criminal complaint for direct bribery was filed
against him.

ISSUE:
Whether or not the search warrant issued was valid.

RULING:
NO. The interrogations conducted by the respondent judge upon the applicant NBI agent showed that
the latter knew nothing, of his own personal knowledge, to show that petitioner had committed any offense.
The statement of Congressman Mate, which was the sole basis for the issuance of the search warrant, was
replete with conclusions and inferences drawn from what he allegedly witnessed when he visited petitioner in
the hospital. It lacked the directness and definiteness which would have been present, had the same statement
dealt with facts which Congressman Mate actually witnessed.
Search warrant issued is declared NULL and VOID.

BACHE VS. RUIZ, FEBRUARY 27, 1971

93
FACTS:
On 24 Feb 1970, Commissioner Vera of Internal Revenue, wrote a letter addressed to J Ruiz requesting
the issuance of a search warrant against petitioners for violation of Sec 46(a) of the NIRC, in relation to all
other pertinent provisions thereof, particularly Sects 53, 72, 73, 208 and 209, and authorizing Revenue
Examiner de Leon make and file the application for search warrant which was attached to the letter. The next
day, de Leon and his witnesses went to CFI Rizal to obtain the search warrant. At that time J Ruiz was hearing
a certain case; so, by means of a note, he instructed his Deputy Clerk of Court to take the depositions of De
Leon and Logronio. After the session had adjourned, J Ruiz was informed that the depositions had already
been taken. The stenographer read to him her stenographic notes; and thereafter, J Ruiz asked respondent
Logronio to take the oath and warned him that if his deposition was found to be false and without legal basis,
he could be charged for perjury. J Ruiz signed de Leon’s application for search warrant and Logronio’s
deposition. The search was subsequently conducted.

ISSUE:
Whether or not there had been a valid search warrant.

HELD:
The SC ruled in favor of Bache on three grounds.
1. J Ruiz failed to personally examine the complainant and his witness.
Personal examination by the judge of the complainant and his witnesses is necessary to enable
him to determine the existence or non-existence of a probable cause.

2. The search warrant was issued for more than one specific offense.
The search warrant in question was issued for at least four distinct offenses under the Tax Code.
As ruled in Stonehill “Such is the seriousness of the irregularities committed in connection with the
disputed search warrants, that this Court deemed it fit to amend Section 3 of Rule 122 of the former
Rules of Court that ‘a search warrant shall not issue but upon probable cause in connection with one
specific offense.’ Not satisfied with this qualification, the Court added thereto a paragraph, directing
that ‘no search warrant shall issue for more than one specific offense.

3. The search warrant does not particularly describe the things to be seized.
The documents, papers and effects sought to be seized are described in the Search Warrant:
“Unregistered and private books of accounts (ledgers, journals, columnar, receipts and
disbursements books, customer’s ledgers); receipts for payments received; certificates of stocks
and securities; contracts, promissory notes and deeds of sale; telex and coded messages;
business communications, accounting and business records; checks and check stubs; records of
bank deposits and withdrawals; and records of foreign remittances, covering the years 1966 to
1970.”

The description does not meet the requirement in Art III, Sec. 1, of the Constitution, and of Sec.
3, Rule 126 of the Revised Rules of Court, that the warrant should particularly describe the things to be
seized.
A search warrant may be said to particularly describe the things to be seized when the
description therein is as specific as the circumstances will ordinarily allow or when the description
expresses a conclusion of fact not of law by which the warrant officer may be guided in making the
search and seizure or when the things described are limited to those which bear direct relation to the
offense for which the warrant is being issued.

JOSEPHINE SORIANO MATA VS. JUDGE BAYONA, GR. NO. 50720, MARCH 26, 1984

FACTS:

94
Soriano Mata was accused under Presidential Decree (PD) 810, as amended by PD 1306, the
information against him alleging that Soriano Mata offered, took and arranged bets on the Jai Alai game by
“selling illegal tickets known as ‘Masiao tickets’ without any authority from government authorities concerned.”
Petitioner discovered that nowhere from the records of the said case could be found the search warrant
and other pertinent papers connected to the issuance of the same. This led Mata to file a motion to quash and
annul the search warrant and for the return of the articles seized, citing and invoking, among others, Section 4
of Rule 126 of the Revised Rules of Court. The motion was denied by the Judge, stating that the court has made
a thorough investigation and examination under oath by the members of the Intelligence Section of Police and
that the fact that the rule does not specify when these documents are to be attached to the records. Mata came
to the Supreme Court and prayed that the search warrant be declared invalid.

ISSUE:
Whether or not the search warrant is valid.

HELD:
NO. We hold that the search warrant is tainted with illegality for being violative of the Constitution and
the Rules of Court.
Section 4 of Rule 126 which provides that the judge must before issuing the warrant personally examine
on oath or affirmation the complainant and any witnesses he may produce and take their depositions in
writing, and attach them to the record, in addition to any affidavits presented to him.
Mere affidavits of the complainant and his witnesses are thus not sufficient. The examining Judge has
to take depositions in writing of the complainant and the witnesses he may produce and to attach them to the
record. Such written deposition is necessary in order that the Judge may be able to properly determine the
existence or non-existence of the probable cause, to hold liable for perjury the person giving it if it will be found
later that his declarations are false.
A deposition is the testimony of a witness, put or taken in writing, under oath or affirmation before a
commissioner, examiner or other judicial officer, in answer to interlocutory and cross interlocutory and usually
subscribed by the witnesses. The examination or investigation which must be under oath may not be in public.
It may even be held in the secrecy of his chambers.

20TH CENTURY FOX FILM V. COURT OF APPEALS, AUGUST 19, 1988

FACTS:
Petitioner 20th Century Fox Film Corporation sought the assistance of the NBI in conducting searches
and seizures in connection with the NBI’s anti-film piracy campaign. Petitioner alleged that certain videotape
outlets all over Metro Manila are engaged in the unauthorized sale and renting out of copyrighted films in
violation of PD No. 49 (the old Intellectual Property Law).
The NBI conducted surveillance and investigation of the outlets pinpointed by the petitioner and
subsequently filed three (3) applications for search warrants against the video outlets owned by the private
respondents. The lower court issued the desired search warrants. The NBI, accompanied by the petitioner's
agents, raided the video outlets and seized the items described in the three warrants.
Private respondents later filed a motion to lift the search warrants and release the seized properties,
which was granted by the lower court. Petitioner’s motion for reconsideration was denied by the lower court.
The CA affirmed the trial court.

ISSUE:
Whether or not the judge properly lifts the search warrants he issued?

HELD:
The Court DISMISSED the petition and AFFIRMED the questioned decision and resolution of the
Court of Appeals.
YES, the judge properly lifted the search warrants he issued.
The lower court lifted the three (3) questioned search warrants in the absence of probable cause that the
private respondents violated P.D. 49. NBI agents who acted as witnesses during the application for search
95
warrant did not have personal knowledge of the subject matter of their testimony, which was the alleged
commission of the offense of piracy by the private respondents. Only the petitioner’s counsel who was also a
witness during the application stated that he had personal knowledge that the confiscated tapes owned by the
private respondents were pirated tapes taken from master tapes belonging to the petitioner. The lower court
lifted the warrants, declaring that the testimony of petitioner’s counsel did not have much credence because the
master tapes of the allegedly pirated tapes were not shown to the court during the application.
The presentation of the master tapes of the copyrighted films, from which the pirated films were
allegedly copied, was necessary for the validity of search warrants against those who have in their possession
the pirated films. The petitioner's argument to the effect that the presentation of the master tapes at the time of
application may not be necessary as these would be merely evidentiary in nature and not determinative of
whether or not a probable cause exists to justify the issuance of the search warrants is not meritorious. The
court cannot presume that duplicate or copied tapes were necessarily reproduced from master tapes that it
owns.
The essence of a copyright infringement is the similarity or at least substantial similarity of the
purported pirated works to the copyrighted work. Hence, the applicant must present to the court the
copyrighted films to compare them with the purchased evidence of the video tapes allegedly pirated to
determine whether the latter is an unauthorized reproduction of the former. This linkage of the copyrighted
films to the pirated films must be established to satisfy the requirements of probable cause. Mere allegations as
to the existence of the copyrighted films cannot serve as basis for the issuance of a search warrant.

LUNA VS. PLAZA


26 SCRA 310 (1968)

FACTS:
This is an appeal from the decision of CFI Surigao del Sur dismissing the petition for a writ of habeas
corpus filed by herein petitioner-appellant Simon LUNA who was charged with murder.
This petition stemmed from a criminal action commenced by the Phil. Constabulary investigator of
Tandag, Surigao del Sur when he filed with respondent MTC Judge Lorenzo PLAZA a criminal case charging
LUNA with the crime of murder.
Supporting the complaint/information were sworn statements of the witnesses for the prosecution,
subscribed and sworn to before the respondent Judge at the time of the filing of the complaint.
Judge PLAZA examined the prosecution witnesses by reading to them "all over again the questions and
answers" in their statements in writing, and the witnesses-affiants declared before said Judge that the
averments contained in their affidavits were made by them. They also swore to the truth thereof. Thereafter,
they signed their respective affidavits in the presence of Judge PLAZA, who also signed after the usual
procedure of administering the oath.
Considering the averments in these affidavits, the stipulation in the complaint/information and on the
strength of other documentary evidence submitted to him (medico-legal report, sketches of the crime scene,
etc.), Judge PLAZA found probable cause and issued the warrant of arrest, specifying therein that no bail is
fixed for the provisional release of the accused.
However, upon motion from LUNA, Judge PLAZA granted bail at P30,000, which, however was later
revoked, in effect again denying LUNA bail.
Thereafter, LUNA waived his right to PI. Hence, the case was remanded to the CFI of Surigao del Sur
for hearing after the Provincial Fiscal filed the necessary information charging LUNA with the crime of murder.
On April 1967, LUNA filed a petition for a writ of habeas corpus with the CFI of Surigao del Sur,
claiming that he was being deprived of liberty without the due process of law, on the ground that the
imprisonment and detention was the result of a warrant of arrest issued by respondent Judge in violation of RA
3828, and praying for the annulment of the order for his arrest and his discharge from confinement.
Herein respondents (Judge PLAZA, the jail warden and the city fiscal) all answered said petition and
alleged that the provisions of RA 3828 (Amendment to the Judiciary Act) have been complied with.
Respondents also averred that a motion to quash, and not a petition for habeas corpus was the proper
remedy for LUNA; but that LUNA's application for bail constituted a waiver of the right to question the validity
of the arrest.

96
As stated, the CFI ruled against LUNA and rendered herein assailed decision denying his petition for
habeas corpus, hence this appeal.

ISSUE: WON CFI erred in denying the writ of habeas corpus

HELD: NO. LUNA’s appeal is devoid of merit. Perforce, the decision of the CFI of Surigao del Sur is
AFFIRMED.

RATIO: On Compliance with RA 3828 and the Constitution


In this appeal, LUNA assails the conduct of the “preliminary examination” by Judge PLAZA as being
irregular as far as RA 3828 is concerned, hence, the warrant of arrest which issued therefrom is also irregular
this is UNTENABLE.
According to RA 3828 (Judiciary Act), judges doing preliminary examination for purposes of issuing
warrants of arrests are required to personally “examine” (i.e. ask searching questions) the complainant and her
witnesses under oath; and then reduce the same into writing (i.e. to make part of the records such
examination).
On the other hand, Art III, Sec 1 (3) of the 1935 Const. provides that no warrant shall issue but upon
probable cause, to be determined by the judge after examination under oath or affirmation of the complainant
and the witnesses he may produce.
A perusal of the records of the case would lead to the conclusion that Judge PLAZA substantially
complied with the preceding requirements of the Constitution and the law (refer to statement of facts).

No Denial of Due Process and Waiver of Rights.


Preliminary examination/investigation is not an essential part of due process of law. This is evidenced
by the fact that our rules of criminal procedure allow its waiver. Therefore, when it is forgone for the reason
thus stated, the same does not constitute a denial of due process. It is merely a statutory right.
As borne by the records, LUNA waived his right to the preliminary examination when he filed a petition
for bail.
Consequently, this conduct of LUNA indicates that he had waived his objection to whatever defect, if
any, in the preliminary examination conducted by respondent Judge PLAZA prior to the issuance of the
warrant of arrest.
Also, no substantial right of LUNA had been violated precisely because he has waived his right to
preliminary investigation after he was arrested, and that he took the step of applying for bail before respondent
Municipal Judge constitute an implied admission on his part that here was a probable cause for the issuance of
the warrant of arrest against him. Those acts of the petitioner constitute a waiver of whatever irregularity, if
any there was, that attended his arrest.

Quashal and not Habeas Corpus is the Proper Remedy


The remedy available to the petitioner LUNA, under the circumstances of this case, is not a petition for
a writ of habeas corpus but a petition to quash the warrant of arrest or a petition for a reinvestigation of the
case by the respondent Municipal Judge PLAZA or by the Provincial Fiscal.

JOSEFINO S. ROAN VS. HONORABLE ROMULO T. GONZALES, NOVEMBER 25, 1986

FACTS:
A search warrant was issued by respondent judge Romulo T. Gonzales on May 10, 1984. The application
for the said search warrant was personally filed by PC Capt. Mauro Quillosa. Together with Quillosa were two
witnesses, Esmael Morada and Jesus Tohilida, who presented to respondent judge their respective affidavits
taken by police investigator Pat. Josue V. Lining. The application was not yet subscribed and sworn to, as such
respondent judge proceeded to examine Quillosa on the contents of the application to ascertain if he knew and
understood the same. Afterwards, Quillosa subscribed and swore the said application before respondent judge.
Roan’s house was searched two days after the issuance of the search warrant. The said search was
conducted by military authorities. Despite none of the articles listed in the warrant was discovered, the officers

97
who performed the search found one Colt Magnum revolver and 18 live bullets which they confiscated. The said
items served as bases for the charge of illegal possession of firearms against the petitioner.

ISSUE:
Whether or not the search warrant be annulled on the ground that it Violates the privacy of one
person’s house.

HELD:
Yes. To be valid, a search warrant must be supported by probable cause to be determined by the judge
or some authorized officer after examining the complainant and the witnesses he may produce. There must be
a specific description of the place to be searched and the things to be seized, to prevent arbitrary and
indiscriminate use of the warrant. Probable cause, as described by Judge Escolin in Burgos v. Chief of staff,
refers to “such facts and circumstances which would lead a reasonably discreet and prudent man to believe that
an offense has been committed and that the objects sought in connection with the offense are in the place
sought to be searched”. The probable cause must refer to only one specific offense.
Capt. Quillosa was asking for the issuance of the search warrant on the basis of mere hearsay and not of
information personally known to him as required by settled jurisprudence. It is axiomatic that the magistrate
must be probing and exhaustive, not merely routinary or pro-forma, if the claimed probable cause is to be
established. The examining magistrate must not simply rehash the contents of the affidavit but must make his
own inquiry on the intent and justification of the application.
Prohibited articles may be seized but only as long as the search is valid. In this case, it was not because
there was no valid search warrant and absent of such warrant, the right thereto was not validly waived by the
petitioner. In short, the military officers who entered the petitioner’s premises had no right to be there and
therefore had no right to seize the pistol and bullets.

NEMESIO PRUDENTE v. HON JUDGE ABELARDO M. DAYRIT, DECEMBER 14, 1989

FACTS:
The Chief of the Intelligence Special Action Division (ISAD) filed with the Regional Trial Court (RTC)
Manila, Judge Abelardo Dayrit, for the issuance of Search Warrant for violation of PD No. 1866 (Illegal
Possession of Firearm, etc). In the deposition of witness (P/Lt. Florencio C. Angeles), it was made mentioned of
“result of our continuous surveillance conducted for several days. We gathered information from verified
sources that the holders of said firearms and explosives as well as ammunitions aren’t licensed to possess said
firearms and ammunition. Further, the premise is a school and the holders of these firearms are not student
who were not supposed to possess firearms, explosives and ammunitions.
Person to be searched in Nemesio Prudente at the Polytechnic University of the Philippines, Sta. Mesa,
Sampaloc, Manila, has in his control or possession firearms, explosives hand grenades and ammunitions which
are illegally possesses at the office of Department of Military Science and Tactics and at the office of the
President.
1. Petitioner moved to quash the Search Warrant. He claimed that:
2. Petitioners, had no personal knowledge of the facts
3. The examination of the said witness was not in form of searching questions and answers
4. Search warrant was a general warrant
5. Violation of Circular No. 19 of the Supreme Court in that the complainant failed to allege under oath
that the issuance of the search warrant on a Saturday, urgent.

ISSUE:
Whether or not the search and seizure was valid?

HELD:
Search Warrant annulled and set aside.
Valid search warrant to issue, there must be probable cause, which is to be determined personally by
the Judge, after examination under oath and affirmation of the complainant, and that witnesses he may
produce and particularly describing the place to be searched and the persons and things to be seized. The
98
probable cause must be in connection with one specific offense and the Judge must, before issuing Search
Warrant, personally examine in the form of searching questions and answers, in writing and under oath, the
complainant and any witnesses he may produce, on facts personally known to them and attach to the record
their sworn statements together with any affidavits submitted.
“Probable Cause” for a valid search warrant, has been defined as such facts and circumstances which
would lead a reasonably discreet and prudent man to believe that an offense has been committed, and that
objects sought in connection which the offense is in the place sought to be searched.
This probable cause must be shown to be personal knowledge and of the complainant and witnesses he
may produce and not based on mere hearsay.

CYNTHIA D. NOLASCO, ET AL. vs. ERNANI CRUZ PAÑO, et al.


OCTOBER 8, 1985

FACTS:
The present case was subject for resolution.
Supreme Court held in a criminal case that the arrest of the petitioners was illegal, annulling the
decision of respondent Judge Paño, and that the seizure of the items by virtue of the warrant by the same
respondent judge are inadmissible as evidence in the Subversive Documents case. However, the Court held that
the items were to be retained in case it would be used as evidence in a separate criminal case pending before
the Special Military Commission No.1, returning the rest which are determined irrelevant by petitioner.
Petitioners questioned the portion of the decision regarding the retention of the properties seized. One
of the petitioners also assailed the respondent’s claim that the search was incidental to her arrest for the crime
of rebellion.

ISSUE:
Whether or not some of the properties seized may be introduced as evidence in a separate criminal case.

HELD:
The Court ruled the propriety of the declaration of the arrest and search as null and void. It was held
that the warrant was one of a general warrant issued in gross violation of the constitutional mandate against
unreasonable searches and seizures. The Bill of rights also orders the absolute exclusion of all illegally obtained
evidence: “Any evidence obtained in violation of this section shall be inadmissible for any purpose in any
proceeding” (Sec. 4[2]).
Since the search was not an incident of an arrest as it was in fact made under a void general warrant,
the seizure of documents could not be justified as an incident of an arrest. Therefore, the Court ordered the
return of all seized items to petitioners.

99
*Search Incidental to Valid Arrest:
Rule 126, Sec. 12
Rule 113, Sec. 5 and Sec. 8
Umil, et al. v. Ramos, et al. (en banc)
G.R. No. 81567, 9 July 1990 (Decision)
187 SCRA 311

FACTS:
On 1 February 1988, the Regional Intelligence Operations Unit of the Capital Command (RIOU-
CAPCOM) received confidential information about a member of the NPA Sparrow Unit being treated for a
gunshot wound in St. Agnes Hospital, Quezon City. Upon verification, it was found that the wounded person
who was listed in the hospital records as Ronnie Javelon is actually petitioner Rolando Dural, a member of the
NPA liquidation squad responsible for the killing of two CAPCOM soldiers the day before in Bagong Barrio,
Caloocan City. In view of this verification, Rolando Dural was transferred to the Regional Medical Services of
the CAPCOM for security reasons. While confined thereat, he was positively identified by eyewitnesses as the
gunman who went on top of the hood of the CAPCOM mobile patrol car and fired at the two CAPCOM soldiers
seated inside. As a consequence of this positive identification, Rolando Dural alias Ronnie Javelon was
charged with the crime of "Double Murder with Assault Upon Agents of Persons in Authority." No bail was
recommended.
Meanwhile, on 6 February 1988, a petition for habeas corpus was filed with this Court on behalf of
Roberto Umil, Rolando Dural, and Renato Villanueva. Thereafter, the parties were heard. However, Roberto
Umil and Renato Villanueva posted bail before the Regional Trial Court of Pasay City, where charges for
violation of the Anti-Subversion Act had been filed against them, and they were accordingly released. The
petition for habeas corpus, insofar as Umil and Villanueva are concerned, is now moot and academic and is
accordingly dismissed, since the writ of habeas corpus does not lie in favor of an accused in a criminal case
who has been released on bail.
As to Rolando Dural, it clearly appears that he was not arrested while in the act of shooting the two (2)
CAPCOM soldiers aforementioned. Nor was he arrested just after the commission of the said offense for his
arrest came a day after the said shooting incident. Seemingly, his arrest without warrant is unjustified. Hence,
this petition.

ISSUE/S:
Whether or not Rolando Dural was lawfully arrested.

RULING/S:
Yes, Rolando Dural was lawfully arrested.
Under the law, the crimes of rebellion, subversion, conspiracy or proposal to commit such crimes, and
crimes or offenses committed in furtherance thereof or in connection therewith constitute direct assaults
against the State and are in the nature of continuing crimes.
In the case at bar, Rolando Dural was arrested for being a member of the New People’s Army (NPA),
an outlawed subversive organization. Subversion being a continuing offense, the arrest of Rolando Dural
without warrant is justified as it can be said that he was committing an offense when arrested. The record,
moreover, shows that the criminal case filed against Rolando Dural and Bernardo Itucal, Jr. for "Double
Murder, etc." was tried in the court below and at the conclusion thereof, Rolando Dural and Bernardo Itucal, Jr.
were found guilty of the charge and sentenced accordingly. Rolando Dural is now serving the sentence
imposed upon him by the trial court. Thus, the writ of habeas corpus is no longer available to him.

DISPOSITIVE PORTION:
In the Supreme Court’s decision dated 9 July 1990, the petition was DISMISSED. No Costs.

Umil, et al. v. Ramos, et al. (en banc)


100
G.R. No. 81567, 3 October 1991 (Resolution)
202 SCRA 251

FACTS:
The petitioners were seeking reconsideration of the Court’s decision promulgated on 9 July 1990 (the
decision, for brevity) which dismissed the petition.

ISSUE/S:
1. Whether or not the assailed decision in upholding the validity of the questioned arrest made without
warrant and in relying on the provisions of the Rules of Court, particularly Section 5 of Rule 113
(Arrest), disregards the fact that such arrest violated the constitutional rights of the persons arrested;
and
2. Whether or not the assailed decision is based on a misappreciation of facts;

RULING/S:
No, the Court did not find merit in the motion for reconsideration.

1. Under Section 5, Rule 113 of the Rules of Court, a valid arrest without warrant can be conducted.
The Court's decision of 9 July 1990 ruled that the arrest of Rolando Dural without warrant is
justified as it can be said that, within the contemplation of Section 5 Rule 113, he (Dural) was
committing an offense, when arrested because Dural was arrested for being a member of the New
People's Army, an outlawed organization where membership penalized, and for subversion which, like
rebellion is, under the doctrine of Garcia vs. Enrile, a continuing offense. Subversion and rebellion are
anchored on an ideological base which compels the repetition of the same acts of lawlessness and
violence until the overriding objective of overthrowing organized government is attained.

2. Under the law, "personal knowledge of facts," in arrests without warrant must be based upon probable
cause, which means an actual belief or reasonable grounds of suspicion. The grounds of suspicion are
reasonable when, in the absence of actual belief of the arresting officers, the suspicion that the person
to be arrested is probably guilty of committing the offense, is based on actual facts. A reasonable
suspicion therefore must be founded on probable cause, coupled with good faith on the part of the
peace officers making the arrest.
These requisites are complied with in the case at bar. Said confidential information received by
the arresting officers, to the effect that an NPA member ("sparrow unit") was being treated for a
gunshot wound in the named hospital, is deemed reasonable and with cause as it was based on actual
facts and supported by circumstances sufficient to engender a belief that an NPA member was truly in
the said hospital. The actual facts supported by circumstances are: first — the day before, two (2)
CAPCOM soldiers were actually killed in Bagong Bario, Caloocan City by five (5) "sparrows" including
Dural; second — a wounded person listed in the hospital records as "Ronnie Javellon" was actually
then being treated in St. Agnes Hospital for a gunshot wound; third — as the records of this case
disclosed later, "Ronnie Javellon" and his address entered in the hospital records were fictitious and the
wounded man was in reality Rolando Dural.

In fine, the confidential information received by the arresting officers merited their immediate attention
and action and, in fact, it was found to be true. As to the condition that "probable cause" must also be coupled
with acts done in good faith by the officers who make the arrest, the Court notes that the peace officers who
arrested Dural are deemed to have conducted the same in good faith, considering that law enforcers are
presumed to regularly perform their official duties. The records show that the arresting officers did not appear
to have been ill-motivated in arresting Dural.
It is therefore clear that the arrest, without warrant of Dural was made in compliance with the
requirements of paragraphs (a) and (b) of Section 5, Rule 113.

DISPOSITIVE PORTION:

101
The Supreme Court DENIED the motion for reconsideration of the decision dated 9 July 1990. This
denial is FINAL.

Roldan, Jr., et al. v. Arca, et al.


G.R. No. L-25434, 25 July 1975 (Decision)
Makasiar, J.:

FACTS:
On April 3, 1964, respondent company filed a civil case against petitioner Fisheries Commissioner
Arsenio N. Roldan, Jr., for the recovery of fishing vessel Tony Lex VI (one of two fishing boats in question)
which had been seized and impounded by petitioner Fisheries Commissioner through the Philippine Navy. The
CFI Manila granted it, thus respondent company took to possession of the vessel Tony Lex VI. Petitioner
requested the Philippine Navy to apprehended vessels Tony Lex VI and Tony Lex III, also respectively called
Srta. Winnie and Srta. Agnes, for alleged violations of some provisions of the Fisheries Act. On August 5, or 6,
1965, the two fishing boats were actually seized for illegal fishing with dynamite.
The Fiscal filed an ex parte motion to hold the boats in custody as instruments and therefore evidence
of the crime, and cabled the Fisheries Commissioner to detain the vessels. On October 2 and 4, likewise, the
Court of First Instance of Palawan ordered the Philippine Navy to take the boats in custody. On 18 October
1965, respondent Judge Francisco Arca issued an order granting the issuance of the writ of preliminary
mandatory injunction and issued the preliminary writ upon the filing by private respondent company of a bond
of P5,000.00 for the release of the two vessels. On 19 October 1965, the petitioners, the Commission and the
Navy, filed a motion for reconsideration of the order issuing the preliminary writ.
Judge Arca denied the said motion for reconsideration. The Commission and the Navy filed a motion
for certiorari and prohibition with preliminary injunction to restrain Judge Arca from enforcing his order dated 18
October 1965, and the writ of preliminary mandatory injunction there under issued.

ISSUE/S:
Whether or not the seizure of the vessel, its equipment and dynamites therein is valid.

RULING/S:
Yes, it is valid.
In the case of Papa v. Mago, 22 SCRA 857, search and seizure without search warrant of vessels and
air crafts for violations of the customs laws have been the traditional exception to the constitutional requirement
of a search warrant, because the vessel can be quickly moved out of the locality or jurisdiction in which the
search warrant must be sought before such warrant could be secured; hence it is not practicable to require a
search warrant before such search or seizure can be constitutionally effected. The same exception should
apply to seizures of fishing vessels breaching our fishery laws. They are usually equipped with powerful motors
that enable them to elude pursuing ships of the Philippine Navy or Coast Guard.
Another exception to the constitutional requirement of a search warrant for a valid search and seizure
as in the case of Alvero v. Dizon, 76 Phil. 637, is a search or seizure as an incident to a lawful arrest. Under
our Rules of Court, a police officer or a private individual may, without a warrant, arrest a person (a) who has
committed, is actually committing or is about to commit an offense in his presence; (b) who is reasonably
believed to have committed an offense which has been actually committed; or (c) who is a prisoner who has
escaped from confinement while serving a final judgment or from temporary detention during the pendency of
his case or while being transferred from one confinement to another (Sec. 6, Rule 113, Revised Rules of
Court).
In the case at bar, the members of the crew of the two vessels were caught in flagrante illegally fishing
with dynamite and without the requisite license. Thus, their apprehension without a warrant of arrest while
committing a crime is lawful. Consequently, the seizure of the vessel, its equipment and dynamites therein was
equally valid as an incident to a lawful arrest.

DISPOSITIVE PORTION:

102
In its 25 July 1975 decision, the Supreme Court GRANTED the petition and the order of respondent
Judge Arca dated 18 October 1965, the writ of preliminary mandatory injunction issued thereunder and the
order dated 23 November 1965, were SET ASIDE as null and void, with costs against private respondent.

People v. Aminnudin
G.R. No. 74869, 6 July 1988 (Decision)
Cruz, J.:

FACTS:
Idel Aminnudin was arrested on June 25, 1984, shortly after disembarking from the M/V Wilcon 9 in
Iloilo City. The PC (Philippine Constabulary) officers who were in fact waiting for him --- after they received a
tip from one of their informers that the accused was on board a vessel bound for Iloilo City and was carrying
marijuana --- simply accosted him, detained him, inspected his bag and finding three kilos of what were later
analyzed as marijuana leaves by the forensic examiner. On the basis of the finding, an information for violation
of the Dangerous Drugs Act was filed against Aminnudin.
In his defense, Aminnudin disclaimed the marijuana. He also alleged that he was arbitrarily arrested
and immediately handcuffed. His bag was confiscated without a search warrant. At the PC headquarters, he
was manhandled to force him to admit he was carrying the marijuana. He insisted he did not even know what
marijuana looked like and that his business was selling watches and sometimes cigarettes. He also argued
that the marijuana he was alleged to have been carrying was not properly identified and could have been any
of several bundles kept in the stock room of the PC headquarters.
The trial court, though, was unconvinced and also rejected his allegations of maltreatment, observing
that he had not sufficiently proved the injuries sustained by him. Hence, this petition.

ISSUE/S:
Whether or not the accused Aminnudin’s constitutional right against unreasonable search and arrest
was violated.

RULING/S:
Yes, it was violated.
Under Section 2 of the Bill of Rights, the right of the people to be secure in their persons, houses,
papers and effects against unreasonable searches and seizures of whatever nature and for any purpose shall
be inviolable, and no search warrant or warrant of arrest shall issue except upon probable cause to be
determined personally by the judge after examination under oath or affirmation of the complainant and the
witnesses he may produce, and particularly describing the place to be searched and the persons or things to
be seized.
In the case at bar, there was no warrant of arrest or search warrant issued by a judge after personal
determination by him of the existence of probable cause, even though the present case presented no such
urgency. Contrary to the averments of the government, the accused-appellant was not caught in flagrante nor
was a crime about to be committed or had just been committed to justify the warrantless arrest allowed under
Rule 113 of the Rules of Court.
Hence, to all appearances, Amminudin was like any of the other passengers innocently disembarking
from the vessel. The said marijuana therefore could not be appreciated as evidence against the defendant, and
furthermore he is acquitted of the crime as charged.

DISPOSITIVE PORTION:
In its 6 July 1988 decision, the Supreme Court REVERSED the decision of the trial court and
ACQUITTED the accused-appellant.

Alih, et al. v. Castro, et al. (en banc)


G.R. No. L-69401, 23 June 1987 (Decision)
Cruz, J.:
103
FACTS:
On 25 November 1984, a contingent of more than two hundred soldiers, who were members of the
Philippine marines and defense forces, implemented a “zona” --- a rounding up of the people in a locality and
arresting the persons fingered by a hooded informer --- raided the compound occupied by petitioners Rizal Alih
and others in Zamboanga City in search of loose firearms, ammunition and explosives. A shoot-out ensued
after petitioners resisted the intrusion by the respondents, killing a number of men. The following morning, the
petitioners were arrested and subjected to finger-printing, paraffin-testing and photographing despite their
objection. Several kinds of rifle, grenades and ammunitions were also confiscated.
The petitioners filed for a petition for prohibition and mandamus with preliminary injunction and
restraining order to have the arms and ammunition, which were taken without a search warrant as required by
the Bill of Rights, returned to them to prevent these from being used as evidence against them, and to
challenge their finger-printing, photographing and paraffin-testing as violative of their right against self-
incrimination, and invoked the provisions on the Bill of Rights. Hence, this petition.

ISSUE/S:
Whether or not the seizing of the items is violative of the Bill of Rights and are inadmissible as evidence
against them.

RULING/S:
Yes, it is violative of the Bill of Rights and are inadmissible as evidence.
Under Article IV, Section 3, of the 1973 Constitution, “the right of the people to be secure in their
persons, houses, papers, and effects against unreasonable searches and seizures of whatever nature and for
any purpose shall not be violated, and no search warrant or warrant of arrest shall issue except upon probable
cause to be determined by the judge, or such other responsible officer as may be authorized by law, after
examination under oath or affirmation of the complainant and the witnesses he may produce, and particularly
describing the place to be searched, and the persons or things to be seized.”
In the case at bar, when the respondents could have easily obtained a search warrant from any of the
TEN civil courts then open and functioning in Zamboanga City, they instead simply barged into the
beleaguered premises on the verbal order of their superior officers. It follows that as the search of the
petitioners' premises was violative of the Constitution, all the firearms and ammunition taken from the raided
compound are inadmissible in evidence in any of the proceedings against the petitioners. These articles are
"fruits of the poisonous tree.

DISPOSITIVE PORTION:
In the Supreme Court’s 23 June 1987 decision, the search of the petitioners' premises on November
25, 1984, was declared ILLEGAL and all the articles seized as a result thereof were inadmissible in evidence
against the petitioners in any proceedings. However, the said articles shall remain in custodia legis pending the
outcome of the criminal cases that have been or may later be filed against the petitioners.

Guazon, et al. v. De Villa, et al. (en banc)


G.R. No. 80508, 30 January 1990 (Decision)
Gutierrez, Jr., J.:

FACTS:
The 41 petitioners alleged that the "saturation drive" or "aerial target zoning" that were conducted in
their place (Tondo Manila) were unconstitutional. They alleged that there is no specific target house to be
search and that there is no search warrant or warrant of arrest served. Most of the policemen are in their
civilian clothes and without nameplates or identification cards. The residents were rudely rouse from their sleep
by banging on the walls and windows of their houses. The residents were at the point of high-powered guns
and herded like cows. Men were ordered to strip down to their briefs for the police to examine their tattoo
marks. The residents complained that their homes were ransacked, tossing their belongings and destroying
their valuables. Some of their money and valuables had disappeared after the operation. The residents also
104
reported incidents of mauling, spot-beatings and maltreatment. Those who were detained also suffered mental
and physical torture to extract confessions and tactical information.
The respondents said that such accusations were all lies. Respondents contends that the Constitution
grants to government the power to seek and cripple subversive movements for the maintenance of peace in
the state. The aerial target zoning was intended to flush out subversives and criminal elements coddled by the
communities were the said drives were conducted. They said that they have intelligently and carefully planned
months ahead for the actual operation and that local and foreign media joined the operation to witness and
record such event.
Hence, this petition.

ISSUE/S:
Whether or not there is impediment to securing search warrants or warrants of arrest.

RULING/S:
No, there is impediment to securing search warrants or warrants of arrest.
Under the provisions of Article IV of the 1973 Constitution: “SEC. 3. The right of the people to be secure
in their persons, houses, papers and effects against unreasonable searches and seizures of whatever nature
and for any purpose shall not be violated, and no search warrant or warrant of arrest shall issue except upon
probable cause to be determined by the judge, or such other responsible officer as may be authorized by law,
after examination under oath or affirmation of the complainant and the witnesses he may produce, and
particularly describing the place to be searched, and the persons or things to be seized.”
In the case at bar, where there is large scale mutiny or actual rebellion, the police or military may go in
force to the combat areas, enter affected residences or buildings, round up suspected rebels and otherwise
quell the mutiny or rebellion without having to secure search warrants and without violating the Bill of Rights.
The areal target zonings in this petition were intended to flush out subversives and criminal elements
particularly because of the blatant assassinations of public officers and police officials by elements supposedly
coddled by the communities where the "drives" were conducted.
It is clear from the pleadings of both petitioners and respondents, however, that there was no rebellion
or criminal activity similar to that of the attempted coup d' etats. There appears to have been no impediment to
securing search warrants or warrants of arrest before any houses were searched or individuals roused from
sleep were arrested. There is no strong showing that the objectives sought to be attained by the "areal zoning"
could not be achieved even as the rights of squatter and low income families are fully protected.
However, the remedy is not an original action for prohibition brought through a taxpayers' suit. Where
not one victim complains and not one violator is properly charged, the problem is not initially for the Supreme
Court. Under the circumstances of the taxpayers' suit, there is no erring soldier or policeman whom the Court
can order prosecuted. In the absence of clear facts ascertained through an orderly procedure, no permanent
relief can be given.

DISPOSITIVE PORTION:
In its 30 January 1990 decision, the Supreme Court REMANDED the Petition to the Regional Trial
Courts of Manila, Malabon, and Pasay City where the petitioners may present evidence supporting their
allegations and where specific erring parties may be pinpointed and prosecuted.
Copies of this decision were likewise forwarded to the Commission on Human Rights, the Secretary of
Justice, the Secretary of National Defense, and the Commanding General PC-INP for the drawing up and
enforcement of clear guidelines to govern police actions intended to abate riots and civil disturbances, flush out
criminal elements, and subdue terrorist activities.
In the meantime, the acts violative of human rights alleged by the petitioners as committed during the
police actions were ENJOINED until such time as permanent rules to govern such actions were promulgated.

Valmonte, et al. v. De Villa, et al. (en banc)


G.R. No. 83988, 29 September 1989 (Decision)
Padilla, J.:

105
FACTS:
On 20 January 1987, the National Capital Region District Command (NCRDC) was activated pursuant
to Letter of Instruction of the Philippine General Headquarters, AFP, with the mission of conducting security
operations within its area of responsibility and peripheral areas, for the purpose of establishing an effective
territorial defense, maintaining peace and order, and providing an atmosphere conducive to the social,
economic and political development of the National Capital Region. As part of its duty to maintain peace and
order, the NCRDC installed checkpoints in various parts of Valenzuela, Metro Manila.
Petitioners aver that, because of the installation of said checkpoints, the residents of Valenzuela are
worried of being harassed and of their safety being placed at the arbitrary, capricious and whimsical disposition
of the military manning the checkpoints, considering that their cars and vehicles are being subjected to regular
searches and check-ups, especially at night or at dawn, without the benefit of a search warrant and/or court
order.
Petitioners further contend that the said checkpoints give the respondents a blanket authority to make
searches and/or seizures without search warrant or court order in violation of the Constitution; and, instances
have occurred where a citizen, while not killed, had been harassed.

ISSUE/S:
Whether or not the installation of military and police checkpoints violates the right of the people against
unreasonable search and seizures.

RULING/S:
No, the installations of military and police checkpoints DO NOT violate the right of the people against
unreasonable search and seizures.
Under the law, the constitutional right against unreasonable searches and seizures is a personal right
invocable only by those whose rights have been infringed, or threatened to be infringed. Also, not all searches
and seizures are prohibited. Those which are reasonable are not forbidden. A reasonable search is not to be
determined by any fixed formula but is to be resolved according to the facts of each case.
In the present case, petitioners' concern for their safety and apprehension at being harassed by the
military manning the checkpoints are not sufficient grounds to declare the checkpoints as per se illegal. No
proof has been presented before the Court to show that, in the course of their routine checks, the military
indeed committed specific violations of petitioners' right against unlawful search and seizure or other rights.
The setting up of the questioned checkpoints in Valenzuela (and probably in other areas) may be
considered as a security measure to enable the NCRDC to pursue its mission of establishing effective
territorial defense and maintaining peace and order for the benefit of the public. Checkpoints may also be
regarded as measures to thwart plots to destabilize the government, in the interest of public security.
Between the inherent right of the state to protect its existence and promote public welfare and an
individual's right against a warrantless search which is however reasonably conducted, the former should
prevail.
True, the manning of checkpoints by the military is susceptible of abuse by the men in uniform, in the
same manner that all governmental power is susceptible of abuse. But, at the cost of occasional
inconvenience, discomfort and even irritation to the citizen, the checkpoints during these abnormal times, when
conducted within reasonable limits, are part of the price we pay for an orderly society and a peaceful
community.

DISPOSITIVE PORTION:
In the Supreme Court’s decision dated 29 September 1989, the petition was DISMISSED.

Valmonte, et al. v. De Villa, et al. (en banc)


G.R. No. 83988, 24 May 1990 (Resolution)
Padilla, J.:

FACTS:

106
Petitioners have filed the instant motion and supplemental motion for reconsideration of said decision
dated 29 September 1989. Before submission of the incident for resolution, the Solicitor General, for the
respondents, filed his comment, to which petitioners filed a reply.

ISSUE/S:
1. Whether or not routine checks can be regarded as violative of an individual's right against
unreasonable search.
2. Whether or not the checkpoints’ warrantless search and seizure is violative of the Constitution.

RULING/S:
No one can be compelled, under our libertarian system, to share with the present government its
ideological beliefs and practices, or commend its political, social and economic policies or performance. But, at
least, one must concede to it the basic right to defend itself from its enemies and, while in power, to pursue its
program of government intended for public welfare; and in the pursuit of those objectives, the government has
the equal right, under its police power, to select the reasonable means and methods for best achieving them.
The checkpoint is evidently one of such means it has selected.

1. No, routine checks cannot be regarded as violative of an individual's right against unreasonable search.
Admittedly, the routine checkpoint stop does intrude, to a certain extent, on motorist's right to
"free passage without interruption", but it cannot be denied that, as a rule, it involves only a brief
detention of travelers during which the vehicle's occupants are required to answer a brief question or
two. For as long as the vehicle is neither searched nor its occupants subjected to a body search, and
the inspection of the vehicle is limited to a visual search, said routine checks cannot be regarded as
violative of an individual's right against unreasonable search.

2. No, the checkpoints’ warrantless search and seizure is not violative of the Constitution.
As already stated, vehicles are generally allowed to pass these checkpoints after a routine
inspection and a few questions. If vehicles are stopped and extensively searched, it is because of some
probable cause which justifies a reasonable belief of the men at the checkpoints that either the motorist
is a law-offender or the contents of the vehicle are or have been instruments of some offense.
Besides these warrantless searches and seizures at the checkpoints are quite similar to
searches and seizures accompanying warrantless arrests during the commission of a crime, or
immediately thereafter.
By the same token, a warrantless search of incoming and outgoing passengers, at the arrival
and departure areas of an international airport, is a practice not constitutionally objectionable because it
is founded on public interest, safety, and necessity.

DISPOSITIVE PORTION:
The Supreme Court DENIED the Motion and Supplemental Motion for Reconsideration. This denial is
FINAL.

Bagalihog v. Fernandez, et al.


G.R. No. 96356, 27 June 1991 (Decision)
Cruz, J.:

FACTS:
On March 17, 1989, Rep. Moises Espinosa was shot to death shortly after disembarking at the Masbate
Airport. Witnesses said one of the gunmen fled on a motorcycle. On the same day, the petitioner's house,
which was near the airport, was searched with his consent to see if the killers had sought refuge there. The
search proved fruitless.
Two days later, Capt. Julito Roxas and his men from the Philippine Constabulary seized the petitioner's
motorcycle and took it to the PC headquarters in Masbate. They had no search warrant. The motorcycle was
impounded on the suspicion that it was one of the vehicles used by the killers.
107
After investigation, the petitioner and several others were charged with multiple murder and frustrated
murder for the killing of Espinosa and three of his bodyguards and the wounding of another person.
On June 21, 1989, petitioner Nonillon Bagalihog filed a complaint against Capt. Roxas for the recovery
of the motorcycle with an application for a writ of replevin, plus damages in the total amount of P55,000.00.
On November 7, 1989, the petitioner filed an urgent manifestation for the deposit of the motorcycle with
the clerk of court of the Regional Trial Court of Masbate, on the ground that PC soldiers were using the vehicle
without authority. The motion was granted on November 10, 1989, by Judge Ricardo Butalid.
Judge Butalid later inhibited himself and Civil Case No. 3878 was transferred to Branch 45, presided by
Judge Gil Fernandez. In the criminal cases, a change of venue was ordered by this Court from Branch 45 of
the Regional Trial Court of Masbate to Branch 56 of the Regional Trial Court of Makati. On October 12, 1990,
Judge Fernandez dismissed Civil Case No. 3878 as he had no jurisdiction over the motorcycle because it was
in custodia legis and only the judge trying the criminal cases (Makati RTC) could order its release.
Reconsideration having been denied, the petitioner now asks the Court to reverse the said order.
Hence, this petition.

ISSUE/S:
Whether or not the motorcycle was illegally seized.

RULING/S:
Yes, the motorcycle was illegally seized.
Under Article III, Section 2, of the Bill of Rights: “The right of the people to be secure in their persons,
houses, papers, and effects against unreasonable searches and seizures of whatever nature and for any
purpose shall be inviolable, and no search warrant or warrant of arrest shall issue except upon probable cause
to be determined personally by the judge after examination under oath or affirmation of the complainant and
the witnesses he may produce, and particularly describing the place to be searched and the persons or things
to be seized.”
In the case at bar, the warrantless seizure of the motorcycle was unquestionably violative of "the right
to be let alone" by the authorities as guaranteed by the Constitution. The vehicle cannot even be detained on
the ground that it is a prohibited article the mere possession of which is unlawful. The circumstance that Judge
Fernandez ordered the motorcycle to be deposited with the clerk of court on motion of the petitioner did not
place the vehicle in custodia legis. The respondent judge had no authority over it because it had not been
lawfully seized nor had it been voluntarily surrendered to the court by the petitioner. The private respondent
observed in his comment that "it is only when the exhibits are offered in evidence and admitted by the court
that they are submitted to the custody of the Court, and, before that, "they are usually in the possession of the
prosecution." Even he agrees therefore that the motorcycle is not in custodia legis.
At that, the vehicle in the case at bar is not admissible as an exhibit even if offered as such because it
is "the fruit of the poisonous tree." Under Article III, Sec. 3(2) "any evidence obtained in violation" of the rule
against unreasonable searches and seizure "shall be inadmissible for any purpose in any proceeding." Also,
the action to recover the motorcycle in the Regional Trial Court of Masbate will not constitute interference with
the processes of the Regional Trial Court of Makati and that, consequently, the complaint should not have
been dismissed by the respondent judge.

DISPOSITIVE PORTION:
The Supreme Court SET ASIDE the order of the respondent judge dated 12 October 1990 and
REINSTATED Civil Case No. 3878 for further proceedings. No costs.

People v. Balingan
G.R. No. 105834, 13 February 13 1995 (Decision)
Puno, J.:

FACTS:
On August 31, 1988, the Narcotics Intelligence Division of the Baguio City Police Station received a
telephone call from an unnamed male informant, alleging that appellant Balingan was going to Manila with a
108
bag filled with marijuana. Acting on such information, police officer Obrera conducted surveillance of petitioner
in different places, such as her house and at bus stations around the city. Upon receiving information that
petitioner boarded a Dangwa Bus, Obrera immediately went to the terminal to confirm the said report. He
boarded the bus and saw petitioner carrying a gray maleta. A prior checkpoint along Kennon Road was set-up
by the police in order to apprehend appellant. Upon arrival at the checkpoint, the bus stopped and yielded to
the police officers. Obrera announced a routine check on petitioner, but petitioner did not respond. The police
officers then grabbed the bag in the overhead compartment of appellant Balingan and opened it. Just as they
suspected, they found approximately 3 kilos of marijuana.
The police officers then requested Balingan to go with them to the police station. However, the
appellant resisted and bit one of the police officers. Eventually, after thirty minutes, they were able to pull
Balingan out of the bus and brought her to the Baguio City Police Station and locked her up in jail.
The marijuana confiscated were then submitted to the PNP Crime Laboratory for a confirmatory test on
the said prohibited drugs. The test yielded a positive result. Appellant alleged that she is innocent and that the
gray bag is not hers. Notwithstanding appellant’s protestations, the trial court found her guilty of the crime of
illegal transportation of prohibited drugs and was charged with a penalty of life imprisonment.

ISSUE/S:
Whether or not there was sufficient probable cause for the police officers to believe that she was then
and there committing a crime so as to justify the warrantless search and seizure of the bag.

RULING/S:
Yes, there was sufficient probable cause for the police officers to believe that she was then and there
committing a crime so as to justify the warrantless search and seizure of the bag.
The search and seizure in the case at bench happened in a moving public vehicle. The warrantless
search does not lack probable cause, as it was the “product” of the surveillance activities conducted by the
Narcotics Division of the Baguio City Police, revealing the appellant as the person that would be bringing the
said prohibited drug to Manila via bus. As the Court ruled, “when the moving public bus was stopped, her bag,
upon inspection, yielded marijuana. Under those circumstances, the warrantless search of appellant’s bag was
not illegal.”

DISPOSITIVE PORTION:
The Supreme Court affirmed in toto the conviction of appellant JEAN BOBBONAN BALINGAN. Cost
against appellant.

Papa, et al. v. Mago, et al. (en banc)


G.R. No. L-27360, 28 February 1968 (Decision)
Zaldivar, J.:

FACTS:
Mago, the owner of the goods that were seized, when the truck transporting the goods was intercepted
by the Bureau of Customs, questioned the validity of the search conducted by them since it was made without
any search warrant and whether the BOC has jurisdiction over the forfeited goods.

ISSUE/S:
Whether or not the search conducted by the Bureau of Customs was valid.

RULING/S:
Yes, it was valid.
Petitioner Martin Alagao and his companion policemen had authority to effect the seizure without any
search warrant issued by a competent court. The Tariff and Customs Code does not require said warrant in the
instant case. The Code authorizes persons having police authority under Section 2203 of the Tariff and
Customs Code to enter, pass through or search any land, enclosure, warehouse, store or building, not being a
dwelling house; and also to inspect, search and examine any vessel or aircraft and any trunk, package, or
109
envelope or any person on board, or to stop and search and examine any vehicle, beast or person suspected
of holding or conveying any dutiable or prohibited article introduced into the Philippines contrary to law, without
mentioning the need of a search warrant in said cases. But in the search of a dwelling house, the Code
provides that said "dwelling house may be entered and searched only upon warrant issued by a judge or
justice of the peace. . .." It is our considered view, therefore, that except in the case of the search of a dwelling
house, persons exercising police authority under the customs law may effect search and seizure without a
search warrant in the enforcement of customs laws.
In, Carroll vs US, it was made lawful for customs officers not only to board and search vessels within
their own and adjoining districts, but also to stop, search and examine any vehicle, beast or person on which or
whom they should suspect there was merchandise which was subject to duty, or had been introduced into the
United States in any manner contrary to law, whether by the person in charge of the vehicle or beast or
otherwise, and if they should find any goods, wares, or merchandise thereon, which they had probably cause
to believe had been so unlawfully brought into the country, to seize and secure the same, and the vehicle or
beast as well, for trial and forfeiture.

DISPOSITIVE PORTION:
The Supreme Court rendered the following decisions: (a) Granting the writ of certiorari and prohibition
prayed for by petitioners; (b) Declaring null and void, for having been issued without jurisdiction, the order of
respondent Judge Hilarion U. Jarencio, dated March 7, 1967, of the Court of First Instance of Manila; (c)
Declaring permanent the preliminary injunction issued by the Court on March 31, 1967 restraining respondent
Judge from executing, enforcing and/or implementing his order of March 7, 1967 of First Instance of Manila,
and from proceeding in any manner in said case; (d) Ordering the dismissal of Civil Case No. 67496 of the
Court of First Instance of Manila; and (e) Ordering the private respondent, Remedios Mago, to pay the costs.

Pasion Vda. de Garcia vs. Locsin


[GR 45950, 20 June 1938]
Laurel (J)

Facts: On 10 November 1934, Mariano G. Almeda, an agent of the Anti-Usury Board, obtained from the justice
of the peace of Tarlac, Tarlac, a search warrant commanding any officer of the law to search the person, house
or store of Leona Pasion Vda. de Garcia at Victoria, Tarlac, for "certain books, lists, chits, receipts, documents
and other papers relating to her activities as usurer." The search warrant was issued upon an affidavit given by
the said Almeda "that he has and there is just and probable cause to believe and he does believe that Leona
Pasion de Garcia keeps and conceals in her house and store at Victoria, Tarlac, certain books, lists, chits,
receipts, documents, and other papers relating to her activities as usurer, all of which is contrary to the statute
in such cases made and provided." On the same date, Almeda, accompanied by a captain of the Philippine
Constabulary, went to the office of Pasion de Garcia in Victoria, Tarlac and, after showing the search warrant to
the latter's bookkeeper, Alfredo Salas, and, without Pasion de Garcia's presence who was ill and confined at the
time, proceeded with the execution thereof. Two packages of records and a locked filing cabinet containing
several papers and documents were seized by Almeda and a receipt therefor issued by him to Salas. The papers
and documents seized were kept for a considerable length of time by the Anti-Usury Board and thereafter were
turned over by it to the provincial fiscal Felix Imperial, who subsequently filed, in the Court of First Instance
(CFI) of Tarlac, 6 separate criminal cases against Pasion de Garcia for violation of the Anti-Usury Law. On
several occasions, after seizure, Pasion de Garcia, through counsel, demanded from the Anti-Usury Board the
return of the documents seized. On January 7, and, by motion, on 4 June 1937, the legality of the search
warrant was challenged by Pasion de Garcia's counsel in the 6 criminal cases and the devolution of the
documents demanded. By resolution of 5 October 1937, Judge Diego Locsin (CFI) denied Pasion de garcia's
motion of June 4 for the reason that though the search warrant was illegal, there was a waiver on the latter's
part. A motion for reconsideration was presented but was denied by order of 3 January 1938. Pasion de Garcia
registered her exception.

Issue: Whether the lack of personal examination of witnesses renders the warrant void.

110
Held: Freedom from unreasonable searches and seizures is declared a popular right and for a search warrant
to be valid, (1) it must be issued upon probable cause; (2) the probable cause must be determined by the judge
himself and not by the applicant or any other person; (3) in the determination of probable cause, the judge
must examine, under oath or affirmation, the complainant and such witnesses as the latter may produce; and
(4) the warrant issued must particularly describe the place to be searched and persons or things to be seized.
These requirements are complemented by the Code of Criminal Procedure, particularly with reference to the
duration of the validity of the search warrant and the obligation of the officer seizing the property to deliver the
same to the corresponding court. Herein, the existence of probable cause was determined not by the judge
himself but by the applicant. All that the judge did was to accept as true the affidavit made by agent Almeda. He
did not decide for himself. It does not appear that he examined the applicant and his witnesses, if any. Even
accepting the description of the properties to be seized to be sufficient and on the assumption that the receipt
issued is sufficiently detailed within the meaning of the law, the properties seized were not delivered to the
court which issued the warrant, as required by law. Instead, they were turned over to the provincial fiscal and
used by him in building up cases against Pasion de Garcia. Considering that at the time the warrant was issued
there was no case pending against Pasion de Garcia, the averment that the warrant was issued primarily for
exploration purposes is not without basis. The search warrant was illegally issued by the justice of the peace of
Tarlac, Tarlac. In any event, the failure on the part of Pasion de Garcia and her bookkeeper to resist or object to
the execution of the warrant does not constitute an implied waiver of constitutional right. It is, as Judge Cooley
observes, but a submission to the authority of the law. As the constitutional guaranty is not dependent upon
any affirmative act of the citizen, the courts do not place the citizen in the position of either contesting an
officer's authority by force, or waiving his constitutional rights; but instead they hold that a peaceful
submission to a search or seizure is not a consent or an invitation thereto, but is merely a demonstration of
regard for the supremacy of the law.

Lopez vs. Commissioner of Customs


[GR L-27968, 3 December 1975]
Fernando (J)

Facts: M/V Jolo Lema had been under strict surveillance by the combined team of agents of the NBI, PC,
RASAC, and City Police of Davao prior to its apprehension at a private wharf in Batjak, Sasa, Davao City. M/V
[Jolo Lema] was skippered (sic) by Capt. Aquilino Pantinople and chartered by Mr. Tomas Velasco. During the
period from the latter part of August to September 18, 1966, the said vessel was in Indonesian waters where it
loaded copra and coffee beans from Taruna, Pitta, and Mangenito, all of Indonesia. In its trip to Indonesia it
brought various merchandise from the Philippines which were exchanged and/or bartered for copra and coffee
beans and subsequently taken to Davao City. Said vessel passed Marore, Indonesia on 18 September 1966 on its
a way to Tahuna, Indonesia before proceeding to Davao City where it was apprehended on 19 September 1966.
At about 3:00 p.m. of the said day, when the vessel was searched and after Captain Pantinople informed the
team that Velasco, the charterer of the vessel, had other documents showing that vessel came from Indonesia
carrying smuggled copra and coffee, a combined team of Constabulary and Regional Anti-Smuggling Center
operatives headed by Earl Reynolds, Senior NBI Agent of Davao, proceeded to the Velasco's room at the
Skyroom Hotel in Davao City, to ask for said document. Velasco was not inside the hotel room when they
entered the room. There are conflicting claims whether the manicurist Teofila Ibañez or whether Velasco's wife,
who was allegedly inside the room at that time, voluntarily allowed the police officers to enter; and whether the
police officers "forcibly opened luggages and boxes from which only several documents and papers were found,
then seized, confiscated and took away the same," or whether Mrs. Velasco volunteered to open the suitcases
and baggages of Velasco and delivered the documents and things contained therein to Reynolds. The Collector
of Customs of Davao seized 1,480 sacks of copra and 86 sacks of coffee from the M/V motor vessel Jolo Lema.
The seizure was declared lawful by the Court of Tax Appeals, and its decision was affirmed by the Supreme
Court on 29 November 1974 in Nasiad vs. Court of Tax Appeals (GR L-29318, November 29, 1974, 61 SCRA
238). In the present special civil action for certiorari, prohibition and mandamus; the only question left then is
whether the search conducted by a party headed by Reynolds without the search warrant for the hotel room of
Velasco, who entered into a contract with Jose G. Lopez, the awardee of such Philippine Reparations
Commission vessel, for its operation and use ostensibly for fishing, is violative of such constitutional provision.

111
Issue: Whether there was consent on the part of the person who was the occupant of the hotel room then
rented by Velasco.

Held: There was an attempt on the part of Lopez and Velasco to counteract the force of the recital of the
written statement of Teofila Ibañez (allegedly wife of Tomas Velasco) by an affidavit of one Corazon Y. Velasco,
who stated that she is the legal wife of Velasco, and another by Velasco himself; reiterating that the person who
was present at his hotel room was one Teofila Ibañez, "a manicurist by occupation." If such indeed were the
case, then it is much more easily understandable why that person, Teofila Ibañez, who could be aptly described
as the wrong person at the wrong place and at the wrong time, would have signified her consent readily and
immediately. Under the circumstances, that was the most prudent course of action. It would save her and even
Velasco himself from any gossip or innuendo. Nor could the officers of the law be blamed if they would act on
the appearances. There was a person inside who from all indications was ready to accede to their request. Even
common courtesy alone would have precluded them from inquiring too closely as to why she was there. Under
all the circumstances, therefore, it can readily be concluded that there was consent sufficient in law to dispense
with the need for a search warrant.

People vs. Damaso


[GR 93516, 12 August 1992]
Medialdea (J)

Facts: On 18 June 1988, Lt. Candido Quijardo, a Philippine Constabulary officer connected with the 152nd PC
Company at Lingayen, Pangasinan, and some companions were sent to verify the presence of CPP/NPA
members in Barangay Catacdang, Arellano-Bani, Dagupan City. In said place, the group apprehended Gregorio
Flameniano, Berlina Aritumba, Revelina Gamboa and Deogracias Mayaoa. When interrogated, the persons
apprehended revealed that there was an underground safehouse at Gracia Village in Urdaneta, Pangasinan.
After coordinating with the Station Commander of Urdaneta, the group proceeded to the house in Gracia
Village. They found subversive documents, a radio, a 1 x 7 caliber .45 firearm and other items. After the raid,
the group proceeded to Bonuan, Dagupan City, and put under surveillance the rented apartment of Rosemarie
Aritumba, sister of Berlina Aritumba whom they earlier arrested. They interviewed Luzviminda Morados, a
visitor of Rosemarie Aritumba. She stated that she worked with Bernie Mendoza/Basilio Damaso. She guided
the group to the house rented by Damaso(@Mendoza). When they reached the house, the group found that it
had already vacated by the occupants. Since Morados was hesitant to give the new address of Damaso
(@Mendoza), the group looked for the Barangay Captain of the place and requested him to point out the new
house rented by Damaso (@Mendoza). The group again required Morados to go with them. When they reached
the house, the group saw Luz Tanciangco outside. They told her that they already knew that she was a member
of the NPA in the area. At first, she denied it, but when she saw Morados she requested the group to go inside
the house. Upon entering the house, the group, as well as the Barangay Captain, saw radio sets, pamphlets
entitled "Ang Bayan," xerox copiers and a computer machine. They also found persons who were companions
of Luz Tanciangco (namely, Teresita Calosa, Ricardo Calosa, Marites Calosa, Eric Tanciangco and Luzviminda
Morados). The group requested the persons in the house to allow them to look around. When Luz Tanciangco
opened one of the rooms, they saw books used for subversive orientation, one M-14 rifle, bullets and
ammunitions, Kenwood radio, artificial beard, maps of the Philippines, Zambales, Mindoro and Laguna and
other items. They confiscated the articles and brought them to their headquarters for final inventory. They
likewise brought the persons found in the house to the headquarters for investigation. Said persons revealed
that Damaso (@Mendoza) was the lessee of the house and owned the items confiscated therefrom. Thus,
Basilio Damaso, was originally charged in an information filed before the Regional Trial Court of Dagupan City
with violation of Presidential Decree 1866 in furtherance of, or incident to, or in connection with the crime of
subversion, together with Luzviminda Morados y Galang @ Ka Mel, Teresita Calosa y Macabangon @ Ka
Tessie, Ricardo Calosa y Perez @ Ka Ric, Marites Calosa y Evangelista @ Ka Tess, Eric Tanciangco y Capira @
Ka Ric and Luz Tanciangco y Pencial @ Ka Luz. Such information was later amended to exclude all other
persons except Damaso from the criminal charge. Upon arraignment, Damaso pleaded not guilty to the crime
charged. Trial on the merits ensued. The prosecution rested its case and offered its exhibits for admission. The
defense counsel interposed his objections to the admissibility of the prosecution's evidence on grounds of its
being hearsay, immaterial or irrelevant and illegal for lack of a search warrant; and thereafter, manifested that
112
he was not presenting any evidence for the accused. On 17 January 1990, the trial court rendered its decision,
finding Damaso guilty beyond reasonable doubt, sentencing the latter to suffer the penalty of Reclusion
Perpetua and to pay the costs of the proceedings. Damaso appealed.

Issue: Whether there was waiver on the part of Damaso to allow the warrantless search of his house.

Held: Damaso was singled out as the sole violator of PD 1866, in furtherance of, or incident to, or in
connection with the crime of subversion. There is no substantial and credible evidence to establish the fact that
the appellant is allegedly the same person as the lessee of the house where the M-14 rifle and other subversive
items were found or the owner of the said items. Even assuming for the sake of argument that Damaso is the
lessee of the house, the case against him still will not prosper, the reason being that the law enforcers failed to
comply with the requirements of a valid search and seizure proceedings. The constitutional immunity from
unreasonable searches and seizures, being a personal one cannot he waived by anyone except the person whose
rights are invaded or one who is expressly authorized to do so in his or her . The records show that Damaso was
not in his house at that time Luz Tanciangco and Luz Morados, his alleged helper, allowed the authorities to
enter it. There is no evidence that would establish the fact that Luz Morados was indeed Damaso's helper or if it
was true that she was his helper, that Damaso had given her authority to open his house in his absence. The
prosecution likewise failed to show if Luz Tanciangco has such an authority. Without this evidence, the
authorities' intrusion into Damaso's dwelling cannot be given any color of legality. While the power to search
and seize is necessary to the public welfare, still it must be exercised and the law enforced without
transgressing the constitutional rights of the citizens, for the enforcement of no statute is of sufficient
importance to justify indifference to the basic principles of government. As a consequence, the search
conducted by the authorities was illegal. It would have been different if the situation here demanded urgency
which could have prompted the authorities to dispense with a search warrant. But the record is silent on this
point. The fact that they came to Damaso's house at nighttime, does not grant them the license to go inside his
house.

Aniag vs. Commission on Elections


[GR 104961, 7 October 1994]
Bellosillo (J)

Facts: In preparation for the synchronized national and local elections scheduled on 11 May 1992, the
Commission on Elections (COMELEC) issued on 11 December 1991 Resolution 2323 ("Gun Ban"),
promulgating rules and regulations on bearing, carrying and transporting of firearms or other deadly weapons,
on security personnel or bodyguards, on bearing arms by members of security agencies or police organizations,
and organization or maintenance of reaction forces during the election period. Subsequently, on 26 December
1991 COMELEC issued Resolution 2327 providing for the summary disqualification of candidates engaged in
gunrunning, using and transporting of firearms, organizing special strike forces, and establishing spot
checkpoints. On 10 January 1992, pursuant to the "Gun Ban," Mr. Serapio P. Taccad, Sergeant-at-Arms, House
of Representatives, wrote Congressman Francisc B. Aniag Jr., who was then Congressman of the 1st District of
Bulacan requesting the return of the 2 firearms issued to him by the House of Representatives. Upon being
advised of the request on 13 January 1992 by his staff, Aniag immediately instructed his driver, Ernesto
Arellano, to pick up the firearms from his house at Valle Verde and return them to Congress. Meanwhile, at
about 5:00 p,.m. of the same day, the Philippine National Police (PNP) headed by Senior Superintendent
Danilo Cordero set up a checkpoint outside the Batasan Complex some 20 meters away from its entrance.
About 30 minutes later, the policemen manning the outpost flagged down the car driven by Arellano as it
approached the checkpoint. They searched the car and found the firearms neatly packed in their gun cases and
placed in a bag in the trunk of the car. Arellano was then apprehended and detained. He explained that he was
ordered by Aniag to get the firearms from the house and return them to Sergeant-at Arms Taccad of the House
of Representatives. Thereafter, the police referred Arellano's case to the Office of the City Prosecutor for
inquest. The referral did not include Aniag as among those charged with an election offense. On 15 January
1992, the City Prosecutor ordered the release of Arellano after finding the latter's sworn explanation
meritorious. On 28 January 1992, the City Prosecutor invited Aniag to shed light on the circumstances
mentioned in Arellano's sworn explanation. Aniag not only appeared at the preliminary investigation to
113
confirm Arellano's statement but also wrote the City Prosecutor urging him to exonerate Arellano. He
explained that Arellano did not violate the firearms ban as he in fact was complying with it when apprehended
by returning the firearms to Congress; and, that he was Aniag's driver, not a security officer nor a bodyguard.
On 6 March 1992, the Office of the City Prosecutor issued a resolution which, among other matters,
recommended that the case against Arellano be dismissed and that the "unofficial" charge against Aniag be also
dismissed. Nevertheless, on 6 April 1992, upon recommendation of its Law Department, COMELEC issued
Resolution 92-0829 directing the filing of information against Aniag and Arellano for violation of Sec. 261, par.
(q), of BP 881 otherwise known as the Omnibus Election Code, in relation to Sec. 32 of RA 7166; and Aniag to
show cause why he should not be disqualified from running for an elective position, pursuant to COMELEC
Resolution 2327, in relation to Secs. 32, 33 and 35 of RA 7166, and Sec. 52, par. (c), of BP 881. On 13 April
1992, Aniag moved for reconsideration and to hold in abeyance the administrative proceedings as well as the
filing of the information in court. On 23 April 1992, the COMELEC denied Aniag's motion for reconsideration.
Aniag filed a petition for declaratory relief, certiorari and prohibition against the COMELEC.

Issue: Whether the search of Aniag’s car that yielded the firarms which were to be returned to the House of
Representatives within the purview of the exception as to the search of moving vehicles.

Held: As a rule, a valid search must be authorized by a search warrant duly issued by an appropriate authority.
However, this is not absolute. Aside from a search incident to a lawful arrest, a warrantless search had been
upheld in cases of moving vehicles and the seizure of evidence in plain view, as well as the search conducted at
police or military checkpoints which we declared are not illegal per se, and stressed that the warrantless search
is not violative of the Constitution for as long as the vehicle is neither searched nor its occupants subjected to a
body search, and the inspection of the vehicle is merely limited to a visual search. As there was no evidence to
show that the policemen were impelled to do so because of a confidential report leading them to reasonably
believe that certain motorists matching the description furnished by their informant were engaged in
gunrunning, transporting firearms or in organizing special strike forces. Nor was there any indication from the
package or behavior of Arellano that could have triggered the suspicion of the policemen. Absent such
justifying circumstances specifically pointing to the culpability of Aniag and Arellano, the search could not be
valid. The action then of the policemen unreasonably intruded into Aniag's privacy and the security of his
property, in violation of Sec. 2, Art. III, of the Constitution. Consequently, the firearms obtained in violation of
Aniag's right against warrantless search cannot be admitted for any purpose in any proceeding.

114
5. Arrest
*Rule 113, Rules of Court
GERONIMO v. RAMOS
G.R. No. 192793: February 22, 2011
PERALTA, J.:

Facts: Petitioner was proclaimed winner for the mayoralty race during the May 10, 2010 Automated Elections
for the Municipality of Cajidiocan, Province of Romblon. The proclamation was based on the COC, but without
the official signed Certificate of Canvass for Proclamation (COCP). This was done with the approval of the
Provincial Board of Canvassers (PBOC) Chairman. Subsequently, private respondent Nicasio Ramos, who was
also a mayoralty candidate in the same election, requested the COMELEC to conduct a manual reconciliation of
the votes cast. The COMELEC then issued Resolution No. 8923, granting said request. As for the COCP for the
previously proclaimed mayoralty and vice-mayoralty candidates, the total number of votes for each of the
candidates remained the same even after the manual reconciliation; hence, only the date was erased and
changed to read "May 20, 2010" to correspond with the date of the manual reconciliation.On May 27, 2010,
private respondent filed an election protest case against petitioner before the RTC. The following day, the court
sheriff went to petitioner's residence to serve summons with a copy of the petition. The Sheriff's Return of
Summons stated that the sheriff was able to serve Summons on petitioner by leaving the same and the attached
copy of the protest with a certain Gerry Rojas, who was then at petitioner's residence. Petitioner, together with
his counsel, Atty. Abner Perez, appeared in court and requested a copy of the summons with a copy of the
election protest. Petitioner filed a Motion to Admit Answer, to which was attached his Answer with Affirmative
Defense and Counterclaim. One of his affirmative defenses was that the electoral protest was filed out of time,
since it was filed more than ten (10) days after the date of proclamation of the winning candidate. The trial
court then issued an order finding the service of Summons on petitioner on May 28, 2010 as valid, and
declaring the Answer as filed out of time.
On July 12, 2010, petitioner filed an Omnibus Motion to: (1) Restore Protestee's Standing in Court; (2)
Motion for Reconsideration of the Order dated June 24, 2010; and (3) Suspend Proceedings Pending
Resolution of Falsification Case Before the Law Department of the COMELEC. However, the trial court denied
petitioner's Omnibus Motion. Hence, the present petition for certiorari and prohibition under Rule 65, alleging
that respondent judge acted without or in excess of jurisdiction or with grave abuse of discretion amounting to
lack or excess of jurisdiction in considering as valid, the Sheriff's Service of Summons on May 28, 2010 on a
person not residing in petitioner's residence. On the other hand, respondents pointed out that the petition for
certiorari should not be filed with this Court but with the COMELEC.

Issue: Whether the respondent judge acted without or in excess of jurisdiction or grave abuse of discretion
amounting to lack or excess of jurisdiction

Held: The decision of the COMELEC is sustained.

POLITICAL LAW jurisdiction of the COMELEC


In election cases involving an act or an omission of a municipal or a regional trial court, the petition
shall be filed exclusively with the Commission on Elections, in aid of its appellate jurisdiction.
The question then is, would taking cognizance of a petition for certiorari questioning an interlocutory
order of the regional trial court in an electoral protest case be considered in aid of the appellate jurisdiction of
the COMELEC? The Court finds in the affirmative. Interpreting the phrase "in aid of its appellate jurisdiction,"
the Court held in J.M. Tuason & Co., Inc. v. Jaramillo, et al. 118 Phil. 1022 (1963) that if a case may be appealed
to a particular court or judicial tribunal or body, then said court or judicial tribunal or body has jurisdiction to
issue the extraordinary writ of certiorari, in aid of its appellate jurisdiction. This was reiterated in De Jesus v.
Court of Appeals, G.R. No. 101630, August 24, 1992 where the Court stated that a court may issue a writ of
certiorari in aid of its appellate jurisdiction if said court has jurisdiction to review, by appeal or writ of error,
the final orders or decisions of the lower court.
Since it is the COMELEC which has jurisdiction to take cognizance of an appeal from the decision of the
regional trial court in election contests involving elective municipal officials, then it is also the COMELEC
115
which has jurisdiction to issue a writ of certiorari in aid of its appellate jurisdiction. Clearly, petitioner erred in
invoking this Court's power to issue said extraordinary writ.
The petition is dismissed.

Pagandaman v. Casar
G.R. No. 71782, April 14, 1988
NARVASA, J.:

Facts: The shooting incident by armed men in Lanao led to the issuance of a warrant of arrest. Petitioners
assert that the respondent Judge issued a warrant of arrest against fifty (50) “John Does” transgressing the
Constitutional provision requiring that such warrants should particularly describe the persons or things to be
seized.

Issue: Whether said warrant is valid

Held: No. Insofar as said warrant is issued against fifty (50) “John Does” not one of whom the witnesses to the
complaint could or would identify, it is of the nature of a general warrant, one of a class of writs long proscribed
as unconstitutional and once anathematized as “totally subversive of the liberty of the subject.”[30] Clearly
violative of the constitutional injunction that warrants of arrest should particularly describe the person or
persons to be seized,[31] the warrant must, as regards its unidentified subjects, be voided.
WHEREFORE, the warrant complained of is upheld and declared valid insofar as it orders the arrest of
the petitioners. Said warrant is voided to the extent that it is issued against fifty (50) “John Does.” The
respondent Judge is directed to forward to the Provincial Fiscal of Lanao del Sur the record of the preliminary
investigation of the complaint in Criminal Case No. 1748 of his court for further appropriate action.

Soliven v. Mckasiar
G.R. No. 82827 November 14, 1988
PER CURIAM:

Facts: Luis Beltran is among the petitioners in this case. He, together with others, was charged with libel by
the then president Corzaon Aquino. Cory herself filed a complaint-affidavit against him and others. Makasiar
averred that Cory cannot file a complaint affidavit because this would defeat her immunity from suit. He
grounded his contention on the principle that a president cannot be sued. However, if a president would sue
then the president would allow herself to be placed under the court’s jurisdiction and conversely she would be
consenting to be sued back. Also, considering the functions of a president, the president may not be able to
appear in court to be a witness for herself thus she may be liable for contempt.

Issue: Whether or not such immunity can be invoked by Beltran, a person other than the president.

Held: No. The rationale for the grant to the President of the privilege of immunity from suit is to assure the
exercise of Presidential duties and functions free from any hindrance or distraction, considering that being the
Chief Executive of the Government is a job that, aside from requiring all of the office-holder’s time, also
demands undivided attention.
But this privilege of immunity from suit, pertains to the President by virtue of the office and may be
invoked only by the holder of the office; not by any other person in the President’s behalf. Thus, an accused like
Beltran et al, in a criminal case in which the President is the complainant cannot raise the presidential privilege
as a defense to prevent the case from proceeding against such accused.
Moreover, there is nothing in our laws that would prevent the President from waiving the privilege.
Thus, if so minded the President may shed the protection afforded by the privilege and submit to the court’s
jurisdiction. The choice of whether to exercise the privilege or to waive it is solely the President’s prerogative. It
is a decision that cannot be assumed and imposed by any other person.

116
People v. Barros
[GR 90640, 29 March 1994]
Feliciano (J)

Facts: On 6 September 1987, M/Sgt. Francis Yag-as and S/Sgt. James Ayan, both members of the P.C.
Mountain Province Command, rode the Dangwa Bus bearing Plate ABZ-242 bound for Sabangan, Mountain
Province. Upon reaching Chackchakan, Bontoc, Mountain Province, the bus stopped and both M/Sgt. Yag-as
and S/Sgt. Ayan, who were seated at the back, saw Bonifacio Barros carrying a carton, board the bus and seated
himself on seat 18 after putting the carton under his seat. Thereafter, the bus continued and upon reaching
Sabangan, M/Sgt. Yag-as and S/Sgt. Ayan before they alighted, it being their station, called C2C [Fernando]
Bongyao to inspect the carton under seat 18. After C2C Bongyao inspected the carton, he found out that it
contained marijuana and he asked the passengers who the owner of the carton was but nobody answered.
Thereafter, C2C Bongyao alighted with the carton and S/Sgt. Ayan and C2C Bongyao invited Barros to the
detachment for questioning as the latter was the suspected owner of the carton containing marijuana. Upon
entering the detachment the carton was opened in the presence of Barros. When Barros denied ownership of
the carton of marijuana, the P.C. officers called for the bus conductor who pinpointed to Barros as the owner of
the carton of marijuana. Barros was charged with violating Section 4 of RA 6425, as amended (Dangerous
Drugs Act of 1972). After trial, the trial court convicted Bonifacio Barros of violation of Section 4 of RA 6425 as
amended and sentenced him to suffer the penalty of reclusion perpetua and to pay a fine of P20,000.00. Barros
appealed.

Issue: Whether the failure of the carton bearer to object to the search made in the moving vehicle, resulting to
his warrantless arrest, constitutes a waiver.

Held: The general rule is that a search and seizure must be carried out through or with a judicial warrant;
otherwise such search and seizure becomes "unreasonable" within the meaning of Section 2, Article III of the
1987 Constitution. The evidence secured thereby — i.e., the "fruits" of the search and seizure — will be
inadmissible in evidence "for any purpose in any proceeding." The requirement that a judicial warrant must be
obtained prior to the carrying out of a search and seizure is, however, not absolute. There are certain
exceptions recognized in our law, one of which relates to the search of moving vehicles. Peace officers may
lawfully conduct searches of moving vehicles — automobiles, trucks, etc. — without need of a warrant, it not
being practicable to secure a judicial warrant before searching a vehicle, since such vehicle can be quickly
moved out of the locality or jurisdiction in which the warrant may be sought. In carrying out warrantless
searches of moving vehicles, however, peace officers are limited to routine checks, that is, the vehicles are
neither really searched nor their occupants subjected to physical or body searches, the examination of the
vehicles being limited to visual inspection. When, however, a vehicle is stopped and subjected to an extensive
search, such a warrantless search would be constitutionally permissible only if the officers conducting the
search have reasonable or probable cause to believe, before the search, that either the motorist is a law offender
or the contents or cargo of the vehicle are or have been instruments or the subject matter or the proceeds of
some criminal offense. The Court has in the past found probable cause to conduct without a judicial warrant an
extensive search of moving vehicles in situations where (1) there had emanated from a package the distinctive
smell of marijuana; (2) agents of the Narcotics Command ("Narcom") of the Philippine National Police ("PNP")
had received a confidential report from informers that a sizeable volume of marijuana would be transported
along the route where the search was conducted; (3) Narcom agents were informed or "tipped off" by an
undercover "deep penetration" agent that prohibited drugs would be brought into the country on a particular
airline flight on a given date; (4) Narcom agents had received information that a Caucasian coming from
Sagada, Mountain Province, had in his possession prohibited drugs and when the Narcom agents confronted
the accused Caucasian, because of a conspicuous bulge in his waistline, he failed to present his passport and
other identification papers when requested to do so; and (5) Narcom agents had received confidential
information that a woman having the same physical appearance as that of the accused would be transporting
marijuana. Herein, there is nothing in the record that any circumstance which constituted or could have
reasonably constituted probable cause for the peace officers to search the carton box allegedly owned by
Barros. The testimony of the law enforcement officers who had apprehended the accused (M/Sgt. Francis Yag-
as and S/Sgt. James Ayan), and who had searched the box in his possession, (C2C Fernando Bongyao), simply
117
did not suggest or indicate the presence of any such probable cause. Further, The accused is not to be presumed
to have waived the unlawful search conducted on the occasion of his warrantless arrest "simply because he
failed to object." To constitute a waiver, it must appear first that the right exists; secondly, that the person
involved had knowledge, actual or constructive, of the existence of such a right; and lastly, that said person had
an actual intention to relinquish the right. The fact that the accused failed to object to the entry into his house
does not amount to a permission to make a search therein. As the constitutional guaranty is not dependent
upon any affirmative act of the citizen, the courts do not place the citizen in the position of either contesting an
officer's authority by force, or waiving his constitutional rights; but instead they hold that a peaceful
submission to a search or seizure is not a consent or an invitation thereto, but is merely a demonstration of
regard for the supremacy of the law. Courts indulge every reasonable presumption against waiver of
fundamental constitutional rights and that we do not presume acquiescence in the loss of fundamental rights.
Accordingly, the search and seizure of the carton box was equally non-permissible and invalid. The "fruits" of
the invalid search and seizure — i.e., the 4) kilos of marijuana — should therefore not have been admitted in
evidence against Barros.

Morano v. Vivo
G.R. No. L-22196 June 30, 1967
SANCHEZ, J.:

Facts: Chan Sau Wah, a Chinese citizen born in Fukien, China arrived in the Philippines on November 1961 to
visit her cousin, Samuel Lee Malaps. She left China and her children by a first marriage: Fu Tse Haw and Fu
Yan Kai both minors, in the care of neighbors in Fukien, China. Chan Sau wah arrived in the Philippines with
Fu Yan Fun, her minor son also by the first marriage. Chan Sau Wah and her minor son Fu Yan Fun were
permitted only into the Philippines under a temporary visitor's visa for two months and after they posted a
cash bond of 4,000 pesos. On January 1962, Chan Sau Wah married Esteban Morano, a native-born Filipino
citizen. Born to this union on September 1962 was Esteban Morano, Jr. To prolong their stay in the
Philippines, Chan Sau Wah and Fu Yan Fun obtained several extensions. The last extension expired on
September 10, 1962. In a letter dated August 31, 1962, the Commissioner of Immigration ordered Chan Sau
Wah and her son, Fu Yan Fun, to leave the country on or before September 10, 1962 with a warning that upon
failure so to do, he will issue a warrant for their arrest and will cause the confiscation of their bond.

Issue: Whether or Not the issuance of the warrant of arrest is unconstitutional.

Ruling: Chan Sau Wah entered the Philippines on a tourist-temporary visitor's visa. She is a non-immigrant.
Under Section 13 just quoted, she may therefore be admitted if she were a qualified and desirable alien and
subject to the provisions of the last paragraph of Section 9. Therefore, first, she must depart voluntarily to some
foreign country; second, she must procure from the appropriate consul the proper visa; and third, she must
thereafter undergo examination by the officials of the Bureau of Immigration at the port of entry for
determination of her admissibility in accordance with the requirements of the immigration Act.
Warrants of arrest may be issued by administrative authorities only for the purpose of carrying out a
final finding of a violation of law, like an order of deportation or an order of contempt, and not for the sole
purpose of investigation or prosecution. It is also held that the requirement of probable cause is not applicable
in deportation proceedings, which are not criminal in nature. The order of deportation is purely administrative,
its purpose being not punishment but the return to his country of the alien who has violated the conditions for
the admission to the local state.

HARVEY V. DEFENSOR-SANTIAGO 162 SCRA 840; G.R. NO. 82544; 28 JUN 1988

Facts: This is a petition for Habeas Corpus. Petitioners are the following: American nationals Andrew Harvey,
52 and Jonh Sherman 72. Dutch Citizen Adriaan Van Den Elshout, 58. All reside at Pagsanjan Laguna
respondent Commissioner Miriam Defensor Santiago issued Mission Orders to the Commission of
Immigration and Deportation (CID) to apprehended petitioners at their residences. The “Operation Report”
read that Andrew Harvey was found together with two young boys. Richard Sherman was found with two
118
naked boys inside his room. While Van Den Elshout in the “after Mission Report” read that two children of
ages 14 and 16 has been under his care and subjects confirmed being live-in for some time now.
Seized during the petitioner’s apprehension were rolls of photo negatives and photos of suspected child
prostitutes shown in scandalous poses as well as boys and girls engaged in sex. Posters and other literature
advertising the child prostitutes were also found.
Petitioners were among the 22 suspected alien pedophiles. They were apprehended 17 February1988
after close surveillance for 3 month of the CID in Pagsanjan, Laguna. 17 of the arrested aliens opted for self-
deportation. One released for lack of evidence, another charged not for pedophile but working with NO VISA,
the 3 petitioners chose to face deportation proceedings. On 4 March1988, deportation proceedings were
instituted against aliens for being undesirable aliens under Sec.69 of Revised Administrative Code.
Warrants of Arrest were issued 7March1988 against petitioners for violation of Sec37, 45 and 46 of
Immigration Act and sec69 of Revised Administrative Code. Trial by the Board of Special Inquiry III
commenced the same date. Petition for bail was filed 11March 1988 but was not granted by the Commissioner
of Immigration. 4 April1988 Petitioners filed a petition for Writ of Habeas Corpus. The court heard the case on
oral argument on 20 April 1988.

Issues:
1) Whether or Not the Commissioner has the power to arrest and detain petitioners pending
determination of existence of probable cause.
2) Whether or Not there was unreasonable searches and seizures by CID agents.
3) Whether or Not the writ of Habeas Corpus may be granted to petitioners.

Held: While pedophilia is not a crime under the Revised Penal Code, it violates the declared policy of the state
to promote and protect the physical, moral, spiritual and social well-being of the youth. The arrest of
petitioners was based on the probable cause determined after close surveillance of 3 months. The existence of
probable cause justified the arrest and seizure of articles linked to the offense. The articles were seized as an
incident to a lawful arrest; therefore, the articles are admissible evidences (Rule 126, Section12 of Rules on
Criminal Procedure).
The rule that search and seizures must be supported by a valid warrant of arrest is not an absolute rule.
There are at least three exceptions to this rule. 1.) Search is incidental to the arrest. 2.) Search in a moving
vehicle. 3.) Seizure of evidence in plain view. In view of the foregoing, the search done was incidental to the
arrest.
The filing of the petitioners for bail is considered as a waiver of any irregularity attending their arrest
and estops them from questioning its validity. Furthermore, the deportation charges and the hearing presently
conducted by the Board of Special Inquiry made their detention legal. It is a fundamental rule that habeas
corpus will not be granted when confinement is or has become legal, although such confinement was illegal at
the beginning.
The deportation charges instituted by the Commissioner of Immigration are in accordance with Sec37
(a) of the Philippine Immigration Act of 1940 in relation to sec69 of the Revised Administrative code. Section
37 (a) provides that aliens shall be arrested and deported upon warrant of the Commissioner of Immigration
and Deportation after a determination by the Board of Commissioners of the existence of a ground for
deportation against them. Deportation proceedings are administrative in character and never construed as a
punishment but a preventive measure. Therefore, it need not be conducted strictly in accordance with ordinary
Court proceedings. What is essential is that there should be a specific charge against the alien intended to be
arrested and deported. A fair hearing must also be conducted with assistance of a counsel if desired.
Lastly, the power to deport aliens is an act of the State and done under the authority of the sovereign
power. It a police measure against the undesirable aliens whose continued presence in the country is found to
be injurious to the public good and tranquility of the people.

CALLANTA VS. VILLANUEVA 77 SCRA 377; G.R. NOS. 24646 & L-24674; 20 JUN 1977

Facts: Two complaints for grave oral defamation were filed against Faustina Callanta. The City Judge of
Dagupan City, Felipe Villanueva, denied the motions to quash the complaints. Thus, petitioner Callanta
brought the suits for certiorari in the Supreme Court. Petitioner questions the validity of the issuance of
119
warrant of arrest by respondent, arguing that the City Fiscal should have conducted the preliminary
investigation. According to petitioner’s counsel, there was jurisdictional infirmity. After the issuance of the
warrants of arrest and the bail fixed at P600, petitioner posted the bail bond, thus obtaining her provisional
liberty. The City Fiscal in this case did not disagree with the judge’s investigation, and agreed with the
complaints filed.

Issue: Whether or Not petitioner’s contentions are to be given merit.

Held: Based on many precedent cases of the Supreme Court, “where the accused has filed bail and waived the
preliminary investigation proper, he has waived whatever defect, if any, in the preliminary examination
conducted prior to the issuance of the warrant of arrest”. In the case at bar, it is futile for the petitioner to
question the validity of the issuance of the warrant of arrest, because she posted the bail bond. Petitioner also
erred in arguing that only the City Fiscal can conduct a preliminary investigation. According to the Charter of
the City of Dagupan, “the City Court of Dagupan City may also conduct preliminary investigation for any
offense, without regard to the limits of punishment, and may release, or commit and bind over any person
charged with such offense to secure his appearance before the proper court”. Petition for certiorari is denied.
Restraining order issued by the Court is lifted and set aside.

120
6. Privacy of Communication and
Correspondence
SALCEDO-ORTANEZ V CA
G.R. No. 110662 | August 4, 1994 | J. Padilla

Facts:
Private respondent Rafael Ortanez filed with the Quezon City RTC a complaint for annulment of
marriage with damages against petitioner Teresita Salcedo-Ortanez, on grounds of lack of marriage license
and/or psychological incapacity of the petitioner.
Among the exhibits offered by private respondent were three (3) cassette tapes of alleged telephone
conversations between petitioner and unidentified persons.
Teresita submitted her Objection/Comment to Rafael’s oral offer of evidence. However, the trial court
admitted all of private respondent’s offered evidence and later on denied her motion for reconsideration,
prompting petitioner to file a petition for certiorari with the CA to assail the admission in evidence of the
aforementioned cassette tapes.
These tape recordings were made and obtained when private respondent allowed his friends from the
military to wiretap his home telephone.
CA denied the petition because (1) Tape recordings are not inadmissible per se. They and any other
variant thereof can be admitted in evidence for certain purposes, depending on how they are presented and
offered and on how the trial judge utilizes them in the interest of truth and fairness and the even handed
administration of justice; and (2) A petition for certiorari is notoriously inappropriate to rectify a supposed
error in admitting evidence adduced during trial. The ruling on admissibility is interlocutory; neither does it
impinge on jurisdiction. If it is erroneous, the ruling should be questioned in the appeal from the judgment on
the merits and not through the special civil action of certiorari. The error, assuming gratuitously that it exists,
cannot be any more than an error of law, properly correctible by appeal and not by certiorari.
Petitioner then filed the present petition for review under Rule 45 of the Rules of Court.

Issue:
1. W/N the recordings of the telephone conversations are admissible in evidence.
2. W/N the remedy of certiorari under Rule 65 of the Rules of Court was properly availed of by the
petitioner in the Court of Appeals.

Held:
1. No. Rep. Act No. 4200 entitled “An Act to Prohibit and Penalize Wire Tapping and Other Related
Violations of the Privacy of Communication, and for other purposes” expressly makes such tape
recordings inadmissible in evidence thus:
Sec. 1. It shall be unlawful for any person, not being authorized by all the parties to any private
communication or spoken word, to tap any wire or cable, or by using any other device or arrangement,
to secretly overhear, intercept, or record such communication or spoken word by using a device
commonly known as a Dictaphone or dictagraph or detectaphone or walkie-talkie or tape-recorder, or
however otherwise described. . ..
Sec. 4. Any communication or spoken word, or the existence, contents, substance, purport, or
meaning of the same or any part thereof, or any information therein contained, obtained or secured by
any person in violation of the preceding sections of this Act shall not be admissible in evidence in any
judicial, quasi-judicial, legislative or administrative hearing or investigation.
Absent a clear showing that both parties to the telephone conversations allowed the recording of
the same, the inadmissibility of the subject tapes is mandatory under Rep. Act No. 4200.

2. Yes and no. The extraordinary writ of certiorari is generally not available to challenge an
interlocutory order of a trial court. The proper remedy in such cases is an ordinary appeal from an
adverse judgment, incorporating in said appeal the grounds for assailing the interlocutory order.

121
However, where the assailed interlocutory order is patently erroneous and the remedy of appeal
would not afford adequate and expeditious relief, the Court may allow certiorari as a mode of redress.

Felipe Navarro vs Court of Appeals

FACTS: Enrique Lingan and Stanley Jalbuena, both radio reporters went to a police station to report for a
blotter. During the course, a heated argument arose between police officer Navarro and the two reporters.
Navarro then poked his cocked firearm on the face of Jalbuena. Lingan interfered, this then irked Navarro and
then and there hit Lingan with the handle of his pistol above the left eyebrow. This caused Lingan to fall on the
floor bloodied.

ISSUE: Whether or not there is an aggravating circumstance against Navarro due to the fact that he
committed such crime in the police station?

HELD: A police station is a place wherein public authorities such as policemen are engaged in the discharge of
their duties. Since Navarro, who is a cop, committed the crime inside the police station, an aggravating
circumstance is appreciated against him.

Cecilia Zulueta vs Court of Appeals and Alfredo Martin


(253 SCRA 699)
GR no. 107383 February 20, 1996

Facts:
Cecilia Zulueta is the Petitioner who offset the private papers of his husband Dr. Alfredo Martin. Dr.
Martin is a doctor of medicine while he is not in his house His wife took the 157 documents consisting of
diaries, cancelled check, greeting cards, passport and photograph, private respondents between her Wife and
his alleged paramours, by means of forcibly opened the drawers and cabinet. Cecilia Zulueta filed the papers
for the evidence of her case of legal separation and for disqualification from the practice of medicine against
her husband.
Dr. Martin brought the action for recovery of the documents and papers and for damages against
Zulueta, with the Regional Trial Court of Manila, Branch X. the trial court rendered judgment for Martin,
declaring him the capital/exclusive owner of the properties described in paragraph 3 of Martin’s Complaint or
those further described in the Motion to Return and Suppress and ordering Zulueta and any person acting in
her behalf to a immediately return the properties to Dr. Martin and to pay him P5,000.00, as nominal
damages; P5,000.00, as moral damages and attorney’s fees; and to pay the costs of the suit. On appeal, the
Court of Appeals affirmed the decision of the Regional Trial Court. Zulueta filed the petition for review with the
Supreme Court.

Issue:
The papers and other materials obtained from forcible intrusion and from unlawful means are
admissible as evidence in court regarding marital separation and disqualification from medical practice.

Ruling/Held:
The documents and papers are inadmissible in evidence. The constitutional injunction declaring “the
privacy of communication and correspondence to be inviolable is no less applicable simply because it is the
wife who thinks herself aggrieved by her husband’s infidelity, who is the party against whom the constitutional
provision is to be enforced.
The only exception to the prohibition in the Constitution is if there is a lawful order from a court or
when public safety or order requires otherwise, as prescribed by law. Any violation of this provision renders the
evidence obtained inadmissible for any purpose in any proceeding. The intimacies between husband and wife
do not justify any one of them in breaking the drawers and cabinets of the other and in ransacking them for any
telltale evidence of marital infidelity. A person, by contracting marriage, does not shed his/her integrity or his
right to privacy as an individual and the constitutional protection is ever available to him or to her. The law
122
insures absolute freedom of communication between the spouses by making it privileged. Neither husband nor
wife may testify for or against the other without the consent of the affected spouse while the marriage subsists.
Neither may be examined without the consent of the other as to any communication received in confidence by
one from the other during the marriage, save for specified exceptions. But one thing is freedom of
communication; quite another is a compulsion for each one to share what one knows with the other. And this
has nothing to do with the duty of fidelity that each owes to the other.

Alejano v. Cabuay, GR 160792, August 25, 2005

FACTS: A directive was issued to all Major Service Commanders to take into custody the military personnel
under their command who took part in the Oakwood incident. Petitioners filed a petition for habeas corpus
with SC. The SC issued a resolution, which required respondents to make a return of the writ and to appear and
produce the persons of the detainees before the CA. CA dismissed the petition because the detainees are
already charged of coup d’etat. Habeas corpus is unavailing in this case as the detainees’ confinement is under
a valid indictment.

ISSUE: What is the objective of the writ of habeas corpus?

HELD: The duty to hear the petition for habeas corpus necessarily includes the determination of the propriety
of the remedy. The remedy of habeas corpus has one objective: to inquire into the cause of detention of a
person. The purpose of the writ is to determine whether a person is being illegally deprived of his liberty. If the
inquiry reveals that the detention is illegal, the court orders the release of the person. If, however, the detention
is proven lawful, then the habeas corpus proceedings terminate. The use of habeas corpus is thus very limited.
It is not a writ of error. Neither can it substitute for an appeal.

123
7.Freedom of Expression, Right to Assembly and
Academic Freedom
FRANCISCO CHAVEZ
vs.
RAUL M. GONZALES, in his capacity as the Secretary of the Department of Justice; and NTC
G.R. No. 168338, February 15, 2008

FACTS: Sometime before 6 June 2005, the radio station dzMM aired the Garci Tapes where the parties to the
conversation discussed “rigging” the results of the 2004 elections to favor President Arroyo. On 6 June 2005,
Presidential spokesperson Bunye held a press conference in Malacañang Palace, where he played before the
presidential press corps two compact disc recordings of conversations between a woman and a man. Bunye
identified the woman in both recordings as President Arroyo but claimed that the contents of the second
compact disc had been “spliced” to make it appear that President Arroyo was talking to Garcillano.
However, on 9 June 2005, Bunye backtracked and stated that the woman’s voice in the compact discs
was not President Arroyo’s after all.3 Meanwhile, other individuals went public, claiming possession of the
genuine copy of the Garci Tapes. Respondent Gonzalez ordered the NBI to investigate media organizations
which aired the Garci Tapes for possible violation of Republic Act No. 4200 or the Anti-Wiretapping Law.
On 11 June 2005, the NTC issued a press release warning radio and television stations that airing the
Garci Tapes is a “cause for the suspension, revocation and/or cancellation of the licenses or authorizations”
issued to them. On 14 June 2005, NTC officers met with officers of the broadcasters group KBP, to dispel fears
of censorship. The NTC and KBP issued a joint press statement expressing commitment to press freedom.
On 21 June 2005, petitioner Francisco I. Chavez (petitioner), as citizen, filed this petition to nullify the
“acts, issuances, and orders” of the NTC and respondent Gonzalez (respondents) on the following grounds: (1)
respondents’ conduct violated freedom of expression and the right of the people to information on matters of
public concern under Section 7, Article III of the Constitution, and (2) the NTC acted ultra vires when it warned
radio and television stations against airing the Garci Tapes.

ISSUE: The principal issue for resolution is whether the NTC warning embodied in the press release of 11 June
2005 constitutes an impermissible prior restraint on freedom of expression.

1. Standing to File Petition


Petitioner has standing to file this petition. When the issue involves freedom of expression, as in
the present case, any citizen has the right to bring suit to question the constitutionality of a government
action in violation of freedom of expression, whether or not the government action is directed at such
citizen. Freedom of expression, being fundamental to the preservation of a free, open and democratic
society, is of transcendental importance that must be defended by every patriotic citizen at the earliest
opportunity.

2. Overview of Freedom of Expression, Prior Restraint and Subsequent Punishment


Freedom of expression is the foundation of a free, open and democratic society. Freedom of
expression is an indispensable condition8 to the exercise of almost all other civil and political rights.
Freedom of expression allows citizens to expose and check abuses of public officials. Freedom of
expression allows citizens to make informed choices of candidates for public office.
Section 4, Article III of the Constitution prohibits the enactment of any law curtailing freedom
of expression:
No law shall be passed abridging the freedom of speech, of expression, or the press, or
the right of the people peaceably to assemble and petition the government for redress of
grievances.

Thus, the rule is that expression is not subject to any prior restraint or censorship because the
Constitution commands that freedom of expression shall not be abridged. Over time, however, courts
have carved out narrow and well defined exceptions to this rule out of necessity.

124
The exceptions, when expression may be subject to prior restraint, apply in this jurisdiction to
only four categories of expression, namely: pornography, false or misleading advertisement, advocacy of
imminent lawless action, and danger to national security.
All other expression is not subject to prior restraint.
Expression not subject to prior restraint is protected expression or high-value expression. Any
content-based prior restraint on protected expression is unconstitutional without exception. A
protected expression means what it says – it is absolutely protected from censorship. Thus, there can be
no prior restraint on public debates on the amendment or repeal of existing laws, on the ratification of
treaties, on the imposition of new tax measures, or on proposed amendments to the Constitution.
If the prior restraint is not aimed at the message or idea of the expression, it is content-neutral
even if it burdens expression. A content-neutral restraint is a restraint which regulates the time, place
or manner of the expression in public places without any restraint on the content of the expression.
Courts will subject content-neutral restraints to intermediate scrutiny. An example of a content-neutral
restraint is a permit specifying the date, time and route of a rally passing through busy public streets. A
content-neutral prior restraint on protected expression which does not touch on the content of the
expression enjoys the presumption of validity and is thus enforceable subject to appeal to the courts.
Expression that may be subject to prior restraint is unprotected expression or low-value
expression. By definition, prior restraint on unprotected expression is content-based since the restraint
is imposed because of the content itself. In this jurisdiction, there are currently only four categories of
unprotected expression that may be subject to prior restraint. This Court recognized false or misleading
advertisement as unprotected expression only in October 2007.
Only unprotected expression may be subject to prior restraint. However, any such prior restraint
on unprotected expression must hurdle a high barrier. First, such prior restraint is presumed
unconstitutional. Second, the government bears a heavy burden of proving the constitutionality of the
prior restraint.
Prior restraint is a more severe restriction on freedom of expression than subsequent
punishment. Although subsequent punishment also deters expression, still the ideas are disseminated
to the public. Prior restraint prevents even the dissemination of ideas to the public.
While there can be no prior restraint on protected expression, such expression may be subject to
subsequent punishment, either civilly or criminally. Similarly, if the unprotected expression does not
warrant prior restraint, the same expression may still be subject to subsequent punishment, civilly or
criminally. Libel falls under this class of unprotected expression.
However, if the expression cannot be subject to the lesser restriction of subsequent punishment,
logically it cannot also be subject to the more severe restriction of prior restraint. Thus, since profane
language or “hate speech” against a religious minority is not subject to subsequent punishment in this
jurisdiction, such expression cannot be subject to prior restraint.
If the unprotected expression warrants prior restraint, necessarily the same expression is
subject to subsequent punishment. There must be a law punishing criminally the unprotected
expression before prior restraint on such expression can be justified.
The prevailing test in this jurisdiction to determine the constitutionality of government action
imposing prior restraint on three categories of unprotected expression – pornography,31 advocacy of
imminent lawless action, and danger to national security – is the clear and present danger test.32 The
expression restrained must present a clear and present danger of bringing about a substantive evil that
the State has a right and duty to prevent, and such danger must be grave and imminent.
Prior restraint on unprotected expression takes many forms – it may be a law, administrative
regulation, or impermissible pressures like threats of revoking licenses or withholding of benefits.34
The impermissible pressures need not be embodied in a government agency regulation, but may
emanate from policies, advisories or conduct of officials of government agencies.

3. Government Action in the Present Case


The government action in the present case is a warning by the NTC that the airing or
broadcasting of the Garci Tapes by radio and television stations is a “cause for the suspension,
revocation and/or cancellation of the licenses or authorizations” issued to radio and television stations.
The NTC warning, embodied in a press release, relies on two grounds. First, the airing of the Garci
Tapes “is a continuing violation of the Anti-Wiretapping Law and the conditions of the Provisional
125
Authority and/or Certificate of Authority issued to radio and TV stations.” Second, the Garci Tapes have
not been authenticated, and subsequent investigation may establish that the tapes contain false
information or willful misrepresentation.
The NTC does not claim that the public airing of the Garci Tapes constitutes unprotected
expression that may be subject to prior restraint. The NTC does not specify what substantive evil the
State seeks to prevent in imposing prior restraint on the airing of the Garci Tapes. The NTC does not
claim that the public airing of the Garci Tapes constitutes a clear and present danger of a substantive
evil, of grave and imminent character, that the State has a right and duty to prevent.
The NTC did not conduct any hearing in reaching its conclusion that the airing of the Garci
Tapes constitutes a continuing violation of the Anti-Wiretapping Law. There is also the issue of whether
a wireless cellular phone conversation is covered by the Anti-Wiretapping Law.
Clearly, the NTC has no factual or legal basis in claiming that the airing of the Garci Tapes
constitutes a violation of the Anti-Wiretapping Law. The radio and television stations were not even
given an opportunity to be heard by the NTC. The NTC did not observe basic due process as mandated
in Ang Tibay v. Court of Industrial Relations.
The NTC concedes that the Garci Tapes have not been authenticated as accurate or truthful. The
NTC also concedes that only “after a prosecution or appropriate investigation” can it be established that
the Garci Tapes constitute “false information and/or willful misrepresentation.” Clearly, the NTC
admits that it does not even know if the Garci Tapes contain false information or willful
misrepresentation.

4. Nature of Prior Restraint in the Present Case


The NTC action restraining the airing of the Garci Tapes is a content-based prior restraint
because it is directed at the message of the Garci Tapes. The NTC’s claim that the Garci Tapes might
contain “false information and/or willful misrepresentation,” and thus should not be publicly aired, is
an admission that the restraint is content-based.

5. Nature of Expression in the Present Case


The public airing of the Garci Tapes is a protected expression because it does not fall under any
of the four existing categories of unprotected expression recognized in this jurisdiction. The airing of
the Garci Tapes is essentially a political expression because it exposes that a presidential candidate had
allegedly improper conversations with a COMELEC Commissioner right after the close of voting in the
last presidential elections.
Obviously, the content of the Garci Tapes affects gravely the sanctity of the ballot. Public
discussion on the sanctity of the ballot is indisputably a protected expression that cannot be subject to
prior restraint. In any event, public discussion on all political issues should always remain uninhibited,
robust and wide open.
The rule, which recognizes no exception, is that there can be no content-based prior restraint on
protected expression. On this ground alone, the NTC press release is unconstitutional. Of course, if the
courts determine that the subject matter of a wiretapping, illegal or not, endangers the security of the
State, the public airing of the tape becomes unprotected expression that may be subject to prior
restraint. However, there is no claim here by respondents that the subject matter of the Garci Tapes
involves national security and publicly airing the tapes would endanger the security of the State.
The airing of the Garci Tapes does not violate the right to privacy because the content of the
Garci Tapes is a matter of important public concern. The Constitution guarantees the people’s right to
information on matters of public concern. The remedy of any person aggrieved by the public airing of
the Garci Tapes is to file a complaint for violation of the Anti-Wiretapping Law after the commission of
the crime. Subsequent punishment, absent a lawful defense, is the remedy available in case of violation
of the Anti-Wiretapping Law.
While there can be no prior restraint on protected expression, there can be subsequent
punishment for protected expression under libel, tort or other laws. In the present case, the NTC action
seeks prior restraint on the airing of the Garci Tapes, not punishment of personnel of radio and
television stations for actual violation of the Anti-Wiretapping Law.

6. Only the Courts May Impose Content-Based Prior Restraint


126
The NTC has no power to impose content-based prior restraint on expression. The charter of the
NTC does not vest NTC with any content-based censorship power over radio and television stations.
In the present case, the airing of the Garci Tapes is a protected expression that can never be
subject to prior restraint. However, even assuming for the sake of argument that the airing of the Garci
Tapes constitutes unprotected expression, only the courts have the power to adjudicate on the factual
and legal issue of whether the airing of the Garci Tapes presents a clear and present danger of bringing
about a substantive evil that the State has a right and duty to prevent, so as to justify the prior restraint.
Any order imposing prior restraint on unprotected expression requires prior adjudication by the
courts on whether the prior restraint is constitutional. This is a necessary consequence from the
presumption of invalidity of any prior restraint on unprotected expression.

7. Government Failed to Overcome Presumption of Invalidity


Respondents did not invoke any compelling State interest to impose prior restraint on the public
airing of the Garci Tapes. The respondents claim that they merely “fairly warned” radio and television
stations to observe the Anti-Wiretapping Law and pertinent NTC circulars on program standards.
Respondents have not explained how and why the observance by radio and television stations of the
Anti-Wiretapping Law and pertinent NTC circulars constitutes a compelling State interest justifying
prior restraint on the public airing of the Garci Tapes.
Violation of the Anti-Wiretapping Law, like the violation of any criminal statute, can always be
subject to criminal prosecution after the violation is committed. Respondents have not explained how
the violation of the Anti-Wiretapping Law, or of the pertinent NTC circulars, can incite imminent
lawless behavior or endanger the security of the State.

8. The NTC Warning is a Classic Form of Prior Restraint


The NTC press release threatening to suspend or cancel the airwave permits of radio and
television stations constitutes impermissible pressure amounting to prior restraint on protected
expression. Whether the threat is made in an order, regulation, advisory or press release, the chilling
effect is the same: the threat freezes radio and television stations into deafening silence. Radio and
television stations that have invested substantial sums in capital equipment and market development
suddenly face suspension or cancellation of their permits. The NTC threat is thus real and potent.

9. Conclusion
In sum, the NTC press release constitutes an unconstitutional prior restraint on protected
expression. There can be no content-based prior restraint on protected expression. This rule has no
exception.

Soriano vs. La Guardia


G.R. No. 164785. April 29, 2009

Facts:
On August 10, 2004, at around 10:00 p.m., petitioner, as host of the program Ang Dating Daan, aired
on UNTV 37, made obscene remarks against INC. Two days after, before the MTRCB, separate but almost
identical affidavit-complaints were lodged by Jessie L. Galapon and seven other private respondents, all
members of the Iglesia ni Cristo (INC), against petitioner in connection with the above broadcast. Respondent
Michael M. Sandoval, who felt directly alluded to in petitioner’s remark, was then a minister of INC and a
regular host of the TV program Ang Tamang Daan.

Issue:
Whether or not Soriano’s statements during the televised “Ang Dating Daan” part of the religious
discourse and within the protection of Section 5, Art. III.

Held:
No. Under the circumstances obtaining in this case, therefore, and considering the adverse effect of
petitioner’s utterances on the viewers’ fundamental rights as well as petitioner’s clear violation of his duty as a
127
public trustee, the MTRCB properly suspended him from appearing in Ang Dating Daan for three months.
Furthermore, it cannot be properly asserted that petitioner’s suspension was an undue curtailment of his right
to free speech either as a prior restraint or as a subsequent punishment. Aside from the reasons given above (re
the paramount of viewers rights, the public trusteeship character of a broadcaster’s role and the power of the
State to regulate broadcast media), a requirement that indecent language be avoided has its primary effect on
the form, rather than the content, of serious communication. There are few, if any, thoughts that cannot be
expressed by the use of less offensive language.

GMA NETWORK, INC., ET AL. vs. JESUS G. BUSTOS, ET AL.


G.R. No. 146848 October 17, 2006

FACTS: A Petition for Mandamus filed by the unsuccessful examinees of the physician’s licensure
examinations before the RTC of Manila to compel the PRC and the board of medical examiners to re-check and
reevaluate the test papers. As alleged, mistakes in the counting of the total scores and erroneous checking of
answers to test questions vitiated the results of the examinations. As news writer and reporter of petitioner
GMA Network, Inc. assigned to gather news from courts, among other beats, its co-petitioner Rey Vidal
covered the filing of the mandamus petition. After securing a copy of the petition, Vidal composed and narrated
the news coverage for the ten o'clock evening news edition of GMA's Channel 7 Headline News, quoting thereof
the allegations of the unsuccessful examiners that the gross, massive, haphazard, whimsical and capricious
checking that must have been going on for years should now be stopped once and for all. Simultaneous with the
news, was an old video footage showing physicians wearing black armbands. Along these lines, respondents
claimed that the said report was false, malicious and one-sided. Vidal and GMA Network, Inc., in reckless
disregard for the truth, defamed them by word of mouth and simultaneous visual presentation on GMA
Network, Inc.'s Channel 7. They added that, the showing of the unrelated old footage was done purposely so as
to make a forceful impact on their audience making it appear that other doctors were supporting and
sympathizing with the complaining unsuccessful examinees.
In defense of the alleged libel, GMA Network argued that the same was but a privileged communication.

ISSUE:
Whether the said news report was within the ambit of privileged communication

HELD:
Yes. The disputed news report consists merely of a summary of the allegations in the said Petition for
Mandamus filed by the medical examinees making the same fall within the protected ambit of privileged
communication. GMA and Vidal cannot be held liable for damages claimed by respondents for simply bringing
to fore information on subjects of public concern.
Privileged matters may be absolute or qualified. Absolutely privileged matters are not actionable
regardless of the existence of malice in fact. In absolutely privileged communications, the mala or bona fides of
the author is of no moment as the occasion provides an absolute bar to the action. On the other hand, in
qualifiedly or conditionally privileged communications, the freedom from liability for an otherwise defamatory
utterance is conditioned on the absence of express malice or malice in fact. The second kind of privilege, in
fine, renders the writer or author susceptible to a suit or finding of libel provided the prosecution established
the presence of bad faith or malice in fact. To this genre belongs "private communications" and "fair and true
report without any comments or remarks" falling under and described as exceptions in Article 354 of the
Revised Penal Code.
However, the enumeration under the aforecited Article 354 is not an exclusive list of conditional
privilege communications as the constitutional guarantee of freedom of the speech and of the press has
expanded the privilege to include fair commentaries on matters of public interest. The news telecast in
question clearly falls under the second kind of privileged matter.

G.R. No. 170643 September 8, 2006


JEJOMAR C. BINAY, for and in behalf of his minor daughter, JOANNA MARIE BIANCA S.
BINAY,
128
petitioner,
vs.
THE SECRETARY OF JUSTICE, GENINI V. FACTAO and VICENTE G. TIROL, respondents.

YNARES-SANTIAGO, J:

Facts:
In the April 15-21, 2001 issue of Pinoy Times Special Edition, an article entitled" ALYAS ERAP JR." was
published regarding the alleged extravagant lifestyle of the Binays and the assets that they acquired while in
public office. Paragraph 25 of the article reads:
Si Joanne Marie Bianca, 13 ang sinasabing ampong anak ng mga Binay, ay bumibili ng panty na
nagkakahalaga ng P1,000 ang isa, ayon sa isang writer ni Binay. Magarboang pamumuhay ng batang ito
dahil naspoiled umano ng kanyang ama.

Based on this article, Elenita S. Binay, mother of the minor Joanna Marie Bianca, filed a complaint for
libel against private respondents Vicente G. Tirol as publisher, and Genivi V. Factao as writer of the article,
with the Office of the City Prosecutor of Makati. Petitioner claims that the article is defamatory as it tends to, if
not actually, injure Joanna’s reputation and diminish the esteem, respect, and goodwill that others have of her.
Petitioner alleges that there is no good intention or justifiable motive in publishing Joanna’s status as an
adopted child which is essentially a private concern and the purchase of an expensive intimate apparel, but to
ridicule and to induce readers to lower their perception of Joanna. On the other hand, private respondents
allege that they did not harp on Joanna’s status as an adopted child as the same was mentioned only once in
the article; that they did not intend to injure her reputation or diminish her self-esteem; that they referred to
the price of the underwear not for the purpose of maligning her or to make her look frivolous in the public’s
eyes, but to show that petitioner and his family lead lavish and extravagant lives; and that this matter is within
the realm of public interest given that petitioner is an aspirant to a public office while his wife is an incumbent
public official.

Issue:
The issue to be resolved is whether there is prima facie evidence showing that the subject article was
libelous.

Held:
Yes. The court cannot discern a legal, moral, or social duty in publishing Joanna’s status as an adopted
daughter. Neither is there any public interest respecting her purchases of panties worth P1,000. Whether she
indeed bought those panties is not something that the public can afford protection against. With this backdrop,
it is obvious that private respondents’ only motive is to embarrass Joanna before the reading public.

G.R. No. 203335, 11 February 2014


Disini, et al.
v.
The Secretary of Justice, et al.,

FACTS
Petitioners lament that libel provisions of the penal code and, in effect, the libel provisions of the
cybercrime law carry with them the requirement of “presumed malice” even when the latest jurisprudence
already replaces it with the higher standard of “actual malice” as a basis for conviction. Petitioners argue that
inferring “presumed malice” from the accused’s defamatory statement by virtue of Article 354 of the penal code
infringes on his constitutionally guaranteed freedom of expression.

ISSUE
Whether or not Section 4(c)(4) of the Cybercrime Prevention Act on cyber libel affected the requirement
of “actual malice” as opposed to “presumed malice” as basis for conviction of libel.

129
RULING
The prosecution bears the burden of proving the presence of actual malice in instances where such
element is required to establish guilt. The defense of absence of actual malice, even when the statement turns
out to be false, is available where the offended party is a public official or a public figure, as in the cases of
Vasquez (a barangay official) and Borjal (the Executive Director, First National Conference on Land
Transportation). Since the penal code and implicitly, the cybercrime law, mainly target libel against private
persons, the Court recognizes that these laws imply a stricter standard of “malice” to convict the author of a
defamatory statement where the offended party is a public figure. Society’s interest and the maintenance of
good government demand a full discussion of public affairs.
But, where the offended party is a private individual, the prosecution need not prove the presence of
malice. The law explicitly presumes its existence (malice in law) from the defamatory character of the assailed
statement. For his defense, the accused must show that he has a justifiable reason for the defamatory statement
even if it was in fact true.

G.R. No. L-69500 July 22, 1985


JOSE ANTONIO U. GONZALEZ in behalf of MALAYA FILMS, LINO BROCKA, JOSE F. LACABA,
and DULCE Q. SAGUISAG, petitioners,
Vs.
CHAIRMAN MARIA KALAW KATIGBAK, GENERAL WILFREDO C. ESTRADA (Ret.), and THE
BOARD OF REVIEW FOR MOTION PICTURES AND TELEVISION (BRMPT), respondents.

Facts: In a resolution of a sub-committee of respondent Board of October 23, 1984, a permit to exhibit the film
Kapit sa Patalim under the classification "For Adults Only," with certain changes and deletions enumerated
was granted. A motion for reconsideration was filed by petitioners stating that the classification of the film "For
Adults Only" was without basis. 4 Then on November 12, 1984, respondent Board released its decision: "Acting
on the applicant's Motion for Reconsideration dated 29 October 1984, the Board, after a review of the
resolution of the sub-committee and an examination of the film, resolves to affirm in toto the ruling of the sub-
committee. Considering, however, certain vital deficiencies in the application, the Board further Resolves to
direct the Chairman of the Board to Withheld the issuance of the Permit to exhibit until these deficiencies are
supplied

Issue: WON the rating made with grave abuse of discretion

Held:
Roth- Sex and obscenity are not synonymous. Obscene material is material which deals with sex in a
manner appealing to prurient interest. The portrayal of sex, e.g., in art, literature and scientific works, is not
itself sufficient reason to deny material the constitutional protection of freedom of speech and press. Sex, a
great and mysterious motive force in human life has indisputably been a subject of absorbing interest to
mankind through the ages; it is one of the vital problems of human interest and public concern.
In the Philippine context, E.O. 876 applied contemporary Filipino cultural values as a standard.
Moreover, as far as the question of sex and obscenity are concerned, it cannot be stressed strongly that the arts
and letters "shall be under the patronage of the State. Given this constitutional mandate, it will be less than
true to its function if any government office or agency would invade the sphere of autonomy that an artist
enjoys. There is no orthodoxy in what passes for beauty or for reality. It is for the artist to determine what for
him is a true representation. It is not to be forgotten that art and belles-lettres deal primarily with imagination,
not so much with ideas in a strict sense. What is seen or perceived by an artist is entitled to respect, unless
there is a showing that the product of his talent rightfully may be considered obscene.
On the question of obscenity, therefore, such standard set forth in Executive Order No. 878 is to be
construed in such a fashion to avoid any taint of unconstitutionality. To repeat, what was stated in a recent
decision in Trinidad- an elementary, a fundamental, and a universal role of construction, applied when
considering constitutional questions, that when a law is susceptible of two constructions' one of which will
maintain and the other destroy it, the courts will always adopt the former. There can be no valid objection to
the controlling standard. There was really a grave abuse of discretion when the Board and its perception of
what obscenity is very restrictive.
130
But, sadly, THERE WERE NOT ENOUGH VOTES TO MAINTAIN THAT THERE WAS GRAVE ABUSE
OF DISCRETION. The supporting evidence was in the fact that some scenes were not for young people. They
might misunderstand the scenes. The respondents offered to make it GP if the petitioners would remove the
lesbian and sex scenes. But they refused. The ruling is to be limited to the concept of obscenity applicable to
motion pictures. It is the consensus of this Court that where television is concerned: a less liberal approach
calls for observance. This is so because unlike motion pictures where the patrons have to pay their way,
television reaches every home where there is a set. It is hardly the concern of the law to deal with the sexual
fantasies of the adult population. It cannot be denied though that the State as parens patriae is called upon to
manifest an attitude of caring for the welfare of the young.

MTRCB v. ABS-CBN
G.R. No. 155282. January 17, 2005
J. Sandoval Gutierrez

Facts:
Respondent abs-cbn aired “Prosti-tuition”, an episode of the TV program “The Inside Story” produced
and hosted by respondent Legarda. It depicted female students moonlighting as prostitutes to enable them to
pay for their tuition fees. PWU was named as the school of some of the students involved and the façade of the
PWU building served as the background of the episode. This caused upsoar in the PWU community and they
filed a letter-complaint to the MTRCB.

MTRCB alleged that respondents


1) Did not submit “the inside story” to petitioner for review
2) Exhibited the same without its permission, thus violating sec 7 of PD 1986 and some sections of MTRCB
rules and regulations

ABS-CBN averred:
1) The Inside Story is a public affairs program, news documentary and socio-political editorial, its airing is
protected by the constitutional provision on freedom of expression and of the press
2) Petitioners has no power, authority and jurisdiction to impose any form of prior restraint upon
respondents.

After hearing and submission of the parties’ memoranda, MTRCB investigating committee ordered the
respondents to pay P20,000 for non-submission of the program MTRCB affirmed the ruling Respondents filed
a special civil action for certiorari with RTC QC. RTC rendered a decision in favor of respondents, annulling
and setting aside the decision and resolution of the MTRCB and declaring and decreeing that certain sections
of PD 1986 & MTRCB do not cover the TV program “Inside Story”, they being a public affairs programs which
can be equated to a newspaper.
Hence, this petition

Issue:
Whether the MTRCB has the power or authority to review the “Inside Story” prior its exhibition or
broadcast by TV.

Held:
Sec 3 of PD 1986 enumerates the powers, functions and duties of the board:
Xxx
b) to screen, review and examine all motion pictures herein defined, TV programs, including
publicity materials

The court in INC v. CA rules that PD 1986 gives MTRCB the power to screen, review and examine ALL
TV PROGRAMS *LESSON* where the law does not make any exceptions, courts may not exempt something
therefrom, unless there is compelling reason apparent in the law to justify it.

131
Thus, when the law says “all TV programs”, the word all covers all tv programs whether religious, public
affairs, news docu, etc. It then follows that since the Inside Story is a TV Program, MTRCB has the power to
review it.
The only exemptions from the MTRCB’s power to review are those mentioned in Sec 7 of PD 1986:
1) TV programs imprinted or exhibited by Phil govt and/or departments and agencies
2) Newsreels

In a desperate attempt to be exempted, respondents content that Inside Story falls under the category of
newsreels.
MTRCB rules and reg defines newsreels as “straight news reporting, as distinguished from analyses,
commentaries, and opinions. Talk shows on a given issue are not considered newsreels.
Clearly, Inside Story is not a newsreel but more of a public affairs program and within petitioner’s
power of review.

Issue related to Consti law:


Petitioner’s power to review television programs under Section 3(b) of P. D. No. 1986 does not amount
to “prior restraint.”

SOCIAL WEATHER STATIONS, INC.


VS.
JUDGE MAXIMIANO C. ASUNCION, REGIONAL TRIAL COURT, BRANCH104, QUEZON CITY

A.M. No. RTJ-93-1049

FACTS:
Published under the by-line of one Marichu Villanueva and titled “Judiciary worse than PNP,” an item
in the June 17, 1993 issue of the Manila Standard, a metropolitan daily, reported that the results of the latest
opinion polls conducted by the Ateneo Social Weather Station, as Social Weather Stations, Inc. (or SWS) is also
known, showed the Judiciary to have an even lower satisfaction rating that the Philippine National Police. The
item went on to state that the President and his Cabinet had been briefed on the results of the survey by
Professors Mahar Mangahas and Felipe Miranda of the SWS, and that Malacanang had expressed concern over
the Judiciary’s law standing. Press Secretary Jesus Sison was also quoted as saying that this was “most
puzzling,” although he could not, recall the exact rating, noting only that the PNP had “a better image that the
judiciary.” Said report appears to have prompted Judge Maximiano C. Asuncion, presiding judge of Branch 104
of the Regional Trial Court at Quezon City, motu proprio to initiate on the same date of June 17, 1993
proceedings ordering the President of the SWS to: “explain why you should not be held in contempt for
distributing to the
general public without prior permission from any court your findings that the people have more confidence
with the police than with judges thereby tending directly or indirectly to degrade the administration of justice”.
On June 21, 1993, Prof. Mahar Mangahas through Atty. Antonio M. Abad, Jr. submitted his comment
and explanation that it was not true that the Social Weather Stations, Inc. distributed to the general public the
alleged survey. Said survey was privately given to Pres. Ramos and the cabinet and was not intended for
publication nor for public consumption and that if ever it reaches the media, he had not authorized anyone to
do so. The hearing was had a scheduled on June 23, 1993, after which Judge Asuncion promulgated an Order
dated July 2, 1993, finding Professor Mangahas’ explanation satisfactory and dismissing the contempt charge
against him. After three weeks or so, or more precisely on July 26, 1993, Professor Mangahas addressed a letter
to the Chief Justice intended “as a formal complaint against Honorable Maximiano C. Asuncion for grave abuse
of authority and gross ignorance of the law, in connection with his issuance of an Order dated 17 June 1993.

ISSUE:
Whether the Order dated 17 June 1993 is violative of the constitutional guarantees of freedom of speech
and freedom from prior restraint.

HELD:
132
No. What was clearly implicit in the newspaper report about the results of the SWS poll - in the words
of Judge Asuncion, “that the people have more confidence with the police than with the judges” – in light of the
fact, of which judicial notice is taken, that said report came out at a time when there already was widespread
publicity adverse to the judiciary, there can be no doubt of its clear tendency to degrade the administration of
justice. Thus, Judge Asuncion can hardly be faulted for what, at a minimum, he must have felt duty-bound to
doing the circumstances.
No question of prior restraint or violation of the guarantee of free speech arises here, what he did being,
in essence, merely to initiate an inquiry into the source and basis of the derogatory news report. And he
forthwith abated the proceedings upon receiving an explanation he deemed satisfactory.

Blo Umpar Adiong


vs.
Commision on Elections
(G.R. No. 1013956, March 31, 1992)

FACTS:
Blo Umpar Adiong, a senatorial candidate in the May 11, 1992 elections assails that the Comelec’s
Resolution which prohibits the posting of decals and stickers in mobile places like cars and other moving
vehicles is violative of Section 82 of the Omnibus Election Code and Section 11(a) of Republic Act No. 6646. In
addition, the petitioner believes that with the ban on radio, television and print political advertisements, he,
being a neophyte in the field of politics stands to suffer grave and irreparable injury with his prohibition. The
posting of decals and stickers on cars and other moving vehicles would be his last medium to inform the
electorate that he is a senatorial candidate in May 11,1992 elections.

ISSUE:
Whether or not the Comelec may prohibit the posting of decals and stickers on “mobile” places, public
or private, and limit their location or publication to the authorized posting areas that it fixes.

RULING:
The portion of Section 15(a) of Resolution No. 2347 of the Comelec providing that “decals and stickers
may be posted only in any authorized posting areas provided in paragraph (f) of Section 21 hereof” is declared
NULL AND VOID. The Comelec’s probation on posting decals and stickers on “mobile” places whether public
or private except in designated areas provided for by the Comelec itself is also NULL AND VOID on
constitutional grounds.
The prohibition unduly infringes on the citizen’s fundamental right of free speech enshrined in the
constitution (Article III, Section 4 of the 1987 Constitution). Significantly, freedom of expression curtailed by
the questioned prohibition is not so much that of candidate or the political party. The regulation strikes at the
freedom of an individual to express his preference and, by displaying it in his car, to convince others to agree
with him.
The constitutional objective to give rich candidate and a poor candidate equal opportunity to inform the
electorate as regards their candidacies, mandated by Article II, Section 26, and Article XIII, Section 1 in
relation to Article IX © Section 4 of the Constitution, is not impaired by posting decals and stickers on cars and
other private vehicles. It is to be reiterated that the posting of decals and stickers on cars, calesas, tricycles, and
other moving vehicles needs consent of the owner of the vehicle. Hence, the preference of the citizen becomes
crucial in this kind of election propaganda not the financial resources of the candidate.
In sum, the prohibition on posting of decals and stickers on “mobile” places whether private or public
except in authorized areas designated by the Comelec becomes censorship which cannot be justified by the
Constitution.

Pablito Sanidad
vs
Commission on Elections

133
73 SCRA 333

Facts:
On 2 Sept 1976, Marcos issued PD No. 991 calling for a national referendum on 16 Oct 1976 for the
Citizens Assemblies (“barangays”) to resolve, among other things, the issues of martial law, the interim
assembly, its replacement, the powers of such replacement, the period of its existence, the length of the period
for the exercise by the President of his present powers. Twenty days after, the President issued another related
decree, PD No. 1031, amending the previous PD No. 991, by declaring the provisions of PD No. 229 providing
for the manner of voting and canvass of votes in “barangays” applicable to the national referendum-plebiscite
of Oct 16, 1976. Quite relevantly, PD No. 1031 repealed inter alia, Sec 4, of PD No. 991. On the same date of 22
Sept 1976, Marcos issued PD No. 1033, stating the questions to he submitted to the people in the referendum-
plebiscite on October 16, 1976. The Decree recites in its “whereas” clauses that the people’s continued
opposition to the convening of the interim National Assembly evinces their desire to have such body abolished
and replaced thru a constitutional amendment, providing for a new interim legislative body, which will be
submitted directly to the people in the referendum-plebiscite of October 16.
On September 27, 1976, Sanidad filed a Prohibition with Preliminary Injunction seeking to enjoin the
Commission on Elections from holding and conducting the Referendum Plebiscite on October 16; to declare
without force and effect Presidential Decree Nos. 991 and 1033, insofar as they propose amendments to the
Constitution, as well as Presidential Decree No. 1031, insofar as it directs the Commission on Elections to
supervise, control, hold, and conduct the Referendum-Plebiscite scheduled on October 16, 1976.Petitioners
contend that under the 1935 and 1973 Constitutions there is no grant to the incumbent President to exercise
the constituent power to propose amendments to the new Constitution. As a consequence, the Referendum-
Plebiscite on October 16 has no constitutional or legal basis. The Soc-Gen contended that the question is
political in nature hence the court cannot take cognizance of it.

ISSUE: Whether or not Marcos can validly propose amendments to the Constitution.

HELD: Yes. The amending process both as to proposal and ratification raises a judicial question. This is
especially true in cases where the power of the Presidency to initiate the amending process by proposals of
amendments, a function normally exercised by the legislature, is seriously doubted. Under the terms of the
1973 Constitution, the power to propose amendments to the Constitution resides in the interim National
Assembly during the period of transition (Sec. 15, Transitory Provisions). After that period, and the regular
National Assembly in its active session, the power to propose amendments becomes ipso facto the prerogative
of the regular National Assembly (Sec. 1, pars. 1 and 2 of Art. XVI, 1973 Constitution).
The normal course has not been followed. Rather than calling the interim National Assembly to
constitute itself into a constituent assembly, the incumbent President undertook the proposal of amendments
and submitted the proposed amendments thru Presidential Decree 1033 to the people in a Referendum-
Plebiscite on October 16. Unavoidably, the regularity of the procedure for amendments, written in lambent
words in the very Constitution sought to be amended, raises a contestable issue. The implementing Presidential
Decree Nos. 991, 1031, and 1033, which commonly purport to have the force and effect of legislation are
assailed as invalid, thus the issue of the validity of said Decrees is plainly a justiciable one, within the
competence of this Court to pass upon.
Section 2 (2) Article X of the new Constitution provides: “All cases involving the constitutionality of a
treaty, executive agreement, or law shall be heard and decided by the Supreme Court en banc and no treaty,
executive agreement, or law may be declared unconstitutional without the concurrence of at least ten Members.
. . ...” The Supreme Court has the last word in the construction not only of treaties and statutes, but also of the
Constitution itself. The amending, like all other powers organized in the Constitution, is in form a delegated
and hence a limited power, so that the Supreme Court is vested with that authority to determine whether that
power has been discharged within its limits.
This petition is however dismissed. The President can propose amendments to the Constitution and he
was able to present those proposals to the people in sufficient time. The President at that time also sits as the
legislature.

ABS-CBN BROADCASTING CORPORATION vs. COMELEC


134
G.R. No. 133486, January 28, 2000

Facts:
ABS-CBN raised a petition for Certiorari under Rule 65 of the Rules of Court assailing Commission on
Elections (COMELEC) en banc Resolution No. 98-14191 dated April 21, 1998. In the said Resolution, the poll
body RESOLVED to approve the issuance of a restraining order to stop ABS-CBN or any other groups, its
agents or representatives from conducting such exit survey and to authorize the Honorable Chairman to issue
the same.
The Resolution was issued by COMELEC the allegedly upon "information from a reliable source that
ABS-CBN (Lopez Group) has prepared a project, with PR groups, to conduct radio-TV coverage of the elections
and to make an exit survey of the vote during the elections for national officials particularly for President and
Vice President, results of which shall be broadcast immediately."
The electoral body believed that such project might conflict with the official COMELEC count, as well as
the unofficial quick count of the National Movement for Free Elections (NAMFREL). It also noted that it had
not authorized or deputized Petitioner ABS-CBN to undertake the exit survey.
On May 9, 1998, the Supreme Court issued the Temporary Restraining Order prayed for by petitioner.
The Supreme Court directed the COMELEC to cease and desist, until further orders, from implementing the
assailed Resolution or the restraining order issued pursuant thereto, if any. In fact, the exit polls were actually
conducted and reported by media without any difficulty or problem.

Issue:
Whether or not the assailed resolution is valid.

Ruling:
No. The Supreme Court ruled that the absolute ban imposed by the COMELEC cannot be justified. It
does not leave open any alternative channel of communication to gather the type of information obtained
through exit polling. On the other hand, there are other valid and reasonable ways and means to achieve the
COMELEC end of avoiding or minimizing disorder and confusion that may be brought about by exit surveys.
A specific limited area for conducting exit polls may be designated. Only professional survey groups may be
allowed to conduct the same. Pollsters may be kept at a reasonable distance from the voting center. They may
be required to explain to voters that the latter may refuse interviewed, and that the interview is not part of the
official balloting process. The pollsters may further be required to wear distinctive clothing that would show
they are not election officials. Additionally, they may be required to undertake an information campaign on the
nature of the exercise and the results to be obtained therefrom.
These measures, together with a general prohibition of disruptive behavior, could ensure a clean, safe
and orderly election. The freedom of expression is a fundamental principle of our democratic government. It "is
a 'preferred' right and, therefore, stands on a higher level than substantive economic or other liberties and this
must be so because the lessons of history, both political and legal, illustrate that freedom of thought and speech
is the indispensable condition of nearly every other form of freedom."
Our Constitution clearly mandates that no law shall be passed abridging the freedom of speech or of the
press. In the landmark case Gonzales v. COMELEC, the Supreme Court enunciated that at the very least, free
speech and a free press consist of the liberty to discuss publicly and truthfully any matter of public interest
without prior restraint. The freedom of expression is a means of assuring individual self-fulfillment, of
attaining the truth, of securing participation by the people in social and political decision-making, and of
maintaining the balance between stability and change. It represents a profound commitment to the principle
that debates on public issues should be uninhibited, robust, and wide open. It means more than the right to
approve existing political beliefs or economic arrangements, to lend support to official measures, or to take
refuge in the existing climate of opinion on any of public consequence.
In exit polls, the contents of the official ballot are not actually exposed. Furthermore, the revelation of
whom an elector has voted for is not compulsory, but voluntary. Voters may also choose not to reveal their
identities. Indeed, narrowly tailored countermeasures may be prescribed by the COMELEC, so as to minimize
or suppress incidental problems in the conduct of exit polls, without transgressing the fundamental rights of
the people.

135
Thus the Supreme Court, granted the petition and the Temporary Restraining Order issued by the Court
on May 9, 1998 is made PERMANENT. The Assailed Minute Resolution No. 98-1419 issued by the COMELEC
en banc on April 21, 1998 is NULLIFIED and SET ASIDE.

SOCIAL WEATHER STATIONS, INCORPORATED and KAMAHALAN PUBLISHING


CORPORATION, doing business as MANILA STANDARD vs. COMMISSION ON ELECTIONS
G.R. No. 147571 May 5, 2001

Facts:
Petitioner, Social Weather Stations, Inc. (SWS), is a private non-stock, non-profit social research
institution conducting surveys in various fields, including economics, politics, demography, and social
development, and thereafter processing, analyzing, and publicly reporting the results thereof.
On the other hand, petitioner Kamahalan Publishing Corporation publishes the Manila Standard, a
newspaper of general circulation, which features news- worthy items of information including election surveys
Petitioners brought this action for prohibition to enjoin the Commission on Elections from enforcing Section
5.4 of RA. No.9006 (Fair Election Act), which provides surveys affecting national candidates, shall not be
published fifteen (15) days before an election and surveys affecting local candidates shall not be published
seven (7) days before an election.
Petitioner SWS states that it wishes to conduct an election survey throughout the period of the elections
both at the national and local levels and release to the media the results of such survey as well as publish them
directly. Petitioner Kamahalan Publishing Corporation, on the other hand, states that it intends to publish
election survey results up to the last day of the elections on May 14, 2001.

Issue:
Whether or not Section 5.4 of R.A. No. 9006 constitutes an unconstitutional abridgment of freedom of
speech, expression, and the press.

Ruling:
What test should then be employed to determine the constitutional validity of Section 5.4. The United
States Supreme Court, through Chief Justice Warren, held in United States v. O 'Brien: A Government
regulation is sufficiently justified when [1] if it is within the constitutional power of the Government; [2] if it
furthers an important or substantial governmental interest; [3] if the governmental interest is unrelated to the
suppression of free expression; and [4] if the incidental restriction on alleged First Amendment freedoms of
speech, expression and press is no greater than is essential to the furtherance of that interest.
This is so far the most influential test for distinguishing content-based from content neutral regulations
and is said to have "become canonical in the review of such laws." is noteworthy that the O 'Brien test has been
applied by this Court in at least two cases First. Sec. 5.4 fails to meet criterion of the O 'Brien test because the
causal connection of expression to the asserted governmental interest makes such interest "not related to the
suppression of free expression."
By prohibiting the publication of election survey results because of the possibility that such publication
might undermine the integrity of the election, section 5.4 actually suppresses a whole class of expression, while
allowing the expression of opinion concerning the same subject matter by newspaper columnists, radio and TV
commentators, armchair theorists, and other opinion takers even if the governmental interest sought to be
promoted is unrelated to the suppression of speech and the resulting restriction of free expression is only
incidental, section 5.4 nonetheless fails to meet criterion [4] of the O'Brien test, namely, that the restriction be
not greater than is necessary to further the governmental interest. As already stated, section 5.4 aims at the
prevention of last-minute pressure on voters, the creation of bandwagon effect, "junking" of weak or "losing"
candidates, and resort to the form of election cheating called "dagdag-bawas."
Praiseworthy as these aims of the regulation might be, they cannot be attained at the sacrifice of the
fundamental right of expression, when such aim can be more narrowly pursued by punishing unlawful acts,
rather than speech because of apprehension that such speech creates the danger of such evils.
To summarize then, the Supreme Court hold that section 5.4 is invalid because (1) it imposes a prior
restraint on the freedom of expression, (2) it is a direct and total suppression of a category of expression even

136
though such suppression is only for a limited period, and (3) the governmental interest sought to be promoted
can be achieved by means other than suppression of freedom of expression.

GMA NETWORK, INC. vs. COMMISSION ON ELECTIONS


G.R. No. 205357, September 2, 2014

Facts:
The five petitions before the Court put in issue the alleged unconstitutionality of Section 9 (a) of
COMELEC Resolution No. 9615 limiting the broadcast and radio advertisements of candidates and political
parties for national election positions to an aggregate total of one hundred twenty (120) minutes and one
hundred eighty (180) minutes, respectively. They contend that such restrictive regulation on allowable
broadcast time violates freedom of the press, impairs the people’s right to suffrage as well as their right to
information relative to the exercise of their right to choose who to elect during the forth coming elections.
Section 9 (a) provides for an “aggregate total” airtime instead of the previous “per station” airtime for political
campaigns or advertisements, and also required prior COMELEC approval for candidates’ television and radio
guestings and appearances.

Issue:
Whether or not Section 9 (a) of COMELEC Resolution No. 9615 on airtime limits violates freedom of
expression, of speech and of the press.

Ruling:
YES. The Supreme Court held that the assailed rule on “aggregate-based” airtime limits is
unreasonable and arbitrary as it unduly restricts and constrains the ability of candidates and political parties to
reach out and communicate with the people.
The adverted reason for imposing the “aggregate-based” airtime limits leveling the playing field does
not constitute a compelling state interest which would justify such a substantial restriction on the freedom of
candidates and political parties to communicate their ideas, philosophies, platforms and programs of
government. And, this is specially so in the absence of a clear-cut basis for the imposition of such a prohibitive
measure.
It is also particularly unreasonable and whimsical to adopt the aggregate-based time limits on broadcast
time when we consider that the Philippines is not only composed of so many islands. There are also a lot of
languages and dialects spoken among the citizens across the country. Accordingly, for a national candidate to
really reach out to as many of the electorates as possible, then it might also be necessary that he conveys his
message through his advertisements in languages and dialects that the people may more readily understand
and relate to. To add all of these airtimes in different dialects would greatly hamper the ability of such
candidate to express himself – a form of suppression of his political speech.

THE DIOCESE OF BACOLOD, REPRESENTED BY THE MOST REV. BISHOP VICENTE M.


NAVARRA and THE BISHOP HIMSELF IN HIS PERSONAL CAPACITY vs. COMMISSION ON
ELECTIONS AND THE ELECTION OFFICER OF BACOLOD CITY, ATTY. MAVIL V.
MAJARUCON
G.R. No. 205728, January 21, 2015

Facts:
On February 21, 2013, petitioners posted two tarpaulins within a private compound housing the San
Sebastian Cathedral of Bacolod. Each tarpaulin was approximately six feet (6′) by ten feet (10′) in size. They
were posted on the front walls of the cathedral within public view. The first tarpaulin contains the message
“IBASURA RH Law” referring to the Reproductive Health Law of 2012 or Republic Act No. 10354.
The second tarpaulin is the subject of the present case. This tarpaulin contains the heading “Conscience
Vote” and lists candidates as either “(Anti-RH) Team Buhay” with a check mark, or “(Pro-RH) Team Patay”
with an “X” mark. The electoral candidates were classified according to their vote on the adoption of Republic

137
Act No. 10354, otherwise known as the RH Law. Those who voted for the passing of the law were classified by
petitioners as comprising “Team Patay,” while those who voted against it form “Team Buhay.”
Respondents conceded that the tarpaulin was neither sponsored nor paid for by any candidate.
Petitioners also conceded that the tarpaulin contains names ofcandidates for the 2013 elections, but not of
politicians who helped in the passage of the RH Law but were not candidates for that election.

Issues:
1. Whether or not the size limitation and its reasonableness of the tarpaulin is a political question, hence
not within the ambit of the Supreme Court’s power of review.
2. Whether or not the petitioners violated the principle of exhaustion of administrative remedies as the
case was not brought first before the COMELEC En Banc or any if its divisions.
3. Whether or not COMELEC may regulate expressions made by private citizens.
4. Whether or not the assailed notice and letter for the removal of the tarpaulin violated petitioners’
fundamental right to freedom of expression.
5. Whether the order for removal of the tarpaulin is a content-based or content-neutral regulation.
6. Whether or not there was violation of petitioners’ right to property.
7. Whether or not the tarpaulin and its message are considered religious speech.

Ruling:

First Issue:
No. The Court ruled that the present case does not call for the exercise of prudence or modesty. There is
no political question. It can be acted upon by this court through the expanded jurisdiction granted to this court
through Article VIII, Section 1 of the Constitution..
The concept of a political question never precludes judicial review when the act of a constitutional organ
infringes upon a fundamental individual or collective right. Even assuming arguendo that the COMELEC did
have the discretion to choose the manner of a regulation of the tarpaulin in question, it cannot do so by
abridging the fundamental right to expression.
Also, the Court said that in our jurisdiction, the determination of whether an issue involves a truly
political and non-justiciable question lies in the answer to the question of whether there are constitutionally
imposed limits on powers or functions conferred upon political bodies. If there are, then our courts are duty-
bound to examine whether the branch or instrumentality of the government properly acted within such limits.
A political question will not be considered justiciable if there are no constitutionally imposed limits on
powers or functions conferred upon political bodies. Hence, the existence of constitutionally imposed limits
justifies subjecting the official actions of the body to the scrutiny and review of this court.
In this case, the Bill of Rights gives the utmost deference to the right to free speech. Any instance that
this right may be abridged demands judicial scrutiny. It does not fall squarely into any doubt that a political
question brings.

Second Issue:
No. The Court held that the argument on exhaustion of administrative remedies is not proper in this
case.
Despite the alleged non-exhaustion of administrative remedies, it is clear that the controversy is already
ripe for adjudication. Ripeness is the “prerequisite that something had by then been accomplished or
performed by either branch or in this case, organ of government before a court may come into the picture.”
Petitioners’ exercise of their right to speech, given the message and their medium, had understandable
relevance especially during the elections. COMELEC’s letter threatening the filing of the election offense
against petitioners is already an actionable infringement of this right. The impending threat of criminal
litigation is enough to curtail petitioners’ speech.
In the context of this case, exhaustion of their administrative remedies as COMELEC suggested in their
pleadings prolongs the violation of their freedom of speech.

THIRD ISSUE: No.


Respondents cite the Constitution, laws, and jurisprudence to support their position that they had the
power to regulate the tarpaulin. However, the Court held that all of these provisions pertain to candidates and
138
political parties. Petitioners are not candidates. Neither do they belong to any political party. COMELEC does
not have the authority to regulate the enjoyment of the preferred right to freedom of expression exercised by a
non-candidate in this case.

FOURTH ISSUE: Yes.


The Court held that every citizen’s expression with political consequences enjoys a high degree of
protection.
Moreover, the respondent’s argument that the tarpaulin is election propaganda, being petitioners’ way
of endorsing candidates who voted against the RH Law and rejecting those who voted for it, holds no water.
The Court held that while the tarpaulin may influence the success or failure of the named candidates
and political parties, this does not necessarily mean it is election propaganda. The tarpaulin was not paid for or
posted “in return for consideration” by any candidate, political party, or party-list group.
By interpreting the law, it is clear that personal opinions are not included, while sponsored messages
are covered.

The content of the tarpaulin is a political speech


Political speech refers to speech “both intended and received as a contribution to public deliberation
about some issue,” “fostering informed and civic minded deliberation.” On the other hand, commercial speech
has been defined as speech that does “no more than propose a commercial transaction.” The expression
resulting from the content of the tarpaulin is, however, definitely political speech.

FIFTH ISSUE: Content-based regulation.


Content-based restraint or censorship refers to restrictions “based on the subject matter of the
utterance or speech.” In contrast, content-neutral regulation includes controls merely on the incidents of the
speech such as time, place, or manner of the speech.
The Court held that the regulation involved at bar is content-based. The tarpaulin content is not easily
divorced from the size of its medium.
Content-based regulation bears a heavy presumption of invalidity, and this court has used the clear and
present danger rule as measure.
Under this rule, “the evil consequences sought to be prevented must be substantive, ‘extremely serious
and the degree of imminence extremely high.’” “Only when the challenged act has overcome the clear and
present danger rule will it pass constitutional muster, with the government having the burden of overcoming
the presumed unconstitutionality.”
Even with the clear and present danger test, respondents failed to justify the regulation. There is no
compelling and substantial state interest endangered by the posting of the tarpaulin as to justify curtailment of
the right of freedom of expression. There is no reason for the state to minimize the right of non-candidate
petitioners to post the tarpaulin in their private property. The size of the tarpaulin does not affect anyone else’s
constitutional rights.

SIXTH ISSUE: Yes.


The Court held that even though the tarpaulin is readily seen by the public, the tarpaulin remains the
private property of petitioners. Their right to use their property is likewise protected by the Constitution.
Any regulation, therefore, which operates as an effective confiscation of private property or constitutes
an arbitrary or unreasonable infringement of property rights is void, because it is repugnant to the
constitutional guaranties of due process and equal protection of the laws.
The Court in Adiong case held that a restriction that regulates where decals and stickers should be
posted is “so broad that it encompasses even the citizen’s private property.” Consequently, it violates Article III,
Section 1 of the Constitution which provides that no person shall be deprived of his property without due
process of law.

SEVENTH ISSUE: No.


The Court held that the churches doctrines relied upon by petitioners are not binding upon this court.
The position of the Catholic religion in the Philippines as regards the RH Law does not suffice to qualify the
posting by one of its members of a tarpaulin as religious speech solely on such basis. The enumeration of

139
candidates on the face of the tarpaulin precludes any doubt as to its nature as speech with political
consequences and not religious speech.

Doctrine of benevolent neutrality


With religion looked upon with benevolence and not hostility, benevolent neutrality allows
accommodation of religion under certain circumstances. Accommodations are government policies that take
religion specifically into account not to promote the government’s favored form of religion, but to allow
individuals and groups to exercise their religion without hindrance. Their purpose or effect therefore is to
remove a burden on, or facilitate the exercise of, a person’s or institution’s religion.
As Justice Brennan explained, the “government may take religion into account . . . to exempt, when
possible, from generally applicable governmental regulation individuals whose religious beliefs and practices
would otherwise thereby be infringed, or to create without state involvement an atmosphere in which voluntary
religious exercise may flourish.”

1-UNITED TRANSPORT KOALISYON (1-UTAK) vs. COMMISSION ON ELECTIONS


G.R. No. 206020, April 14, 2015

Facts:
On January 15, 2013, the COMELEC promulgated Resolution No. 9615, which provided for the rules
implementing R.A. No. 9006 in connection with the May 13, 2013 national and local elections and subsequent
elections. Section 7 thereof, which enumerates the prohibited forms of election propaganda, pertinently
provides:

SEC. 7. Prohibited Forms of Election Propaganda. – During the campaign period, it is unlawful:
(f) To post, display or exhibit any election campaign or propaganda material outside of authorized
common poster areas, in public places, or in private properties without the consent of the owner
thereof.
(g) Public places referred to in the previous subsection (f) include any of the following:
5. Public utility vehicles such as buses, jeepneys, trains, taxi cabs, ferries, pedicabs and tricycles,
whether motorized or not;
6. Within the premises of public transport terminals, such as bus terminals, airports, seaports, docks,
piers, train stations, and the like.

The violation of items [5 and 6] under subsection (g) shall be a cause for the revocation of the public
utility franchise and will make the owner and/or operator of the transportation service and/or terminal liable
for an election offense under Section 9 of Republic Act No. 9006 as implemented by Section 18 (n) of these
Rules.
Petitioner sought for clarification from COMELEC as regards the application of Resolution No. 9615
particularly Section 7(g) items (5) and (6), in relation to Section 7(f), vis-à-vis privately owned public utility
vehicles (PUVs) and transport terminals. The petitioner then requested the COMELEC to reconsider the
implementation of the assailed provisions and allow private owners of PUVs and transport terminals to post
election campaign materials on their vehicles and transport terminals.
The COMELEC en banc issued Minute Resolution No. 13-0214, which denied the petitioner’s request to
reconsider the implementation of Section 7(g) items (5) and (6), in relation to Section 7(f), of Resolution No.
9615.

Issue:
Whether or not Section 7(g) items (5) and (6), in relation to Section 7(f), of Resolution No. 9615 are
constitutional.

Ruling:
The Supreme Court held that the said provisions of Resolution No. 9615 are null and void for being
repugnant to Sections 1 and 4, Article III of the 1987 Constitution.

140
Section 7(g) items (5) and (6), in relation to Section 7(f), of Resolution No. 9615 are prior
restraints on speech
Section 7(g) items (5) and (6), in relation to Section 7(f), of Resolution No. 9615 unduly infringe on the
fundamental right of the people to freedom of speech. Central to the prohibition is the freedom of individuals,
i.e., the owners of PUVs and private transport terminals, to express their preference, through the posting of
election campaign material in their property, and convince others to agree with them.
Pursuant to the assailed provisions of Resolution No. 9615, posting an election campaign material
during an election period in PUVs and transport terminals carries with it the penalty of revocation of the public
utility franchise and shall make the owner thereof liable for an election offense.
The prohibition constitutes a clear prior restraint on the right to free expression of the owners of PUVs
and transport terminals. As a result of the prohibition, owners of PUVs and transport terminals are forcefully
and effectively inhibited from expressing their preferences under the pain of indictment for an election offense
and the revocation of their franchise or permit to operate.

The assailed prohibition on posting election campaign materials is an invalid content-neutral


regulation repugnant to the free speech clause.
A content-neutral regulation, i.e., which is merely concerned with the incidents of the speech, or
one that merely controls the time, place or manner, and under well-defined standards, is constitutionally
permissible, even if it restricts the right to free speech, provided that the following requisites concur:
1. The government regulation is within the constitutional power of the Government;
2. It furthers an important or substantial governmental interest;
3. The governmental interest is unrelated to the suppression of free expression; and
4. The incidental restriction on freedom of expression is no greater than is essential to the furtherance of
that interest.

Section 7(g) items (5) and (6) of Resolution No. 9615 are content-neutral regulations since they
merely control the place where election campaign materials may be posted. However, the prohibition is still
repugnant to the free speech clause as it fails to satisfy all of the requisites for a valid content-neutral
regulation.
Section 7(g) items (5) and (6), in relation to Section 7(f), of Resolution No. 9615, are not
within the constitutionally delegated power of the COMELEC under Section 4, Article IX-C of
the Constitution. Also, there is absolutely no necessity to restrict the right to free speech of the owners of
PUVs and transport terminals.

The COMELEC may only regulate the franchise or permit to operate and not the
ownership per se of PUVs and transport terminals.

In the instant case, the Court further delineates the constitutional grant of supervisory and regulatory
powers to the COMELEC during an election period. As worded, Section 4, Article IX-C of the Constitution only
grants COMELEC supervisory and regulatory powers over the enjoyment or utilization “of all franchises or
permits for the operation,” inter alia, of transportation and other public utilities. The COMELEC’s
constitutionally delegated powers of supervision and regulation do not extend to the ownership per se of PUVs
and transport terminals, but only to the franchise or permit to operate the same.

Section 7(g) items (5) and (6) of Resolution No. 9615 are not within the constitutionally
delegated power of the COMELEC to supervise or regulate the franchise or permit to operate of
transportation utilities. The posting of election campaign material on vehicles used for public transport or
on transport terminals is not only a form of political expression, but also an act of ownership – it has nothing to
do with the franchise or permit to operate the PUV or transport terminal.
Section 7(g) items (5) and (6) of Resolution No. 9615 are not justified under the captive-
audience doctrine.
The captive-audience doctrine states that when a listener cannot, as a practical matter, escape from
intrusive speech, the speech can be restricted. The “captive-audience” doctrine recognizes that a listener has a
right not to be exposed to an unwanted message in circumstances in which the communication cannot be
avoided.
141
A regulation based on the captive-audience doctrine is in the guise of censorship, which undertakes
selectively to shield the public from some kinds of speech on the ground that they are more offensive than
others. Such selective restrictions have been upheld only when the speaker intrudes on the privacy of the home
or the degree of captivity makes it either impossible or impractical for the unwilling viewer or auditor to avoid
exposure.
Thus, a government regulation based on the captive-audience doctrine may not be justified if the
supposed “captive audience” may avoid exposure to the otherwise intrusive speech. The prohibition under
Section 7(g) items (5) and (6) of Resolution No. 9615 is not justified under the captive-audience doctrine;
the commuters are not forced or compelled to read the election campaign materials posted on PUVs and
transport terminals. Nor are they incapable of declining to receive the messages contained in the posted
election campaign materials since they may simply avert their eyes if they find the same unbearably intrusive.

Prohibiting owners of PUVs and transport terminals from posting election campaign materials
violates the equal protection clause.
Section 7(g) items (5) and (6) of Resolution No. 9615 do not only run afoul of the free speech clause, but
also of the equal protection clause. One of the basic principles on which this government was founded is that of
the equality of right, which is embodied in Section 1, Article III of the 1987 Constitution.
It is conceded that the classification under Section 7(g) items (5) and (6) of Resolution No. 9615 is not
limited to existing conditions and applies equally to the members of the purported class. However, the
classification remains constitutionally impermissible since it is not based on substantial distinction and is not
germane to the purpose of the law. A distinction exists between PUVs and transport terminals and private
vehicles and other properties in that the former, to be considered as such, needs to secure from the government
either a franchise or a permit to operate. Nevertheless, as pointed out earlier, the prohibition imposed under
Section 7(g) items (5) and (6) of Resolution No. 9615 regulates the ownership per se of the PUV and transport
terminals; the prohibition does not in any manner affect the franchise or permit to operate of the PUV and
transport terminals.
As regards ownership, there is no substantial distinction between owners of PUVs and transport
terminals and owners of private vehicles and other properties. As already explained, the ownership of PUVs
and transport terminals, though made available for use by the public, remains private. If owners of private
vehicles and other properties are allowed to express their political ideas and opinion by posting
election campaign materials on their properties, there is no cogent reason to deny the same
preferred right to owners of PUVs and transport terminals. In terms of ownership, the
distinction between owners of PUVs and transport terminals and owners of private vehicles
and properties is merely superficial. Superficial differences do not make for a valid
classification.
The fact that PUVs and transport terminals are made available for use by the public is
likewise not substantial justification to set them apart from private vehicles and other
properties. Admittedly, any election campaign material that would be posted on PUVs and transport
terminals would be seen by many people. However, election campaign materials posted on private vehicles and
other places frequented by the public, e.g., commercial establishments, would also be seen by many people.
Thus, there is no reason to single out owners of PUVs and transport terminals in the prohibition against
posting of election campaign materials.

Social Weather Stations vs COMELEC

Facts:
Petitioner, Social Weather Stations, Inc. (SWS) is a private non-stock, non-profit social research
institution conducting surveys in various fields. On the other hand, petitioner Kamahalan Publishing
Corporation publishes the Manila Standard, a newspaper of general circulation.
Petitioners brought this action for prohibition to enjoin the Commission on Elections from enforcing
Section 5.4 of RA. No.9006 (Fair Election Act), which provides that: “Surveys affecting national candidates
shall not be published fifteen (15) days before an election and surveys affecting local candidates shall not be
published seven (7) days before an election”.

142
Petitioners argue that the restriction on the publication of election survey results constitutes a prior
restraint on the exercise of freedom of speech without any clear and present danger to justify such restraint.
They claim that SWS and other pollsters conducted and published the results of surveys prior to the 1992, 1995,
and 1998 elections up to as close as two days before the election day without causing confusion among the
voters and that there is neither empirical nor historical evidence to support the conclusion that there is an
immediate and inevitable danger to tile voting process posed by election surveys. No similar restriction is
imposed on politicians from explaining their opinion or on newspapers or broadcast media from writing and
publishing articles concerning political issues up to the day of the election. They contend that there is no reason
for ordinary voters to be denied access to the results of election surveys, which are relatively objective.
Respondent Commission on Elections justifies the restrictions in §5.4 of R.A. No. 9006 as necessary to
prevent the manipulation and corruption of the electoral process by unscrupulous and erroneous surveys just
before the election. It contends that (1) the prohibition on the publication of election survey results during the
period proscribed by law bears a rational connection to the objective of the law, i.e., the prevention of the
debasement of the electoral process resulting from manipulated surveys, bandwagon effect, and absence of
reply; (2) it is narrowly tailored to meet the "evils" sought to be prevented; and (3) the impairment of freedom
of expression is minimal, the restriction being limited both in duration, i.e., the last 15 days before the national
election and the last 7 days before a local election, and in scope as it does not prohibit election survey results
but only require timeliness.

Issue:
Whether or not Section 5.4 of RA 9006 constitutes an unconstitutional abridgment of freedom of
speech, expression and the press.

Ruling:
Yes. It constitutes an unconstitutional abridgement of freedom of expression, speech and the press. To
summarize, the Supreme Court held that §5.4 is invalid because (1) it imposes a prior restraint on the freedom
of expression, (2) it is a direct and total suppression of a category of expression even though such suppression
is only for a limited period, and (3) the governmental interest sought to be promoted can be achieved by means
other than suppression of freedom of expression.
It has been held that mere legislative preferences or beliefs respecting matters of public convenience
may well support regulation directed at other personal activities, but be insufficient to justify such as
diminishes the exercise of rights so vital to the maintenance of democratic institutions.

Pharmaceutical and Health Care Association of the Philippines vs. Duque III

Facts:
Named as respondents are the Health Secretary, Undersecretaries, and Assistant Secretaries of the
Department of Health (DOH). For purposes of herein petition, the DOH is deemed impleaded as a co-
respondent since respondents issued the questioned RIRR in their capacity as officials of said executive agency.
Executive Order No. 51 (Milk Code) was issued by President Corazon Aquino on October 28, 1986 by
virtue of the legislative powers granted to the president under the Freedom Constitution. One of the
preambular clauses of the Milk Code states that the law seeks to give effect to Article 112 of the International
Code of Marketing of Breast milk Substitutes (ICMBS), a code adopted by the World Health Assembly (WHA)
in 1981.
From 1982 to 2006, the WHA adopted several Resolutions to the effect that breastfeeding should be
supported, promoted and protected, hence, it should be ensured that nutrition and health claims are not
permitted for breast milk substitutes. In 1990, the Philippines ratified the International Convention on the
Rights of the Child. Article 24 of said instrument provides that State Parties should take appropriate measures
to diminish infant and child mortality, and ensure that all segments of society, specially parents and children,
are informed of the advantages of breastfeeding.
On May 15, 2006, the DOH issued herein assailed RIRR which was to take effect on July 7, 2006.

Issue:

143
Whether Administrative Order or the Revised Implementing Rules and Regulations (RIRR) issued by
the Department of Health (DOH) is not constitutional;

Ruling:
YES. Under Article 23, recommendations of the WHA do not come into force for members, in the same
way that conventions or agreements under Article 19 and regulations under Article 21 come into force. Article
23 of the WHO Constitution reads:
The Health Assembly shall have authority to make recommendations to Members with respect to any
matter within the competence of the Organization for an international rule to be considered as customary law,
it must be established that such rule is being followed by states because they consider it obligatory to comply
with such rules Under the 1987 Constitution, international law can become part of the sphere of domestic law
either by transformation or incorporation. The transformation method requires that an international law be
transformed into a domestic law through a constitutional mechanism such as local legislation. The
incorporation method applies when, by mere constitutional declaration, international law is deemed to have
the force of domestic law.
Consequently, legislation is necessary to transform the provisions of the WHA Resolutions into
domestic law. The provisions of the WHA Resolutions cannot be considered as part of the law of the land that
can be implemented by executive agencies without the need of a law enacted by the legislature.

Bayan vs. Executive Secretary Ermita


488 SCRA 226, April 25, 2006

Facts:
Rallies of September 20, October 4, 5 and 6, 2005 is at issue. BAYAN’s rally was violently dispersed.
Petitioners were injured, arrested and detained when a peaceful mass action they was preempted and violently
dispersed by the police.
KMU asserts that the right to peaceful assembly, are affected by Batas Pambansa No. 880 and the policy
of “Calibrated Preemptive Response” (CPR) being followed to implement it. KMU, et al., claim that on October
4, 2005, a rally KMU co-sponsored was to be conducted at the Mendiola bridge but police blocked them along
C.M. Recto and Lepanto Streets and forcibly dispersed them, causing injuries to several of their members. They
further allege that on October 6, 2005, a multi-sectoral rally which KMU also co-sponsored was scheduled to
proceed along España Avenue in front of the UST and going towards Mendiola bridge. Police officers blocked
them along Morayta Street and prevented them from proceeding further. They were then forcibly dispersed,
causing injuries on one of them. Three other rallyists were arrested.
All petitioners assail Batas Pambansa No. 880, the Public Assembly Act of 1985, some of them in toto
and others only Sections 4, 5, 6, 12, 13(a), and 14(a), as well as the policy of CPR. They seek to stop violent
dispersals of rallies under the “no permit, no rally” policy and the CPR policy announced on Sept. 21, 2005.
Petitioners Bayan, et al., contend that BP 880 is clearly a violation of the Constitution and the
International Covenant on Civil and Political Rights and other human rights treaties of which the Philippines is
a signatory.
They argue that B.P. No. 880 requires a permit before one can stage a public assembly regardless of the
presence or absence of a clear and present danger. It also curtails the choice of venue and is thus repugnant to
the freedom of expression clause as the time and place of a public assembly form part of the message for which
the expression is sought.
Petitioners Jess del Prado, et al., in turn, argue that B.P. No. 880 is unconstitutional as it is a
curtailment of the right to peacefully assemble and petition for redress of grievances because it puts a condition
for the valid exercise of that right. It also characterizes public assemblies without a permit as illegal and
penalizes them and allows their dispersal. Thus, its provisions are not mere regulations but are actually
prohibitions. Regarding the CPR policy, it is void for being an ultra vires act that alters the standard of
maximum tolerance set forth in B.P. No. 880, aside from being void for being vague and for lack of publication.
KMU, et al., argue that the Constitution sets no limits on the right to assembly and therefore B.P. No.
880 cannot put the prior requirement of securing a permit. And even assuming that the legislature can set
limits to this right, the limits provided are unreasonable: First, allowing the Mayor to deny the permit on clear
and convincing evidence of a clear and present danger is too comprehensive. Second, the five-day requirement
144
to apply for a permit is too long as certain events require instant public assembly, otherwise interest on the
issue would possibly wane.As to the CPR policy, they argue that it is preemptive, that the government takes
action even before the rallyists can perform their act, and that no law, ordinance or executive order supports
the policy. Furthermore, it contravenes the maximum tolerance policy of B.P. No. 880 and violates the
Constitution as it causes a chilling effect on the exercise by the people of the right to peaceably assemble.
Respondents argued that petitioners have no standing. BP 880 entails traffic re-routing to prevent
grave public inconvenience and serious or undue interference in the free flow of commerce and trade. It is
content-neutral regulation of the time, place and manner of holding public assemblies. According to Atienza
RA. 7160 gives the Mayor power to deny a permit independently of B.P. No. 880. and that the permit is for the
use of a public place and not for the exercise of rights; and that B.P. No. 880 is not a content-based regulation
because it covers all rallies.

Issue:
Whether or Not BP 880 and the CPR Policy unconstitutional.

Ruling:
No question as to standing. Their right as citizens to engage in peaceful assembly and exercise the right
of petition, as guaranteed by the Constitution, is directly affected by B.P. No. 880. B.P. 880 is not an absolute
ban of public assemblies but a restriction that simply regulates the time, place and manner of the assemblies. It
refers to all kinds of public assemblies that would use public places. The reference to “lawful cause” does not
make it content-based because assemblies really have to be for lawful causes; otherwise they would not be
“peaceable” and entitled to protection. Maximum tolerance1 is for the protection and benefit of all rallyists and
is independent of the content of the expressions in the rally. There is, likewise, no prior restraint, since the
content of the speech is not relevant to the regulation.
The so-called calibrated preemptive response policy has no place in our legal firmament and must be
struck down as a darkness that shrouds freedom. It merely confuses our people and is used by some police
agents to justify abuses. Insofar as it would purport to differ from or be in lieu of maximum tolerance, this was
declared null and void.
The Secretary of the Interior and Local Governments are DIRECTED to take all necessary steps for the
immediate compliance with Section 15 of Batas Pambansa No. 880 through the establishment or designation of
at least one suitable freedom park or plaza in every city and municipality of the country. After thirty (30) days
from the finality of this Decision, subject to the giving of advance notices, no prior permit shall be required to
exercise the right to peaceably assemble and petition in the public parks or plazas of a city or municipality that
has not yet complied with Section 15 of the law.

Integrated Bar of the Philippines vs. Mayor Lito Atienza


613 SCRA 518

Facts:
In June 2006, the Integrated Bar of the Philippines (IBP) filed an application for a rally permit with the
office of Manila Mayor Jose “Lito” Atienza. The IBP sought their rally to be staged at the Mendiola Bridge.
Atienza granted the permit but indicated thereon that IBP is only allowed to stage their rally at the Plaza
Miranda, a freedom park.
IBP President Jose Anselmo Cadiz received the rally permit on the day before the scheduled rally. Cadiz
immediately went to the Court of Appeals to assail the permit because what Atienza did was only a partial grant
which was alleged to be a violation of the constitutional right to freedom of expression and a grave abuse of
discretion on the part of Atienza.
Meanwhile, IBP pushed through with the rally not at Plaza Miranda but at the Mendiola Bridge.
Subsequently, the Manila Police District (MPD) filed a criminal case against Cadiz for allegedly violating the
Public Assembly Act or specifically, for staging a rally in a place different from what was indicated in the rally
permit.
The Court of Appeals ruled in favor of Atienza. The CA ruled that what Atienza did was within his
power; that freedom of expression is not absolute.

145
Cadiz appealed before the Supreme Court. Cadiz also prayed for the suspension of the criminal case
against him on the ground that the certiorari case he filed against Atienza is a prejudicial question to the
criminal case.

Issues:
1. Whether or not the certiorari case Cadiz filed against Atienza is a prejudicial question to the criminal
case filed against him (Cadiz).
2. Whether or not it is within Mayor Jose Atienza’s power to modify the rally permit without consulting
with the IBP.

Ruling:
1. No. It is improper for Cadiz to raise the issue of prejudicial question at this stage and in this certiorari
case. Under the Rules of Court, a prejudicial question is a ground to suspend the criminal proceeding.
However, Cadiz must first file a petition to suspend the criminal proceeding in the said criminal case.
The determination of the pendency of a prejudicial question should be made at the first instance in the
criminal action, and not before the Supreme Court in an appeal from the civil action.
2. No. In modifying a rally permit or in granting a rally permit which contains a time and place different
from that applied for, the mayor must first consult with the applicant at the earliest opportunity. This is
in order to give the applicant some time to determine if such change is favorable to him or adverse (and
if adverse, he can seek judicial remedies) – Section 6 of the Public Assembly Act.
It is an indispensable condition to such refusal or modification that the clear and present danger
tests be the standard for the decision reached. If he is of the view that there is such an imminent and
grave danger of a substantive evil, the applicant must be heard on the matter. In this case, Atienza did
not consult with the IBP. Atienza capriciously and whimsically changed the venue without any reason
therefor. Such is a grave abuse of discretion and a violation of the freedom of expression.

Miriam College Foundation, Inc. v CA


348 SCRA 265 December 15, 2000

FACTS:
The members of the editorial board of the Miriam College Foundation’s school paper were subjected to
disciplinary sanction by the College Discipline Committee after letters of complaint were filed before the Board
following the publication of the school paper that contains obscene, vulgar, and sexually explicit contents. Prior
to the disciplinary sanction to the defendants they were required to submit a written statement to answer the
complaints against them to the Discipline Committee but the defendants, instead of doing so wrote to the
Committee to transfer the case to the DECS which they alleged to have the jurisdiction over the issue. Pushing
through with the investigation ex parte the Committee found the defendants guilty and imposed upon them
disciplinary sanctions. Defendants filed before the court for prohibition with preliminary injunction on said
decision of the Committee questioning the jurisdiction of said Discipline Board over the defendants.

ISSUE:
Whether or not the Discipline Board of Miriam College has jurisdiction over the defendants.

HELD:
The court resolved the issue before it by looking through the power of DECS and the Disciplinary
Committee in imposing sanctions upon the defendants. Section 5 (2), Article XIV of the Constitution
guarantees all institutions of higher learning academic freedom. This institutional academic freedom includes
the right of the school or college to decide for itself, its aims and objectives, and how best to attain them free
from outside coercion or interference save possibly when the overriding public welfare calls for some restraint.
Such duty gives the institution the right to discipline its students and inculcate upon them good values, ideals
and attitude. The right of students to free speech in school is not always absolute. The court upheld the right of
students for the freedom of expression but it does not rule out disciplinary actions of the school on the conduct
of their students. Further, Sec. 7 of the of the Campus Journalism Act provides that the school cannot suspend
or expel a student solely on the basis of the articles they write EXCEPT when such article materially disrupts
146
class work of involve substantial disorder or invasion of the rights of others. Therefore the court ruled that the
power of the school to investigate is an adjunct of its power to suspend or expel. It is a necessary corollary to
the enforcement of rules and regulations and the maintenance of a safe and orderly educational environment
conducive to learning. That power, like the power to suspend or expel, is an inherent part of the academic
freedom of institutions of higher learning guaranteed by the Constitution. The court held that Miriam College
has the authority to hear and decide the cases filed against respondent students.

Re: Letter of the UP Law Faculty entitled “Restoring Integrity: A Statement by the Faculty of the
University of the Philippines College of Law on the Allegations of Plagiarism and
Misrepresentation in the Supreme Court.”

Facts:
Allegations of plagiarism were hurled by Atty. Harry L. Roque, Jr. and Atty. Romel R. Bagares against
Justice Mariano C. Del Castillo for his ponencia in the case of Vinuya v. Executive Secretary. In said case, the
Court denied the petition for certiorari filed by Filipino comfort women to compel certain officers of the
executive department to espouse their claims for reparation and demand apology from the Japanese
government for the abuses committed against them by the Japanese soldiers during World War II. Attys.
Roque and Bagares represent the comfort women in Vinuya v. Executive Secretary, which is presently the
subject of a motion for reconsideration.

UP Law Faculty
37 members of the faculty of the University of the Philippines College of Law published a statement on
the allegations of plagiarism and misrepresentation relative to the Court’s decision in Vinuya v. Executive
Secretary. Essentially, the faculty of the UP College of Law, headed by its dean, Atty. Marvic M.V.F. Leonen,
calls for the resignation of Justice Del Castillo in the face of allegations of plagiarism in his work.
Notably, while the statement was meant to reflect the educators’ opinion on the allegations of
plagiarism against Justice Del Castillo, they treated such allegation not only as an established fact, but a truth.
In particular, they expressed dissatisfaction over Justice Del Castillo’s explanation on how he cited the primary
sources of the quoted portions and yet arrived at a contrary conclusion to those of the authors of the articles
supposedly plagiarized.
The insult to the members of the Court was aggravated by imputations of deliberately delaying the
resolution of the said case, its dismissal on the basis of “polluted sources,” the Court’s alleged indifference to
the cause of petitioners, as well as the supposed alarming lack of concern of the members of the Court for even
the most basic values of decency and respect.
The Court could hardly perceive any reasonable purpose for the faculty’s less than objective comments
except to discredit the Decision in the Vinuya case and undermine the Court’s honesty, integrity and
competence in addressing the motion for its reconsideration. As if the case on the comfort women’s claims is
not controversial enough, the UP Law faculty would fan the flames and invite resentment against a resolution
that would not reverse the said decision. This runs contrary to their obligation as law professors and officers of
the Court to be the first to uphold the dignity and authority of this Court, to which they owe fidelity according
to the oath they have taken as attorneys, and not to promote distrust in the administration of justice.

Issue:
Does the Show Cause Resolution deny respondents their freedom of expression?

Held:
No. A reading of the show cause resolution will plainly show that it was neither the fact that respondents
had criticized a decision of the court nor that they had charged one of its members of plagiarism that motivated
the said resolution. Respondents who are neither parties nor counsels in the vinuya case, have expressed their
opinions in favor of the petitioners in the said pending case for the proper disposition and consideration of the
court that gave rise to the said resolution. The show cause resolution painstakingly enumerated the statements
that the court considered excessive and uncalled for under the circumstances surrounding the issuance,
publication and letter submission to this court of the UP law faculty’s restoring integrity statement.

147
8. Freedom of Religion
ANG LADLAD VS. COMELEC
618 SCRA 32 (2010)

FACTS:
Petitioner is a national organization which represents the lesbians, gays, bisexuals, and trans-genders.
It filed a petition for accreditation as a party-list organization to public respondent. However, due to moral
grounds, the latter denied the said petition. To buttress their denial, COMELEC cited certain biblical and
quranic passages in their decision. It also stated that since their ways are immoral and contrary to public
policy, they are considered nuissance. In fact, their acts are even punishable under the Revised Penal Code in
its Article 201.
A motion for reconsideration being denied, Petitioner filed this instant Petition on Certiorari under
Rule 65 of the ROC.
AngLadlad argued that the denial of accreditation, insofar as it justified the exclusion by using religious
dogma, violated the constitutional guarantees against the establishment of religion. Petitioner also claimed that
the Assailed Resolutions contravened its constitutional rights to privacy, freedom of speech and assembly, and
equal protection of laws, as well as constituted violations of the Philippines’ international obligations against
discrimination based on sexual orientation.
In its Comment, the COMELEC reiterated that petitioner does not have a concrete and genuine national
political agenda to benefit the nation and that the petition was validly dismissed on moral grounds. It also
argued for the first time that the LGBT sector is not among the sectors enumerated by the Constitution and RA
7941, and that petitioner made untruthful statements in its petition when it alleged its national existence
contrary to actual verification reports by COMELEC’s field personnel.

ISSUES:
Whether or not the respondent violated the Non-establishment clause of the Constitution;
Whether or not respondent erred in denying Petitioners application on moral and legal grounds.

HELD:
Respondent mistakenly opines that our ruling in Ang Bagong Bayani stands for the proposition that
only those sectors specifically enumerated in the law or related to said sectors (labor, peasant, fisher folk,
urban poor, indigenous cultural communities, elderly, handicapped, women, youth, veterans, overseas
workers, and professionals) may be registered under the party-list system. As we explicitly ruled in Ang Bagong
Bayani-OFW Labor Party v. Commission on Elections, “the enumeration of marginalized and under-
represented sectors is not exclusive”. The crucial element is not whether a sector is specifically enumerated, but
whether a particular organization complies with the requirements of the Constitution and RA 7941.
Our Constitution provides in Article III, Section 5 that “[n]o law shall be made respecting an
establishment of religion, or prohibiting the free exercise thereof.” At bottom, what our non-establishment
clause calls for is “government neutrality in religious matters.” Clearly, “governmental reliance on religious
justification is inconsistent with this policy of neutrality.” We thus find that it was grave violation of the non-
establishment clause for the COMELEC to utilize the Bible and the Koran to justify the exclusion of Ang
Ladlad. Be it noted that government action must have a secular purpose.
Respondent has failed to explain what societal ills are sought to be prevented, or why special protection
is required for the youth. Neither has the COMELEC condescended to justify its position that petitioner’s
admission into the party-list system would be so harmful as to irreparably damage the moral fabric of society.
We also find the COMELEC’s reference to purported violations of our penal and civil laws flimsy, at
best; disingenuous, at worst. Article 694 of the Civil Code defines a nuisance as “any act, omission,
establishment, condition of property, or anything else which shocks, defies, or disregards decency or morality,”
the remedies for which are a prosecution under the Revised Penal Code or any local ordinance, a civil action, or
abatement without judicial proceedings. A violation of Article 201 of the Revised Penal Code, on the other
hand, requires proof beyond reasonable doubt to support a criminal conviction. It hardly needs to be
emphasized that mere allegation of violation of laws is not proof, and a mere blanket invocation of public

148
morals cannot replace the institution of civil or criminal proceedings and a judicial determination of liability or
culpability.
As such, we hold that moral disapproval, without more, is not a sufficient governmental interest to
justify exclusion of homosexuals from participation in the party-list system. The denial of AngLadlad’s
registration on purely moral grounds amounts more to a statement of dislike and disapproval of homosexuals,
rather than a tool to further any substantial public interest.

IGLESIA NI CRISTO VS. COURT OFAPPEALS


[259 SCRA 529; G.R. NO. 119673; 26 JUL 1996]

FACTS:
Petitioner has a television program entitled" Ang Iglesia ni Cristo" aired on Channel 2 every Saturday
and on Channel 13 every Sunday. The program presents and propagates petitioner’s religious beliefs, doctrines
and practices often times incomparative studies with other religions. Petitioner submitted to the respondent
Board of Review for Moving Pictures and Television the VTR tapes of its TV program Series Nos. 116, 119, 121
and 128. The Board classified the series as "X" or not for public viewing on the ground that they "offend and
constitute an attack against other religions which is expressly prohibited by law." On November 28, 1992, it
appealed to the Office of the President the classification of its TV Series No. 128 which allowed it through a
letter of former Executive Secretary Edelmiro A. Amante, Sr., addressed for Henrietta Mendez reversing the
decision of the respondent Board. According to the letter the episode in is protected by the constitutional
guarantee of free speech and expression and no indication that the episode poses any clear and present danger.
Petitioner also filed Civil Case. Petitioner alleged that the respondent Board acted without jurisdiction or with
grave abuse of discretion in requiring petitioner to submit the VTR tapes of its TV program and in x-rating
them. It cited its TV Program Series Nos. 115,119, 121 and 128. In their Answer, respondent Board invoked its
power under PD No. 19861 in relation to Article 201 of the Revised Penal Code. The Iglesia ni Cristo insists on
the literal translation of the bible and says that our (Catholic) veneration of the Virgin Mary is not to be
condoned because nowhere it is found in the bible. The board contended that it outrages Catholic and
Protestant's beliefs. RTC ruled in favor of petitioners. CA however reversed it hence this petition.

ISSUE:
Whether or Not the "ang iglesia ni cristo" program is not constitutionally protected as a form of
religious exercise and expression.

HELD:
Yes. Any act that restrains speech is accompanied with presumption of invalidity. It is the burden of the
respondent Board to overthrow this presumption. If it fails to discharge this burden, its act of censorship will
be struck down. This is true in this case. So-called "attacks" are mere criticisms of some of the deeply held
dogmas and tenets of other religions. RTC’s ruling clearly suppresses petitioner’s freedom of speech and
interferes with its right to free exercise of religion. “attack” is different from “offend “any race or religion. The
respondent Board may disagree with the criticisms of other religions by petitioner but that gives it no excuse to
interdict such criticisms, however, unclean they may be. Under our constitutional scheme, it is not the task of
the State to favor any religion by protecting it against an attack by another religion. Religious dogmas and
beliefs are often at war and to preserve peace among their followers, especially the fanatics, the establishment
clause of freedom of religion prohibits the State from leaning towards any religion. Respondent board cannot
censor the speech of petitioner Iglesia ni Cristo simply because it attacks other religions, even if said religion
happens to be the most numerous church in our country. The basis of freedom of religion is freedom of thought
and it is best served by encouraging the marketplace of dueling ideas. It is only where it is unavoidably
necessary to prevent an immediate and grave danger to the security and welfare of the community that
infringement of religious freedom may be justified, and only to the smallest extent necessary to avoid the
danger. There is no showing whatsoever of the type of harm the tapes will bring about especially the gravity
and imminence of the threatened harm. Prior restraint on speech, including religious speech, cannot be
justified by hypothetical fears but only by the showing of a substantive and imminent evil. It is inappropriate to
apply the clear and present danger test to the case at bar because the issue involves the content of speech and
not the time, place or manner of speech. Allegedly, unless the speech is first allowed, its impact cannot be
149
measured, and the causal connection between the speech and the evil apprehended cannot be established. The
determination of the question as to whether or not such vilification, exaggeration or fabrication falls within or
lies outside the boundaries of protected speech or expression is a judicial function which cannot be arrogated
by an administrative body such as a Board of Censors." A system of prior restraint may only be validly
administered by judges and not left to administrative agencies.

ESTRADA VS. ESCRITOR


[492 SCRA 1 ; AM NO P-02-1651; 22 JUN 2006]

FACTS:
Escritor is a court interpreter since 1999 in the RTC of Las Pinas City. She has been living with Quilapio,
a man who is not her husband, for more than twenty five years and had a son with him as well. Respondent’s
husband died a year before she entered into the judiciary while Quilapio is still legally married to another
woman.
Complainant Estrada requested the Judge of said RTC to investigate respondent. According to
complainant, respondent should not be allowed to remain employed therein for it will appear as if the court
allows such act.
Respondent claims that their conjugal arrangement is permitted by her religion—the Jehovah’s
Witnesses and the Watch Tower and the Bible Trace Society. They allegedly have a ‘Declaration of Pledging
Faithfulness’ under the approval of their congregation. Such a declaration is effective when legal impediments
render it impossible for a couple to legalize their union.

ISSUE:
Whether or not the State could penalize respondent for such conjugal arrangement.

HELD:
No. The State could not penalize respondent for she is exercising her right to freedom of religion. The
free exercise of religion is specifically articulated as one of the fundamental rights in our Constitution. As
Jefferson put it, it is the most inalienable and sacred of human rights. The State’s interest in enforcing its
prohibition cannot be merely abstract or symbolic in order to be sufficiently compelling to outweigh a free
exercise claim. In the case at bar, the State has not evinced any concrete interest in enforcing the concubinage
or bigamy charges against respondent or her partner. Thus the State’s interest only amounts to the symbolic
preservation of an unenforced prohibition.
Furthermore, a distinction between public and secular morality and religious morality should be kept in
mind. The jurisdiction of the Court extends only to public and secular morality.
The Court further states that our Constitution adheres the benevolent neutrality approach that gives
room for accommodation of religious exercises as required by the Free Exercise Clause. This benevolent
neutrality could allow for accommodation of morality based on religion, provided it does not offend compelling
state interests. Assuming arguendo that the OSG has proved a compelling state interest, it has to further
demonstrate that the state has used the least intrusive means possible so that the free exercise is not infringed
any more than necessary to achieve the legitimate goal of the state. Thus the conjugal arrangement cannot be
penalized for it constitutes an exemption to the law based on her right to freedom of religion.

150
9. Liberty of Abode and Freedom of Movement
Marcos vs. Manglapus,
[G.R. # 88211 September 15, 1989]

FACTS:
Ferdinand E. Marcos was deposed from the presidency and was forced into exile. Corazon Aquino’s
ascension into presidency was challenged by failed coup attempts as well as by plots of Marcos loyalists and the
Marcoses themselves. Marcos, in his deathbed, has signified his wish to return to the Philippines to die. But
President Aquino, considering the dire consequences to the nation of his return has stood firmly on the
decision to bar the return of Mr. Marcos and his family. Hence, this petition for mandamus and prohibition
asks the Courts to order the respondents to issue travel documents to Mr. Marcos and the immediate members
of his family and to enjoin the implementation of the President's decision to bar their return to the Philippines.

ISSUE:
Whether or not the President has the power to bar the return of Marcos to the Philippines.

HELD:
Petition Dismissed. The request of the Marcoses must not be treated only in the light of constitutional
provisions, it must be treated as a matter that is appropriately addressed to those residual unstated powers of
the President which are implicit in to the paramount duty residing in that office to safeguard and protect
general welfare. Such request or demand should submit to the exercise of a broader discretion on the part of
the President to determine whether it must be granted or denied. It is found by the Court that from the
pleadings filed by the parties, from their oral arguments, and the facts revealed during the briefing in chambers
by the Chief of Staff of the Armed Forces of the Philippines and the National Security Adviser, wherein
petitioners and respondents were represented, that there exist factual bases for the President's decision. Hence,
this act cannot be said to have been done arbitrarily or capriciously. Further, the ponencia (the coups, the
communist threat, peace and order issues especially in Mindanao, Marcos loyalists plotting) bolsters the
conclusion that the return of Marcos will only exacerbate the situation in the country.
Another reason of the Court...
“We cannot also lose sight of the fact that the country is only now beginning to recover from the
hardships brought about by the plunder of the economy attributed to the Marcoses and their close
associates and relatives, many of whom are still here in the Philippines in a position to destabilize the
country, while the Government has barely scratched the surface, so to speak, in its efforts to recover the
enormous wealth stashed away by the Marcoses in foreign jurisdictions.

Gudani vs. Senga


498 SCRA 671 (2006)

FACTS:
Senator Rodolfo Biazon invited several senior officers of the AFP, including Gen. Gudani and Col.
Balutan, to appear at a public hearing before the Senate Committee on National Defense and Security to shed
light on the “Hello Garci” controversy. Gudani and Balutan were directed by AFP Chief of Staff Gen. Senga, per
instruction of Pres. Arroyo, not testify before said Committee. On the very day of the hearing, President Gloria-
Macapagal-Arroyo issued Executive Order No. 464 enjoining officials of the executive department including
the military establishment from appearing in any legislative inquiry without her approval. However, the two
testified before the Senate, prompting Gen. Senga to issue an order directing Gudani and Balutan to appear
before the Office of the Provost Marshal General (OPMG) on 3 October 2005 for investigation. The following
day, Gen. Gudani was compulsorily retired from military service. After investigation, the OPMG recommended
that the two be charged with violation of Article of War 65, on willfully disobeying a superior officer. Thus,
Gudani and Balutan filed a petition for certiorari and prohibition seeking that (1) the order of President Arroyo
be declared unconstitutional; (2) the charges against them be quashed; and (3) Gen. Senga and their

151
successors-in-interest or persons acting for and on their behalf or orders, be permanently enjoined from
proceeding against them, as a consequence of their having testified before the Senate.

ISSUES:
1. Whether or not may the President prevent a member of the armed forces from testifying before a
legislative inquiry?
2. Whether or the members of the military be compelled to attend legislative inquiries even if the
President desires otherwise?
3. Whether the court-martial have jurisdiction over Gudani considering his retirement last 4 October
2005?

HELD:
1. Yes. The President has constitutional authority to do so, by virtue of her power as commander-in-chief,
and that as a consequence a military officer who defies such injunction is liable under military justice.
Our ruling that the President could, as a general rule, require military officers to seek presidential
approval before appearing before Congress is based foremost on the notion that a contrary rule unduly
diminishes the prerogatives of the President as commander-in-chief. Congress holds significant control
over the armed forces in matters such as budget appropriations and the approval of higher-rank
promotions, yet it is on the President that the Constitution vests the title as commander-in-chief and all
the prerogatives and functions appertaining to the position. Again, the exigencies of military discipline
and the chain of command mandate that the President’s ability to control the individual members of the
armed forces be accorded the utmost respect. Where a military officer is torn between obeying the
President and obeying the Senate, the Court will without hesitation affirm that the officer has to choose
the President. After all, the Constitution prescribes that it is the President, and not the Senate, who is
the commander-in-chief of the armed forces.

2. At the same time, the refusal of the President to allow members of the military to appear before
Congress is still subject to judicial relief. The Constitution itself recognizes as one of the legislature’s
functions is the conduct of inquiries in aid of legislation. Inasmuch as it is ill-advised for Congress to
interfere with the President’s power as commander-in-chief, it is similarly detrimental for the President
to unduly interfere with Congress’s right to conduct legislative inquiries. The impasse did not come to
pass in this petition, since petitioners testified anyway despite the presidential prohibition. Yet the
Court is aware that with its pronouncement today that the President has the right to require prior
consent from members of the armed forces, the clash may soon loom or actualize.
We believe and hold that our constitutional and legal order sanctions a modality by which
members of the military may be compelled to attend legislative inquiries even if the President desires
otherwise, a modality which does not offend the Chief Executive’s prerogatives as commander-in-chief.
The remedy lies with the courts.
The fact that the executive branch is an equal, coordinate branch of government to the
legislative creates a wrinkle to any basic rule that persons summoned to testify before Congress must do
so. There is considerable interplay between the legislative and executive branches, informed by due
deference and respect as to their various constitutional functions. Reciprocal courtesy idealizes this
relationship; hence, it is only as a last resort that one branch seeks to compel the other to a particular
mode of behavior. The judiciary, the third coordinate branch of government, does not enjoy a similar
dynamic with either the legislative or executive branches. Whatever weakness inheres on judicial power
due to its inability to originate national policies and legislation, such is balanced by the fact that it is the
branch empowered by the Constitution to compel obeisance to its rulings by the other branches of
government.

3. An officer whose name was dropped from the roll of officers cannot be considered to be outside the
jurisdiction of military authorities when military justice proceedings were initiated against him before
the termination of his service. Once jurisdiction has been acquired over the officer, it continues until his
case is terminated. Military jurisdiction has fully attached to Gen. Gudani inasmuch as both the acts
complained of and the initiation of the proceedings against him occurred before he compulsorily retired
on 4 October 2005.
152
10. Right to Information
Tanada, et al. v. Tuvera, et al.

FACTS:
Petitioners Senator Lorenzo Tañada, Atty. Abraham Sarmiento, and the Movement of Attorneys for
Brotherhood, Integrity and Nationalism, Inc. invoked due process in demanding the disclosure of a number of
presidential decrees which they claimed had not been published as required by law. The government argued
that while publication was necessary as a rule, it was not so when it was “otherwise provided,” as when the
decrees themselves declared that they were to become effective immediately upon their approval. In the
decision of this case on April 24, 1985, the Court affirmed the necessity for the publication of some of these
decrees. The Court ordered the respondents to publish in the Official Gazette all unpublished Presidential
Issuances which are of general force and effect. The petitioners suggest that there should be no distinction
between laws of general applicability and those which are not; that publication means complete publication;
and that the publication must be made forthwith in the Official Gazette.

ISSUES:
1) Whether or not a distinction be made between laws of general applicability and laws which are not as to
their publication.
2) Whether or not a publication shall be made in publications of general circulation.

RULINGS:
1) No, a distinction cannot be made between laws of general applicability and laws which are not as to
their publication.
“Laws” should refer to all laws and not only to those of general application, for strictly speaking,
all laws relate to the people in general albeit there are some that do not apply to them directly. A law
without any bearing on the public would be invalid as an intrusion of privacy or as class legislation or as
an ultra vires act of the legislature. To be valid, the law must invariably affect the public interest even if
it might be directly applicable only to one individual, or some of the people only, and not to the public
as a whole.

2) Yes, a publication shall be made in publications of general circulation.


All statutes, including those of local application and private laws, shall be published as a
condition for their effectivity, which shall begin 15 days after publication unless a different effectivity
date is fixed by the legislature. Publication must be in full or it is no publication at all, since its purpose
is to inform the public of the content of the law. Article 2 of the Civil Code provides that publication of
laws must be made in the Official Gazette, and not elsewhere, as a requirement for their effectivity.

DISPOSITIVE PORTION:
In its December 29, 1986 Resolution, the Supreme Court hereby declared that all laws as above defined
shall immediately upon their approval, or as soon thereafter as possible, be published in full in the Official
Gazette, to become effective only after fifteen days from their publication, or on another date specified by the
legislature, in accordance with Article 2 of the Civil Code.

In Re: Production of Court Records and Documents and the Attendance of Court officials and
employees as witnesses under the subpoenas of February 10, 2012 and the various letters for
the Impeachment Prosecution Panel dated January 19 and 25, 2012 (En banc Resolution)

Facts:
During the impeachment proceedings against Chief Justice Corona, the prosecution Panel manifested
in a COMPLIANCE that it would present about 100 witnesses which included Justices of the Supreme Court,
and Court officials and employees who will testify on matters internal to the Court and almost a thousand
documents.
153
Letters were sent to the SC asking for the examination of records, and the issuance of certified true
copies of the rollos and the Agenda and Minutes of the Deliberations of various cases decided by the SC for
purposes of the -Impeachment Complaint. Subpoena Ad Testificandum et Duces Tecum And Subpoena Ad
Testificandum were also issued against Clerks of Court of the SC. In light of the subpoenas served, the urgent
need for a court ruling and based on the Constitution, the pertinent laws and of the Court’s rules and policies,
we shall now determine how the Court will comply with the subpoenas and the letters of the Prosecution
Impeachment Panel.

Issues:
1. Whether or not court records are accessible as a policy of transparency.
2. Are Court Records considered confidential?
3. Are Court deliberations recognized as privileged communication?

Ruling:
1. Underlying every request for information is the constitutional right to information (a right granted to
the people) that Article III, Section 7 of the Constitution provides:
Section 7. The right of the people to information on matters of public concern shall be
recognized. Access to official records, and to documents and papers pertaining to officials acts,
transactions, or decisions, as well as to government research data used as basis for policy
development, shall be afforded the citizen, subject to such limitation as may be provided by law.

The right to information, by its very nature and by the Constitution’s own terms, is not absolute.
In line with the public’s constitutional right to information, the Court has adopted a policy of
transparency with respect to documents in its possession or custody, necessary to maintain the integrity
of its sworn duty to adjudicate justiciable disputes. This policy is embodied in terms of Court Rules.
The rule grants access to court records to any person, subject to payment of fees and compliance
with rules; it is not necessary that the request be made by a party to the case. This grant, however, is not
as open nor as broad as its plain terms appear to project, as it is subject to the limitations the laws and
the Court’s own rules provide.

2. In the Judiciary, privileges against disclosure of official records “create a hierarchy of rights that protect
certain confidential relationships over and above the public’s evidentiary need” or “right to every man’s
evidence.” Accordingly, certain information contained in the records of cases before the Supreme Court
are considered confidential and are exempt from disclosure. To reiterate, the need arises from the
dictates of the integrity of the Court’s decision-making function which may be affected by the disclosure
of information.
The Internal Rules of the Supreme Court (IRSC) prohibits the disclosure of
1) the result of the raffle of cases,
2) the actions taken by the Court on each case included in the agenda of the Court’s
session,
3) the deliberations of the Members in court sessions on cases and matters
pending before it.

3. Court deliberations are traditionally recognized as privileged communication. Section 2,


Rule 10 of the IRSC provides:
Section 2. Confidentiality of court sessions. – Court sessions are executive in character,
with only the Members of the Court present. Court deliberations are confidential and
shall not be disclosed to outside parties, except as may be provided herein or as
authorized by the Court.

The privilege is not exclusive to the Judiciary. We have in passing recognized the claim of
this privilege by the two other branches of government in Chavez v. Public Estates Authority (speaking
through J. Carpio) when the Court declared that –
[t]he information x x x like internal deliberations of the Supreme Court and
other collegiate courts, or executive sessions of either house of Congress, are
154
recognized as confidential. This kind of information cannot be pried open by a co-equal
branch of government. A frank exchange of exploratory ideas and assessments, free
from the glare of publicity and pressure by interested parties, is essential to
protect the independence of decision-making of those tasked to exercise
Presidential, Legislative and Judicial power.

Under the law, therefore, the Members of the Court may not be compelled to testify in
the impeachment proceedings against the Chief Justice or other Members of the Court
about information they acquired in the performance of their official function of
adjudication, such as information on how deliberations were conducted or the material
inputs that the justices used in decision-making, because the end-result would be the
disclosure of confidential information that could subject them to criminal prosecution.
Such act violates judicial privilege (or the equivalent of executive privilege) as it pertains to the exercise
of the constitutional mandate of adjudication.
Jurisprudence implies that justices and judges may not be subject to any compulsory
process in relation to the performance of their adjudicatory functions.
This ruling was dictated in no small measure by the principle of comity mentioned above.
Inter-departmental courtesy demands that the highest levels of each department be
exempt from the compulsory processes of the other departments on matters related to
the functions and duties of their office.
With respect to Court officials and employees, the same rules on confidentiality that
apply to justices and judges apply to them. They are barred from disclosing (1) the result of the
raffle of cases, (2) the actions taken by the Court on each case included in the agenda of the Court’s
session, and (3) the deliberations of the Members in court sessions on cases and matters pending before
it. They are subject as well to the disqualification by reason of privileged communication and the sub
judice rule. As stated above, these rules extend to documents and other communications which cannot
be disclosed.
These privileges, incidentally, belong to the Judiciary and are for the Supreme Court (as the
representative and entity speaking for the Judiciary), and not for the individual justice, judge, or court
official or employees to waive. Thus, every proposed waiver must be referred to the Supreme Court for
its consideration and approval.

RE: REQUEST FOR COPY OF 2008 STATEMENT OF ASSETS, LIABILITIES AND NETWORTH
[SALN] AND PERSONAL DATA SHEET OR CURRICULUM VITAE OF THE JUSTICES OF THE
SUPREME COURT AND OFFICERS AND EMPLOYEES OF THE JUDICIARY. 672 SCRA 27 A. M.
No. 09-8-6-SC, June 13, 2012. (En banc Resolution)

Facts:
Rowena Paraan, Research Director of the PCIJ, sought copies of the SALN of the Justices of the
Supreme Court for the year 2008. She also requested for copies of the Personal Data Sheet of the Justices of
this Court for the purpose of updating their database of information on government officials.

Issue #1:
Can the SALN of justices be accessed via the right to information?

Ruling:
Yes. The right to information goes hand-in-hand with the constitutional policies of full public disclosure
and honesty in the public service

Issue #2:
What are the limitations on the constitutional right to information?

Ruling:

155
The right to information is not absolute. It is further subject to such limitations as may be provided by
law. Jurisprudence has provided the following limitations to that right:(1) national security matters and
intelligence information;(2) trade secrets and banking transactions;(3) criminal matters; and(4) other
confidential information such as confidential or classified information officially known to public officers and
employees by reason of their office and not made available to the public as well as diplomatic correspondence,
closed door Cabinet meetings and executive sessions of either house of Congress, and the internal deliberations
of the Supreme Court.

VILLANUEVA VS. JBC, (En banc)


G.R. No. 211833, April 07, 2015 (Decision)

FACTS:
Presiding Judge Ferdinand R. Villanueva (petitioner) applied for the vacant position of Presiding Judge
in Regional Trial Courts (RTC’s) but was not included in the list of candidates for the said position because of
the Judicial and Bar Council’s (JBC) long standing policy of opening the chance for promotion to second-level
courts to, among others, incumbent judges who have served in their current position for at least five years, and
since the petitioner has been a judge only for more than a year, he was excluded from the list. This caused the
petitioner to take recourse to the Supreme Court.

ISSUE:
Whether or not the policy of JBC requiring five years of service as judges of first-level courts before they
can qualify as applicant to second-level courts violates the equal protection clause.

RULING:
No. The equal protection clause of the Constitution does not require the universal application of the
laws to all persons or things without distinction; what it requires is simply equality among equals as
determined according to a valid classification. Consideration of experience by JBC as one factor in choosing
recommended appointees does not constitute a violation of the equal protection clause. The JBC does not
discriminate when it employs number of years of service to screen and differentiate applicants from the
competition. The number of years of service provides a relevant basis to determine proven competence which
may be measured by experience, among other factors.
The classification created by the challenged policy satisfies the rational basis test. Substantial
distinctions do exist between lower court judges with five year experience and those with less than five years of
experience, like the petitioner, and the classification enshrined in the assailed policy is reasonable and relevant
to its legitimate purpose. The Court, thus, rules that the questioned policy does not infringe on the equal
protection clause as it is based on reasonable classification intended to gauge the proven competence of the
applicants. Therefore, the said policy is valid and constitutional.

DISPOSITIVE PORTION:
The Supreme Court in its April 7, 2015 Decision DISMISSED the petition. The Court, however,
DIRECTS that the Judicial and Bar Council comply with the publication requirement of (1) the assailed policy
requiring five years of experience as judges of first-level courts before they can qualify as applicant to the
Regional Trial Court, and (2) other special guidelines that the Judicial and Bar Council is or will be
implementing.

Antolin v. Abelardo T. Domondon, Jose A. Gangan, and Violeta J. Josef (1st Div.)
623 SCRA 163 July 5, 2010 (Decision)
Del Castillo, J.:

Facts:
Petitioner Hazel Ma. C. Antolin took the 1997 CPA Board Exams but failed, receiving failing grades
from four out of seven subjects. Convinced that she deserved to pass, she wrote to respondent Abelardo
Domondon, Acting Chairman of the Board of Accountancy, and requested that her answer sheets be re-
156
corrected. Her answer sheets were shown but these consisted merely of shaded marks. She requested for copies
of the questionnaire, their respective answer keys, and an explanation of the grading system used in each
subject. Respondent denied the request.

Issue:
Whether or not Antolin has a right to obtain copies of the examination papers.

Petitioner:
Primarily, petitioner filed a petition for mandamus with damages against the Board of Accountancy and
its members before the Manila RTC, praying that the court would order the board to furnish her with copies of
the examination papers and other documents and materials. She later amended her petition, pleading a cause
of action for the access of the documents requested for. However, the RTC dismissed the petition on the ground
that the petition had already become moot and academic since she already passed the 1998 CPA Board Exams.
However, an omnibus order of the trial court reconsidered her case. The CA, however, ruled that (i) the PRC
regulation preventing her from gaining access to said documents were valid limitations on petitioner’s right to
information and access to government documents; (ii) that the examination documents were not of public
concern; (iii) it was not the function of the respondents to review and reassess the answers to exam questions
of a failing examinee; (iv) the case was moot and academic as petitioner already passed the 1998 CPA Board
Exams; (v) that petitioner failed to exhaust administrative remedies, having not elevated the matter to the PRC
before seeking judicial intervention. Petitioner insists she has the Constitutional right to gain access to said
examination documents, that she did not need to exhaust administrative remedies since no recourse to the PRC
was available as only a pure question of law is involved in the case and that her petition was not rendered moot
and academic when she passed the 1998 CPA Board Exams.

Respondents:
Respondent primarily denied the request of petitioner on two grounds: first, the PRC rules only
permitted access to the petitioner’s answer sheet and that reconsideration of rating shall be effected only on
grounds of mechanical error in grading the answer sheets or malfeasance; secondly, he clarified that the Board
was precluded from releasing the exam papers as such act were considered unprofessional by the PRC
resolution. The Board did not find any mechanical error in the grading of petitioner’s test papers. Nonetheless,
the petitioner elevated the case to the RTC wherein respondents argue that petitioner was not entitled for the
relief sought, among others. They also filed to dismiss the petition on damages since (1) petitioner failed to
exhaust administrative remedies, (2) the petition stated no cause of action as there was no ministerial duty to
release the information demanded, (3) and the constitutional right to information on matters of public concern
is subject to the limitation set forth by the PRC Resolution No. 338. Also, they added that the petition had
become moot and academic since petitioner already passed the 1998 CPA Board Exams.

Dispositive Portion:
IN VIEW OF THE FOREGOING, the petitions are GRANTED. The December 11, 2006 and
February 16, 2004 Decisions of the Court of Appeals in CA-GR SP No. 76546 and CA-GR SP No. 76498,
respectively, are hereby SET ASIDE. The November 11, 2002 and January 30, 2003 Orders of the Regional
Trial Court of Manila, Branch 33, in Civil Case No. 98-86881 are AFFIRMED. The case is remanded to the
Regional Trial Court for further proceedings.

Court:
The Court rules in favor of the petitioner. Section 28, Article 2 of the Constitution provides that the
State may adopt policies in the disclosure of all its transactions involving public interest while Section 7, Article
3 provides the right of the people to information on matters of public interest. It is clear that the people’s right
to information is limited to matters of public concern and subject to such limitations as may be provided by
law. The Court, nonetheless, conceded that the CPA Board Exams are matters of public concern. The examinees
in particular, would understandably be interested in the fair and competent administration of these exams in
order to ensure that only those qualified are admitted into the accounting profession.
Furthermore, on the issue of mootness, the Court held that the petitioner’s belated passing of the Board
Exams does not automatically mean that her interest in the examination papers has become mere superfluity.

157
Lastly, CA erred in ruling that petitioner should have exhausted administrative remedies before seeking
judicial intervention because issues of law cannot be resolved with finality by an administrative officer.

158
11. Right of Association
SOCIAL SECURITY SYSTEM EMPLOYEE ASSOCIATION (SSSEA), V. COURT OF APPEALS
(Third Division)
175 SCRA 686 July 28, 1989
Cortes, J.:

FACTS:
On June 11, 1987, the SSS filed with the Regional Trial Court of Quezon City a complaint for damages
with a prayer for a writ of preliminary injunction against petitioners, alleging that on June 9, 1987, the officers
and members of SSSEA staged an illegal strike and barricaded the entrances to the SSS Building preventing
non-striking employees from reporting for work and SSS members from transacting business with the SSS that
the strike was reported to the Public Sector Labor – Management Council, which ordered the strikes to return
to work, that the strikers refused to return to work, and that the SSS suffered damages as a result of the strike.
The complaint prayed that a writ of preliminary injunction be issued to enjoin the strike and that the strikers
be ordered to return to work, that the defendants (petitioners herein) be ordered to pay damages, and that the
strike be declared illegal.
It appears that the SSSEA went on strike after the SSS failed to act on the union’s demands, which
included implementation of the provisions of the old SSS-SSSEA collective bargaining agreement (CBA) on
check-off of union dues, payment of accrued overtime pay, night differential pay and holiday pay, conversion of
temporary or contractual employees with six (6) months or more of service into regular and permanent
employees and their entitlement to the same salaries, allowances and benefits given to other regular employees
of the SSS, and payment of the children’s allowance of ₧ 30.00, and after the SSS deducted certain amounts
from the salaries of the employees and allegedly committed acts of discrimination and unfair labor practices.

ISSUE:
Whether or not employees of the Social Security System (SSS) have the right to strike.

RULING:
The 1987 Constitution, in the Article on Social Justice and Human Rights provides that the State “shall
guarantee the rights of all workers to self-organization, collective bargaining and negotiations, and peaceful
concerted activities including the right to strike in accordance with law” (Art. XIII, Sec. 31). Resort to the intent
of the framers of the organic law becomes helpful in understanding the meaning of these provisions. A reading
of the proceedings of the Constitutional Commission that drafted the 1987 Constitution would show that in
recognizing the right of government employees to organize the commissioners intended to limit the right to the
formation of unions or association only without including the right to strike.
Considering that under the 1987 Constitution “the civil service embraces all branches, subdivisions,
instrumentalities, and agencies of the Government, including government-owned or controlled corporations
with original charters”. (Art. IX (B), Sec 2(1) see also Sec 1 of E.O. No. 180 where the employees in the civil
service are denominated as “government employees”) and that the SSS is one such government-controlled
corporation with an original charter having been created under R.A. No. 1161, its employees are part of the civil
service (NASECO v. NLRC G. R. Nos. 69870 & 70295, November 24, 1988) and are covered by the Civil Service
Commission’s memorandum prohibiting strikes. This being the case, the strike staged by the employees of the
SSS was illegal.

DISPOSITIVE PORTION:
The Supreme Court decided that no reversible error having been committed by the Court of Appeals
and denied the instant petition for review and the decision of the appellate court dated March 9, 1988 in CA-G.
R. SP No. 13192 is affirmed. Petitioners’ “Petition/Application for Preliminary and Mandatory Injunction”
dated December 13, 1988 is DENIED.

In re Edillon
84 SCRA 554, August 3, 1978
159
Facts:
The respondent Marcial A. Edillon is a duly licensed practicing attorney in the Philippines. On
November 29, 1975, the Integrated Bar of the Philippines (IBP) Board of Governors unanimously adopted
Resolution No. 75-65 in Administrative Case No. MDD-1 (In the Matter of the Membership Dues Delinquency
of Atty. Marcial A. Edillon) recommending to the Court the removal of the name of the respondent from its Roll
of Attorneys for "stubborn refusal to pay his membership dues" to the IBP since the latter's constitution
notwithstanding due notice.
The core of the respondent's arguments is that the above provisions constitute an invasion of his
constitutional rights in the sense that he is being compelled, as a pre-condition to maintaining his status as a
lawyer in good standing, to be a member of the IBP and to pay the corresponding dues, and that as a
consequence of this compelled financial support of the said organization to which he is admittedly personally
antagonistic, he is being deprived of the rights to liberty and property guaranteed to him by the Constitution.
Hence, the respondent concludes, the above provisions of the Court Rule and of the IBP By-Laws are void and
of no legal force and effect.
The respondent similarly questions the jurisdiction of the Court to strike his name from the Roll of
Attorneys, contending that the said matter is not among the justiciable cases triable by the Court but is rather
of an "administrative nature pertaining to an administrative body."

Issue:
Whether or not the respondent should be disbarred due to refusal to pay his membership dues?

Ruling:
It is the unanimous sense of the Court that the respondent Marcial A. Edillon should be as he is hereby
disbarred, and his name is hereby ordered stricken from the Roll of Attorneys of the Court.

Ratio Decidendi:
To compel a lawyer to be a member of the Integrated Bar is not violative of his constitutional freedom to
associate. Integration does not make a lawyer a member of any group of which he is not already a member. He
became a member of the Bar when he passed the Bar examinations. All that integration actually does is to
provide an official national organization for the well-defined but unorganized and in cohesive group of which
every lawyer is a ready a member. Bar integration does not compel the lawyer to associate with anyone. He is
free to attend or not attend the meetings of his Integrated Bar Chapter or vote or refuse to vote in its elections
as he chooses. The only compulsion to which he is subjected is the payment of annual dues. The Supreme
Court, in order to further the State's legitimate interest in elevating the quality of professional legal services,
may require that the cost of improving the profession in this fashion be shared by the subjects and beneficiaries
of the regulatory program - the lawyers. Moreover, there is nothing in the Constitution that prohibits Court,
under its constitutional power and duty to promulgate rules concerning the admission to the practice of law
and the integration of the Philippine Bar (Article X, Section 5 of the 1973 Constitution), from requiring
members of a privileged class, such as lawyers are, to pay a reasonable fee toward defraying the expenses of
regulation of the profession to which they belong. It is quite apparent that the fee is indeed imposed as a
regulatory measure, designed to raise funds for carrying out the objectives and purposes of integration. Also, it
clear that under the police power of the State, and under the necessary powers granted to the Court to
perpetuate its existence, the respondent's right to practice law before the courts of this country should be and is
a matter subject to regulation and inquiry. And, if the power to impose the fee as a regulatory measure is
recognize, then a penalty designed to enforce its payment, which penalty may be avoided altogether by
payment, is not void as unreasonable or arbitrary. It is sufficient to state then that the matters of admission,
suspension, disbarment and reinstatement of lawyers and their regulation and supervision have been and are
indisputably recognized as inherent judicial functions and responsibilities, and the authorities holding such are
legion. Thus, the Court's jurisdiction was greatly reinforced by our 1973 Constitution when it explicitly granted
to the Court the power to "Promulgate rules concerning pleading, practice ... and the admission to the practice
of law and the integration of the Bar ... (Article X, Sec. 5(5) the power to pass upon the fitness of the respondent
to remain a member of the legal profession is indeed undoubtedly vested in the Court.

160
BPI VS BPI EMPLOYEES UNION – Davao Chapter-Federation of Unions in BPI Unibank (En
banc)
627 SCRA 590 August 10, 2010 (Decision)
Leonardo-De Castro, J:

Facts:
The BSP approved the Articles of Merger executed on January 20, 2000 by and between BPI, and
FEBTC. This Article and Plan of Merger was approved by the SEC on April 7, 2000. Pursuant to the Article and
Plan of Merger, all the assets and liabilities of FEBTC were transferred to and absorbed by BPI as the surviving
corporation. FEBTC employees, including those in its different branches across the country, were hired by
petitioner as its own employees, with their status and tenure recognized and salaries and benefits maintained.
Respondent BPI Employees Union-Davao Chapter-Federation of Unions in BPI Unibank is the
exclusive bargaining agent of BPI’s rank and file employees in Davao City. The former FEBTC rank-and-file
employees in Davao City did not belong to any labor union at the time of the merger. Prior to the effectivity of
the merger, respondent union invited said FEBTC employees to a meeting regarding the Union Shop Clause of
the existing CBA between petitioner BPI and respondent union. The parties both advert to certain provisions of
the existing CBA.
After the meeting called by the union, some of the former FEBTC employees joined the union, while
others refused. Later, however, some of those who initially joined retracted their membership. Respondent
union then sent notices to the former FEBTC employees who refused to join, as well as those who retracted
their membership and called them to a hearing regarding the matter. When these former FEBTC employees
refused to attend the hearing, the president of the Union requested BPI to implement the Union Shop Clause of
the CBA and to terminate their employment.
After two months of management inaction on the request, respondent informed petitioner of its
decision to refer the issue of the implementation of the Union Shop Clause of the CBA to the Grievance
Committee. However, the issue remained unresolved at this level and so it was subsequently submitted for
voluntary arbitration by the parties. Voluntary Arbitrator ruled in favor of petitioner BPI. Respondent Union
filed a motion for reconsideration, but the voluntary arbitrator denied the same. It appealed to the CA and the
CA reversed and set aside the decision of the voluntary arbitrator. Hence, this petition.

Issue:
May a corporation invoke its merger with another corporation as a valid ground to exempt its absorbed
employees from the coverage of a union shop clause contained in its existing CBA with its own certified labor
union.

Employment Contracts
Significantly, too, the Articles of Merger and Plan of Merger dated April 7, 2000 did not contain any
specific stipulation with respect to the employment contracts of existing personnel of the non-surviving entity
which is FEBTC. Unlike the Voluntary Arbitrator, this Court cannot uphold the reasoning that the general
stipulation regarding transfer of FEBTC assets and liabilities to BPI as set forth in the Articles of Merger
necessarily includes the transfer of all FEBTC employees into the employ of BPI and neither BPI nor the
FEBTC employees allegedly could do anything about it. Even if it is so, it does not follow that the absorbed
employees should not be subject to the terms and conditions of employment obtaining in the surviving
corporation.

Ruling:
The rule is that unless expressly assumed, labor contracts such as employment contracts and collective
bargaining agreements are not enforceable against a transferee of an enterprise, labor contracts being in
personam, thus binding only between the parties. A labor contract merely creates an action in personam and
does not create any real right which should be respected by third parties. This conclusion draws its force from
the right of an employer to select his employees and to decide when to engage them as protected under our
Constitution, and the same can only be restricted by law through the exercise of the police power. (BANK OF
THE PHILIPPINE ISLANDS v. BPI EMPLOYEES UNION-DAVAO CHAPTER-FEDERATION OF UNIONS IN
BPI UNIBANK, G.R. No. 164301, August 10, 2010 Equality)

161
12. Eminent Domain
Heirs of Juancho Ardona v. Reyes (En banc)
125 SCRA 220, October 26, 1983
Gutierrez, Jr., J.:

Facts:
The Philippine Tourism Authority filed 4 complaints with the Court of First Instance of Cebu City for
the expropriation of 282 ha of rolling land situated in barangays Malubog and Babag, Cebu City for the
development into integrated resort complexes of selected and well-defined geographic areas with potential
tourism value. The PTA will construct a sports complex, club house, golf course, playground and picnic area on
said land. An electric power grid will also be established by NPC as well as deep well and drainage system.
Complimentary support facilities (malls, coffee shops, etc.) will also be created. The defendants alleged that the
taking is allegedly not impressed with public use under the Constitution. Also, assuming that PTA has such
power, the intended use cannot be paramount to the determination of the land as a land reform area; that
limiting the amount of compensation by legislative fiat is constitutionally repugnant; and that since the land is
under the land reform program, it is the Court of Agrarian Relations and not the Court of First Instance, that
has jurisdiction over the expropriation cases. The Philippine Tourism Authority having deposited with the
PNB, an amount equivalent to 10% of the value of the properties pursuant to PD1533, the lower court issued
separate orders authorizing PTA to take immediate possession of the premises and directing the issuance of
writs of possession. The petitioners who are occupants of the lands, filed a petition for certiorari in the SC. They
contended that (1) the taking was not for public use; (2) the land was covered by the land reform program; and
(3) expropriation would impair the obligation of contracts.

Issue: Whether or not the public use requirement has been complied with.

Ruling:
Yes, there are three provisions of the Constitution which directly provide for the exercise of the power of
eminent domain. Sec 2, Article IV states that private property shall not be taken for public use without just
compensation. Section 6, Article XIV allows the State, in the interest of national welfare or defense and upon
payment of just compensation to transfer to public ownership, utilities and other private enterprises to be
operated by the government. Section 13, Article XIV states that the Batasang Pambansa may authorize upon
payment of just compensation the expropriation of private lands to be subdivided into small lots and conveyed
at cost to deserving citizens.
The concept of public use is not limited to traditional purposes for the construction of roads, bridges,
and the like. The idea that "public use" means "use by the public" has been discarded. As long as the purpose of
the taking is public, then the power of eminent domain comes into play. It is accurate to state then that at
present whatever may be beneficially employed for the general welfare satisfies the requirement of public use.
The petitioners have not shown that the area being developed is land reform area and that the affected persons
have been given emancipation patents and certificates of land transfer. The contract clause has never been
regarded as a barrier to the exercise of the police power and likewise eminent domain.

Makasiar, J, concurring and dissenting


It appearing that the petitioners are not tenants of the parcels of land in question and therefore do not
fall within the purview of the Land Reform Code, the petition should be dismissed on that score alone. There is
no need to decide whether the power of the PTA to expropriate the land in question predicated on the police
power of the State shall take precedence over the social justice guarantee in favor of tenants and the landless.
The welfare of the landless and small land owners should prevail over the right of the PTA to expropriate the
lands just to develop tourism industry, which benefit the wealthy only. Such a position would increase the
disenchanted citizens and drive them to dissidence. The government is instituted primarily for the welfare of
the governed and there are more poor people in this country than the rich. The tourism industry is not essential
to the existence of the government, but the citizens are, and their right to live in dignity should take precedence
over the development of the tourism industry.

162
Hacienda Luisita Inc. (HLI) v. Presidential Agrarian Reform Council (PARC), et al

I. FACTS
On July 5, 2011, the Supreme Court en banc voted unanimously (11-0) to DISMISS/DENY the petition
filed by HLI and AFFIRM with MODIFICATIONS the resolutions of the PARC revoking HLI’s Stock
Distribution Plan (SDP) and placing the subject lands in Hacienda Luisita under compulsory coverage of the
Comprehensive Agrarian Reform Program (CARP) of the government.
The Court however did not order outright land distribution. Voting 6-5, the Court noted that there are
operative facts that occurred in the interim and which the Court cannot validly ignore. Thus, the Court declared
that the revocation of the SDP must, by application of the operative fact principle, give way to the right of the
original 6,296 qualified farmworkers-beneficiaries (FWBs) to choose whether they want to remain as HLI
stockholders or [choose actual land distribution]. It thus ordered the Department of Agrarian Reform (DAR) to
“immediately schedule meetings with the said 6,296 FWBs and explain to them the effects, consequences and
legal or practical implications of their choice, after which the FWBs will be asked to manifest, in secret voting,
their choices in the ballot, signing their signatures or placing their thumb marks, as the case may be, over their
printed names.”
The parties thereafter filed their respective motions for reconsideration of the Court decision.

II. ISSUES
1) Is the operative fact doctrine available in this case?
2) Is Sec. 31 of RA 6657 unconstitutional?
3) Can’t the Court order that DAR’s compulsory acquisition of Hacienda Lusita cover the full 6,443
hectares allegedly covered by RA 6657 and previously held by Tarlac Development Corporation
(Tadeco), and not just the 4,915.75 hectares covered by HLI’s SDP?
4) Is the date of the “taking” (for purposes of determining the just compensation payable to HLI)
November 21, 1989, when PARC approved HLI’s SDP?
5) Has the 10-year period prohibition on the transfer of awarded lands under RA 6657 lapsed on May 10,
1999 (since Hacienda Luisita were placed under CARP coverage through the SDOA scheme on May 11,
1989), and thus the qualified FWBs should now be allowed to sell their land interests in Hacienda
Luisita to third parties, whether they have fully paid for the lands or not?
6) THE CRUCIAL ISSUE: Should the ruling in the July 5, 2011 Decision that the qualified FWBs be given
an option to remain as stockholders of HLI be reconsidered?

III. RULING
[The Court PARTIALLY GRANTED the motions for reconsideration of respondents PARC, et al. with
respect to the option granted to the original farmworkers-beneficiaries (FWBs) of Hacienda Luisita to remain
with petitioner HLI, which option the Court thereby RECALLED and SET ASIDE. It reconsidered its earlier
decision that the qualified FWBs should be given an option to remain as stockholders of HLI, and
UNANIMOUSLY directed immediate land distribution to the qualified FWBs.]

1) YES, the operative fact doctrine is applicable in this case.


[The Court maintained its stance that the operative fact doctrine is applicable in this case since,
contrary to the suggestion of the minority, the doctrine is not limited only to invalid or unconstitutional
laws but also applies to decisions made by the President or the administrative agencies that have the
force and effect of laws. Prior to the nullification or recall of said decisions, they may have produced
acts and consequences that must be respected. It is on this score that the operative fact doctrine should
be applied to acts and consequences that resulted from the implementation of the PARC Resolution
approving the SDP of HLI. The majority stressed that the application of the operative fact doctrine by
the Court in its July 5, 2011 decision was in fact favorable to the FWBs because not only were they
allowed to retain the benefits and homelots they received under the stock distribution scheme, they
were also given the option to choose for themselves whether they want to remain as stockholders of HLI
or not.]

2) NO, Sec. 31 of RA 6657 NOT unconstitutional.


163
[The Court maintained that the Court is NOT compelled to rule on the constitutionality of Sec.
31 of RA 6657, reiterating that it was not raised at the earliest opportunity and that the resolution
thereof is not the lis mota of the case. Moreover, the issue has been rendered moot and academic since
SDO is no longer one of the modes of acquisition under RA 9700. The majority clarified that in its July
5, 2011 decision, it made no ruling in favor of the constitutionality of Sec. 31 of RA 6657, but found
nonetheless that there was no apparent grave violation of the Constitution that may justify the
resolution of the issue of constitutionality.]

3) NO, the Court CANNOT order that DAR’s compulsory acquisition of Hacienda Lusita cover the full
6,443 hectares and not just the 4,915.75 hectares covered by HLI’s SDP.
[Since what is put in issue before the Court is the propriety of the revocation of the SDP, which
only involves 4,915.75 has. of agricultural land and not 6,443 has., then the Court is constrained to rule
only as regards the 4,915.75 has. of agricultural land. Nonetheless, this should not prevent the DAR,
under its mandate under the agrarian reform law, from subsequently subjecting to agrarian reform
other agricultural lands originally held by Tadeco that were allegedly not transferred to HLI but were
supposedly covered by RA 6657.
However since the area to be awarded to each FWB in the July 5, 2011 Decision appears too
restrictive – considering that there are roads, irrigation canals, and other portions of the land that are
considered commonly-owned by farmworkers, and these may necessarily result in the decrease of the
area size that may be awarded per FWB – the Court reconsiders its Decision and resolves to give the
DAR leeway in adjusting the area that may be awarded per FWB in case the number of actual qualified
FWBs decreases. In order to ensure the proper distribution of the agricultural lands of Hacienda Luisita
per qualified FWB, and considering that matters involving strictly the administrative implementation
and enforcement of agrarian reform laws are within the jurisdiction of the DAR, it is the latter which
shall determine the area with which each qualified FWB will be awarded.
On the other hand, the majority likewise reiterated its holding that the 500-hectare portion of
Hacienda Luisita that have been validly converted to industrial use and have been acquired by
intervenors Rizal Commercial Banking Corporation (RCBC) and Luisita Industrial Park Corporation
(LIPCO), as well as the separate 80.51-hectare SCTEX lot acquired by the government, should be
excluded from the coverage of the assailed PARC resolution. The Court however ordered that the
unused balance of the proceeds of the sale of the 500-hectare converted land and of the 80.51-hectare
land used for the SCTEX be distributed to the FWBs.]

4) YES, the date of “taking” is November 21, 1989, when PARC approved HLI’s SDP.
[For the purpose of determining just compensation, the date of “taking” is November 21, 1989
(the date when PARC approved HLI’s SDP) since this is the time that the FWBs were considered to own
and possess the agricultural lands in Hacienda Luisita. To be precise, these lands became subject of the
agrarian reform coverage through the stock distribution scheme only upon the approval of the SDP, that
is, on November 21, 1989. Such approval is akin to a notice of coverage ordinarily issued under
compulsory acquisition. On the contention of the minority (Justice Sereno) that the date of the notice of
coverage [after PARC’s revocation of the SDP], that is, January 2, 2006, is determinative of the just
compensation that HLI is entitled to receive, the Court majority noted that none of the cases cited to
justify this position involved the stock distribution scheme. Thus, said cases do not squarely apply to
the instant case. The foregoing notwithstanding, it bears stressing that the DAR's land valuation is only
preliminary and is not, by any means, final and conclusive upon the landowner. The landowner can file
an original action with the RTC acting as a special agrarian court to determine just compensation. The
court has the right to review with finality the determination in the exercise of what is admittedly a
judicial function.]

5) NO, the 10-year period prohibition on the transfer of awarded lands under RA 6657 has NOT lapsed on
May 10, 1999; thus, the qualified FWBs should NOT yet be allowed to sell their land interests in
Hacienda Luisita to third parties.
[Under RA 6657 and DAO 1, the awarded lands may only be transferred or conveyed after 10
years from the issuance and registration of the emancipation patent (EP) or certificate of land
ownership award (CLOA). Considering that the EPs or CLOAs have not yet been issued to the qualified
164
FWBs in the instant case, the 10-year prohibitive period has not even started. Significantly, the
reckoning point is the issuance of the EP or CLOA, and not the placing of the agricultural lands under
CARP coverage. Moreover, should the FWBs be immediately allowed the option to sell or convey their
interest in the subject lands, then all efforts at agrarian reform would be rendered nugatory, since, at
the end of the day, these lands will just be transferred to persons not entitled to land distribution under
CARP.]

6) YES, the ruling in the July 5, 2011 Decision that the qualified FWBs be given an option to remain as
stockholders of HLI should be reconsidered.
[The Court reconsidered its earlier decision that the qualified FWBs should be given an option to
remain as stockholders of HLI, inasmuch as these qualified FWBs will never gain control [over the
subject lands] given the present proportion of shareholdings in HLI. The Court noted that the share of
the FWBs in the HLI capital stock is [just] 33.296%. Thus, even if all the holders of this 33.296%
unanimously vote to remain as HLI stockholders, which is unlikely, control will never be in the hands of
the FWBs. Control means the majority of [sic] 50% plus at least one share of the common shares and
other voting shares. Applying the formula to the HLI stockholdings, the number of shares that will
constitute the majority is 295,112,101 shares (590,554,220 total HLI capital shares divided by 2 plus
one [1] HLI share). The 118,391,976.85 shares subject to the SDP approved by PARC substantially fall
short of the 295,112,101 shares needed by the FWBs to acquire control over HLI.]

Export Processing Zone Authority


vs Hon. Ceferino Dulay

Facts:
January 1957, president of the Philippines issued proclamation no 1811 reserving a parcel of land in
Lapu-lapu for the establishment of an export processing zone. Not all the reserved land was public. EPZA then
offered to purchase the lands from its registered owners, in the valuation set by PD 464,as amended. The
owners did not agree.
EPZA filed with CFI-Cebu a complaint for expropriation with a prayer for the issuance of a writ of
possession against the landowners. Judge Dulay, later issued a writ of possession authorizing EPZA to take
immediate possession of the premises.
After the recommendation of the appointed commissioners as to the just compensation, EPZA filed for
a motion for reconsideration saying that the PD 464, as amended, superseded the rules of court. The trial court
denied the motion. EPZA then filed for certiorari and mandamus with preliminary restraining order.

Issue: Whether PD 76, 464, 794 and 1533 have repealed the Revised Rules of Court, such that in determining
just compensation in expropriation shall be based only in its market value as declared by the owner or by
assessor, whichever is lower.

Ruling:
PD 464 on just compensation is unconstitutional and void. The method of just compensation provided
by PD 464 is an encroachment on judicial prerogatives, contradicting the Constitution which reserved the
power to determine just compensation to the Court's final determination. We are convinced and so rule that
the trial court correctly stated that the valuation in the decree may only serve as a guiding principle or one of
the factors in determining just compensation but it may not substitute the court's own judgment as to what
amount should be awarded and how to arrive at such amount.
Just compensation means the value of the property at the time of the taking. It means a fair and full
equivalent for the loss sustained. All the facts as to the condition of the property and its surroundings, its
improvements and capabilities, should be considered. The determination of "just compensation" in eminent
domain cases is a judicial function. The executive department or the legislature may make the initial
determinations but when a party claims a violation of the guarantee in the Bill of Rights that private property
may not be taken for public use without just compensation, no statute, decree, or executive order can mandate
that its own determination shall prevail over the court's findings. Much less can the courts be precluded from
looking into the "just-ness" of the decreed compensation.
165
Republic vs. Lim

Facts: Chule Y. Lim, respondent, was an illegitimate child of a Chinese father and a Filipino mother. She filed
a petition to the court for correction of four erroneous entries in her birth certificate to wit: 1) her surname Yu
was misspelled as Yo 2) her father’s name was written as Yo Diu To (Co Tian) when it should have
been Yu Dio To 3) her nationality was entered as Chinese when it should have been Filipino 4)that she was
a legitimate child when she should have been described as illegitimate considering that her parents were never
married. After the trial court conducted the appropriate proceeding, it granted the petition sought by
respondent to set the records straight and in their proper perspective. However, petitioner herein filed an
appeal specifically on the correction of her citizenship (from Chinese to Filipino) not having complied with the
legal requirements for election of citizenship. It cited Article IV, Sec 1(3) of the 1935Constitution and Sec 1, CA
No. 625 which provides the election of citizenship of a legitimate child of a Filipino mother and alien father
upon reaching the age of maturity.

Issue:
Whether or not respondent needs to elect Filipino citizenship upon reaching the age of majority?

Holding:
The constitutional and statutory requirements of electing Filipino citizenship apply only to legitimate
children. The case at bar clearly states that respondent is an illegitimate child of a Filipino mother and alien
father. By being an illegitimate child of a Filipino mother, respondent automatically became a Filipino upon
birth.

MACTAN-CEBU INTERNATIONAL AIRPORT AUTHORITY v. BERNARDO L. LOZADA, GR No.


176625, 2010-02-25

Facts:
Subject of this case is Lot No. 88-SWO-25042 (Lot No. 88), with an area of 1,017 square meters, more
or less, located in Lahug, Cebu City.
Its original owner was Anastacio Deiparine when the same was subject to expropriation proceedings,
initiated by the Republic of the Philippines (Republic), represented by the then Civil Aeronautics
Administration (CAA), for the expansion and improvement of the Lahug Airport. The case was filed with the
then Court of First Instance of Cebu, Third Branch, and docketed as Civil Case No. R-1881.
During the pendency of the expropriation proceedings, respondent Bernardo L. Lozada, Sr. acquired
Lot No. 88 from Deiparine.
On December 29, 1961, the trial court rendered judgment in favor of the Republic and ordered the latter
to pay Lozada the fair market value of Lot No. 88, adjudged at P3.00 per square meter, with consequential
damages by way of legal interest computed from November 16, 1947--the... time when the lot was first occupied
by the airport.
Lozada, with the other landowners, contacted then CAA Director Vicente Rivera, Jr., requesting to
repurchase the lots, as per previous agreement.
On November 29, 1989, then President Corazon C. Aquino issued a Memorandum to the Department of
Transportation, directing the transfer of general aviation operations of the Lahug Airport to the Mactan
International Airport before the end of 1990 and, upon such transfer, the... closure of the Lahug Airport.
From the date of the institution of the expropriation proceedings up to the present, the public purpose
of the said expropriation (expansion of the airport) was never actually initiated, realized, or implemented.
Thus, on June 4, 1996, petitioners initiated a complaint for the recovery of possession and reconveyance
of ownership of Lot No. 88.
On October 22, 1999, the RTC rendered its Decision, disposing as follows... he Court hereby renders
judgment in favor of the plaintiffs, Bernardo L. Lozada, Sr.,... Aggrieved, petitioners interposed an appeal to the
CA.
166
After the filing of the necessary appellate briefs, the CA rendered its assailed Decision dated February
28, 2006, denying petitioners' appeal and affirming in toto the Decision of the RTC, Branch 57, Cebu City.

Issues:
1) the respondents utterly failed to prove that there was a repurchase agreement or compromise
settlement between them and the Government;
2) the judgment in Civil Case No. R-1881 was absolute and unconditional, giving title in fee simple to... the
Republic;

Ruling:
The petition should be denied.
Indeed, the Decision in Civil Case No. R-1881 should be read in its entirety, wherein it is apparent that
the acquisition by the Republic of the expropriated lots was subject to the condition that the Lahug Airport
would continue its operation. The condition not having... materialized because the airport had been
abandoned, the former owner should then be allowed to reacquire the expropriated property.
More particularly, with respect to the element of public use, the expropriator should commit to use the
property pursuant to the purpose stated in the petition for expropriation filed, failing which, it should file
another petition for the new purpose.
In light of these premises, we now expressly hold that the taking of private property, consequent to the
Government's exercise of its power of eminent domain, is always subject to the condition that the property be
devoted to the specific public purpose for which it was taken.
Corollarily, if this particular purpose or intent is not initiated or not at all pursued, and is peremptorily
abandoned, then the former owners, if they so desire, may seek the reversion of the property, subject to the
return of the amount of just compensation received. In such... a case, the exercise of the power of eminent
domain has become improper for lack of the required factual justification.
It bears stressing that both the RTC, Branch 57, Cebu and the CA have passed upon this factual issue
and have declared, in no uncertain terms, that a compromise agreement was, in fact, entered into between the
Government and respondents, with the former undertaking to resell Lot No. 88 to the latter if the improvement
and expansion of the Lahug Airport would not be pursued.
The testimony of Lozada was based on... personal knowledge as the assurance from the government was
personally made to him.
As regards the position of petitioners that respondents' testimonial evidence violates the Statute of
Frauds, suffice it to state that the Statute of Frauds operates only with respect to executory contracts, and does
not apply to contracts which have been completely or partially... performed,... The right of respondents to
repurchase Lot No. 88 may be enforced based on a constructive trust constituted on the property held by the
government in favor of the former.
WHEREFORE, the petition is DENIED.

Principles:
If x x x land is expropriated for a particular purpose, with the condition that when that purpose is ended
or abandoned the property shall return to its former owner, then, of course, when the purpose is terminated or
abandoned the former owner reacquires the... property so expropriated. If x x x land is expropriated for a
public street and the expropriation is granted upon condition that the city can only use it for a public street,
then, of course, when the city abandons its use as a public street, it returns to the former... owner, unless there
is some statutory provision to the contrary. x x x. If, upon the contrary, however, the decree of expropriation
gives to the entity a fee simple title, then, of course, the land becomes the absolute property of the expropriator,
whether it be the State, a... province, or municipality, and in that case the non-user does not have the effect of
defeating the title acquired by the expropriation proceedings. x x x.
When land has been acquired for public use in fee simple, unconditionally, either by the exercise of
eminent domain or by purchase, the former owner retains no right in the land, and the public use may be
abandoned, or the land may be devoted to a different use, without... any impairment of the estate or title
acquired, or any reversion to the former owner. x x x

167
NATIONAL POWER CORPORATION, Petitioner, vs.HEIRS OF SATURNINO Q. BORBON, AND
COURT OF APPEALS, Respondents.

Facts: NAPOCOR entered a property located in Barangay San Isidro, Batangas City in order to construct and
maintain transmission lines. Respondents heirs of Saturnino Q. Borbon owned the property. NAPOCOR filed a
complaint for expropriation in the Regional Trial Court in Batangas City (RTC), seeking the acquisition of an
easement of right of way over a portion of the property.
The respondents staunchly maintained that NAPOCOR had not negotiated with them before entering
the property and that the entry was done without their consent; nonetheless, they tendered no objection to
NAPOCOR’s entry provided it would pay just compensation not only for the portion sought to be expropriated
but for the entire property whose potential was greatly diminished, if not totally lost, due to the project.
During the pendency of an appeal, NAPOCOR filed a Manifestation and Motion to Discontinue
Expropriation Proceedings, informing that the parties failed to reach an amicable agreement; that the property
sought to be expropriated was no longer necessary for public purpose because of the intervening retirement of
the transmission lines installed on the respondents’ property; that because the public purpose for which such
property would be used thereby ceased to exist, the proceedings for expropriation should no longer continue,
and the State was now duty-bound to return the property to its owners; and that the dismissal or
discontinuance of the expropriation proceedings was in accordance with Section 4, Rule 67 of the Rules of
Court.

Issue: Whether or not the expropriation proceedings should be discontinued or dismissed pending appeal.

Ruling: The dismissal of the proceedings for expropriation at the instance of NAPOCOR is proper, but,
conformably with Section 4, Rule 67 of the Rules of Court, the dismissal or discontinuance of the proceedings
must be upon such terms as the court deems just and equitable.
Before anything more, we remind the parties about the nature of the power of eminent domain. The
right of eminent domain is “the ultimate right of the sovereign power to appropriate, not only the public but the
private property of all citizens within the territorial sovereignty, to public purpose.” But the exercise of such
right is not unlimited, for two mandatory requirements should underlie the Government’s exercise of the power
of eminent domain, namely: (1) that it is for a particular public purpose; and (2) that just compensation be paid
to the property owner. These requirements partake the nature of implied conditions that should be complied
with to enable to condemn or to keep the property expropriated.
Public use, in common acceptation, means “use by the public.” However, the concept has expanded to
include utility, advantage or productivity for the benefit of the public. “Public use” has now been held to be
synonymous with “public interest,” “public benefit,” and “public convenience.”
It is essential that the element of public use of the property be maintained throughout the
proceedings for expropriation. The effects of abandoning the public purpose were explained in Mactan-
Cebu International Airport Authority v. Lozada, Sr., to wit:
More particularly, with respect to the element of public use, the expropriator should commit to use the
property pursuant to the purpose stated in the petition for expropriation filed, failing which, it should file
another petition for the new purpose. If not, it is then incumbent upon the expropriator to return the
said property to its private owner, if the latter desires to reacquire the same. Otherwise, the
judgment of expropriation suffers an intrinsic flaw, as it would lack one indispensable element
for the proper exercise of the power of eminent domain, namely, the particular public purpose
for which the property will be devoted. Accordingly, the private property owner would be
denied due process of law, and the judgment would violate the property owner’s right to justice,
fairness and equity.
It is not denied that the purpose of the plaintiff was to acquire the land in question for public use. The
fundamental basis then of all actions brought for the expropriation of lands, under the power of eminent
domain, is public use. That being true, the very moment that it appears at any stage of the proceedings that the
expropriation is not for a public use, the action must necessarily fail and should be dismissed, for the reason
that the action cannot be maintained at all except when the expropriation is for some public use. That must be
true even during the pendency of the appeal or at any other stage of the proceedings. If, for example, during the
trial in the lower court, it should be made to appear to the satisfaction of the court that the expropriation is not
for some public use, it would be the duty and the obligation of the trial court to dismiss the action. And even
168
during the pendency of the appeal, if it should be made to appear to the satisfaction of the appellate court that
the expropriation is not for public use, then it would become the duty and the obligation of the appellate court
to dismiss it.
Verily, the retirement of the transmission lines necessarily stripped the expropriation
proceedings of the element of public use. To continue with the expropriation proceedings
despite the definite cessation of the public purpose of the project would result in the rendition
of an invalid judgment in favor of the expropriator due to the absence of the essential element
of public use.
Accordingly, the Court grants the motion to discontinue the proceedings subject to the conditions to be
shortly mentioned hereunder, and requires the return of the property to the respondents. Having said that, we
must point out that NAPOCOR entered the property without the owners’ consent and without paying just
compensation to the respondents. Neither did it deposit any amount as required by law prior to its entry. The
Constitution is explicit in obliging the Government and its entities to pay just compensation before depriving
any person of his or her property for public use. Considering that in the process of installing transmission lines,
NAPOCOR destroyed some fruit trees and plants without payment, and the installation of the transmission
lines went through the middle of the land as to divide the property into three lots, thereby effectively rendering
the entire property inutile for any future use, it would be unfair for NAPOCOR not to be made liable to the
respondents for the disturbance of their property rights from the time of entry until the time of restoration of
the possession of the property.
In view of the discontinuance of the proceedings and the eventual return of the property
to the respondents, there is no need to pay “just compensation” to them because their property
would not be taken by NAPOCOR. Instead of full market value of the property, therefore,
NAPOCOR should compensate the respondents for the disturbance of their property rights
from the time of entry until the time of restoration of the possession by paying to them actual
or other compensatory damages.
This should mean that the compensation must be based on what they actually lost as a result and by
reason of their dispossession of the property and of its use, including the value of the fruit trees, plants and
crops destroyed by NAPOCOR’s construction of the transmission lines. Considering that the dismissal of the
expropriation proceedings is a development occurring during the appeal, the Court now treats the dismissal of
the expropriation proceedings as producing the effect of converting the case into an action for damages. For
that purpose, the Court remands the case to the court of origin for further proceedings. The court of origin shall
treat the case as if originally filed as an action for damages.

National Power Corporation vs. Ibrahim

Facts:
Ibrahim owns a parcel of land located in Lanao del Norte.
In 1978, NAPOCOR took possession of the sub-terrain area of the land and constructed underground
tunnels on the said property.
The tunnels were apparently being used by NAPOCOR in siphoning the water of Lake Lanao and in the
operation of NAPOCOR’s Agus projects.
In 1991, Maruhom (one of the co-heirs of Ibrahim) requested Marawi City Water District for a permit to
construct or install a motorized deep well on the parcel of land but it was rejected on the grounds that the
construction would cause danger to lives and property by reason of the presence of the underground tunnels.
Maruhom demanded NAPOCOR to pay damages and to vacate the sub-terrain portion of the land.

Issue: WON Ibrahim is the rightful owner of the sub-terrain area of the land. If yes, are they entitled to the
payment of just compensation.

Held: YES. The sub-terrain portion of the property belongs to Ibrahim.


The Supreme Court cited Article 437 of the Civil Code which provides that: The owner of a parcel of
land is the owner of its surface and of everything under it, and he can construct thereon any works or make any
plantations and excavations which he may deem proper, without detriment to servitudes and subject to special
laws and ordinances. Xxx
169
Hence, the ownership of land extends to the surface as well as to the subsoil under it. Therefore,
Ibrahim owns the property as well as the sub-terrain area of the land where the underground tunnels were
constructed.
On the issue of just compensation, the Supreme Court also said that Ibrahim should be paid a just
compensation.
Ibrahim could have dug upon their property and built motorized deep wells but was prevented from
doing so by the authorities because of the construction of the tunnels underneath the surface of the land.
Ibrahim still had a legal interest in the sub-terrain portion insofar as they could have excavated the
same for the construction of the deep wells. It has been shown that the underground tunnels have deprived the
plaintiffs of the lawful use of the land and considerably reduced its value.
It was held that: If the government takes property without expropriation and devotes the property to
public use, after many years, the property owner may demand payment of just compensation in the event
restoration of possession is neither convenient nor feasible. This is in accordance with the principle that
persons shall not be deprived of their property except by competent authority and for public use and always
upon payment of just compensation.

NATIONAL POWER CORPORATION, Petitioner, v. HEIRS OF MACABANGKIT SANGKAY,


NAMELY: CEBU, BATOWA-AN, ET AL., ALL SURNAMED MACABANGKIT, Respondents.

FACTS:
Pursuant to its legal mandate under Republic Act No. 6395 (An Act Revising the Charter of the National
Power Corporation), NPC undertook the Agus River Hydroelectric Power Plant Project in the 1970s to generate
electricity for Mindanao. The project included the construction of several underground tunnels to be used in
diverting the water flow from the Agus River to the hydroelectric plants.
On November 21, 1997, the respondents as the owners of land with an area of 221,573 square meters
situated in Ditucalan, Iligan City, sued NPC in the RTC for the recovery of damages and of the property, with
the alternative prayer for the payment of just compensation. They alleged that they had belatedly discovered
that one of the underground tunnels of NPC that diverted the water flow of the Agus River for the operation of
the Hydroelectric Project in Agus V, Agus VI and Agus VII traversed their land; that their discovery had
occurred in 1995 after Atty. Saidali C. Gandamra, President of the Federation of Arabic Madaris School, had
rejected their offer to sell the land because of the danger the underground tunnel might pose to the proposed
Arabic Language Training Center and Muslims Skills Development Center; that such rejection had been
followed by the withdrawal by Global Asia Management and Resource Corporation from developing the land
into a housing project for the same reason; that Al-Amanah Islamic Investment Bank of the Philippines had
also refused to accept their land as collateral because of the presence of the underground tunnel; that the
underground tunnel had been constructed without their knowledge and consent; that the presence of the
tunnel deprived them of the agricultural, commercial, industrial and residential value of their land; and that
their land had also become an unsafe place for habitation because of the loud sound of the water rushing
through the tunnel and the constant shaking of the ground, forcing them and their workers to relocate to safer
grounds.
In its answer with counterclaim, NPC countered that the Heirs of Macabangkit had no right to
compensation under section 3(f) of Republic Act No. 6395, under which a mere legal easement on their land
was established; that their cause of action, should they be entitled to compensation, already prescribed due to
the tunnel having been constructed in 1979; and that by reason of the tunnel being an apparent and continuous
easement, any action arising from such easement prescribed in five years.
After trial, the RTC ruled in favor of the plaintiffs (Heirs of Macabangkit).
Earlier, on August 18, 1999, the Heirs of Macabangkit filed an urgent motion for execution of judgment
pending appeal. The RTC granted the motion and issued a writ of execution, prompting NPC to assail the writ
by petition for certiorari in the CA. On September 15, 1999, the CA issued a temporary restraining order (TRO)
to enjoin the RTC from implementing its decision.The Heirs of Macabangkit elevated the ruling of the CA (G.R.
No. 141447), but the Court upheld the CA on May 4, 2006.
On October 5, 2004, the CA affirmed the decision of the RTC.

ISSUE:
170
1) Whether the CA and the RTC erred in holding that there was an underground tunnel traversing the
Heirs of Macabangkits land constructed by NPC; and
2) Whether the Heirs of Macabangkits right to claim just compensation had prescribed under section 3(i)
of Republic Act No. 6395, or, alternatively, under Article 620 and Article 646 of the Civil Code.

HELD: We uphold the liability of NPC for payment of just compensation.

five-year prescriptive period under Section 3(i) of Republic Act No. 6395 does not apply to
claims for just compensation
Without necessarily adopting the reasoning of the CA, we uphold its conclusion that prescription did
not bar the present action to recover just compensation.
Section 3 (i) of Republic Act No. 6395, the cited law, relevantly provides:
Section 3. Powers and General Functions of the Corporation. The powers, functions, rights and
activities of the Corporation shall be the following: xxx
(i) To construct works across, or otherwise, any stream, watercourse, canal, ditch, flume, street, avenue,
highway or railway of private and public ownership, as the location of said works may require: Provided, That
said works be constructed in such a manner as not to endanger life or property; And provided, further, That the
stream, watercourse, canal ditch, flume, street, avenue, highway or railway so crossed or intersected be
restored as near as possible to their former state, or in a manner not to impair unnecessarily their usefulness.
Every person or entity whose right of way or property is lawfully crossed or intersected by said works shall not
obstruct any such crossings or intersection and shall grant the Board or its representative, the proper authority
for the execution of such work. The Corporation is hereby given the right of way to locate, construct and
maintain such works over and throughout the lands owned by the Republic of the Philippines or any of its
branches and political subdivisions. The Corporation or its representative may also enter upon private property
in the lawful performance or prosecution of its business and purposes, including the construction of the
transmission lines thereon; Provided, that the owner of such property shall be indemnified for any actual
damage caused thereby; Provided, further, That said action for damages is filed within five years after the
rights of way, transmission lines, substations, plants or other facilities shall have been established; Provided,
finally, That after said period, no suit shall be brought to question the said rights of way, transmission lines,
substations, plants or other facilities;
A cursory reading shows that Section 3(i) covers the construction of works across, or otherwise, any
stream, watercourse, canal, ditch, flume, street, avenue, highway or railway of private and public ownership, as
the location of said works may require. It is notable that Section 3(i) includes no limitation except those
enumerated after the term works. Accordingly, we consider the term works as embracing all kinds of
constructions, facilities, and other developments that can enable or help NPC to meet its objectives of
developing hydraulic power expressly provided under paragraph (g) of Section 3.The CAs restrictive construal
of Section 3(i) as exclusive of tunnels was obviously unwarranted, for the provision applies not only to
development works easily discoverable or on the surface of the earth but also to subterranean works like
tunnels. Such interpretation accords with the fundamental guideline in statutory construction that when the
law does not distinguish, so must we not. Moreover, when the language of the statute is plain and free from
ambiguity, and expresses a single, definite, and sensible meaning, that meaning is conclusively presumed to be
the meaning that the Congress intended to convey.
Even so, we still cannot side with NPC.
We rule that the prescriptive period provided under Section 3(i) of Republic Act No. 6395 is applicable
only to an action for damages, and does not extend to an action to recover just compensation like this case.
Consequently, NPC cannot thereby bar the right of the Heirs of Macabangkit to recover just compensation for
their land.

just compensation
The action to recover just compensation from the State or its expropriating agency differs from the
action for damages. The former, also known as inverse condemnation, has the objective to recover the value of
property taken in fact by the governmental defendant, even though no formal exercise of the power of eminent
domain has been attempted by the taking agency. Just compensation is the full and fair equivalent of the
property taken from its owner by the expropriator. The measure is not the takers gain, but the owner's loss. The
word just is used to intensify the meaning of the word compensation in order to convey the idea that the
171
equivalent to be rendered for the property to be taken shall be real, substantial, full, and ample. On the other
hand, the latter action seeks to vindicate a legal wrong through damages, which may be actual, moral, nominal,
temperate, liquidated, or exemplary. When a right is exercised in a manner not conformable with the norms
enshrined in Article 19 and like provisions on human relations in the Civil Code, and the exercise results to the
damage of another, a legal wrong is committed and the wrongdoer is held responsible.
The two actions are radically different in nature and purpose. The action to recover just compensation
is based on the Constitution while the action for damages is predicated on statutory enactments. Indeed, the
former arises from the exercise by the State of its power of eminent domain against private property for public
use, but the latter emanates from the transgression of a right. The fact that the owner rather than the
expropriator brings the former does not change the essential nature of the suit as an inverse condemnation, for
the suit is not based on tort, but on the constitutional prohibition against the taking of property without just
compensation. It would very well be contrary to the clear language of the Constitution to bar the recovery of
just compensation for private property taken for a public use solely on the basis of statutory prescription.
Due to the need to construct the underground tunnel, NPC should have first moved to acquire the land
from the Heirs of Macabangkit either by voluntary tender to purchase or through formal expropriation
proceedings. In either case, NPC would have been liable to pay to the owners the fair market value of the land,
for Section 3(h) of Republic Act No. 6395 expressly requires NPC to pay the fair market value of such property
at the time of the taking, thusly:
(h)To acquire, promote, hold, transfer, sell, lease, rent, mortgage, encumber and otherwise dispose of
property incident to, or necessary, convenient or proper to carry out the purposes for which the Corporation
was created:Provided, That in case a right of way is necessary for its transmission lines, easement of right of
way shall only be sought:Provided, however,That in case the property itself shall be acquired by purchase, the
cost thereof shall be the fair market value at the time of the taking of such property.

NPCs construction of the tunnel constituted taking of the land, and entitled owners to just
compensation
The Court held in National Power Corporation v. Ibrahim that NPC was liable to pay not merely an
easement fee but rather the full compensation for land traversed by the underground tunnels, viz:
In disregarding this procedure and failing to recognize respondents ownership of the sub-terrain
portion, petitioner took a risk and exposed itself to greater liability with the passage of time. It must be
emphasized that the acquisition of the easement is not without expense. The underground tunnels impose
limitations on respondents use of the property for an indefinite period and deprive them of its ordinary use.
Based upon the foregoing, respondents are clearly entitled to the payment of just compensation.
Notwithstanding the fact that petitioner only occupies the sub-terrain portion, it is liable to pay not merely an
easement fee but rather the full compensation for land. This is so because in this case, the nature of the
easement practically deprives the owners of its normal beneficial use. Respondents, as the owner of the
property thus expropriated, are entitled to a just compensation which should be neither more nor less,
whenever it is possible to make the assessment, than the money equivalent of said property.
Here, like in National Power Corporation v. Ibrahim, NPC constructed a tunnel underneath the land of
the Heirs of Macabangkit without going through formal expropriation proceedings and without procuring their
consent or at least informing them beforehand of the construction. NPCs construction adversely affected the
owners rights and interests because the subterranean intervention by NPC prevented them from introducing
any developments on the surface, and from disposing of the land or any portion of it, either by sale or
mortgage.
We agree with both the RTC and the CA that there was a full taking on the part of NPC, notwithstanding
that the owners were not completely and actually dispossessed. It is settled that the taking of private property
for public use, to be compensable, need not be an actual physical taking or appropriation. Indeed, the
expropriators action may be short of acquisition of title, physical possession, or occupancy but may still
amount to a taking. Compensable taking includes destruction, restriction, diminution, or interruption of the
rights of ownership or of the common and necessary use and enjoyment of the property in a lawful manner,
lessening or destroying its value. It is neither necessary that the owner be wholly deprived of the use of his
property, nor material whether the property is removed from the possession of the owner, or in any respect
changes hands.

172
As a result, NPC should pay just compensation for the entire land. In that regard, the RTC pegged just
compensation at P500.00/square meter based on its finding on what the prevailing market value of the
property was at the time of the filing of the complaint, and the CA upheld the RTC.

reckoning point of just compensation on the value at the time the owners commenced these
inverse condemnation proceedings is entirely warranted.
We rule that the reckoning value is the value at the time of the filing of the complaint, as the RTC
provided in its decision. Compensation that is reckoned on the market value prevailing at the time either when
NPC entered or when it completed the tunnel, as NPC submits, would not be just, for it would compound the
gross unfairness already caused to the owners by NPCs entering without the intention of formally expropriating
the land, and without the prior knowledge and consent of the Heirs of Macabangkit. NPCs entry denied
elementary due process of law to the owners since then until the owners commenced the inverse condemnation
proceedings. The Court is more concerned with the necessity to prevent NPC from unjustly profiting from its
deliberate acts of denying due process of law to the owners. As a measure of simple justice and ordinary
fairness to them, therefore, reckoning just compensation on the value at the time the owners commenced these
inverse condemnation proceedings is entirely warranted.
In National Power Corporation v. Court of Appeals, a case that involved the similar construction of an
underground tunnel by NPC without the prior consent and knowledge of the owners, and in which we held that
the basis in fixing just compensation when the initiation of the action preceded the entry into the property was
the time of the filing of the complaint, not the time of taking, we pointed out that there was no taking when the
entry by NPC was made without intent to expropriate or was not made under warrant or color of legal
authority.

WHEREFORE, the Court AFFIRMS the decision promulgated on October 5, 2004 by the
Court of Appeals, subject to the following MODIFICATIONS, to wit:
a) Interest at the rate of 12%per annum is IMPOSED on the principal amount of
P113,532,500.00 as just compensation, reckoned from the filing of the complaint on
November 21, 1997 until the full liability is paid;
b) The awards of P30,000.00 as rental fee,P200,000.00 as moral damages, and
P200,000.00 as exemplary damages are DELETED; and
c) The award of 15% attorneys fees decreed to be paid by National Power Corporation to the
Heirs of Macabangkit is DELETED.

The Court PARTLY GRANTED the motion to register attorney's lien filed by Atty.
Macarupung Dibaratun, and FIXES Atty. Dibaratun's attorney's fees on the basis of quantum
meruit at 10% of the principal award of P113,532,500.00.
The motion to register attorney's lien of Atty. Manuel D.Ballelos is PARTLY GRANTED,
and Atty. Ballelos is DECLARED ENTITLED TO RECOVER from Cebu, Batowa-an, Sayana,
Nasser, Manta and Edgar, all surnamed Macabangkit, the amount of P5,000.00 as attorneys
fees on the basis of quantum meruit.

173
13. Contract Clause
PHILIPPINE VETERANS BANK EMPLOYEES UNION VS JUDGE VEGA (Liquidator of the
Philippine Veterans Bank respondent)

FACTS:
On January 2, 1992, the Congress enacted R.A. 7169 providing for the rehabilitation of Philippine
Veterans Bank. It was published in the Official Gazette in February 24, 1992. Thereafter, petitioners filed with
the labor tribunals their residual claims for benefits and for reinstatement upon reopening the bank.
In May 1992, the Central Bank issued a certificate of authority allowing the PVB to reopen despite the
late mandate for rehabilitation and reopening, Judge Vega continued with the liquidation proceedings of the
bank alleging further that RA 7169 became effective only on March 10, 1992 or 15 days after its publication in
the Official Gazette on February 24, 1992.

ISSUE:
Whether or not RA 7169 became effective on January 2, 1992.

RULING:
Yes. RA 7169 expressly provided that it should take effect upon its approval. Aquino signed it into law
on January 2, 1992. Thereafter, said law became effective on said date. Its subsequent publication was not
necessary for its effectivity. RA 7169 is of internal nature and not have general application thus it took effect on
the date provided for and hence was rightfully invoked by the petitioners. The Supreme Court upheld that
while as a rule laws take effect after 15 days following completion of their publication in the Official Gazette or
in a newspaper of general circulation in the Philippines, the legislature has the authority to provide for
exceptions as indicated in the clause “unless otherwise provided”

LEPANTO CONSOLIDATED MINING COMPANY v. WMC RESOURCES INTERNATIONAL PTY.


LTD. and WMC (PHILIPPINES), INC.

In a contract denominated as “Tampakan Option Agreement”, respondent WMC Resources


International Pty. Ltd. (WMC), through its local subsidiary Western Mining Corporation (Philippines), Inc.
(WMCP), acquired the mining claims in Tampakan, South Cotabato of the Tampakan Companies. The
“Tampakan Option Agreement” was amended by subsequent agreements under which the Tampakan
Companies were given preferential option to acquire the shares of WMC in WMCP and Hillcrest Inc. in the
event WMC decided to sell them. WMC, by a Sale and Purchase Agreement, sold to Lepanto Consolidated
Mining Company (Lepanto) its shares of stock. As the Tampakan Companies later availed of their preferential
right under the “Tampakan Option Agreement,” a Sale and Purchase Agreement was concluded between WMC
and the Tampakan Companies over the same shares of stock priorly purchased by Lepanto.
The Tampakan Companies notified the Director of the Mines and Geosciences Bureau (MGB) of the
DENR of the exercise of their preemptive right to buy WMC‘s equity in WMCP and Hillcrest, Inc. Lepanto
wrote the DENR Secretary about the invalidity of said agreement and reiterated its request for the approval of
its acquisition of the disputed shares.
Lepanto subsequently filed before the Regional Trial Court (RTC) of Makati a complaint against WMC,
WMCP, Tampakan Companies. WMC et al. filed before the RTC a Joint Motion to Dismiss on the ground of
forum shopping. The RTC denied WCM et al.‘s Motion to Dismiss. On appeal, the CA granted the petition of
respondents ruling that Lepanto is guilty of forum shopping. Petitioners filed a motion for reconsideration with
the CA. The CA denied said motion.

ISSUE:
Whether or not Lepanto is guilty of forum shopping

HELD:

174
It is clear from the proceedings before the DENR, specifically before the MGB, that the issue of which –
– between petitioner and respondent Tampakan Companies –– possesses the better right to acquire the mining
rights, claims and interests held by WMC through its local subsidiary WMCP, especially with respect to the
1995 FTAA, had been brought to the fore. The MGB cannot just assess the qualifications of petitioner and of the
Tampakan Companies as potential transferee or assignee of the rights and obligations of WMCP under the
FTAA without also resolving the issue of which has priority of right to become one.
True, the questioned agreements of sale between Lepanto and WMC on one hand and between WMC
and the Tampakan Companies on the other pertain to transfer of shares of stock from one entity to another.
But said shares of stock represent ownership of mining rights or interest in mining agreements. Hence, the
power of the MGB to rule on the validity of the questioned agreements of sale, which was raised by Lepanti
before the DENR, is inextricably linked to the very nature of such agreements over which the MGB has
jurisdiction under the law. Unavoidably, there is identity of reliefs that Lepanto seeks from both the MGB and
the RTC.
Forum shopping exists when both actions involve the same transactions, same essential facts and
circumstances and raise identical causes of actions, subject matter, and issues. Such elements are evidently
present in both the proceedings before the MGB and before the trial court. The case instituted with the RTC
was thus correctly ordered dismissed by the appellate court on the ground of forum shopping. Besides, not only
did Lepanto commit forum shopping but it also failed to exhaust administrative remedies by opting to go ahead
in seeking reliefs from the court even while those same reliefs were appropriately awaiting resolution by the
MGB.

175
14. Poverty and Legal Protection
RE: QUERY OF MR. ROGER C. PRIORESCHI RE EXEMPTION FROM LEGAL AND FILING FEES
OF THE GOOD SHEPHERD FOUNDATION, INC.

Facts. In his letter dated May 22, 2009 addressed to the Chief Justice, Mr. Roger C. Prioreschi, administrator
of the Good Shepherd Foundation, Inc., wrote:
The Good Shepherd Foundation, Inc. is very grateful for your 1rst. Indorsement to pay a nominal fee of
Php 5,000.00 and the balance upon the collection action of 10 million pesos, thus giving us access to the
Justice System previously denied by an up-front excessive court fee. The Good Shepherd Foundation, Inc.
reached-out to the poorest among the poor, to the newly born and abandoned babies, to children who never
saw the smile of their mother, to old people who cannot afford a few pesos to pay for common prescriptions, to
broken families who returned to a normal life. In other words, we have been working hard for the very Filipino
people, that the Government and the society cannot reach to, or have rejected or abandoned them.
The Hon. Court Administrator Jose Perez pointed out to the need of complying with OCA Circular No.
42-2005 and Rule 141 that reserves this privilege to indigent persons. While judges are appointed to interpret
the law, this type of law seems to be extremely detailed with requirements that do not leave much room for
interpretations.

Issue: Whether the Courts can grant to the Good Shepherd Foundation who works for indigent and
underprivileged people, the same option granted to indigent people?

Held: No, the Courts cannot grant to foundations like the Good Shepherd Foundation, Inc. the same
exemption from payment of legal fees granted to indigent litigants even if the foundations are working for
indigent and underprivileged people.
The basis for the exemption from legal and filing fees is the free access clause, embodied in Sec. 11, Art.
III of the 1987 Constitution, thus:
Sec. 11. Free access to the courts and quasi judicial bodies and adequate legal assistance shall not be
denied to any person by reason of poverty.

The importance of the right to free access to the courts and quasi judicial bodies and to adequate legal
assistance cannot be denied. A move to remove the provision on free access from the Constitution on the
ground that it was already covered by the equal protection clause was defeated by the desire to give
constitutional stature to such specific protection of the poor.[1]
In implementation of the right of free access under the Constitution, the Supreme Court promulgated
rules, specifically, Sec. 21, Rule 3, Rules of Court, and Sec. 19, Rule 141, Rules of Court, which respectively state
thus:
Sec. 21. Indigent party. A party may be authorized to litigate his action, claim or defense as an indigent
if the court, upon an ex parte application and hearing, is satisfied that the party is one who has no
money or property sufficient and available for food, shelter and basic necessities for himself and his
family. Such authority shall include an exemption from payment of docket and other lawful fees, and of
transcripts of stenographic notes which the court may order to be furnished him. The amount of the
docket and other lawful fees which the indigent was exempted from paying shall be a lien on any
judgment rendered in the case favorable to the indigent, unless the court otherwise provides.

Sec. 19. Indigent litigants exempt from payment of legal fees. Indigent litigants (a) whose gross
income and that of their immediate family do not exceed an amount double the monthly minimum
wage of an employee and (b) who do not own real property with a fair market value as stated in the
current tax declaration of more than three hundred thousand (P300,000.00) pesos shall be exempt
from payment of legal fees.

The legal fees shall be a lien on any judgment rendered in the case favourable to the indigent litigant
unless the court otherwise provides.

176
To be entitled to the exemption herein provided, the litigant shall execute an affidavit that he and his
immediate family do not earn a gross income abovementioned, and they do not own any real property with the
fair value aforementioned, supported by an affidavit of a disinterested person attesting to the truth of the
litigants affidavit. The current tax declaration, if any, shall be attached to the litigants affidavit.
Any falsity in the affidavit of litigant or disinterested person shall be sufficient cause to dismiss the
complaint or action or to strike out the pleading of that party, without prejudice to whatever criminal liability
may have been incurred.

177
15. Rights of Suspects
*R.A. No. 9745, Anti-Torture Act of 2003

People v. Jose, 37 SCRA 450

Facts: On June 26, 1967, four principal-accused Jaime Jose, Basilio Pineda Jr., Eduardo Aquino and Rogelio
Cañal conspired together, confederated with and mutually helped one another, then and there, to willfully,
unlawfully and feloniously, with lewd design to forcibly abduct Magdalena “Maggie” dela Riva, 25 years old and
single, a movie actress by profession at the time of the incident, where the four principal accused, by means of
force and intimidation using a deadly weapon, have carnal knowledge of the complainant against her will, and
brought her to the Swanky Hotel in Pasay City, and hence committed the crime of Forcible Abduction with
Rape.
Having established the element of conspiracy, the trial court finds the accused guilty beyond reasonable
doubt of the crime of forcible abduction with rape and sentences each of them to the death penalty.

Issue: Whether or not the trial court made a proper ruling of the case considering the element of conspiracy.

Held:
No, the trial court’s ruling was not proper. The SC ruled that since the element of conspiracy was
present, where the act of one is the act of all, each of the accused is also liable for the crime committed by each
of the other persons who conspired to commit the crime. The SC modified the judgment as follows: appellants
Jaime Jose, Basilio Pineda Jr., and Eduardo Aquino are guilty of the complex crime of forcible abduction with
rape and each and every one of them is likewise convicted of three (3) other crimes of rape. As a consequence
thereof, each of them is likewise convicted with four death penalties and to indemnify the victim of the sum of
P10,000 in each of the four crimes. The case against Rogelio Cañal was dismissed only in so far as the criminal
liability is concerned due to his death in prison prior to promulgation of judgment.

PEOPLE VS. GALIT (1985)

FACTS:
The prisoner was arrested for killing the victim oil the occasion of a robbery. He had been detained and
interrogated almost continuously for five days, to no avail. He consistently maintained his innocence. There
was no evidence to link him to the crime. Obviously, something drastic had to be done. A confession was
absolutely necessary. So the investigating officers began to maul him and to torture him physically. Still the
prisoner insisted on his innocence. His will had to be broken. A confession must be obtained. So they continued
to maltreat and beat him. 'They covered his face with a rag and pushed his face into a toilet bowl full of human
waste. The prisoner could not take any more. His body could no longer endure the pain inflicted on him and
the indignities he had to suffer. His will had been broken. He admitted what the investigating officers wanted
him to admit and he signed the confession they prepared. Later, against his will, he posed for pictures as
directed by his investigators, purporting it to be a re enactment.

ISSUE:
Whether or not the accused was informed of his constitutional rights to remain silent and to counsel,
and that any statement he might make could be used against him.

RULING:
Such a long question followed by a monosyllabic answer does not satisfy the requirements of the law
that the accused be informed of his rights under the Constitution and our laws. Instead there should be several
short and clear questions and every right explained in simple words in a dialect or language known to the
person under investigation. Accused is from Samar and there is no showing that he understands Tagalog.
Moreover, at the time of his arrest, accused was not permitted to communicate with his lawyer, a relative, or a
178
friend. In fact, his sisters and other relatives did not know that he had been brought to the NBI for
investigation and it was only about two weeks after he had executed the salaysay that his relatives were allowed
to visit him. His statement does not even contain any waiver of right to counsel and yet during the investigation
he was not assisted by one. At the supposed re-enactment, again accused was not assisted by counsel of his
choice. These constitute gross violations of his rights.

PEOPLE OF THE PHILIPPINES VS SAMUEL MARRA, ET. AL.

FACTS:
Jimmy Din and Nelson Tandoc were conversing with each other in front of the hotel owned by Din’s
father when a man passed by on the opposite side of the street waving a dirty sign with his finger. Din and
Tandoc followed until they caught up with the man and demanded an explanation but they were not given any.
They were joined by two other men and a brawl ensued when Tandoc slapped one of them. The fight ended
when their opponents ran away.
When Din and Tando were on their way to the hotel, they men with whom they just had a fight ran after
them which pressed them to hide in the annex of the hotel. After a few minutes, thinking that there was no
longer any danger, Tandoc decided to go home. However, he was shot by a revolver by a man wearing a security
guard’s uniform when he opened the door. Din witnessed the shooting and was able to take a good look at the
man who fatally shot Tandoc.
Samuel Marra was identified as the man who shot Tandoc and was approached by responding
policemen while he was eating at an eatery shortly after the incident. A series of questions were asked,
particularly his tour of duty and where his issued firearm might be. He brought the policemen to his house and
showed them his revolver with five live bullets and an empty shell. Marra denied shooting Tandoc at first but
admitted to the crime and said that he did so in self-defense. He was then taken to the police station where he
was positively identified by Din and was consequentially detained.

ISSUE:
Whether or not there was violation of rights of suspects under custodial investigation when policemen
asked the appellant questions prior to his arrest.

RULING:
No. In the case at bar, appellant was not under custodial investigation when he made the admission.
There was no coercion whatsoever to compel him to make such as statement. He could have refused to answer
questions from the very start. The police inquiry had not yet reached a level wherein they considered him as a
particular suspect. Thus, there was no violation of Section 12, Article III of the Constitution or the
constitutional procedure on custodial investigation.
In addition, the law provides that the declaration of an accused acknowledging his guilt of the offense
charged or of any offense necessarily included therein may be given in evidence against him and, in certain
circumstances, this admission may be considered as part of the res gestae or the facts that may be admitted as
evidence.
The judgment finding accused-appellant Samuel Marra guilty of the crime of murder was affirmed.

People of the Philippines vs Ting Lan Uy


G.R. No. 157399 November 17, 2005

Facts:
The accused, Uy, Gamus and Ochoa, public officers being employed by the National Power Corporation
(NAPOCOR), was charged for allegedly diverting andcollecting funds of the National Power Corporation (NPC)
intended for the purchase of US Dollars from the United Coconut Planters Bank (UCPB) for the amount of
P183, 805,291.25 was indicted before the Sandiganbayan for the complex crime of Malversation through
Falsification of Commercial Documents for conspiring, confederating with the private co-accused where they
falsify or cause to be falsified the NPC’s application for the managers check with the Philippine National Bank
(PNB). Sandigan Bayan rendered a decision acquitting Uy, and Ochoa being found guilty for the said crime and
179
is ordered to pay the equal amount malversed solidarily with Uy. Ochoa then appealed, He claims that his
conviction was based on the alleged sworn statement and the transcript of stenographic notes of a supposed
interview with appellant NPC personnel and the report of the National Bureau of Investigation (NBI).
Appellant maintains that he signed the sworn statement while confined at the Heart Center and upon
assurance it would not be used against him. He was not assisted by counsel nor was he apprised of his
constitutional rights when he executed his affidavit.

Issue:
Whether or not the constitutional rights of the accused were violated?

Held:
The decision of the Sandiganbayan is affirmed. Considering that his statement was taken during the
administrative investigation of NPC’s audit team and before he was taken into custody. As such inquest was
still a general inquiry into an unsolved offense. Appellant cannot claim that he is in police custody because he
was confined at the time at Heart Center and he gave this statement to NPC personnel, not to police
authorities. The interview where the sworn statement is based was conducted by NPC personnel for NPC’s
administrative investigation. Any investigation conducted by the NBI is a separate proceeding, distinct and
independent from the NPC inquiry and should not be confused or lumped together with the latter.

PEOPLE VS. DOMINGO REYES

FACTS:
On July 16, 1999, at Sitio Lambakin, San Jose del Monte, Bulacan, accused-appellant, conspiring,
confederating and mutually helping one another and grouping themselves together with Juanito Pataray
Cayaban, Federico Pataray Cabayan and Rommel Libarnes Acejo, who are still at large, did then and there
wilfully, unlawfully and feloniously, by means of force and intimidation and with use of firearms, carry away
and deprive Robert Yao, Yao San, Chua Ong Ping Sim, Raymond Yao, Ronald Matthew Yao, Lennie Yao,
Charlene Yao, Jona Abagatnan ang Josephine Ortea against their will and consent on board their Mazda MVP
van for the purpose of extorting money in the amount of Five Million Pesos (P5,000,000.00), that during the
detention of Chua Ong Ping Sim and Raymong Yao, said accused with intent to kill, willfully and unlawfully
strangled Chua Ong Ping Sim and Raymond Yao to death to the damage and prejudice of their heirs.
Appellant Arnaldo surrendered to the Presidential Anti-Organized Crime Task Force (PAOCTF) at
Camp Crame. Thereupon, appellant Arnaldo, with the assistance of Atty. Uminga, executed a written extra-
judicial confession narrating his participation in the incident. Appellant Arnaldo identified appellants Reyes
and Flores. Afterwards, appellant Flores, with the assistance of Atty. Rous, executed a written extra-judicial
confession detailing his participation in the incident.
After trial, the RTC rendered a Decision convicting appellants of the special complex crime of
kidnapping for ransom with homicide and sentencing each of them to suffer the supreme penalty of death.
Appellants were also ordered to pay jointly and severally the Yao family ₱150,000.00 as civil indemnity,
₱500,000.00 as moral damages and the costs of the proceedings.

ISSUES:
1. Were the extra-judicial confessions of the accused admissible in evidence?
2. Was he afforded the right to counsel of his own choice?

RULING:
1. Yes. The Pasubali of appellants Arnaldo and Flores’s written extra-judicial confessions clearly shows
that before they made their respective confessions, the PAOCTF investigators had informed them that
the interrogation about to be conducted on them referred to the kidnapping of the Yao family.
Since the prosecution has sufficiently established that the respective extra-judicial confessions
of appellant Arnaldo and appellant Flores were obtained in accordance with the constitutional
guarantees, these confessions are admissible. They are evidence of a high order because of the strong
presumption that no person of normal mind would deliberately and knowingly confess to a crime,

180
unless prompted by truth and conscience. Consequently, the burden of proving that undue pressure or
duress was used to procure the confessions rests on appellants Arnaldo and Flores.
We have held that an extra-judicial confession is admissible in evidence if the following
requisites have been satisfied: (1) it must be voluntary; (2) it must be made with the assistance of
competent and independent counsel; (3) it must be express; and (4) it must be in writing.

2. Yes. The phrase “preferably of his own choice” does not convey the message that the choice of a lawyer
by a person under investigation is exclusive as to preclude other equally competent and independent
attorneys from handling the defense. A lawyer provided by the investigators is deemed engaged by the
accused when he does not raise any objection to the counsel’s appointment during the course of the
investigation, and the accused thereafter subscribes to the veracity of the statement before the swearing
officer. Appellants Arnaldo and Flores did not object to the appointment of Atty. Uminga and Atty.
Rous as their lawyers, respectively, during their custodial investigation. Appellants Arnaldo and Flores
are deemed to have engaged the services of Atty. Uminga and Atty. Rous, respectively.

People vs. Judge Ayson


175 SCRA 216/ Rights of the Accused

Facts:
Felipe Ramos was a ticket freight clerk of the Philippine Airlines, assigned at its Baguio City station. It
was alleged that he was involved in irregularities in the sales of plane tickets, the PAL management notified
him of an investigation to be conducted. That investigation was scheduled in accordance with PAL's Code of
Conduct and Discipline, and the Collective Bargaining Agreement signed by it with the Philippine Airlines
Employees' Association (PALEA) to which Ramos pertained. A letter was sent by Ramos stating his willingness
to settle the amount of P76, 000. The findings of the Audit team were given to him, and he refuted that he
misused proceeds of tickets also stating that he was prevented from settling said amounts. He proffered a
compromise however this did not ensue. Two months after a crime of estafa was charged against Ramos.
Ramos pleaded not guilty. Evidence by the prosecution contained Ramos’ written admission and statement, to
which defendants argued that the confession was taken without the accused being represented by a lawyer.
Respondent Judge did not admit those stating that accused was not reminded of his constitutional rights to
remain silent and to have counsel. A motion for reconsideration filed by the prosecutors was denied. Hence this
appeal.

Issue:
Whether or not the respondentJudge iscorrect in making inadmissible as evidence the admission and
statement of accused.

Ruling:
No. Section 20 of the 1987 constitution provides that the right against self-incrimination (only to
witnesses other than accused, unless what is asked is relating to a different crime charged- not present in case
at bar).
This is accorded to every person who gives evidence, whether voluntarily or under compulsion of
subpoena, in any civil, criminal, or administrative proceeding. The right is not to "be compelled to be a witness
against himself.” It prescribes an "option of refusal to answer incriminating questions and not a prohibition of
inquiry." the right can be claimed only when the specific question, incriminatory in character, is actually put to
the witness. It cannot be claimed at any other time. It does not give a witness the right to disregard a subpoena,
to decline to appear before the court at the time appointed, or to refuse to testify altogether. It is a right that a
witness knows or should know. He must claim it and could be waived.
Rights in custodial interrogation as laid down in Miranda v. Arizona: the rights of the accused include:
1) He shall have the right to remain silent and to counsel, and to be informed of such right.
2) Nor force, violence, threat, intimidation, or any other means which vitiates the free will shall be
used against him.
3) Any confession obtained in violation of these rights shall be inadmissible in evidence.

181
The individual may knowingly and intelligently waive these rights and agree to answer or make a
statement. But unless and until such rights and waivers are demonstrated by the prosecution at the trial, no
evidence obtained as a result of interrogation can be used against him.

People vs. Juanario 7 February 1997 Ponente: Panganiban

Facts: Januario and Canape were involved in carnapping. Their involvement varies from different versions of
what happened. They gave verbal confessions to the crime, which was only later on executed through a written
statement and as assisted by counsel. This was considered inadmissible since it was a “fruit of the poisonous
tree”. Topic: Police Investigation; Exclusionary rule/ “Fruit of the poisonous tree” doctrine Statement of the
Case: - This is an appeal from the decision of the RTC in Cavite, which held Rene Januario and Efren Canape
guilty of violation of Sec. 14 last sentence of RA 6539, known as the Anti-Carnapping Law. Santiago Cid, in the
same decision, is acquitted for lack of evidence. - 7 November 1988: Assistant Provincial Fiscal Jose Velasco,
Jr. filed against Rene Januario, Efren Canape, Santiago Cid, Eliseo Sarita and Eduardo Sarinos for violation of
the Anti-Carnapping Law - 4 September 1987: Januario and Canape, with Sarita and Sarinos, after stabbing
driver Geronimo Malibago and conductor Andrew Patriarca, took one Isuzu passenger type jeep owned by
Doris and Victor Wolf. - 7 February 1989: Januario and Canape pleaded not guilty. - 30 May 1989: Cid pleaded
not guilty. - Sarita and Sarinos remain at large. Statement of Facts: - Vicente Pons story: o March 1988: Cid
went to the house of prosecution witness Vicente Pons, Cid's cousin. He asked Pons if he wanted to buy a jeep.
Pons said he had no money but he could look for a buyer who can pay 50,000. o So Pons offered to look for a
buyer provided that Cid would entrust the jeep to him. o He offered it to Myrna Temporas who agreed to buy it
for 65,000, which later became 48,500 only. - Myrna Temporas story: o According to her, Pons told her that
the jeep belonged to her niece, Doris Wolf. o Pons, upon Doris Wolf's instruction, borrowed from Myrna
48,500 and used the jeep as collateral. o Pons failed to pay back the 48,500, and also failed to produce a deed
of sale covering the jeep. Myrna filed a complaint. She just found out during the complaint that the driver and
the conductor of the jeep had been killed by kidnappers. Upon NBI's investigation, they found that the
carnapping of the jeep and the killing of the driver and the conductor were done by Januario, Canape, Sarita
and Sarinos. The jeep was disposed of through Cid. From an oral investigation of Januario and Canape, NBI
found out that the driver and the conductor were killed inside a sugar plantation. A lawyer who was just
around, Atty. Carlos Saunar, was asked to assist the two during the investigation. Confession of Januario: o
Januario said that 2 weeks before September, he was in the house of Canape to procure chicken and kalawit for
his business. He also went there because his new friends, Sarita and Samera, with Canape, wanted him to look
for a buyer of a jeep. He asked for a photo of the jeep but he was told that he'll have it later that night after they
have drinks at Toto's house. o At about 5am, the group hailed a jeep. Here, Januario described how Canape,
Sarita and Sarinos tied up the conductor and the driver of the jeep and took control of the vehicle. The jeep
stopped after a while, and brought the conductor and driver down a sugar plantation. Januario described how
he heard growls, but did not witness what happened. He also saw the bloodied hand Sarita and Sarinos. Upon
reaching Libmanan, Januario said they went to Cid with whom Januario had earlier conferred regarding the
sale of the jeep. He got 1,000 cash and rice and eggs worh 600. Januario signed this statement and swore
berfore NBI Executive Director Salvador Ranin. Also signed by Atty. Carlos Saunar as counsel. o Confession of
Canape: o Sarita and Sarinos told him to look for a buyer of a jeep. He looked for a buyer with Januario. They
saw Cid as an interested buyer. o They told Sarita and Sarinos about it. They drank, then at 5am, hailed a jeep,
wrote it and was asked by Sarita and Sarinos to take out a knife and point at the driver and conductor of the
jeep. o They stopped at a certain point. Januario, Sarita and Sarinos brought the driver and the conductor
down the jeep at a sugar plantation, with Sarita later saying that everything was already fixed "Ayos na". o After
this, they went to Cid and gave the jeep to him for 25,000. o He also said that Cid and Pons knew that the jeep
was just going to be stolen. He also admitted that he himself knew that when they were looking for a buyer, the
jeep they will be selling will also be just stolen. Canape signed, subscribed and swore to this statement. 12
September 1989: Prosecution offered evidence, which the court admitted. Defense manifested its intention to
file a demurrer to evidence. 21 November 1989: Since defense has not presented Cid yet, the court ordered the
cancellation of his bail bond and gave his surety 30 days within which to show cause why judgement against
the bond should not be rendered. 22 December 1989: Court issued an order stating that the demurrer to
evidence may not be allowed anymore for failure to appear at the scheduled hearings. 26 December 1989:
Defense mailed a demurrer to evidence or motion to dismiss on insufficiency of evidence. 10 January 1990:
182
Trial Court dismissed the motion, since the demurrer failed to contain a compelling reason to recall the
previous order. 6 February 1990: The court issued an order considering the cases terminated against Januario
and Canape, but granted a reservation to present evidence as regards Cid. 9 March 1990: Defense presented
Cid as witness. He said that a certain Raul Repe, Toto Sarita and Digo Sarreal approached him about the sale of
a jeep. He referred them to Vicente Pons who he thought would buy the jeep. 27 March 1990: The court denied
defense counsel’s motion to cancel the hearing that day. Since Atty. Saunar was present, the trial court ordered
that his testimony be heard that day. Here, Saunar said that Atty. Vela, an NBI agent, approached him. Vela,
along with Atty. Toribio told him that Januario, Canape and Sid verbally confessed to participation in a crime,
and they were about the execute their sworn statements, so they needed his assistance. Saunar agreed and
explained to the three the consequences of their confession. He told them their constitutional rights, the
Miranda rights, specifically. Prosecution reminded the court that Saunar can’t be presented as witness, so they
consider him only as “additional evidence for the prosecution and/or rebuttal” testimony. 11 May 1990:
Defense manifested that it was closing its case.

Issue:
1. Was the admission of the testimony of Atty. Carlos Saunar proper?
2. Were the extra judicial confessions of Januario and Canape admissible as evidence?

Held:
1. Yes.
2. No.

Rationale:
Rule 119 of the Rules of Court shows the order of trial. The order is followed, but strict observance of the
rules depends upon the circumstances of the case, at the discretion of the trial judge. Therefore, the court may
allow the prosecutor to still present involuntarily omitted evidence. Saunar's testimony was considered as a
rebuttal witness with respect to Cid, so it was considered. - Atty. Saunar was not the choice of Januario as his
custodial investigation counsel. - Even if he can be considered as a competent counsel, he is not independent
because at that time, he was applying for a position in the NBI, so his loyalty would not be to the accused but to
NBI. Section 12(1) of Article II of the Constitution states that admission of facts related to a crime must be
obtained with assistance of counsel, otherwise it would be inadmissible. An admission, under Section 26 of
Rule 130 is "an act, declaration, or omission of a party as to a relevant fact". This is different from a confession,
which is defined in Section 33 as a "declaration of an accused acknowledging guilt of he offense. Januario and
Canape made verbal admissions of complicity in the crime. But such verbal admissions must be made with
assistance of counsel. They were not made with assistance of counsel when they made it in Naga City. People vs
Alicando: There is a "libertarian exclusionary rule known as the fruit of the poisonous tree, where once the
primary source (the ‘tree’) is shown to have been unlawfully attained, any secondary or derivative evidence (the
‘fruit’) derived from it is also inadmissible. Judgement: Judgement: Januario and Canape are acquitted.

People v Deniega 251 SCRA 626 (637)

Facts: The accused-appellants were convicted of rape and homicide. The prosecution was based solely on the
alleged extrajudicial confessions taken by the police officers without the presence of a counsel during custodial
investigation. It was also notable that the prosecution did not present any witness to the actual commission of
the crime and the basis of the lower court’s conviction to the accused was based on their alleged extrajudicial
confessions.

Issue: Whether or not the lower court erred in convicting the appellants based on their extra-judicial
confession.

Held: The court held that under rules laid down by the Constitution and existing law and jurisprudence, a
confession to be admissible must satisfy all of four fundamental requirements: 1) the confession must be
voluntary 2) the confession must be made with the assistance of competent and independent counsel; 3) the
confession must be express and 4) the confession must be in writing.
183
The court noted that the assistance of a counsel provided for the accused was inadequate to meet the
standard requirements of the constitution for custodial investigation. It seems that the lawyers were not
around throughout the custodial investigation. Citing People vs Javar, the court reiterated that any statement
obtained in violation of the constitutional provision, or in part, shall be inadmissible in evidence. “Even if the
confession speaks the truth, if it was made without the assistance of counsel, it becomes inadmissible in
evidence regardless of the absence of coercion or even if it had been voluntarily given.” Thus, because of these
defects in observing the proper procedural requirements of the constitution on custodial investigation the
accused-appellants were acquitted.

16. Rights of the Accused

17. Writ of Habeas Corpus and Writ of Amparo


R.A. 9851 (2009), Philippine Act on Crimes Against International Humanitarian Law, Genocide, and
Other Crimes Against Humanity

18. Self-Incrimination Clause

19. Double Jeopardy

20. Ex Post Facto Laws and Bills of Attainder

21. Citizenship

184

You might also like